Sunteți pe pagina 1din 550

SAT Critical Reading

and Writing Workbook

Triumph College Admissions


Skill Lesson

Quiz Time!
A

Hints!

Answers!
A

*Throughout this documentation, and the software: College Board and SAT are registered trademarks of the College Entrance Examination Board. PSAT/NMSQT
is a trademark of the College Entrance Examination Board and National Merit Scholarship Corporation. ACT is a trademark of ACT, Inc. None of these entities are
affiliated with the production of, nor endorse these materials.
Copyright 2009, TCA, LLC.

Triumph College AdmissionsTM License Agreement


Any student, teacher, or employee of a school that has licensed this copy of the Skills Workbook from
Triumph College Admissions may make a copy of this workbook for direct personal use.
Any copies made of the workbook or portions of this workbook MUST CONTAIN THIS PAGE.
This workbook is protected by both U. S. copyright law and international treaty provisions. Providing
copies to persons not affiliated with the school licensed to use this material is expressly forbidden.
This license is not assignable and is non-transferable, except according to the provisions provided for above.

Warranty
Except as provided for above, Triumph College Admissions specifically disclaims all warranties, expressed or
implied, of merchantability and fitness for a particular purpose with respect to defects in these materials, and
the license granted herein in particular, and without regarding limitation use of the workbook license with respect
to any particular application, use, or purpose. In no event will Triumph College Admissions be liable for
damages resulting from use of this product, including (but not limited to) loss of profit and special, incidental,
consequential, or other similar claims or damages. This statement shall be construed, interpreted, and
governed by the laws of the State of Virginia.

Copyright and Trademark Notices


Copyright 2007 by TCA, LLC, dba Triumph College Admissions (800) 233-4728
All rights reserved. Except as noted in the License Agreement above, no part of this publication may be
reproduced, stored in a retrieval system, or transmitted in any form or by any means, electronic, mechanical,
photocopying, recorded, or otherwise without prior written permission of TCA, LLC. Printed in the
United States of America. Portions of math sections are copyright Julie Chedester, Arizona. Portions of
reading sections are copyright Elizabeth S. Carson, Arizona. Some source material used under license of
Harcourt Brace & Company. The reading passages have been edited for suitability for testing purposes.
Triumph Coach is a trademark of Triumph Learning, used under license agreement. Triumph College Admissions
is not affiliated with Triumph Learning.
Throughout this documentation and the software College Board and SAT are registered trademarks of the
College Entrance Examination Board. PSAT/NMSQT is a trademark of the College Entrance Examination
Board and National Merit Scholarship Corporation. ACT is a trademark of ACT, Inc. None of these entities are
affiliated with the production of, nor endorse, these materials.

Triumph College Admissions Policy Concerning Workbook Errors


It is our policy to produce error-free documentation and products. If you find an error, please report it to Customer
Support at(800) 233-4728, extension 5. We will attempt to correct reported errors and make such corrections
available at little or no cost to current registered customers.

Copyright 2009, TCA, LLC.

Critical Reading Lesson #1


Sentence Completions

Triumph College Admissions


Skill Lesson

Quiz Time!
A

Hints!

Answers!
A

*Throughout this documentation, and the software: College Board and SAT are registered trademarks of the College Entrance Examination Board. PSAT/NMSQT
is a trademark of the College Entrance Examination Board and National Merit Scholarship Corporation. ACT is a trademark of ACT, Inc. None of these entities are
affiliated with the production of, nor endorse these materials.
Copyright 2009, TCA, LLC.

Copyright 2009, TCA, LLC.

Studyguide for the SAT Skill Review: Sent. Comp.

SENTENCE COMPLETION OVERVIEW


You must use reasoning to determine the relationship between sentence parts
in the Sentence Completion subsections.
The skill lessons and accompanying quizzes that follow this overview are
designed to provide you with strategies, concepts, important signal
words, and practices before you take the SAT.
The review and practice sections that follow this overview contain the
four types of sentence completions found on the SAT: definitional,
contrast, cause-effect, and synonym. In each of the four review lessons,
you will find important information related to one specific type of
sentence completion. You will also find links to all strategies you
should use for sentence completions and an extensive list of the most
frequently occurring vocabulary used on the SAT. You can also access
strategies from the Table of Contents, and be sure to review the
all-important SAT vocabulary.

Key Components in Sentence Completion Review Lessons


Each sentence completion review lesson takes you through the following
strategies for success:
Signal Words
Key Words and Phrases
Positive and Negative Words
Sentence Direction
In addition, each lesson contains tips, hints, and explanations to
increase your understanding of the sentence completion type before you
take the quizzes.

Copyright
2007, TCA,
TCA, LLC.
LLC.
Copyright 2009,

55

Studyguide for the SAT Skill Review: Sent. Comp.


The Sentence Completion Structure
Each sentence completion question consists of a single sentence with one
or two blanks in it. This is followed by a group of five possible
answers. The sentence may be short and grammatically simple or it may be
long and complex. Each answer will contain the word or words that you
are to plug into the blanks in the sentence. If the sentence contains
one blank, possible answers will look like this:
(A) bellicose
If it's a two-blank sentence, the answer will look like this:
(A) bellicose...argument

What the Questions Require


Sentence completions are designed to test your knowledge of word
meanings and your verbal reasoning ability. This means that you must
have a strong vocabulary and that you must understand the logic of a
sentence.
Strategize to Maximize
Attack each sentence completion in the following way:
1. Read the sentence carefully.
2. Locate the signal word or words.
3. Locate the key words or phrases.
4. Map the direction of the sentence.
5. Use the positive-and-negative-word strategy.
6. Recall vocabulary skills, especially regarding prefixes, suffixes, and roots.

Copyright 2009, TCA, LLC.

Studyguide for the SAT Skill Review: Sent. Comp.


These practices and strategies will be developed in greater detail in
each sentence completion lesson that follows this general overview.
Although this may seem like an extensive list of things to do, with
practice you will be surprised at how quickly you can scan for these. By
using these procedures, you are strategizing to maximize your success.
Your improved scores will please you!
Two-Blank Sentences
When a sentence completion contains two blanks, you have to plug in the
words in the same order as they appear in the answer; the first word
goes in the first blank and the second word goes in the second blank.
You may not switch them around.
Now you are ready to dig into the actual lessons and take practice
quizzes on the four types of sentence completions.

Copyright 2009, TCA, LLC.

Copyright 2009, TCA, LLC.

Studyguide for the SAT Skill Review: Definitional

DEFINITION SENTENCES
In a definition sentence, a part of the sentence defines the meaning of
the missing word. Your task is to find the definition tucked within the
sentence and choose a word that matches that definition.
Most of the time, the definition for the missing word will be found in
key words within the sentence. The first step in selecting the correct
word is to locate the key definition words.

Use Key Words


The following example is a definition sentence.
Gary was confused by the dilemma in which he was caught so he sought his
mentor, a man who was wise and experienced in solving such problems.
The first step is to locate the key definition wordsin this example, a
man who was wise and experienced. These key words define a mentor,
someone who provides guidance and is wise and experienced.

Examples
Here are two more examples of definition sentences that contain key
words to guide you to the correct answer. Before you look at the
solutions, try to fill in the blank with your own word choice.
Example 1
The attorneys - - - - the contract, adding to the terms, expanding the
permits, and altering some of the language.
(A) curtailed
(B) amended
(C) enacted
(D) voided
(E) compressed

Copyright 2009, TCA, LLC.

Studyguide for the SAT Skill Review: Definitional

Explanation:
The answer is (B) amended.
(A) Curtail means to curb or rein in. This word does not fit the
definition because the attorneys are adding.
(B) Amended means to change. This word fulfills the definition of
adding, expanding, and altering the contract.
(C) Enact means to decree, legislate, or pass a law. This word is
related to the legal world, but according to the definition within the
key words, you are seeking a word that suggests changing a contract.
(D) Voided means to declare something invalid. The attorneys are adding
and refashioning, not passing a judgment to cancel the contract.
(E) Compressed means to condense, but the key words indicate that the
attorneys are adding terms. This answer communicates the opposite of
the sentence's direction.

Example 2
She was honest, open, and frank during her testimony, so much so that
her - - - - helped solve the mystery.
(A) reservation
(B) blunder
(C) excitement
(D) candor
(E) reluctance

Tip: Sometimes signal words are also used in definition sentences, but
they do not appear with as much frequency as signal words for other
types of sentence completions. It is, however, important to recognize
them when they do appear. Examples of these signal words are that is, in
that it, in other words, and likewise.

Hint:
The key words that define the missing word are honest, open, and frank.
What word is defined by these key words?

Copyright 2009, TCA, LLC.

10

Studyguide for the SAT Skill Review: Definitional

Explanation:
The answer is (D) candor.
(A) Reservation suggests reluctance. This choice is the opposite of the
key words openness, honesty, and frankness.
(B) Blunder means to make an error. The direction of the sentence is
the opposite.
(C) Excitement has nothing to do with being truthful and open, so there
is no logical relationship between this answer choice and the sentence.
(D) Candor means truthfulness, honesty, and openness. Thus, the key
words in the opening of the sentence are the definition of candor.
(E) Reluctance means hesitance or unwillingness. Someone who is
reluctant is unlikely to behave with openness and frankness.

Use the Positive-and-Negative-Word Strategy


Sometimes, it may be difficult to isolate specific key words. In these
types of definition sentences, using the Positive-and-Negative-Word
Strategy to map the direction of the sentence can provide you with the
correct answer.

How to Map Sentence Direction


If you are not familiar with the idea of mapping the direction of a
sentence, read the following information carefully.
When you map the direction of a sentence, what you are looking for is an
order within the flow of sentence parts. All sentences have this order.
Some move from positive (+) to negative (-); some move from positive (+)
to positive (+); some move from negative (-) to negative (-); some move
from negative (-) to positive (+). In other words, + to - means that the
sentence opens with a positive idea then shifts to a negative one.

Copyright 2009, TCA, LLC.

11

Studyguide for the SAT Skill Review: Definitional


If the sentence completion has only one blank and the direction of the
sentence is positive, look for a + word in the answer choices and
quickly eliminate any - words. For example:
These barbs by the chairman are - - - - and create hurt feelings
in the committee members.
(A) specific
(B) caustic
(C) generous
(D) casual
(E) careful
The movement of the sentence is negative, so we are looking for a
negative word. Only answer (B), caustic, is negative. Eliminate the
other answers quickly.
Use the same strategy if the sentence completion has two blanks.

Tip: When taking the SAT sentence completion section, it is a good


strategy to map the direction of the sentence first. Simply place a +
or - in the blank or blanks in your test booklet, then proceed with your
answer choice.

Use Vocabulary Skills


Some definition sentence completions contain challenging vocabulary.
Remember to use the skills you learned in vocabulary lessons to help
select the correct answer. Knowledge of prefixes, suffixes, and roots
can provide the required boost needed to make the correct choice. Study
the following example. Before you look at the solution, try to make your
own answer choice.
Example 3
The ancient Greeks and Romans were - - - - , worshipping any
number of gods and goddesses.
(A) agnostic
(B) moralistic
(C) monotheistic
(D) polytheistic
(E) hedonistic

Copyright 2009, TCA, LLC.

12

Studyguide for the SAT Skill Review: Definitional

Hint:
Study the prefixes used in the answer choices. What word uses a prefix
that means "many"?

Explanation:
The prefix poly means many. Because the ancient Greeks and Romans
worshipped many gods and goddesses, they are said to be polytheistic.
The worship of numerous gods and goddesses is the definition of
polytheistic.

Before you take a quiz on definition sentences, remember these important


practices:
*Use key words that define the missing word.
*Use the Positive-and-Negative-Word strategy.
*Map the direction of the sentence.
*Use vocabulary skills.

Copyright 2009, TCA, LLC.

13

Copyright 2009, TCA, LLC.

14

Studyguide for the SAT Skill Review: Contrast

CONTRAST SENTENCES
In a contrast sentence, one part of the sentence contrasts with
another sentence part. Because all sentences move in some type of
direction, in contrast sentences it is very important to identify this
direction.
The most important concept that you should recognize when working with a
contrast sentence is that one sentence part REVERSES another sentence
part. Two other words associated with this concept are CONTRADICTS and
OPPOSES.
Contrast Signal Words
A good strategy to use for all sentence completion types is to begin by
locating the key signal words. Notice how signal words for contrast
sentences not only indicate a contrast, but also establish the direction
of the sentence. The most common signal words for contrast sentences are
the following:
on the other hand
rather than
conversely
however
yet
instead of

Copyright 2009, TCA, LLC.

but
despite this/the
even though
unless
nevertheless

15

Studyguide for the SAT Skill Review: Contrast


Key Words or Phrases
Another important strategy to use for all sentence completion types is
to identify the key words or phrases in the sentence. This is very
important in contrast sentences. The missing word or words you have to
select will contrast with the key word or words. Because of this
contrast, you will be able to see a reversal in the direction of the
sentence.
Study the following example of a contrast sentence. The signal word is
underlined, and the key words are in italics.
Rather than study the material in depth, Juanita gave the papers a
- - - - glance.
The signal words rather than establish the sentence direction, and the
missing word will reverse that direction by creating a contrast to the
key words in depth. Consider the answer choices below. What word is the
best contrast for the key words in depth ?
(A) detailed (B) careful (C) studied (D) cursory (E) remedial

Did you select choice (D), cursory? Cursory means hastily or


quickly. The best contrast to the key words in depth is choice (D).
Notice how cursory reverses the direction of the sentence. You can
quickly eliminate detailed, careful, and studied as possible answers
because they mean the same as the key words in depth. It is impossible
to give anything a remedial glance. This choice is not logical.
Here is another example of a contrast sentence. The signal word is
underlined and the key word is in italics.
The scientists had expected the plants to - - - - with the new
fertilizer; instead, the leaves began to wither and the stem to wilt.
The signal word instead establishes the sentence direction, and the
missing word will reverse that direction by creating a contrast to the
key words wither and wilt. Consider the answer choices below. What word
is the best contrast to the key words wither and wilt?
(A) die (B) flourish (C) desiccate (D) terminate (E) resist

Copyright 2009, TCA, LLC.

16

Studyguide for the SAT Skill Review: Contrast


Did you select choice (B), flourish? Flourish means to thrive. It is
the best contrast word to the key words wither and wilt. Notice how
flourish will also change the direction of the sentence. The other
choices can be quickly eliminated because we associate them with
withering and wilting. They are not contrasts.
Now you try it. Read the following two contrast sentences. Look for the
signal words, sentence direction, and key words to aid in selecting the
correct answer.
Example 1.
At first the doctors were - - - - by the patient's symptoms;
however, the test results clarified the nature of the illness.
(A) scornful
(B) intrigued
(C) baffled
(D) impressed
(E) harassed

Hint:
Notice the signal word however and the key word clarified. What
word will change the direction of the sentence and contrast with the key
word clarified ?

Explanation:
The correct answer is (C) baffled.
(A) Scornful suggests contempt and disdain. Its contrast would be
accepting , not clarified . (B) intrigued suggests interest, attraction,
or enchantment. It does not contrast with clarified . (C) Baffled means
puzzled, confused, or confounded. This contrasts with the key word
clarified, meaning to make clear. (D) Impressed means to be mentally or
emotionally moved, stirred, or affected. Impressed does not contrast
with clarified . (E) Harassed means annoyed or bothered. Because the
symptoms are puzzling, the doctors may feel annoyed, but harassed does
not directly contrast with clarified.

Copyright 2009, TCA, LLC.

17

Studyguide for the SAT Skill Review: Contrast

Example 2.
The - - - - storms that raged for nearly a month were replaced by
- - - - days of warm afternoons and cool nights.
(A) tempestuous...temperate
(B) violent...turbulent
(C) noisy...boisterous
(D) irritating...lackluster
(E) languid...sluggish

Tip: You will not find a signal word in this sentence. Not all
sentences have them; instead, you will find words that suggest one of
the signal words. In this sentence, the words were replaced by take the
place of a signal word.

Hint:
The key words are storms, raged, warm, and cool. What set of words
best contrasts with these key words?

Explanation:
The correct answer is (A), tempestuous...temperate.
(A) Tempestuous suggests violence and turbulence. In contrast,
something that is temperate is moderate and mild. (B) Turbulent can be
described as violent. Thus the two words in this answer choice move in
the same direction. You are looking for words that contrast with each
other. (C) Noisy and boisterous are synonyms. You are looking for words
with opposite meanings. (D) Irritating and lackluster have little to do
with each other. Irritating means annoying, and lackluster means dull
or lacking in brilliance. (E), Languid and sluggish, are synonyms. You
are looking for a contrasting pair of words.

Copyright 2009, TCA, LLC.

18

Studyguide for the SAT Skill Review: Contrast


Before you take the first quiz on contrast sentences, remember the
important strategies we have reviewed:
Locate the signal words. Identify the direction in which the sentence is
moving. Locate the key word or words.
Now you should be ready to take the first quiz and be successful in
practicing your skills at selecting the correct answer for contrast
sentences.

Copyright 2009, TCA, LLC.

19

Copyright 2009, TCA, LLC.

20

Studyguide for the SAT Skill Review: Cause and Effect

CAUSE AND EFFECT SENTENCES


Cause and Effect Signal Words
Always begin by looking for the signal word that helps to identify the
type of sentence completion. The list below contains the most common
signal words for cause and effect sentences.
because
as a result
since
thus
consequently if...then

hence
therefore

in order to
so

Examine the sentence below.


Because she was - - - - with tools and machines, she earned a good
reputation for tuning engines effectively.
(A) capable
(B) adept
(C) clumsy
(D) able
(E) awkward

Hint:
Notice the major signal word because is used to introduce the causeeffect relationship. Also, note the key words with tools and machines
and for tuning engines effectively. What is the answer that best fits
the logic of the sentence?

Explanation:
The signal word because points to the cause, and the second part of the
sentence states the effect. She is adept, or skillful, so this results
in her earning a good reputation.

Copyright 2009, TCA, LLC.

21

Studyguide for the SAT Skill Review: Cause and Effect


Use the Positive-and-Negative-Word Strategy
It is always important to remember that words carry more than their
denotation or dictionary definition. They also carry connotations that
are mental associations or suggestions. These associations or
suggestions are either negative or positive.
For example, the word malignant has a very negative denotation.
Likewise, the words war, infuriate, ornery, miserly, forlorn, and
careless have negative connotations.
On the other hand, we associate positive thoughts with such words as
lively, charitable, supportive, applaud, and generosity. In other
words, they have positive connotations.
By determining if the key words carry a positive or negative denotation or
connotation, you can quickly eliminate some answers.
Degree of Word Intensity
Have you ever wondered why the word you selected fits into the sentence
but another word is considered better? This often happens to students
and frequently occurs in the Sentence Completion section of the SAT.
All words, in addition to having denotation and connotation, also have a
degree of intensity. Always select the highest degree if more than one
word appears to be the correct answer.
For example, in the answer choices supplied for the first sentence we
examined, the words capable, adept, and skillful are all positive and
appear to be correct, but of the three, only adept has the highest degree
of intensity for the general concept involving ability. Adept means
clever or expert and has a higher degree of intensity than being just capable or
skillful .

Copyright 2009, TCA, LLC.

22

Studyguide for the SAT Skill Review: Cause and Effect


Double-Blank Sentence Completions
Because cause and effect sentence completions frequently use two blanks, it
is important to know a strategy that can help you eliminate incorrect
answers. Once again, the positive-and-negative-words strategy
comes to the rescue.
Step 1: Determine whether the blanks in the sentence require a positive
or a negative word. Place a + in the blank if you need a positive word
or a - in the blank if you need a negative word. Do this for both
blanks. You now know the direction of the sentence.
Step 2: All you need to do is scan the first word of the pairs in each
answer. If the first word does not match the direction of the first
blank, eliminate the entire answer. With those answers that remain,
quickly scan in order to eliminate choices in which the second word
moves in a different direction from that of the second blank.
Result: You should be able to eliminate at least two answers very
quickly through simple connotations of + or -. In fact, you don't even
need to have a complete understanding of each word's meaning! Learn to
rely on your general feeling as to whether a word seems positive or
negative. You will be right far more times than wrong.
Tip: Follow the same directions for single blank sentences. In
this case you only have to fill one blank with a + or - !

Here are more examples for additional practice before you take the first
cause and effect quiz.
Remember:
1. Determine the signal word.
2. Locate the key words.
3. Use the positive-and-negative-word strategy.
4. Determine the degree of intensity of the word or words in the answer choices.

Copyright 2009, TCA, LLC.

23

Studyguide for the SAT Skill Review: Cause and Effect


Example 1
Because the investigators can rely on the - - - - of the
witnesses' testimony, the case will be solved quickly.
(A) summary
(B) accuracy
(C) graciousness
(D) gallantry
(E) sparseness

Hint:
Notice the major signal word because and the key words can rely on,
witnesses' testimony, and solved quickly. The blank also requires a +
word.

Explanation:
The answer is (B), accuracy.
(A) A summary will be brief and condensed. Logically, it does not
follow that because a testimony is brief it will help quickly solve
a case. Eliminate this answer. (B), accuracy, means exactness and
correctness. It is logical that accurate testimony will result in a
case being solved quickly. (C), graciousness, suggests kindness or
courtesy. Such testimony may be delivered politely, but it does not
follow logically that it will help solve a case. Eliminate this answer.
(D), gallantry, suggests bravery and heroism; however, gallantry will not
result in a solution. Again, eliminate this answer. (E), sparseness
suggests sketchiness and meagerness. This answer choice moves in the
opposite direction from the desired effect. It is negative (-).
Eliminate this answer.

Copyright 2009, TCA, LLC.

24

Studyguide for the SAT Skill Review: Cause and Effect


Example 2
The referendum was so controversial that it - - - - the voters,
creating a rift that divided the city.
(A) startled
(B) humiliated
(C) gratified
(D) snubbed
(E) polarized

Hint:
Notice the signal word so and the key words controversial, rift, and
divided. These words are negative (-) and require a negative word in the blank.

Explanation:
The answer is (E), polarized.
(A) Something that is controversial may be startling or surprising;
however, surprise does not necessarily result in a rift or division.
Eliminate this answer. (B) Similarly, a controversial referendum may
embarrass some voters. It may even humiliate them. It does not
logically follow that a rift will develop. (C), gratified is a + word.
Eliminate this answer. (D), snubbed means to treat with scorn or
contempt. Individuals may snub others, but things such as referendums
do not. This answer does not make sense. (E), polarized means to
separate into strongly opposing groups. If a referendum is
controversial, then voters may become divided on the issue. The likely
result is a rift.

Copyright 2009, TCA, LLC.

25

Studyguide for the SAT Skill Review: Cause and Effect


Example 3
Although earthquakes are among the most - - - - of natural
disasters, these shifts in the earth's plates are often difficult to
- - - - , either because they are deep beneath the surface or
because determining when an earthquake will occur is still not a precise
science.
(A) remote...observe
(B) ominous...detect
(C) predictable...foresee
(D) ancient...determine
(E) violent...disregard

Hint:
Notice the major signal word because is used twice. Also
notice the key words disasters and difficult. The first blank requires a
negative (-) word and the second blank requires a positive (+) word.

Explanation:
The answer is (B), ominous...detect.
(A) Earthquakes are always thought of as extremely negative. Remote
does not suggest this intensity. Eliminate this answer. (B), ominous
means threatening or foreboding. It is the most negative word in the
list. (C) Earthquakes are not yet predictable. This word contradicts
the sentence meaning and is not logical. Eliminate this answer.
(D) Although earthquakes have occurred since ancient times, there is no
negative value here. Eliminate this answer. (E) Yes, earthquakes are
violent, but we do not disregard their destructive power. Eliminate this
answer.

Copyright 2009, TCA, LLC.

26

Studyguide for the SAT Skill Review: Synonym

SYNONYM SENTENCES
In a synonym sentence completion, the missing word is a synonym or is
thought to be synonymous with a key word or phrase in the sentence. When
you select the correct answer to fill the blank(s), the missing word
will reinforce or emphasize the key word by adding additional synonymous
support.
Hint: In some synonym sentence completions,
the missing word(s) is(are) not an exact synonym,
but rather a word closely related to the key word.

Use Key Words


In synonym sentence completions, it is critical to identify the key word
because the word you are looking for is a synonym or closely related
word. Read the sentence that follows. The key word is in italics and the
synonym is underlined.
The inexperienced artist so admired the paintings of Thomas More that he
attempted to imitate the style; in fact, he mimicked every brushstroke
in order to create copies of the original paintings.
Because the artist attempted to imitate More's style, he mimicked every
brushstroke. Mimic means to imitate something closely. You can see how
this word reinforces and continues to emphasize the key word imitate.

Copyright 2009, TCA, LLC.

27

Studyguide for the SAT Skill Review: Synonym


Now you try it. Read the sentence that follows. Locate the key word
first, then review the answer choices to determine which word is the
best synonym or a word closely related to the key word.
Shanika scolded her brother for damaging her CD's; furthermore, she
- - - - him for not asking permission to borrow them in the
first place.
(A) forgave
(B) reprimanded
(C) commended
(D) appeased
(E) apprehended

Hint:
The key word is scolded. What word is the best synonym for the
key word scolded?

Explanation:
Choice (B), reprimanded, is the correct answer.
If you reprimand someone, in effect, you scold that person. You can
quickly eliminate the words forgave, commended, and appeased because
they have opposite meanings from the key word scold. Choice (E) cannot
be correct because apprehend means to locate and take someone into
custody. Shanika knew where to find her brother!

Copyright 2009, TCA, LLC.

28

Studyguide for the SAT Skill Review: Synonym

Use Signal Words


Locating the signal words for synonym sentence completions is an
important strategy. The most common signal words for synonym sentences
are listed below.
and
also
furthermore
beyond this
in fact
in a similar manner

additionally
as well as
moreover
indeed
in a like manner
just as

Hint: By far the most frequently used signal word


for synonym sentence completions is the word and.

Typically, in a synonym sentence completion, the missing word will


follow the key word closely. The important signal word and will
be used to join the key word and the answer. Study the following sentence. What word
is the best synonym for the key word reverse ?
Franklin Roosevelt created numerous government programs during the
1930's to help reverse and - - - - the Depression.
(A) antagonize
(B) ridicule
(C) dominate
(D) unwind
(E) nullify

Hint:
Notice the signal word and along with the key word reverse. What
word is closely related to the key word reverse ?

Copyright 2009, TCA, LLC.

29

Studyguide for the SAT Skill Review: Synonym

Explanation:
The correct answer is (E), nullify. To nullify means to
void something. Reversing the Depression would help to nullify it. By
studying the other answer choices, you will see that the words have no
synonymous relationship to the key word reverse.
Use the Positive-and-Negative-Word Strategy
Remember that all sentences move in some specified direction. Another
strategy to help you select the correct answer in synonym sentence
completions, especially those answer choices containing challenging
vocabulary, is the use of the positive-and-negative-word strategy.
In a synonym sentence completion question, the nature of the missing
word will match the key word. If the key word is negative, the missing
word will be negative. If the key word is positive, the missing word
will be positive.
Study the following sentence and try using the positive-andnegative-word strategy to identify the correct answer.
The angry old hermit was notorious in his town for being spiteful and
----.
(A) benevolent
(B) benign
(C) amicable
(D) malevolent
(E) gregarious

Hint:
The key word spiteful is a negative (-) word. What word is
synonymous with spiteful and is also a negative word?

Copyright 2009, TCA, LLC.

30

Studyguide for the SAT Skill Review: Synonym

Explanation:
Did you select choice (D), malevolent , as the correct answer?
Malevolent means hateful. Both spiteful and malevolent are
negative words. By using the positive-and-negative-word strategy, you
can see that the other answer choices are all positive and can be
quickly eliminated, even if you do not know the meaning of all of them.

Hint: You will never see a positive (+) key word and a negative (-)
missing word. That kind of sentence direction would contradict the
concept of a synonymous or similar idea.

Copyright 2009, TCA, LLC.

31

Copyright 2009, TCA, LLC.

32

Studyguide for the SAT Skill Quiz A: Sent. Comp.

Quiz Time!
A

Question 1.
In time, the settlers all became - - - - : the many gods they
once worshipped were ultimately replaced by a single deity.
(A)
(B)
(C)
(D)
(E)

agnostics
charlatans
monotheists
capricious
expatriates

Question 2.
The newscaster's public - - - - reflect his neurotic need to impress
others and, perhaps, to mask his insecurities.
(A)
(B)
(C)
(D)
(E)

affectations
courtesies
flummeries
segues
banalities

Question 3.
The climber was - - - - in her ascent of the sheer granite wall;
nothing short of - - - - would stop her from reaching the top.
(A)
(B)
(C)
(D)
(E)

Copyright 2009, TCA, LLC.

timorous...apathy
inflexible...blisters
resolute...calamity
notorious...clemency
relentless...acclaim

33

Studyguide for the SAT Skill Quiz A: Sent. Comp.


Question 4.
The normally - - - - Mr. Gonzalez stunned his office
colleagues by buying them all lunch.
(A)
(B)
(C)
(D)
(E)

amiable
magnanimous
worldly
lax
miserly

Question 5.
At one time, rural America was the target of - - - - who
came to town to sell their phony panaceas.
(A)
(B)
(C)
(D)
(E)

assassins
quacks
toadies
juntas
prospectors

Question 6.
Cows are very - - - - animals, preferring to remain in herds
rather than venture from the barn alone.
(A)
(B)
(C)
(D)
(E)

solitary
despondent
perfidious
furtive
gregarious

Question 7.
Withdrawn and shy by nature, Juan became even more - - - - when
confronted by strangers.
(A)
(B)
(C)
(D)
(E)

Copyright 2009, TCA, LLC.

succinct
taciturn
despondent
apathetic
extroverted

34

Studyguide for the SAT Skill Quiz A: Sent. Comp.

Hint for Question 1:

(Definitional): The key group of words is a single deity. Which word


describes people who believe in one god?

Hint for Question 2:

characterizes a behavior an insecure person might show?

Hint for Question 3:

(Definitional): Use the positive-and-negative-word strategy to


determine the correct answer. The direction of this sentence is
positive (+) moving to negative (-). Which set of words follows such a
pattern?

Hint for Question 4:

(Definitional): The key group of words is stunned his office colleagues


by buying them all lunch . This description is offset by the contrast
word normally . Using this contrast, the word you are looking for is
negative (-). What word describes a person who normally would not buy
lunch for anyone?

Hint for Question 5:

(Definitional): The key words are phony panaceas. Panacea means cureall. What word is associated with someone who tries to sell phony
medicine?

Hint for Question 6:

(Definitional): The key words are preferring to remain in herds. What


word describes animals that prefer to stay together?

Copyright 2009, TCA, LLC.

35

Studyguide for the SAT Skill Quiz A: Sent. Comp.


Hint for Question 7:

(Definitional): The key words are withdrawn and shy. What word
describes a shy person confronted by strangers?

Copyright 2009, TCA, LLC.

36

Studyguide for the SAT Skill Quiz A: Sent. Comp.

Answers!
A

Question 1.
The answer is (C).
Explanation for Question 1:

People who believe in one god are called monotheists. Remember from
vocabulary study that mono means one.

Question 2.
The answer is (A).
Explanation for Question 2:

An insecure person may show affectations in an attempt to impress


others. Remember from vocabulary study to look for root words; in this
case, the word affect means to make an impression on.

Question 3.
The answer is (C).
Explanation for Question 3:

Only answer (C) follows the pattern of positive (+) and then negative (-).
Although answer (E) may appear correct, the second word choice is
positive instead of negative. To be resolute is a positive attribute
meaning to be strongly determined, and a calamity is a very strong word
to characterize some type of disaster. Falling from a sheer granite
wall would be described as a calamity.

Question 4.
The answer is (E).
Explanation for Question 4:

Only two words are negative, lax and miserly. Eliminate the other
answers. Someone who normally would not buy lunch for anyone must want
to hoard money, the definition of the word miserly. Eliminate lax as a
faulty definition.

Copyright 2009, TCA, LLC.

37

Studyguide for the SAT Skill Quiz A: Sent. Comp.


Question 5.
The answer is (B).
Explanation for Question 5:

This question contains challenging vocabulary. By process of


elimination, assassins, toadies, juntas, and prospectors are not
associated with medicine. This leaves only answer (B), quacks. This
negative term was given to people who tried to sell phony panaceas in
rural America.

Question 6.
The answer is (E).
Explanation for Question 6:

Gregarious means to prefer the company of one's kind. Because cows


prefer to remain in herds, they are said to be gregarious animals.

Question 7.
The answer is (B).
Explanation for Question 7:

Taciturn means not inclined to talk. Someone who is normally withdrawn


and shy would also be taciturn when confronted by strangers.

Copyright 2009, TCA, LLC.

38

Studyguide for the SAT Skill Quiz B: Sent. Comp.

Quiz Time!
A

Question 1.
The - - - - generated by the benefit concert will be applied to the
charitable organization's operating costs.
(A)
(B)
(C)
(D)
(E)

goodwill
expenses
publicity
revenues
commitments

Question 2.
When the fire bell rang, the new teacher followed the directions
specified in the faculty handbook: she calmly - - - - her fourthgraders to go quietly to the designated safety area.
(A)
(B)
(C)
(D)
(E)

begged
berated
instructed
forced
enticed

Question 3.
One - - - - issue for a professor employed by a university for
more than five years is whether she will be given - - - - , for this
virtually guarantees her employment for the rest of her career.
(A)
(B)
(C)
(D)
(E)

Copyright 2009, TCA, LLC.

important...degrees
conflicting...substance
inane...benefits
redundant...compensation
critical...tenure

39

Studyguide for the SAT Skill Quiz B: Sent. Comp.


Question 4.
Noise performs a - - - - in nature; it may - - - - as a
warning or a means of identification.
(A)
(B)
(C)
(D)
(E)

deterrent...suffice
disservice...operate
function...serve
process...decry
job...officiate

Question 5.
The - - - - of the chairperson was predictable; his successor
possessed a superior record of sales.
(A)
(B)
(C)
(D)
(E)

ouster
promotion
mitigation
commendation
eulogizing

Question 6.
According to some - - - - television viewers, the news media has
lost touch with what reporting should be: - - - - substantive information
with stylistically slick, often emotionally charged sound bites that
command an audience's attention while requiring little or no thought.
(A)
(B)
(C)
(D)
(E)

Copyright 2009, TCA, LLC.

angry...inducing
pensive...inculcating
disgruntled...supplanting
mawkish...presenting
captivated...superseding

40

Studyguide for the SAT Skill Quiz B: Sent. Comp.


Question 7.
Archeologists excavating the Meadowcroft Rock Shelter in Pennsylvania
have found numerous - - - - : arrowheads, spear points, and
carving stones used for practical purposes in daily life by prehistoric
Native Americans.
(A)
(B)
(C)
(D)
(E)

Copyright 2009, TCA, LLC.

tools
artifacts
objects
discoveries
layers

41

Copyright 2009, TCA, LLC.

42

Studyguide for the SAT Skill Quiz B: Sent. Comp.

Hints!
A

Hint for Question 1:

(Definitional): Notice the key words operating costs. What can a


concert produce that would pay the organization's bills?

Hint for Question 2:

(Definitional): Notice the key words faculty handbook. Why do school


administrators give faculty handbooks to all teachers?

Hint for Question 3:

(Definitional): Notice the key words issue for a professor employed by


a university for more than five years and virtually guarantees her
employment for the rest of her career. What word is defined by a reward
for five years of successful teaching?

Hint for Question 4:

(Definitional): Notice the key word performs. What word indicates the
correct movement of the sentence?

Hint for Question 5:

(Definitional): Notice the key words his successor possessed a superior


record of sales. More important, the movement of this sentence is
negative. What word fits this negative movement of the sentence?

Hint for Question 6:

(Definitional): Use the positive-and-negative-word strategy for this


sentence. The sentence movement is from negative to negative. What
words show this type of sentence movement?

Copyright 2009, TCA, LLC.

43

Studyguide for the SAT Skill Quiz B: Sent. Comp.


Hint for Question 7:

(Definitional): Notice the key words arrowheads, spear points, and


carving stones used for practical purposes in daily life. What word is
defined as things used by people for practical purposes ?

Copyright 2009, TCA, LLC.

44

Studyguide for the SAT Skill Quiz B: Sent. Comp.

Answers!
A

Question 1.
The answer is (D).
Explanation for Question 1:

Using the key words operating costs, only choice (D), revenues, is
correct. All other choices have nothing to do with paying bills.

Question 2.
The answer is (C).
Explanation for Question 2:

A faculty handbook is used to provide instructions regarding all aspects


of a school. One of those aspects is what to do in case of a fire.
You can also use the strategy of positive and negative words for this
question. A teacher would not beg, berate, force, or entice students.
These words are all negative. Only choice (C), instructed, is positive.

Question 3.
The answer is (E).
Explanation for Question 3:

Only the word tenure in choice (E) is defined as a guarantee of


employment for successful professors or teachers. None of the other
answers is defined in this way.

Question 4.
The answer is (C).
Explanation for Question 4:

By using the key word performs, you will see that the only words that
will fit the sentence movement are those in choice (C), function...serve. The
sentence movement is from positive to positive. You cannot perform a
deterrent or a process. The word disservice is negative, so eliminate it.
Noise can perform a job, but it cannot officiate, so eliminate this
answer as well.

Copyright 2009, TCA, LLC.

45

Studyguide for the SAT Skill Quiz B: Sent. Comp.


Question 5.
The answer is (A).
Explanation for Question 5:

Only choice (A), ouster, is a negative word. All of the other word
choices are positive and are not logical to the movement of the
sentence. By using the key words, it is logical to expect that someone
with a superior record could oust, or take the place of, someone who has
a less than superior record.

Question 6.
The answer is (C).
Explanation for Question 6:

Eliminate choices (A), (B), (D), and (E) because the words do not fit
the negative-to-negative movement of the sentence. Only choice (C),
disgruntled...supplanting, follows this type of negative-to-negative
sentence movement.

Question 7.
The answer is (B).
Explanation for Question 7:

Arrowheads, spear points, and carving stones used for practical purposes
in daily life define the word artifacts. None of the other choices
denote the word artifact.

Copyright 2009, TCA, LLC.

46

Studyguide for the SAT Skill Quiz C: Sent. Comp.

Quiz Time!
A

Question 1.
Henry thought of himself as a - - - - , but those who had watched
him devour any food set before him thought of Henry as a - - - - .
(A)
(B)
(C)
(D)
(E)

victual...raconteur
gourmand...glutton
chef...fishmonger
lummox...celebrity
bon vivant...egoist

Question 2.
Although his colleagues thrive on the - - - - of big-city life,
Professor Takimora feels more comfortable surrounded by the - - - of a secluded mountain valley.
(A)
(B)
(C)
(D)
(E)

vehemence...heat
ruination...chaos
excitement...strangeness
cacophony...dullness
vibrancy...tranquility

Question 3.
She was persistently - - - - by shouted barbs, but the
prisoner's calm demeanor never wavered as the sheriff marched her
through the village.
(A)
(B)
(C)
(D)
(E)

Copyright 2009, TCA, LLC.

regaled
taunted
bemused
depressed
embarrassed

47

Studyguide for the SAT Skill Quiz C: Sent. Comp.


Question 4.
In her letter to the editor, she detailed what she found - - - - ,
even though she - - - - the basic tenets of the editorial.
(A)
(B)
(C)
(D)
(E)

erroneous...disputed
authentic...understood
offensive...supported
objectionable...lambasted
accurate...applauded

Question 5.
While he - - - - the thought of daily medication, he realizes that
his ailment is a - - - - problem that will subside in a few
weeks.
(A)
(B)
(C)
(D)
(E)

appreciates...permanent
opposes...irrevocable
comprehends...transitional
detests...temporary
relishes...transient

Question 6.
Unlike Senator Bartlett, who - - - - the new trade agreement as an
economic boon for both countries, Representative Oldakowski rarely
misses the opportunity to - - - - it.
(A)
(B)
(C)
(D)
(E)

chastises...revoke
lauds...decry
assails...tout
extols...infuriate
assuage...oppose

Question 7.
After twenty years as director of the research project, he still does
not - - - - the fact that no definitive - - - - have been
reached.
(A)
(B)
(C)
(D)
(E)

Copyright 2009, TCA, LLC.

condone...questions
believe...obstacles
acknowledge...conclusions
comprehend...enigmas
deny...postulates

48

Studyguide for the SAT Skill Quiz C: Sent. Comp.

Hints!
A

Hint for Question 1:


(Contrast): Notice the signal word but and the key phrase devour any
food. Which answer choice contains the best set of words to contrast
Henry's view of himself with that of his peers?

Hint for Question 2:


(Contrast): Notice the signal word although and the key phrases bigcity life and secluded mountain valley. What sort of environmental
qualities establish a contrast between big cities and secluded mountain
valleys?

Hint for Question 3:


(Contrast): Notice the signal word but and the key phrases shouted
barbs and prisoner's calm demeanor. Which word choice creates the best
contrast between the shouted barbs and the surprisingly calm demeanor of
the prisoner?

Hint for Question 4:


(Contrast): Notice the signal words even though. Which words best
establish a contrast between negative and positive?

Hint for Question 5:


(Contrast): Notice the signal word while and the key phrases daily
medication and subside in a few weeks. Which answer choice contains
words that establish the best contrast between the man's attitude toward
daily medication and his understanding that the ailment will subside in a
few weeks?

Hint for Question 6:


(Contrast): Notice the signal word unlike. Which set of words
establishes the best contrast between Bartlett's and Oldakowski's
positions?

Copyright 2009, TCA, LLC.

49

Studyguide for the SAT Skill Quiz C: Sent. Comp.


Hint for Question 7:
(Contrast): There is no exact signal word in this question, so use the
contrast established by the key phrases he still does not, the fact
that, and no definitive to help make the correct choice. Which answer
choice establishes the best contrast between the director's position
after twenty years and what he is unwilling to realize?

Copyright 2009, TCA, LLC.

50

Studyguide for the SAT Skill Quiz C: Sent. Comp.

Answers!
A

Question 1.
The answer is (B).
Explanation for Question 1:

Gourmand...glutton. By using the contrast signal word but and the key
phrase devour any food, the best set of contrast words is choice (A),
gourmand...glutton. Someone who is a glutton will eat anything and
everything. None of the other choices works in the context of the
sentence.

Question 2.
The answer is (E).
Explanation for Question 2:
Vibrancy...tranquility. In this question, the best answer will use the
second word as an antonym for the first. Only choice (E) contains words
that establish a perfect contrast between big-city life and a secluded
mountain valley. Although choice (D) may appear correct, only the first
word could fit the sentence context. Dullness is not an antonym for
cacophony. Choices (A), (B), and (C) do not work in the sentence
context.

Question 3.
The answer is (B).
Explanation for Question 3:
Taunted. Although choice (A) may appear correct, regaled does not
have the negative meaning of taunted. Only choice (B) is correct. Choices
(C), (D), and (E) do not work in the sentence context.

Question 4.
The answer is (C).
Explanation for Question 4:
(Contrast): Only choice (C) uses words that create an accurate negative
then positive position regarding the editorial.

Copyright 2009, TCA, LLC.

51

Studyguide for the SAT Skill Quiz C: Sent. Comp.


Question 5.
The answer is (D).
Explanation for Question 5:
Detests...temporary. Only choice (D) establishes the correct contrast
between attitude and understanding that the ailment will subside in a
few weeks. No other set of words comes close to completing the sentence
context.

Question 6.
The answer is (B).
Explanation for Question 6:
Lauds...decry. Only choice (B) contains words that clearly establish a
precise contrast in two different positions. Again, this question asks the
test taker to use the concept of antonyms to select the correct words. Only
decry is the accurate antonym for lauds .

Question 7.
The answer is (C).
Explanation for Question 7:
Acknowledge...conclusions. The end goal of research is to arrive at
definitive conclusions. Only choice (C) establishes a sense of
contradiction in the research director's position after twenty years,
which is his unwillingness to acknowledge the fact that no definitive
conclusions have been reached. None of the other choices establishes this
contrast relationship.

Copyright 2009, TCA, LLC.

52

Studyguide for the SAT Skill Quiz D: Sent. Comp.

Quiz Time!
A

Question 1.
The comedian's brilliant - - - - brought a smile to the face of
even the most - - - - heckler in the audience.
(A)
(B)
(C)
(D)
(E)

joke...tolerant
repartee...churlish
discussion...reticent
treatise...loathsome
expression...difficult

Question 2.
Although a praying mantis looks - - - - , it is, in fact, a docile
insect that is beautifully colored and beneficial to plants because it
eats harmful insects.
(A)
(B)
(C)
(D)
(E)

furtive
amiable
menacing
exuberant
silky

Question 3.
Scientists warned that global warming would create new
problems on earth, some of them - - - - but others very
unexpected.
(A)
(B)
(C)
(D)
(E)

Copyright 2009, TCA, LLC.

unanticipated
accidental
predictable
costly
unhealthy

53

Studyguide for the SAT Skill Quiz D: Sent. Comp.


Question 4.
The importance of the Declaration of Independence is not its
- - - - signers, but rather their general belief in democratic
principles.
(A)
(B)
(C)
(D)
(E)

stoic
specific
debatable
conservative
strict

Question 5.
Although music video critics - - - - the new release as
uninspired and weak in lyrics and visual imagery, sales soared
in the 18 to 25 year-old market.
(A)
(B)
(C)
(D)
(E)

suppressed
valued
applauded
publicized
berated

Question 6.
Although many see it as exceedingly ugly, the octopus is actually
- - - - when seen close up in the aquarium.
(A)
(B)
(C)
(D)
(E)

beautiful
dangerous
unremarkable
unattractive
docile

Question 7.
Native Americans did not immediately become voting citizens of the United
States; on the contrary, full citizenship was a - - - - process that took
place over the course of many centuries.
(A)
(B)
(C)
(D)
(E)

Copyright 2009, TCA, LLC.

popular
quick
bold
gradual
superficial

54

Studyguide for the SAT Skill Quiz D: Sent. Comp.

Hints!
A

Hint for Question 1:

(Contrast): This sentence has two blanks, so begin with the second blank
first. Notice the change in sentence direction created by the signal
word even. What set of words shows how the comedian reversed the
attitude of even the worst heckler in the crowd?

Hint for Question 2:

(Contrast): Notice the signal word although. What word creates the best
contrast to the key word docile ?

Hint for Question 3:

(Contrast): Notice the signal word but. What word creates the best
contrast to the key word unexpected ?

Hint for Question 4:

(Contrast): Notice the signal word but. What word creates the best
contrast to the key word general ?

Hint for Question 5:

(Contrast): Notice the signal word although. What word best contrasts
with the key words sales soared ?

Hint for Question 6:

(Contrast): Notice the signal word although. What word best contrasts
with the key words exceedingly ugly ?

Copyright 2009, TCA, LLC.

55

Studyguide for the SAT Skill Quiz D: Sent. Comp.


Hint for Question 7:

(Contrast): Notice the signal words on the contrary. What word best
contrasts with the key word immediately ?

Copyright 2009, TCA, LLC.

56

Studyguide for the SAT Skill Quiz D: Sent. Comp.

Answers!
A

Question 1.
The answer is (B).
Explanation for Question 1:

Eliminate choices (A) and (C) because the second word does not
characterize a heckler. Eliminate choice (D) because the first word is
not what comedians perform. Eliminate choice (E) because the words are
too vague. Choice (B) is correct because the brilliant repartee, or
a quick and witty reply, was able to amuse even the most churlish,
or rude, heckler.

Question 2.
The answer is (C).
Explanation for Question 2:

Eliminate choices (A), (B), (D), and (E) because they do not contrast
with the key word docile. Something that looks menacing is in sharp
contrast to the docile behavior of the mantis.

Question 3.
The answer is (C).
Explanation for Question 3:

The best contrast to the key word unexpected is choice (C), predictable.
Eliminate the other choices because they do not create the required
contrast.

Question 4.
The answer is (B).
Explanation for Question 4:

Eliminate choices (A), (C), (D), and (E) because they do not contrast
with the key word general. Choice (B), specific, is always
the correct contrast word to the word general.

Copyright 2009, TCA, LLC.

57

Studyguide for the SAT Skill Quiz D: Sent. Comp.


Question 5.
The answer is (E).
Explanation for Question 5:

The best contrast choice is (E), berated. If critics berate or belittle


the music video, you would not expect sales to continue to soar.
Eliminate the other choices because they do not contrast with the idea
of sales soaring.

Question 6.
The answer is (A).
Explanation for Question 6:

Only choice (A), beautiful, contrasts with the key words exceedingly ugly.
The other choices do not establish such a clear reversal of the sentence
idea.

Question 7.
The answer is (D).
Explanation for Question 7:

Only choice (D), gradual, contrasts with the key word immediate. The
other choices do not establish such a contrast.

Copyright 2009, TCA, LLC.

58

Studyguide for the SAT Skill Quiz E: Sent. Comp.

Quiz Time!
A

Question 1.
The recently published biography was not - - - - ; therefore,
it could not be endorsed by those closest to the subject.
(A)
(B)
(C)
(D)
(E)

appreciated
arbitrary
invasive
stable
authorized

Question 2.
The attorney for the defense will - - - - the decision and will
await his client's sentencing.
(A)
(B)
(C)
(D)
(E)

repeal
conceal
abide by
confide in
appeal

Question 3.
The - - - - statement given to the detective served its
intended purpose: it - - - - the investigator and sent him off
on a wild goose chase.
(A)
(B)
(C)
(D)
(E)

Copyright 2009, TCA, LLC.

credible...deceived
sworn...amused
fallacious...misled
antiquated...rebuked
symbolic...admonished

59

Studyguide for the SATSkill Quiz E: Sent. Comp.


Question 4.
The greedy land barons - - - - parcels of property, thereby
- - - - the tenant farmers without a means to support themselves.
(A)
(B)
(C)
(D)
(E)

expropriated...leaving
stole...presenting
culled...rewarding
donated...rendering
acquired...entertaining

Question 5.
Because poor weather had greatly increased the potential for disaster,
the stunt pilot opted for - - - - over - - - - in planning his maneuvers
for the air show.
(A)
(B)
(C)
(D)
(E)

tradition...custom
caution...daring
luck...logic
legerdemain...skill
nuance...strategy

Question 6.
To - - - - their images as charitable enterprises, many corporations
have initiated - - - - programs to aid victims of natural disasters.
(A)
(B)
(C)
(D)
(E)

bolster...relief
magnify...educational
minimize...remuneration
deter...loan
ascertain...self-help

Question 7.
Because the dictator - - - - the citizenry's pleas for basic
freedoms, the populace believed it had no - - - - but to
revolt against the tyrant.
(A)
(B)
(C)
(D)
(E)

Copyright 2009, TCA, LLC.

heard...answer
renounced...incursion
dismissed...recourse
applauded...obligation
contemplated...response

60

Studyguide for the SAT Skill Quiz E: Sent. Comp.

Hints!
A

Hint for Question 1:

(Cause-effect): Typically, friends and family of the person about whom


a biography is written would endorse the book if the subject had given
permission to the author to proceed with his work. Notice the signal
word therefore and the key phrase could not be endorsed.

Hint for Question 2:

(Implied cause-effect): Can a lawyer repeal, conceal, or confide in a


jury's decision?

Hint for Question 3:

(Cause-effect): What kind of statement could cause the detective to look


for clues in all the wrong places? Notice the key words served and
intended purpose.

Hint for Question 4:

(Cause-effect): If land barons took away land previously used by tenant


farmers, could those farmers continue to support themselves? Notice the
key word greedy and the signal word thereby.

Hint for Question 5:

(Cause-effect): Even a hotshot flyer sometimes needs to tone it down


when the odds are against him! Notice the signal word because and the
key phrases poor weather and potential for disaster.

Copyright 2009, TCA, LLC.

61

Studyguide for the SAT Skill Quiz E: Sent. Comp.


Hint for Question 6:

(Cause-effect): Most corporations are very concerned about the image


they present to the public. Notice the key word to. When to is
followed by a verb, as in to sing, to dance, etc., this infinitive
establishes a cause. Also notice the key phrases such as charitable
enterprises and to aid victims.

Hint for Question 7:

(Cause-effect): Notice the signal word because and the key words
dictator, basic freedoms, and revolt. Which answer choice contains a
word associated with a dictator's typical behavior and a word to
indicate what the populace believed?

Copyright 2009, TCA, LLC.

62

Studyguide for the SAT Skill Quiz E: Sent. Comp.

Answers!
A

Question 1.
The answer is (E).
Explanation for Question 1:

Authorized. The signal word therefore establishes a cause-effect


relationship, and the key phrase could not be endorsed requires the word
authorized for the correct cause-effect relationship. The other choices
do not work in the context of the sentence.

Question 2.
The answer is (C).
Explanation for Question 2:

Abide by. A lawyer cannot repeal, conceal, or confide in a jury's


decision. Although answer (E) may appear correct, a lawyer cannot
appeal a decision until sentencing is pronounced. Only choice (C) is
correct. Until sentencing, the lawyer must abide by the jury's decision.

Question 3.
The answer is (C).
Explanation for Question 3:
Fallacious...misled. Only a fallacious statement could mislead the
detective and cause him to look for clues in all the wrong places. None
of the other choices works in the context of the sentence.

Question 4.
The answer is (A).
Explanation for Question 4:
Expropriated...leaving. The signal word thereby establishes the causeeffect relationship, and the key word greedy establishes the context
focus for the first blank in the sentence. Eliminate choices (C), (D),
and (E) because they do not work in the context of the sentence.
Although choice (B) may appear correct, only the first word in the pair
would be correct. Because the land barons were greedy, they
expropriated the tenants' land, thereby leaving them without a means to
support themselves.

Copyright 2009, TCA, LLC.

63

Studyguide for the SAT Skill Quiz E: Sent. Comp.


Question 5.
The answer is (B).
Explanation for Question 5:
Caution...daring. The signal word because establishes the cause-effect
relationship, and the phrases poor weather and potential for disaster
establish the context requirement for choice (B). Eliminate all other
choices because they do not work in the context of the sentence.

Question 6.
The answer is (A).
Explanation for Question 6:
Bolster...relief. Although choice (B) may seem correct, only the first
word could possibly work in the sentence context. Choices (C), (D), and
(E) can be eliminated because they do not work in the sentence context.
Bolster means to make stronger or support, and relief programs aid
victims of natural disasters.

Question 7.
The answer is (C).
Explanation for Question 7:
Dismissed...recourse. Eliminate choices (A), (D), and (E) because the
first word in each is not a behavior typical of a dictator. Although
choice (B) may appear correct, only the first word would work in the
sentence context. Typically, dictators dismiss pleas from citizens,
leaving them no recourse or other action but revolt.

Copyright 2009, TCA, LLC.

64

Studyguide for the SAT Skill Quiz F: Sent. Comp.

Quiz Time!
A

Question 1.
The principal's curriculum review committee had problems coming to a
consensus on what school subjects to delete, because their viewpoints are
derived from many - - - - sources.
(A)
(B)
(C)
(D)
(E)

similar
divergent
confused
precise
blunt

Question 2.
Because the old almanac rarely made a mistake predicting the weather,
many farmers came to believe that it was - - - - and began to
- - - - its prognostications without question.
(A)
(B)
(C)
(D)
(E)

perfect...ignore
foolproof...shun
infallible...follow
undependable...accept
unreliable...imitate

Question 3.
Gustav Flaubert's style is characterized by an - - - - of
language; he never wrote a - - - - word.
(A)
(B)
(C)
(D)
(E)

Copyright 2009, TCA, LLC.

eloquence...comprehensible
ingenuity...clever
ambiguity...vague
economy...superfluous
arrogance...vain

65

Studyguide for the SAT Skill Quiz F: Sent. Comp.


Question 4.
We lost confidence in the CEO's leadership because he repeatedly
- - - - the statements he made.
(A)
(B)
(C)
(D)
(E)

forgot about
tired of
lied about
reneged on
delivered on

Question 5.
The Pre-Clovis theory has presented scholars with new ideas on when
Native Americans first reached America because it - - - significant data that was previously - - - - .
(A)
(B)
(C)
(D)
(E)

explores...noticed
ignores...considered
examines...neglected
mocks...ridiculed
tolerates...supported

Question 6.
Because of the - - - - of the new video game, one store began
selling it at - - - - prices.
(A)
(B)
(C)
(D)
(E)

dearth...reasonable
surplus...excessive
popularity...fair
obscurity...extravagant
scarcity...exorbitant

Question 7.
As a result of people continuing their exodus from America's big cities,
the amount of farmland continues to - - - -, thereby
- - - - agricultural productivity.
(A)
(B)
(C)
(D)
(E)

Copyright 2009, TCA, LLC.

diminish...protecting
dwindle...threatening
expand...exposing
shrink...shielding
flourish...endangering

66

Studyguide for the SAT Skill Quiz F: Sent. Comp.

Hints!
A

Hint for Question 1:

(Cause-effect): Notice the signal word because and the key words
problems, consensus, and viewpoints. Which choice best explains
why the curriculum committee could not arrive at a consensus?

Hint for Question 2:

(Cause-effect): Notice the signal word because and the key words rarely
made a mistake and without question. Which answer choice contains the
best word for rarely made a mistake ?

Hint for Question 3:

(Cause-effect): Notice the key words


language and he never wrote. Which answer choice contains antonyms that
that best demonstrate cause and effect?

Hint for Question 4:

(Cause-effect): Notice the signal word because and the key words lost
confidence and repeatedly. Using the strategy of positive and negative
words, which answer choice best explains why we lost confidence in the
CEO's leadership?

Hint for Question 5:

(Cause-effect): Notice the signal word because and the key words new
ideas and significant data. What was the effect on scholars of
new ideas concerning when Native Americans first reached America?

Copyright 2009, TCA, LLC.

67

Studyguide for the SAT Skill Quiz F: Sent. Comp.


Hint for Question 6:

(Cause-effect): Notice the signal word because and the key words new
video game and prices. If the video game just came out, what might
a store do if the supply were not equal to the demand?

Hint for Question 7:

(Cause-effect): Notice the signal words as a result of and the key


words exodus from America's big cities and farmland. As more and more
people leave America's big cities for the country, what effect will that
exodus have on available farmland?

Copyright 2009, TCA, LLC.

68

Studyguide for the SAT Skill Quiz F: Sent. Comp.

Answers!
A

Question 1.
The answer is (B).
Explanation for Question 1:

Divergent. Notice how the key words suggest the correct answer:
problems, consensus, viewpoints. Only choice (B) is the best word to
explain the cause-effect relationship. The committee had problems
arriving at a consensus because the members had so many divergent
viewpoints. The other choices are incorrect because they do not
work with the word consensus .

Question 2.
The answer is (C).
Explanation for Question 2:

Infallible...follow. If something rarely makes a mistake, people often


believe it to be infallible and follow it without question. Although the
first words in choices (A) and (B) may seem the same in meaning, the
second word contradicts the first. Eliminate choices (D) and (E)
immediately because the first words are opposite in meaning to the
phrase rarely made a mistake.

Question 3.
The answer is (D).
Explanation for Question 3:

Economy...superfluous. Only choice (D) is correct. Flaubert's style is


remarkable for economy of language; he never wrote a superfluous
word. Although choices (A) and (B) may seem correct using the first
word, the second word creates a contradiction. Eliminate choices (C) and
(E) because the second word will not work in the sentence context.

Copyright 2009, TCA, LLC.

69

Studyguide for the SAT Skill Quiz F: Sent. Comp.


Question 4.
The answer is (D).
Explanation for Question 4:

Reneged on. Eliminate choice (E) immediately because it is positive and


the entire sentence is negative in tone. The remaining choices may all
appear to be correct, but only (D) contains the strongest negative
phrase.

Question 5.
The answer is (C).
Explanation for Question 5:

Examines...neglected. Only choice (C) is correct because the word


choices create the most logical cause and effect. The other choices all
contain at least one word that will not establish a logical cause and
effect.

Question 6.
The answer is (E).
Explanation for Question 6:

Scarcity...exorbitant. Only choice (E) is correct because the word pair


explains why with a scarcity of new games a store might try to charge
exorbitant prices. Although choice (B) may appear correct, with a
surplus of games a store could not charge exorbitant prices. The
remaining choices contain at least one word in the pair that
contradicts the sentence context.

Question 7.
The answer is (B).
Explanation for Question 7:

Dwindle...threatening. Only choice (B) is correct. As more and more people


leave America's big cities for the country, farmland will dwindle,
thereby threatening agricultural productivity. Although choice (D) may
appear correct because farmland will shrink but agricultural productivity will
not be shielded. Choice (D) does not make sense in the sentence context.
Eliminate the other choices because at least one of the words clearly
contradicts the sentence meaning.

Copyright 2009, TCA, LLC.

70

Studyguide for the SAT Skill Quiz G: Sent. Comp.

Quiz Time!
A

Question 1.
The earthquake - - - - the valley floor and left an enormous
fissure running through it.
(A)
(B)
(C)
(D)
(E)

rent
discharged
muted
ameliorated
imbued

Question 2.
Tidal waves are caused by sudden, violent shifts in the earth's plates
and are frightening examples of the - - - - and dangerous weather
conditions faced by people living in coastal regions.
(A)
(B)
(C)
(D)
(E)

decorous
fragile
calm
turbulent
tranquil

Question 3.
Though her behavior appears to be outgoing in public, people who know
her realize that Tamika is actually serious and - - - - by
nature.
(A)
(B)
(C)
(D)
(E)

Copyright 2009, TCA, LLC.

frivolous
flippant
unfettered
subdued
boisterous

71

Studyguide for the SAT Skill Quiz G: Sent. Comp.


Question 4.
Indirection and - - - - are significant characteristics of the
poetry of Ezra Pound.
(A)
(B)
(C)
(D)
(E)

praise
detail
boldness
allusion
paradox

Question 5.
The theory that the Greenhouse Effect will radically alter earth's
temperature and weather patterns has been met with skepticism and
- - - - on the part of some politicians.
(A)
(B)
(C)
(D)
(E)

sarcasm
incredulity
encouragement
conformity
amity

Question 6.
Farmers have been unable to plant this spring because the continual
rainfall has left the ground soggy and - - - - .
(A)
(B)
(C)
(D)
(E)

arid
abundant
sodden
placid
fragile

Question 7.
The Civil War took a terrible toll on the lives of both Southerners and
Northerners because it reduced and - - - - their families.
(A)
(B)
(C)
(D)
(E)

Copyright 2009, TCA, LLC.

subdued
decimated
exulted
unfettered
lauded

72

Studyguide for the SAT Skill Quiz G: Sent. Comp.

Hints!
A

Hint for Question 1:

(Synonym): Notice the signal word and along with the key word fissure.
What word is the best synonym for fissure?

Hint for Question 2:

(Synonym): Notice the signal word and along with the key words
dangerous weather conditions. What word is synonymous with dangerous
weather conditions?

Hint for Question 3:

(Synonym): Notice the signal word and along with the key word serious.
What answer is the synonym for serious?

Hint for Question 4:

(Synonym): Notice the signal word and along with the key word
indirection. What word is a synonym for indirection?

Hint for Question 5:

(Synonym): Notice the signal word and along with the key word
skepticism. What word is a synonym for skepticism?

Hint for Question 6:

(Synonym): Notice the signal word and along with the key word soggy.
What word is the best synonym for soggy?

Copyright 2009, TCA, LLC.

73

Studyguide for the SAT Skill Quiz G: Sent. Comp.


Hint for Question 7:

(Synonym): Notice the signal word and along with the key word reduced.
What word is a synonym for reduced?

Copyright 2009, TCA, LLC.

74

Studyguide for the SAT Skill Quiz G: Sent. Comp.

Answers!
A

Question 1.
The answer is (A).
Explanation for Question 1:

The correct answer is choice (A). A fissure is a tear in the earth's


surface and rent means some type of split. Earthquakes tear apart the
earth's crust. None of the other answer choices are synonyms for
fissure.

Question 2.
The answer is (D).
Explanation for Question 2:

Quickly eliminate choices (A), (C), and (E) because they are opposites
for dangerous weather conditions. Choice (B) does not make sense.
Choice (D), turbulent, is correct because it is synonymous with dangerous
weather conditions.

Question 3.
The answer is (D).
Explanation for Question 3:

Only choice (D), subdued, is correct. Serious people behave in a


subdued manner. The other choices are incorrect because they are the
opposite of serious.

Question 4.
The answer is (D).
Explanation for Question 4:

Choice (D), allusion, is correct because an allusion in writing is


an indirect reference. None of the other choices is a synonym for
indirection. Thus, the other choices do not make sense.

Copyright 2009, TCA, LLC.

75

Studyguide for the SAT Skill Quiz G: Sent. Comp.


Question 5.
The answer is (B).
Explanation for Question 5:

Choice (B), incredulity, is correct. Incredulity means not willing to


accept what is offered as true. Someone who reacts with incredulity
would obviously be skeptical. Although choice (A) sarcasm may seem
correct, it is not synonymous with skepticism. Choices (C) and (E) are
the opposite of skepticism. Eliminate choice (D) because a skeptic would
not be willing to conform.

Question 6.
The answer is (C).
Explanation for Question 6:

Choice (C), sodden, is correct because sodden means filled or saturated


with water. Choice (A) is the opposite of soggy, and none of the other
choices makes sense.

Question 7.
The answer is (B).
Explanation for Question 7:
Choice (B), decimated, is correct because if something is decimated, it is
drastically reduced. Since the other choices are not synonymous with
reduced, they do not make sense.

Copyright 2009, TCA, LLC.

76

Studyguide for the SAT Skill Quiz H: Sent. Comp.

Quiz Time!
A

Question 1.
Always outgoing and - - - - , Julio demonstrated even more
gregarious behavior at the school dance.
(A)
(B)
(C)
(D)
(E)

affable
reclusive
solitary
belligerent
deceptive

Question 2.
Neil Simon's plays are packed with characters who are charming as well
as - - - - .
(A)
(B)
(C)
(D)
(E)

bland
vivacious
lethargic
merciless
erratic

Question 3.
Until recently, the dinosaur we call T Rex was considered carnivorous
and - - - - in nature, but current theory holds that it was
really not this way at all.
(A)
(B)
(C)
(D)
(E)

Copyright 2009, TCA, LLC.

erratic
exuberant
pragmatic
apprehensive
predatory

77

Studyguide for the SAT Skill Quiz H: Sent. Comp.


Question 4.
Oprah Winfrey is known for her praise and - - - - of gifted
American authors.
(A)
(B)
(C)
(D)
(E)

ridicule
derision
encouragement
sarcasm
caricature

Question 5.
St. Francis led an ascetic life because he was committed to the values
of sanctity and - - - - .
(A)
(B)
(C)
(D)
(E)

hedonism
self-denial
extravagance
revenge
dominance

Question 6.
Pharmaceutical research clearly proved that the new drug produced
harmful and - - - - results on the nervous systems of rats.
(A)
(B)
(C)
(D)
(E)

nontoxic
refreshing
devastating
invigorating
healthy

Question 7.
The store manager was fired because he treated many customers in a rude
and - - - - manner.
(A)
(B)
(C)
(D)
(E)

Copyright 2009, TCA, LLC.

discourteous
efficient
gracious
verbose
entertaining

78

Studyguide for the SAT Skill Quiz H: Sent. Comp.

Hints!
A

Hint for Question 1:


(Synonym): Notice the signal word and along with the key word outgoing.
What word is the best synonym for outgoing?

Hint for Question 2:


(Synonym): Notice the signal words as well as along with the key word
charming. What word is synonymous with charming?

Hint for Question 3:


(Synonym): Notice the signal word and along with the key word
carnivorous. What word is the best synonym for carnivorous?

Hint for Question 4:


(Synonym): Notice the signal word and along with the key word praise.
What word is synonymous with praise?

Hint for Question 5:


(Synonym): Notice the signal word and along with the key word sanctity.
What word is synonymous with sanctity?

Hint for Question 6:


(Synonym): Notice the signal word indeed along with the key word
harmful. What word is the best synonym for harmful?

Hint for Question 7:


(Synonym): Notice the key word and. Was the manager fired for a
positive or negitive manner?

Copyright 2009, TCA, LLC.

79

Studyguide for the SAT Skill Quiz H: Sent. Comp.

Answers!
A

Question 1.
The answer is (A).
Explanation for Question 1:

Choice (A) is correct because affable means pleasant and at ease when
talking to others. Quickly eliminate choices (B), (C), and (D) because
they are the opposite in meaning of outgoing. Choice (E) does not make
sense.

Question 2.
The answer is (B).
Explanation for Question 2:

Choice (B) is correct because vivacious means full of life. Choices (A)
and (C) contrast with the idea of charming. The remaining choices do not
make sense.

Question 3.
The answer is (E).
Explanation for Question 3:

Choice (E) is correct because predatory means something that preys on


others. None of the other choices make sense.

Question 4.
The answer is (C).
Explanation for Question 4:

By a quick process of elimination, choices (A), (B), (D), and (E) are found
to be opposite in meaning to the key word praise. Only choice (C) can
be correct.

Copyright 2009, TCA, LLC.

80

Studyguide for the SAT Skill Quiz H: Sent. Comp.


Question 5.
The answer is (B).
Explanation for Question 5:

Similar to question 4, because no choice except (B), self-denial is


synonymous with sanctity, so you can quickly arrive at the correct
answer.

Question 6.
The answer is (C).
Explanation for Question 6:

Choice (C), devastating, is synonymous with the key word harmful. All of
the other choices are opposite in meaning to harmful.

Question 7.
The answer is (A).
Explanation for Question 7:

Eliminate choice (C) immediately because gracious is opposite in meaning


to the key word rude. Choice (D), verbose, means someone who uses more
words than necessary. This is not synonymous with rude. Choices (B) and
(E) also contrast with rude. Only choice (A) is correct. Discourteous is
clearly synonymous with rude.

Copyright 2009, TCA, LLC.

81

Copyright 2009, TCA, LLC.

82

Studyguide for the SAT Skill Quiz I: Sent. Comp.

Quiz Time!
A

Question 1.
Although the recipe looked very tasty, the actual cake was very
----.
(A)
(B)
(C)
(D)
(E)

palatable
unsavory
brusque
elusive
unorthodox

Question 2.
As a result of conservation efforts by biologists and wildlife experts,
the river otters of Ohio are no longer - - - - .
(A)
(B)
(C)
(D)
(E)

fragile
relevant
endangered
frantic
fugitive

Question 3.
Like a real expert, Sandra was - - - - in all of the verbal
skills required for success on the SAT.
(A)
(B)
(C)
(D)
(E)

Copyright 2009, TCA, LLC.

proficient
inexperienced
specious
inept
practiced

83

Studyguide for the SAT Skill Quiz I: Sent. Comp.


Question 4.
In theory, the new president's policy appeared unique and
- - - - , but the proposals he suggested were actually routine
and - - - - .
(A)
(B)
(C)
(D)
(E)

daring...novel
innovative...repetitive
unique...special
commonplace...unusual
mundane...predictable

Question 5.
The multimillionaire was known for being a - - - - who
gave generously to support numerous social causes.
(A)
(B)
(C)
(D)
(E)

hypocrite
demagogue
philanthropist
loiterer
miscreant

Question 6.
The paintings of Salvador Dali are often - - - - most people
because he used abstract symbols that are purposely - - - - .
(A)
(B)
(C)
(D)
(E)

Copyright 2009, TCA, LLC.

incomprehensible to...obscure
insensitive to...obstinate
objectionable to...obsolete
supported by...obscene
denounced by...obvious

84

Studyguide for the SAT Skill Quiz I: Sent. Comp.


Question 7.
In his comedy routines, Jack Benny often pretended to be a real
- - - - , a penny-pincher who never gave out money.
(A)
(B)
(C)
(D)
(E)

Copyright 2009, TCA, LLC.

miser
dolt
sage
zealot
rogue

85

Copyright 2009, TCA, LLC.

86

Studyguide for the SAT Skill Quiz I: Sent. Comp.

Hints!
A

Hint for Question 1:


Use the signal word although and the key words very tasty to determine
the correct answer.

Hint for Question 2:


Use the signal words as a result of and the key words conservation efforts
to determine the correct answer.

Hint for Question 3:


Use the key word expert to determine the correct answer.

Hint for Question 4:


Use the signal word and along with the key words unique and routine to
determine the correct answer.

Hint for Question 5:


Use the key words gave generously to support numerous social causes
determine the correct answer. Also use the strategy of positive and
negative words.

to

Hint for Question 6:


Use the signal word because and the key words abstract symbols to
determine the correct answer. Try to fill the second blank first.

Hint for Question 7:


Use the key word penny-pincher to determine the correct answer.

Copyright 2009, TCA, LLC.

87

Copyright 2009, TCA, LLC.

88

Studyguide for the SAT Skill Quiz I: Sent. Comp.

Question 1.
The answer is (B).
Explanation for Question 1:

The signal word although indicates a contrast sentence. The correct answer
must be a contrast to the key words very tasty. Only choice (B) is a
contrast word for tasty.

Question 2.
The answer is (C).
Explanation for Question 2:

The signal words as a result indicate a cause-effect sentence. The


correct answer must be the result of the key words conservation efforts.
Only choice (C) is the correct word to show the effect of the
conservation efforts to protect the otters. None of the other choices
creates this logical relationship.

Question 3.
The answer is (A).
Explanation for Question 3:

The key word expert indicates that you are looking for a definition. An
expert is someone who is proficient in a certain area. Only choice (A)
is correct. None of the other choices offers a correct definition.

Question 4.
The answer is (B).
Explanation for Question 4:
The signal word and indicates a synonym sentence. You are looking for
synonyms for the key words unique and routine. Only choice (B) contains
correct synonyms for these key words.

Copyright 2009, TCA, LLC.

89

Studyguide for the SAT Skill Quiz I: Sent. Comp.


Question 5.
The answer is (C).
Explanation for Question 5:

The key words gave generously to support numerous social causes define
someone who is a philanthropist. In addition, these are positive words,
so the correct choice must also be a positive word. Only choice (C) is
correct. All of the other choices are negative words.

Question 6.
The answer is (A).
Explanation for Question 6:

The signal word because indicates a cause-effect sentence. If you solve


for the second blank first and use the key words abstract symbols, the
only correct choice is the word obscure, and incomprehensible to fits
the first blank. Although choice (C) may appear correct, obsolete is not
a result of using abstract symbols. None of the other choices conveys
logical cause-effect results.

Question 7.
The answer is (A).
Explanation for Question 7:

The key word penny-pincher defines a miser. Only choice (A) is correct.
None of the other choices offers a correct definition for the key word.

Copyright 2009, TCA, LLC.

90

Studyguide for the SAT Skill Quiz J: Sent. Comp.

Quiz Time!
A

Question 1.
The recently released movie was filled with scenes of - - - violence; consequently, it could not be given a G rating.
(A)
(B)
(C)
(D)
(E)

supercilious
untenable
placid
specious
covert

Question 2.
Darcell's most unique trait is his quick adaptability and - - - - .
(A)
(B)
(C)
(D)
(E)

resilience
diffidence
ambivalence
impulsiveness
deviousness

Question 3.
Today, many garden vegetables are - - - - , containing genes
from a variety of similar plants.
(A)
(B)
(C)
(D)
(E)

Copyright 2009, TCA, LLC.

hybrids
misconceptions
toxins
frauds
convoluted

91

Studyguide for the SAT Skill Quiz J: Sent. Comp.


Question 4.
Because the CEO expected - - - - at all of his board
meetings, he was angry when so many people were constantly kept waiting.
(A)
(B)
(C)
(D)
(E)

irregularity
animation
speculation
punctuality
empathy

Question 5.
Constant - - - - at the monthly committee meetings developed
into a division between those members who were supportive of the
chairman's requests and those who were against them.
(A)
(B)
(C)
(D)
(E)

diffidence
affability
dissension
tenaciousness
fraud

Question 6.
The tropical island was a rare oasis of - - - - in a world
dominated by - - - - billboards and flashing lights.
(A)
(B)
(C)
(D)
(E)

Copyright 2009, TCA, LLC.

serenity...garish
extravagance...refined
tranquility...sedate
dignity...elegant
commotion...tawdry

92

Studyguide for the SAT Skill Quiz J: Sent. Comp.


Question 7.
Although he pretended to be affable and - - - - , Simon was actually
extremely shy and preferred to be alone.
(A)
(B)
(C)
(D)
(E)

Copyright 2009, TCA, LLC.

sedate
benign
gregarious
affluent
banal

93

Copyright 2009, TCA, LLC.

94

Studyguide for the SAT Skill Quiz J: Sent. Comp.

Hints!
A

Hint for Question 1:


Use the signal word consequently and the key words could not be given a
G rating to determine the correct answer.

Hint for Question 2:


Use the signal word and along with the key word adaptability to
determine the correct answer.

Hint for Question 3:


Use the key word variety to determine the correct answer.

Hint for Question 4:


Use the signal word because and the key words constantly kept waiting
to determine the correct answer.

Hint for Question 5:


Use the key word division to determine the correct answer.

Hint for Question 6:


Use the key words oasis and flashing lights to determine the correct
answer. Try to fill the second blank first by using the positive-andnegative-word strategy.

Hint for Question 7:


Use the signal word although and the key words shy and preferred to be
alone to determine the correct answer.

Copyright 2009, TCA, LLC.

95

Copyright 2009, TCA, LLC.

96

Studyguide for the SAT Skill Quiz J: Sent. Comp.

Answers!
A

Question 1.
The answer is (B).
Explanation for Question 1:

The signal word consequently indicates a cause-effect sentence. Because


the movie contained (B), untenable scenes of violence, the result
was that it did not qualify for a G rating. None of the other choices
creates a logical cause and effect relationship to the key words.

Question 2.
The answer is (A).
Explanation for Question 2:

The signal word and indicates that you are looking for a synonym or
synonymous idea. Only choice (A), resilience, is synonymous with
adaptability. The other choices contrast with the key word.

Question 3.
The answer is (A).
Explanation for Question 3:

The key word variety signals that you are looking for a definition.
Only choice (A), hybrid, is correct. None of the other answer
choices offers a logical definition using the key word variety.

Question 4.
The answer is (D).
Explanation for Question 4:

The signal word because indicates a contrast sentence. Choice (D),


punctuality, is correct because it indicates why the CEO would be angry
when people were constantly kept waiting. None of the other choices
creates a logical contrast relationship.

Copyright 2009, TCA, LLC.

97

Studyguide for the SAT Skill Quiz J: Sent. Comp.


Question 5.
The answer is (C).
Explanation for Question 5:

The key word division indicates that you are looking for a definition.
Only choice (C), dissension, is defined by the key word division. None of
the other choices forms this definitional relationship.

Question 6.
The answer is (A).
Explanation for Question 6:

Only choice (A) is correct. Flashing lights would be garish in a world


thought to be serene. Also, you need a positive word for the first
blank and a negative word for the second blank. None of the other
choices fits this pattern.

Question 7.
The answer is (C).
Explanation for Question 7:

The signal word although indicates a contrast sentence. You are looking
for a word to contrast to the key words shy and preferred to be alone.
Only choice (C), gregarious, contrasts with the key words. None of the
other choices creates this sharp contrast.

Copyright 2009, TCA, LLC.

98

Critical Reading Lesson #2


Reading

Triumph College Admissions


Skill Lesson

Quiz Time!
A

Hints!

Answers!
A

*Throughout this documentation, and the software: College Board and SAT are registered trademarks of the College Entrance Examination Board. PSAT/NMSQT
is a trademark of the College Entrance Examination Board and National Merit Scholarship Corporation. ACT is a trademark of ACT, Inc. None of these entities are
affiliated with the production of, nor endorse these materials.
Copyright 2009, TCA, LLC.

99

Copyright 2009, TCA, LLC.

100

Studyguide for the SAT Skill Review: Critical Reading

CRITICAL READING OVERVIEW


This skill lesson provides an overview of the Critical Reading
section of the SAT. Please review this material
before working on the eight specific skill lessons.
If you have already reviewed this skill lesson, you may jump to the
last page and work on the specific skill lessons or take the skill quizzes.

The Content
Passages are drawn from the following three subject areas:
The Humanities, such as art, literature, philosophy, and history
The Natural Sciences, such as biology, chemistry, and geology
The Social Sciences, such as psychology, technology, and sociology
The form of these may be
Narrative...a storytelling mode
Expository...an explanation
Argumentative....a verbal sparring match on debatable issues
TIP: Knowing the form of the passage may provide valuable
hints as to the author's purpose.
Close Reading
What's "close reading"? Close reading is involved reading. As you read,
you're analyzing. As you read, you're asking yourself questions similar
to the following:
What's this passage about?
What's the author's attitude here?
How does the author want me to feel about this issue?
What does this detail mean?
What are the implications of this piece of information?
What's the "big picture"? So what?

Copyright 2009, TCA, LLC.

101

Studyguide for the SAT Skill Review: Critical Reading


In other words you're mentally engaged in the passage
on the page. Passive reading is following the words as the author leads you
along the sentences. Passive reading is following the words, lines, sentences,
and paragraphs, and ending with a big "HUH?" The problem with passive
reading on the SAT is that you're going to be asked questions about what
you've mentally processed from the passage!
If you have read passively, then you have NOT mentally processed much and
come empty-handed (-brained?) to the set of questions.
If you practice "close reading," then you're prepared to answer the questions.
If you read passively, you must reread segments of the passage, try to
connect them to the questions, reread the questions, and then identify the
best answers. You'll eat up the clock.

Annotating the Text


Annotating is similar to highlighting. It is the process of marking up a
reading passage by:
* making notes in the margin of the text,
* underlining words and sentences,
* adding symbols that remind you that there's something
important to return to
As you read, mark statements you think reflect the author's attitude,
a main idea, or transitional words and phrases that signal the
movement of the passage.

Decide for yourself what symbols you wish to use to annotate the text:
underlining, stars, checkmarks, brackets, etc. The purpose of annotations
is to help you find information quickly. Your notations should be made
quickly and should highlight only what is most important to
understanding the main idea and the structure (movement) of the passage, and
key phrases that you may want to return to. Be careful not to spend too
much time marking the passage: annotating is meant to save time in finding
information.
Practice annotating so that it becomes a mentally automatic process.
As you practice, you'll find it easier to focus and concentrate on the
passage.
Becoming engaged will get easierboth mentally and physically
with that pencil.

Copyright 2009, TCA, LLC.

102

Studyguide for the SAT Skill Review: Critical Reading


Key Words
Key words are words that can be used to find specific information quickly in a passage.
Finding and identifying key words can help you sort through irrelevant information and
find answers fast to content questions that are likely to be asked on the SAT.
To identify keywords, ask yourself the following questions while reading a passage:
* Which words in this passage identify the subject?
* Which words are likely to state or support the author's thesis in the passage?
* Are there words that give clues to the author's attitude or tone?
* What words give clues as to the order in which events happened?
* Are there words that indicate a cause-and-effect relationship between events?
Of course, not every student will circle the same words, but you should train yourself to
look for any terms that would help you understand the passage better.
Print the following passages and annotate them by underlining the main idea and circling
potentially useful key words. Jot down brief notes to yourself.

Sample passage 1: Civil War Times


When Lincoln was voted President of the United States in 1860, South Carolina seceded
from the Union. Then Mississippi, Alabama and Florida did so, and Georgia followed
close behind on January 19, 1861. Sadly, by February 1 of that year, Louisiana and
Texas joined in. Delegates from these seven states formed the Confederate States of
America at a meeting in Montgomery, Alabama on February 4. This Southern
Confederacy voted Jefferson Davis as its provisional president, A.H. Stevens as its vicepresident, and it then began drafting a constitution.

Copyright 2009, TCA, LLC.

103

Studyguide for the SAT Skill Review: Critical Reading


Main idea underlined, key words circled, notes in italics
cause/effect?
When Lincoln was voted President of the United States in 1860, South Carolina seceded
from the Union. Then Mississippi, Alabama and Florida did so, and Georgia followed
close behind on January 19, 1861. Sadly, by February 1 of that year, Louisiana and
Texas joined in. Delegates from these seven states formed the Confederate States of
America at a meeting in Montgomery, Alabama on February 4. This Southern Confederacy
voted Jefferson Davis as its provisional president, A.H. Stevens as its vice-president,
and it then began drafting a constitution.

Explanation:
"voted President": important event
"South Carolina seceded from the Union": may be a cause of the forming of the
Confederate States of America.
"Sadly": indicates author's opinion
"Delegates from these seven states formed the Confederate States of America":
phrase is underlined because it tells the main idea of the passage.

this

"Southern Confederacy": important name of the newly formed group

Sample Passage 2: T.S. Eliot Interview

Some poets start early. T. S. Eliot, the great early-twentieth-century poet,


once told an interviewer that he began writing poetry at the age of fourteen. He
said those poems had been influenced by a gloomy poem he had read, and he never
showed them to anyone. He did, in fact, destroy them.
Eliot went on to write poetry in high school and at Harvard, becoming editor of the
Harvard Advocate. He also told the interviewer that he had been heavily influenced
by both Baudelaire and Laforgue during his years at Harvard.

Copyright 2009, TCA, LLC.

104

Studyguide for the SAT Skill Review: Critical Reading


Main idea underlined, key words circled, notes in italics
opinion
Some poets start early. T. S. Eliot, the great early-twentieth-century poet,
once told an interviewer that he began writing poetry at the age of fourteen. He
said those poems had been influenced by a gloomy poem he had read, and he never
showed them to anyone. He did, in fact, destroy them.

Eliot went on to write poetry in high school and at Harvard, becoming editor of the
Harvard Advocate. He also told the interviewer that he had been heavily influenced
by both Baudelaire and Laforgue during his years at Harvard.

Explanation:

"great": indicates author's opinion


"fourteen": notable because this is a very young age to start writing
"gloomy": suggests the reason that he destroyed his poems
"heavily influenced by Baudelaire and Laforgue": indicates one of the sources of
Eliot's inspiration
Sample Passage 3 : Learn to Drive in Your Living Room
Driver's education may undergo a radical change in the coming years. Instead of driving
around a parking lot, students may learn how to control a vehicle while sitting in a
classroom. Once they hit the road, they may have already logged several hours of
practice.
Virtual-reality computer programs can replicate thousands of a car's characteristics.
When combined with a steering wheel and brake and gas pedals, these driving simulators
can come so close to the real thing that playing games might someday result in safer
drivers.

Copyright 2009, TCA, LLC.

105

Studyguide for the SAT Skill Review: Critical Reading


Main idea underlined, key words circled, notes in italics
Driver's education may undergo a radical change in the coming years. Instead of driving
around a parking lot, students may learn how to control a vehicle while sitting in a classroom.
Once they hit the road, they may have already logged several hours of practice.
Virtual-reality computer programs can replicate thousands of a car's characteristics.
When combined with a steering wheel and brake and gas pedals, these driving simulators
main goal
can come so close to the real thing that playing games might someday result in safer
drivers.

Explanation:

"Driver's education may undergo a radical change in the coming years.":


This is the main idea of the passage.
"sitting in the classroom": emphasizes important aspect of the change
"Virtual-reality computer programs": key element in the new driver education
program
"Safer": effect of new program

Using the Passage to Answer Questions


Read the question and all of the answers carefully. Look back at the
passage to find pertinent information that will aid you in
answering the question. (Here's where your annotations will help you
quickly locate the information.)
Think about how that information aligns with the answer choices.
Eliminate those answers that do not fit. Select the best answer,
the one that can be supported by information from the passage.
Remember, the passage will always provide support for the best answer,
so use the passage to verify your answer choice.

Copyright 2009, TCA, LLC.

106

Studyguide for the SAT Skill Review: Vocabulary in Context

VOCABULARY IN CONTEXT
These questions are usually the easiest. You're asked to determine
the meaning of a word from the contextual clues surrounding that word.
Sometimes the word is abstract, taking on its special shade of meaning
from the sentences surrounding it. At other times, the word is fairly
difficultone you may not recognize. You're to infer its meaning from
definitional clues placed near the word.

The question stem will provide the line number where the word appears;
however, the contextual clues that suggest meaning may be in lines above
or below the given word.

Contextual Clues
There are four types of contextual clues that will help you decipher
vocabulary in context questions:
Definitions
Synonyms
Antonyms
Implied Definitions

Definitions
In the passage, an unusual word may have its definition nearby.
These are often found in passages taken from a specialized field
where the jargon is probably unfamiliar to the audience.
Choose the best answer.
A hurricane has wind speeds in excess of 64 knots (nautical miles per hour).
In the above line, the word "knots" means
(A) difficulties
(B) tight groups
(C) a measure of speed
(D) dark spots in wood
(E) interlaced strings

Copyright 2009, TCA, LLC.

107

Studyguide for the SAT Skill Review: Vocabulary in Context

All of the choices are definitions of the word "knots," but we know
from the definition, nautical miles per hour, that in this context choice
(C) is correct.
Synonyms
An unusual word may have a synonym or a paraphrase to help you
determine the meaning.
Choose the best answer.
The Great Doctor Magnifico was a complete charlatan, as phony as
they come.
In the above line, the word "charlatan" means
a fraud
(A)
a physician
(B)
a gentleman
(C)
a tall person
(D)
a weakling
(E)

Your clue should come from the paraphrase "as phony as they come."
Choice (A) is the correct answer.
Antonyms
By determining what a word does not mean, sometimes you can determine what
the word does mean.
Choose the best answer
My job was easyall I had to do was rake leaves. On the other hand, my
brother had the arduous task of mowing our three-acre lawn.
In the passage above the word "arduous" means:
(A) simple
(B) burdensome
(C) enjoyable
(D) lofty
(E) important

Copyright 2009, TCA, LLC.

108

Studyguide for the SAT Skill Review: Vocabulary in Context

The words "on the other hand" provide a contrast between the two tasks.
Because one task was easy the other must be difficult, or burdensome.
(B) is the correct choice.
Implied Definitions
This can be the most difficult type of vocabulary question. You will
have to tie two pieces of information together to determine the meaning
of the word in question.
Choose the best answer.
As she gazed at the painting, Wendy wondered if the stippled pattern in front
of her would be recognized by a blind person as letters.
In the passage above, the word "stippled" means
(A) made up of different shapes
(B) made up of stripes
(C) made up of dots
(D) made up of different textures
(E) made up of different colors

Blind people are able to read using the Braille alphabet, which is made up
of patterns of dots to represent letters. Stippled means made up of dots.
(C) is the correct answer.
Another way the test makers might try to test your ability to
understand the meaning of words in context is to give definitions
for similar sounding words.
Choose the best answer.
Because she had lied so often before, I was incredulous when I
heard she had won the lottery.
The word "incredulous" in this sentence means
(A) unbelievable
(B) astonishing
(C) disbelieving
(D) bored
(E) free of guilt

Copyright 2009, TCA, LLC.

109

Studyguide for the SAT Skill Review: Vocabulary in Context

The word incredulous means disbelieving.


Definitions (A) and (B) are for the word incredible.

In the Middle Ages, merchants and artisans formed groups called guilds
in order to protect themselves and their trades. These appeared in the
year 1000, and by the twelfth century, analogous trades, like
wool, spice, and silk dealers, had formed their own guilds. By that
time, towns like Florence, Italy boasted as many as 50 merchants'
guilds.
Apprenticeship became a complex system with the advent of guilds.
Apprentices were to be taught certain things and prove they possessed
certain skills, determined by the guild. Each guild decided the length
of time required for an apprentice to work for a master tradesman
before being admitted to the trade.

Practice Exercise
Question 1.
As it is used in the passage, the word "advent" means
(A)

activity

(B)

arrival

(C)

discretion

(D)

composition

(E)

confusion

The answer is B. That it "became" a complex system should clue you in to


the fact that it had recently arrived.

Copyright 2009, TCA, LLC.

110

Studyguide for the SAT Skill Review: Vocabulary in Context


Question 2.
In the passage above, the word "analogous" means

(A)

difficult

(B)

hectic

(C)

detached

(D)

obscure

(E)

similar

The answer is E.
The fact that all three are "dealers" should clue you in to the fact
that they were somewhat alike.

Copyright 2009, TCA, LLC.

111

Copyright 2009, TCA, LLC.

112

Studyguide for the SAT Skill Review: Finding the Main Idea

FINDING THE MAIN IDEA


Some questions ask you to state the main idea. You will not find this
stated verbatim in the passage, but that should be okay because you have
read the passage knowing that you would probably have to answer this
question. The main idea is a generalized statement that summarizes the
author's intent and what it is that he or she is trying to communicate
to you.
The main idea of a passage ties together most or all of its content.
Everything in the passage should relate in some way to the main idea.
If large sections of the passage don't relate to what you think is the
main idea, you have probably misunderstood something.
How Do I Find the Main Idea?
The main idea of a passage is often stated at or near the beginning. Look
for this statement, sometimes called the thesis statement , and underline it
with your pencil. You will then be able to refer to it later and will also
fix the main idea in your memory. Reading with a pencil in your hand is always
a good idea.
Main Ideas of Paragraphs
The main ideas of paragraphs are found in the topic sentence, typically
near the beginning of each paragraph. These secondary ideas help
support, explain, or develop the main idea. You might want to underline
these as well.

Copyright 2009, TCA, LLC.

113

Studyguide for the SAT Skill Review: Finding the Main Idea

The following is from a speech by a noted member of the Progressive


movement, a political movement that was most active in the United
States in the period before World War I.
There has been something crude and heartless and unfeeling in our
haste to succeed and be great. Our thought has been "Let every man
look out for himself, let every generation look out for itself,"
while we reared giant machinery which made it impossible that any but
those who stood at the levers of control should have a chance to look
out for themselves. We had not forgotten our morals.
We remembered well enough that we had set up a policy which was meant
to serve the humblest as well as the most powerful, with an eye single
to the standards of justice and fair play, and remembered it with pride.
But we were very heedless and in a hurry to be great.

We have come now to the sober second thought. The scales of heedlessness
have fallen from our eyes. We have made up our minds to square every
process of our national life again with the standards we so proudly set
up at the beginning and have always carried at our hearts. Our work is
a work of restoration.
We have itemized with some degree of particularity the things that ought
to be altered and here are some of the chief items: A tariff which cuts
us off from our proper part in the commerce of the world, violates the
just principles of taxation, and makes the Government a facile instrument
in the hands of private interest; a banking and currency system based upon
the necessity of the Government to sell its bonds fifty years ago and

perfectly adapted to concentrating cash and restricting credits; an


industrial system which, taken on all its sides, financial as well as
administrative, holds capital in leading strings, restricts the liberties
and limits the opportunities of labor, and exploits without renewing or
conserving the natural resources of the country; a body of agricultural
activities never yet given the efficiency of great business undertakings
or served as it should be through the instrumentality of science taken
directly to the farm, or afforded the facilities of credit best suited to
its practical needs; watercourses undeveloped, waste places unreclaimed,
forests untended, fast disappearing...

Copyright 2009, TCA, LLC.

114

Studyguide for the SAT Skill Review: Finding the Main Idea

Here is a question of this kind based on the sample reading passage.


Sample:
This passage shows that the Progressives were motivated by a desire to
(A)
(B)
(C)
(D)

alleviate all human suffering


humanize the Industrial Revolution
clear the way for greater power to the industrialists
eliminate rural poverty

This question is asking you, in effect, to find and identify the


main idea of the passage, even though the words "main idea" do not
appear in the question. Look, then, for the topic sentence of the
passage, which most often will be located in the first paragraph.
Answer (C) can be eliminated right away because it is the opposite
of what the author is discussing. Answer (D) may have been one goal
of this author, but it is not specifically mentioned, so it will be
eliminated if there is a better answer.
Answer (A) was quite possibly a goal of the Progressives but
is perhaps too all-encompassing for the topic under discussion.
What the author does mention in the first paragraph is "giant machinery,"
which is a clue that answer (B), which makes direct reference to the
Industrial Revolution, is the best possible answer.

The Main Idea in Double Passages


Questions that ask you to examine the interrelationships between two theories
or concepts are related to main-idea questions because you have to identify
the main idea of a passage before you can see how it relates to something else.
Code phrases that often introduce this kind of question include
"The main difference between (or similarity to)..." and "What role did
these ideas play in...?"

Copyright 2009, TCA, LLC.

115

Studyguide for the SAT Skill Review: Finding the Main Idea

In the Middle Ages, merchants and artisans formed groups called guilds
in order to protect themselves and their trades. These appeared in the
year 1000, and by the twelfth century, analogous trades, like wool,
spice, and silk dealers, had formed their own guilds. By that time,
towns like Florence, Italy, boasted as many as 50 merchants' guilds.
Apprenticeship became a complex system with the advent of guilds.
Apprentices were to be taught certain things and prove they possessed
certain skills, determined by the guild. Each guild decided the length
of time required for an apprentice to work for a master tradesman
before being admitted to the trade.
Practice Exercise
Question 1.
What is the main idea of the passage?
(A)

Guilds were formed only for silk, wool, and spice dealers.

(B)

Guilds were formed to protect trades and train new workers.

(C)

Merchants had to protect themselves from artisans.

(D)

Apprenticeship was the first step in forming a trade guild.

(E)

Very few towns were allowed to have merchants' guilds.

The answer is B.
The first paragraph discusses the use of guilds to protect trades, and
the second discusses the training of apprentices through guilds.

Copyright 2009, TCA, LLC.

116

Studyguide for the SAT Skill Review: Finding the Main Idea

Alaskan sled dogs, known as Huskies, are some of the hardest-working


dogs in the world. These are not your ordinary lap dogs. No way! These
dogs live in their own spacious quarters, not in the sled driver'sor
musher'shome.
They train relentlessly from the time they are quite young. Mushers
get these dogs used to their harness and to pulling small things like
wooden boards. The dogs have heavy fur coats and padded paws that
protect them in the cold weather conditions under which they work.
Pulling sleds is their passion and their life.
These dogs spend their lives pulling sleds, sometimes in very harsh
conditions, and according to those involved in sleddingthe dogs love
their work. These beautiful, smart, strong, and loyal animals consider
their work play.
Question 2.
What is the main idea of the passage?
(A)

Sled dogs work hard at what they love.

(B)

Training sled dogs is a difficult task.

(C)

Huskies live outside in cold weather.

(D)

Sleds are always pulled in harsh weather.

(E)

Sled dogs always pull small loads.

The answer is A.
The passage emphasizes the hard work and the fact that the dogs love it.

Copyright 2009, TCA, LLC.

117

Copyright 2009, TCA, LLC.

118

Studyguide for the SAT Skill Review: Recognizing a Paraphrase

RECOGNIZING A PARAPHRASE

What is the relationship between the following phrases?


asked the boy
questioned the youngster
walked up the trail
ascended the slope
young at heart
children of all ages
The phrase in the second column is a rewording or paraphrase of the first.
The answers to SAT reading questions are often paraphases
of material in the passage.
Types of Questions for Recognizing a Paraphrase
You may be required to paraphrase the main idea of the passage,
or you may have to identify a rewording of the author's opinion.
Questions that require you to recognize a paraphrase often have
the forms:
According to the author...
To the author...
The ... referred to in line 33 is the
Be sure your answer is in the passage. Don't let your personal
opinion or any previous knowledge influence your choice.

Example:
Henderson's films have great historical exactness, but they come
up short on entertainment value. On the other hand, Oliver Stone's
historical movies are entertaining even if they are not entirely accurate.
To the author, Oliver Stone's historical movies are
(A) boring
(B) serious
(C) comical
(D) redundant
(E) imprecise

Copyright 2009, TCA, LLC.

119

Studyguide for the SAT Skill Review: Recognizing a Paraphrase

"Not entirely accurate" is another way of saying imprecise.


(E) is correct.

In the Middle Ages, merchants and artisans formed groups called guilds
in order to protect themselves and their trades. These appeared in the
year 1000, and by the twelfth century, analogous trades, like wool,
spice, and silk dealers, had formed their own guilds. By that time,
towns like Florence, Italy, boasted as many as 50 merchants' guilds.
Apprenticeship became a complex system with the advent of guilds.
Apprentices were to be taught certain things and prove they possessed
certain skills, determined by the guild. Each guild decided the length
of time required for an apprentice to work for a master tradesman
before being admitted to the trade.
Practice Exercise
Question 1.
According to the passage, apprenticeships
(A)

allowed merchants to have free labor

(B)

existed because there were so many trades

(C)

existed because time limits needed to be set

(D)

were created in order to train skilled workers

(E)

began so that older workers could retire

The answer is D.
The fact that the apprentices learned skills and worked with master
tradesmen tells you that this is training.

Copyright 2009, TCA, LLC.

120

Studyguide for the SAT Skill Review: Recognizing a Paraphrase


Question 2.
According to the author, trade guilds

(A)

kept the secrets of their skills to themselves

(B)

only existed for a short amount of time

(C)

helped tradesmen profit

(D)

were threatened by artisans

(E)

had grown in numbers in two centuries

The answer is E.
The passage states that trade guilds began in about 1000 and had grown
in number by 1200.

Copyright 2009, TCA, LLC.

121

Copyright 2009, TCA, LLC.

122

Studyguide for the SAT Skill Review: Making Inferences

MAKING INFERENCES
Sometimes the answer won't be stated, but there will be one or more hints
for you. For example, if the author clearly dislikes modern architecture,
you could safely infer or conclude that the author would prefer traditional
wood furniture instead of metal and plastic furniture.
You have no way of knowing for sure, of course, but you could logically
infer that if you were asked. What you are essentially doing is
"reading between the lines" by making logical, intelligent inferences
about the author's preferences.
To infer means to deduce or conclude something based on facts. For example, if you
leave school early on Tuesday looking pale and weak and do not come to school at all on
Wednesday, your classmates can infer that you are at home with an illness. They put the
facts together and come to a logical conclusion based on those facts. You must do the
same when you are reading critically.
When reviewing the passage, a good question to ask yourself is, "What
reasonable assumptions can I make based on this passage?"
Something else to keep in mind when looking for inferences is the denotative or
connotative meaning of a word. Remember that writers use their words carefully to
express exactly what they mean by using very particular words. Knowing this will help
you uncover an author's intents.
A denotation is the dictionary definition of a word; it is the thing or idea that the
word refers to literally. An example of denotation is found in the following sentence.
"He carried a red book to school with him." The word "red" is used here in its literal,
denotative form. Sometimes, however, this same word implies anger or, when associated
with temperature, heat. Those are the connotative meanings of this word.
A connotation is the meaning, association, or emotion often implied by the word.
Certain words often bring agreed-upon associations and feelings with them. Use of these
words can convey an author's feeling about the topic without stating it outright.
Connotation is a tool that writers use skillfully. The strategic use of one word can change
the inference within a sentence.

Copyright 2009, TCA, LLC.

123

Studyguide for the SAT Skill Review: Making Inferences


Example:
"She is a very determined young woman" is a way of saying that the young
woman does not give up easily, and further implies that that is an admirable and
positive trait.
"She is a very stubborn young woman" also means that the young woman
does not give up easily, but carries a negative connotation.
In this example, the young woman is seen in a different light with the use of the word
"stubborn," which connotes an illogical or disrespectful refusal to do something.
If you remain aware of the fact that words can be used both connotatively and
denotatively, you will be on the road to a better understanding of inference. You will
prepare yourself better for inference.
This kind of question is difficult to answer because very often you are looking
for something that is not directly stated in the reading passage. Signs that you
have encountered this kind of question include code phrases such as
"This passage was probably written in order to..."
"This passage most resembles..."
"Based on this passage, one can infer that..."
"The author suggests that . . ."
"Based on this passage, one can conclude that . . ."
"In the final paragraph, the author implies that . . ."
"Based on this passage, a (person/animal) would most likely . . ."
"Based on the passage, one can conclude that . . ."
Questions asking you to tell when a passage was written or to speculate about
who wrote it frequently are inference questions. So too are most questions
that contain the word "because"for example, "The author supports (or opposes)
idea X because..."

The following is from a speech by a noted member of the Progressive


movement, a political movement that was most active in the United
States in the period before World War I.
There has been something crude and heartless and unfeeling in our
haste to succeed and be great. Our thought has been "Let every man
look out for himself, let every generation look out for itself,"
while we reared giant machinery which made it impossible that any but
those who stood at the levers of control should have a chance to look
out for themselves. We had not forgotten our morals.
We remembered well enough that we had set up a policy which was meant
to serve the humblest as well as the most powerful, with an eye single
to the standards of justice and fair play, and remembered it with pride.
But we were very heedless and in a hurry to be great.

Copyright 2009, TCA, LLC.

124

Studyguide for the SAT Skill Review: Making Inferences


We have come now to the sober second thought. The scales of heedlessness
have fallen from our eyes. We have made up our minds to square every
process of our national life again with the standards we so proudly set
up at the beginning and have always carried at our hearts. Our work is
a work of restoration.
We have itemized with some degree of particularity the things that ought
to be altered and here are some of the chief items: A tariff which cuts
us off from our proper part in the commerce of the world, violates the
just principles of taxation, and makes the Government a facile instrument
in the hands of private interest; a banking and currency system based upon
the necessity of the Government to sell its bonds fifty years ago and
perfectly adapted to concentrating cash and restricting credits; an
industrial system which, taken on all its sides, financial as well as
administrative, holds capital in leading strings, restricts the liberties
and limits the opportunities of labor, and exploits without renewing or
conserving the natural resources of the country; a body of agricultural
activities never yet given the efficiency of great business undertakings
or served as it should be through the instrumentality of science taken
directly to the farm, or afforded the facilities of credit best suited to
its practical needs; watercourses undeveloped, waste places unreclaimed,
forests untended, fast disappearing...

Here is a question of this kind based on the reading passage:


Sample:
Based on this passage, one can infer that one change the
Progressives did not advocate was
(A)
(B)
(C)
(D)

Copyright 2009, TCA, LLC.

the creation of a new social order


based on new principles.
reform of the financial system.
the rationalization of agriculture.
changes in the tariff structure.

125

Studyguide for the SAT Skill Review: Making Inferences

Searching through the passage, you can find a list of changes that
the Progressives advocated. However, there is no list of changes
that they did not advocatethese you must infer.
Clearly they did support reform of the financial system (B)
and changes in the tariff structure (D). You may also infer
that they supported the "rationalization" of agriculture (C)
because the author speaks of the desirability of giving farming
"the efficiency of great business undertakings." Nowhere, however,
is there any expressed desire to remake society according to new
principles.
Answer (A) is correct.

In fact, the contrary is true: the author proclaims that the


work of the Progressives is to bring about a return to the moral
principles and standards upon which they believed American society
had originally been founded. You may or may not have known about the
philosophical foundations of Progressivism from your classwork or
your own reading. If you did not, however, you could have inferred
this answer from the passage with little difficulty.

Copyright 2009, TCA, LLC.

126

Studyguide for the SAT Skill Review: Making Inferences


Practice Exercise
Passage 1
Climbing high mountains is an exhilarating adventure; mountaineers continuously
attempt to conquer a higher one. Some may find this a ridiculous pursuit. Serious
climbers train to ascend. Many of these healthy people attend exercise classes
specifically designed to instruct them in mountain climbing. In these classes, the
participants perform muscle-strengthening and endurance exercises. They do this without
stopping, to simulate the conditions of climbing. When they take a break, it is timed to
approximate the estimated time one can climb without a break. Mountain climbing is not
for the weekend warrior. It is an activity that requires discipline and skill.

Question 1.
From the passage, one can conclude that the author
(A) is an experienced climber.
(B) thinks mountaineers are foolish.
(C) admires mountaineers.
(D) enjoys climbing.
(E) is physically fit.
Ask yourself, "What assumption can I make about the author based on the passage?"
The answer is C.
In the first sentence the author says that mountain climbing is "exhilarating." He then
goes on to explain the discipline and skill involved in training to climb, reinforcing this
with the final sentence.

Question 2.
The author suggests that
(A) mountain climbing is a fun activity.
(B) one must be serious about climbing to do so.
(C) mountaineers must train twelve months a year.
(D) one must be more than a warrior to climb.
(E) mountain climbing is for everyone.
Read the second paragraph, which tells how mountaineers train.

The answer is B.
The second paragraph details what climbers do to train for climbing mountains. Their
training is rigorous, and therefore requires commitment to prepare for the climb.

Copyright 2009, TCA, LLC.

127

Studyguide for the SAT Skill Review: Making Inferences


Passage 2
Jasmine squinted her eyes at the sun glistening on the surface of the river, adjusted her
goggles for the hundredth time, and tried to calm her breathing. It was a good day to do
this: though it was only ten o'clock, the temperature was already past eighty, and
climbing quickly. May in this part of the country could be unpredictable, with highs
ranging from the sixties to the nineties. She told herself that the generously warm outside
air would warm the river, making it easier to maintain her body temperature and spend
her energy on stroking as quickly as possible toward the finish.

Question 1.
Based on this passage, one can assume that Jasmine is getting ready to
(A)
(B)
(C)
(D)
(E)

watch an event near a river.


compete in a rowing race.
take a ferry across the river.
search for something in the river.
compete in a swimming race.

Look for nouns and verbs that identify the activity Jasmine will take part in.

The answer is E.
The writer mentions adjusting goggles, maintaining body temperature in the water, and
stroking quickly toward the finish, all of which combined imply that Jasmine will be
competing in a swimming race.

Question 2.
Based on information from the passage, how does Jasmine feel?
(A)
(B)
(C)
(D)
(E)

frightened, jumpy
nervous, tense
strong, confident
detached, neutral
calm, relaxed

Look adjectives describing Jasmine: her behavior, attire, what she says and how she says
it.
The answer is B.
Jasmine adjusts her goggles for the hundredth time, calms her breathing, and tries to
convince herself that the water will be warm and that she will do well. These details
convey nervousness, but not fright or a desire to run away.

Copyright 2009, TCA, LLC.

128

Studyguide for the SAT Skill Review: Making Inferences


Passage 3
Have you ever heard sand sing? All over the world, it celebrates with song. Most people
aren't aware of that, unless they've wandered alone along certain beaches or deserts and
listened attentively.
Musical sand divides into two types: singing and booming. Singing sand is found along
shorelines, such as the Hebrides Islands above Scotland. Booming sand is found in desert
dunesin the Sahara Desert, for instance.
Singing sands' songs range from a rich and deep sound to a high and squeaky one. These
grains of sand don't sing solos, but when someone comes along and grains of sand are
sifted through fingers, or stepped on, a note is struck and the celebration begins.

Question 1.
From the passage, one can conclude that the author
(A)
(B)
(C)
(D)
(E)

has never heard singing sand


prefers the "rich and deep sound" the sand produces
enjoys hearing singing sand
would like to hear a grain of sand sing a solo
loves visiting Scotland

Pay attention to the verb "celebrates" and the noun "celebration."

The answer is C.
In the first paragraph, the author refers to the sand as something that celebrates, and in the
last sentence of the passage, songs of the sand are referred to as a "celebration." Both
words connote pleasure and enjoyment at the sound of sand singing.

Question 2.
This passage was probably written in order to
(A)
(B)
(C)
(D)
(E)

introduce readers to the existence of singing sand


tell readers that there are two kinds of singing sand
encourage readers to take a vacation in the Sahara Desert
explain to readers what the Hebrides are like
tell readers about many different uses of sand

Re-read the passage, paying attention to what is said in each paragraph


The answer is A.
The introduction asks the reader a question about singing sand.
Each paragraph speaks of a different aspect of the sand.

Copyright 2009, TCA, LLC.

129

Studyguide for the SAT Skill Review: Making Inferences


Passage 4
One of the most successful country music singers in the late 1940's and early 1950's was
Hank Williams. His remarkable career spanned only about four years, from 1949 until his
death at age twenty-nine on January 1, 1953.
Williams' songs have been sung by every important country-western and many pop
and rock-and-roll singers in the United States. The songs talk of soured love and
reckless living, elements that eventually became associated with country-western singers.

Question 1.
Based on this passage, one can infer that Hank Williams
(A) died young because of his lifestyle
(B) sang love songs most of the time
(C) was only famous for four years
(D) sang songs about life when he felt sad
(E) spent his time teaching songs to other singers
As you look at the passage, search for clues as to Williams' character and actions.
The answer is A.
That he died young because of his lifestyle is implied in the mention of "his death at age
twenty-nine," and his songs of "reckless living."

Question 2.
The author suggests that
(A) Hank Williams was the best singer this country has ever had.
(B) no other country singer has been as successful as Williams.
(C) Hank Williams' music is old-fashioned.
(D) all of the songs written by Williams are depressing.
(E) Hank Williams was very good at what he did.
Pay attention to the adjectives describing Williams and his songs.
The answer is E.
Williams is described as "successful," his career as "remarkable, " and "important"
country and popular singers have sung his songs. This implies his having been good at
what he did.

Copyright 2009, TCA, LLC.

130

Studyguide for the SAT Skill Review: Making Inferences


Passage 5
In her book The Writing Life, author Annie Dillard describes her daily adventures as a
writer. She has had a typewriter erupt and leave burn marks on a table, singeing her shirt
in the process. She wrote one of her books by night in a sparsely lit library, where she had
to count the library rows by touch in order to find her desk.
In this work, she also gives other writers information about the craft, and she does all of
this with the light touch of one comfortable writing in image. She uses metaphor
flawlessly, yet still manages to report the monotonous day-to-day details a writer often
faces.

Question 1.
Based on this passage, one can infer that the author
(A) has carefully studied the work of Annie Dillard
(B) admires Annie Dillard's writing skills
(C) thinks the life of a writer is a boring one
(D) wishes to live an adventurous life
(E) thinks metaphors are important in all writing
As you look at the passage, pay attention to all of the ways the author describes Dillard.
The answer is B.
Although some silliness is reported, it is clear in the second paragraph that the author
considers Dillard a good writer.

Question 2.
Based on this passage, one can infer that Annie Dillard
(A) often places herself in dangerous situations
(B) writes poetry, using metaphors constantly
(C) loves her work in spite of occasional challenges
(D) doesn't like writing in libraries
(E) is a teacher of writing
Pay attention to the challenges she faces and the way she writes.
The answer is C.
Although Dillard has faced challenges in her writing life (such as writing in the dark and
surviving her erupting typewriter), the passage makes it clear that she writes well and
probably enjoys it.

Copyright 2009, TCA, LLC.

131

Copyright 2009, TCA, LLC.

132

Studyguide for the SAT Skill Review: Attitude Mood and Tone

Recognizing Attitude, Mood, and Tone


The terms "attitude" and "tone" refer to the author's slant on a
subject, emotions, or feelings. The author's treatment of a subject
often reflects his or her attitude toward the subject. Think "tone of
voice."
Think of how you indicate your feelings with volume, pauses, pitch,
and emphasis of certain words or syllables when you speak. None of
these are useful in writing, so authors instead rely on their choice
of words to express their feelings.
Tone is often described as the way the author feels about both the
topic and the reader. If, for example, a selection is written in
colloquial (conversational) language rather than formal language, one
might perceive that the author is "talking down" to the reader. This
is not necessarily the case, and only a careful reading will give the
reader clues as to authorial intent.
The tone in a selection can be formal, casual, sincere, playful,
serious, sarcastic, or any other attitude one can imagine. This is
where it helps to remember connotative and denotative language.
Writers use words carefully to create the desired effect.

Look carefully at the author's choice of words. Authors will reveal


their slant on a subject through their choice of negatively or
positively charged words. For example, "slim" and "skinny" both may
have the same dictionary meaning (denotation), but they have different
suggested or associated meanings (connotation). "Slim" is a positive
word whereas "skinny" is usually used in a more negative way.

Look carefully at the choice of details. The author has a purpose, and
you must figure this out. This purpose will drive the choice of
details and then will reveal an attitude or tone. Is the author's
purpose to arouse anger, evoke sympathy, astonish, or amuse? The
author chose those details to serve a purpose. You need to look at
those details to determine that purpose, which then will lead you to
understanding the author's attitude.

Copyright 2009, TCA, LLC.

133

Studyguide for the SAT Skill Review: Attitude Mood and Tone
Typical Attitude, Mood and Tone questions
What is the author's attitude about this subject?
Which description best portrays the author's feelings?
Which of these words adds to the mood (tone/attitude) of the
selection?
What is the author's overall tone in the last paragraph?
Read the following paired passages, noting any specific words that
indicate the authors' attitude, mood, and tone.

Passage 1
I find all the discussion of "saving" our trees a bit pointless. Those
oversized sticks grow again once you chop them down, so why worry?
Besides, trees are everywhere you look. They're a plentiful source of
all kinds of energy and materials, so it would be silly not to use
them. Sure, they provide shade in summer and a home for squirrels in
the winter. But we have houses, buildings, and porches for shade, and
holes in the ground protect all kinds of animals. Look, I'm not an
ogre, but I'm not one of those tree-hugging, nothing-to-do-butcomplain kinds of people, either. I call it like I see it, and that's
that.

Copyright 2009, TCA, LLC.

134

Studyguide for the SAT Skill Review: Attitude Mood and Tone

The tone of Passage 1 is critical and disparaging (derogatory). The


author puts the word saving in quotation marks to indicate that while
others call it saving, he does not really believe it. The words
"pointless" and "silly" show that he does not approve of the
environmentalists' efforts. "Tree-hugging" is a negative way of
portraying environmentalists. The author shows his arrogant attitude
in the last sentence, when he says, "I call it like I see it, and
that's that."
Passage 1
I find all the discussion of "saving" our trees a bit
pointless . Those oversized sticks grow again once you
chop them down, so why worry? Besides, trees are everywhere
you look. They're a plentiful source of all kinds of energy and
materials, so it would be silly not to use them. Sure, they
provide shade in summer and a home for squirrels in the winter. But we
have houses, buildings, and porches for shade, and holes in the
ground protect all kinds of animals. Look, I'm not an ogre, but
I'm not one of those tree-hugging, nothing-to-do-but-complain kinds
of people , either. I call it like I see it, and that's that.

Copyright 2009, TCA, LLC.

135

Studyguide for the SAT Skill Review: Attitude Mood and Tone

Passage 2
I'd never seen a tree so lush and generous. It offered delicate
blossoms in the spring and summer, mouth-watering apples late summer
into fall, and a stark contrast to the white all around us in winter.
To cut that tree down, to execute such a fine specimen, was a foolish
whim. The scent was enough to convince me of the tree's healing and
soothing powers. Sweet smells of blossoms awakened me on balmy spring
mornings, and apple scents lingered long past the first turned leaf of
autumn.
The tone of passage 2 is exactly the opposite: it is a tone of respect
and admiration. The author's appreciative attitude for the trees is
evident in the choice of positive words, such as "delicate," "lush,"
and "mouth-watering." The word "execute," a much stronger, more
emotional version of the term "cut down," indicates that the writer
has a protective attitude toward the tree.
Passage 2
I'd never seen a tree so lush and generous . It offered
delicate blossoms in the spring and summer, mouth-watering
apples late summer into fall, and a stark contrast to the white all
around us in winter. To cut that tree down, to execute such a fine
specimen , was foolish whim . The scent was enough to convince me
of the tree's healing and soothing powers. Sweet smells of
blossoms awakened me on balmy spring mornings, and apple scents
lingered long past the first turned leaf of autumn.

Practice Exercise
Alaskan sled dogs, known as Huskies, are some of the hardest-working
dogs in the world. These are not your ordinary lap dogs. No way! These
dogs live in their own spacious quarters, not in the sled driver'sor
musher'shome.
They train relentlessly from the time they are quite young. Mushers
get these dogs used to their harness and to pulling small things like
wooden boards. The dogs have heavy fur coats and padded paws that
protect them in the cold weather conditions under which they work.
Pulling sleds is their passion and their life.
These dogs spend their lives pulling sleds, sometimes in very harsh
conditions, and according to those involved in sleddingthe dogs love
their work. These beautiful, smart, strong, and loyal animals consider
their work play.

Copyright 2009, TCA, LLC.

136

Studyguide for the SAT Skill Review: Attitude Mood and Tone

Question 1.
The author's attitude toward sled dogs is generally

(A)

neutral

(B)

mocking

(C)

fearful

(D)

admiring

(E)

friendly

The answer is D.
These dogs are described as "beautiful, smart, strong, and loyal,"
implying that the author admires the dogs' many positive traits.

Passage 1
Developers are trying to buy up all of the forestland at the edge of
our town. Not only will this become an eyesore of more houses,
apartments, and shopping centers, but it will also destroy the homes
of the beautiful animals that populate this ecosystem. Where will they
go? When we destroy these homes, we can never replace them. Let's
protect the land for our grandchildren by turning it into a wilderness
preserve.
Passage 2
City taxes keep rising, and our schools are not well funded. Money
from the sale of the forestland will help our community in the form of
a new roof for the high school, higher pay for teachers, and new
computers in our classrooms. I agree that a park would be lovely. But
unfortunately, we cannot afford it! A vote against the park is a vote
for your children's education.

Copyright 2009, TCA, LLC.

137

Studyguide for the SAT Skill Review: Attitude Mood and Tone

Question 2.
What is the relationship of the writer's attitude in the first passage
to the writer's attitude in the second in the second passage?
(A)
(B)
(C)

(D)
(E)

Both writers both have positive attitudes


about development.
Both writers have negative attitudes
about development.
The first writer has a positive attitude
about development while the second writer has a negative
attitude.
The first writer has a negative attitude
while the second writer has a positive attitude.
Both writers have negative attitudes
about development.

The answer is D.
The first writer is against developing the site, while the second
feels it is necessary.

Copyright 2009, TCA, LLC.

138

Studyguide for the SAT Skill Review: Text Development

TEXT DEVELOPMENT
These questions are designed to see if you understand the thought process
and techniques used by the writer. You will be asked to determine the purpose
or function of a portion of a passage, or you will need to determine the
relationship of one part of the passage to another.

Common Structures of SAT Passages


It will help you to be familiar with the types of structures used in SAT
passages. There are six common structures used:
Contrast or Comparison
Main Idea Plus Examples
Main Idea Plus Supporting Arguments
Several Different Aspects of One Idea
Pro and Con
Chronological
By identifying these structures, you will not only be able to answer text
development questions, but also have a better understanding of the
main ideas in the passage.
Contrast or Comparison
Comparisons between two scientific theories or between two authors writing
about the same subject are common examples of this structure found on the
SAT. A passage with this structure may begin with a paragraph about one theory
and then have a second paragraph devoted to the second theory. Another method
is to compare the two theories point by point throughout the passage.

Main Idea Plus Examples


This type of passage usually starts with a main idea and then follows with
several examples to support and clarify the idea.
Main Idea Plus Supporting Arguments
This type also usually begins with the main idea and then develops arguments
and evidence to support the idea.

Copyright 2009, TCA, LLC.

139

Studyguide for the SAT Skill Review: Text Development

Several Different Aspects of One Idea


Passages of this type cover different aspects of a central topic in each
paragraph. For instance, a passage about 18th-century painting might include
a paragraph about the styles, a paragraph about the types of paint, and a
paragraph about the artists' lives.
Pro and Con
This structure is often used for dealing with controversial issues.
One side of an argument is developed, then an alternative viewpoint is
presented. Be aware at all times of whose point of view is being presented.
Chronological
In this type of passage, events are presented in the order in which they occurred.
Narrative and historical passages often fit into this structure.
Key Words in Text Development Questions
The following words are often found in questions relating to
text development:
serves to
the function of
relates
in order to
reason why
the purpose of
Questions will often take the following forms:
The relationship between the first and second paragraphs is....
Lines 23-28 serve primarily to develop....
The author makes his point by....

Copyright 2009, TCA, LLC.

140

Studyguide for the SAT Skill Review: Text Development


Literary Devices
Writers use literary devices to enhance the imagery in their writing. These tools help
make the details in what they've written seem more vivid. Several commonly used
literary devices are:
Simile
Metaphor
Personification
By identifying these literary devices, you will be able to answer
questions about them, and will also have a better understanding of the
author's intent.
Simile
A simile is a comparison of unlike things, but this comparison uses words such as "like,"
"as," "resembles," or "than" when making the comparison. An example is the familiar
"My love is like a red, red rose."
Practice Exercise
Question 1.
Which of the following is a simile?
(A)

The day grew darker as the sun disappeared.

(B)

His normally blue eyes were as dark as thunderheads.

The answer is B. It uses "as ___ as" to compare a person's eyes to


clouds. Option A also uses "as," but in a different way. It uses "as"
to relate two events that happen at the same time.

Question 2.
Which of the following is a simile?

(A)

She looked like her mother in more ways than one.

(B)

The baby's wail sounded like a World War II air-raid siren.

Copyright 2009, TCA, LLC.

141

Studyguide for the SAT Skill Review: Text Development

The answer is B.
This is a comparison of unlike things (a baby's cry and a siren) using the word "like."
Choice A uses "like" to compare two similar things. This is not a simile.

Metaphor
A metaphor is a comparison of two unlike things in which the author says that one thing
is another. The author does not use "like" or "as" in a metaphor. An example is: "My
uncle Jimmy is a bear." This comparison gives the reader the image of a man with a
large stature and features.
That metaphor was direct. Other metaphors might be implied. An example of an
implied metaphor occurs in the sentence, "She bloomed gracefully as she aged." This
does not actually say she is a flower; rather, it implies it, since we most often think of
flowers as blooming.

Practice Exercise
Question 1.
Which of the following is a metaphor?
(A)

A heavy blanket of silence covered the room after the concert.

(B)

The flower waited until the first warm day to bloom.

The answer is A.
In this sentence, silence is being compared to a blanket. In Choice B, the flower is not
being compared to something else. It is blooming, which is what flowers naturally do.

Question 2.
She rose to face them, a palace among cottages. They watched as she walked away,
raising their arms and bidding her farewell.
What is the purpose of the metaphor in the paragraph?

(A)

It makes a farewell appear grander than it usually is.

(B)

It emphasizes how the woman is different from the others.

Copyright 2009, TCA, LLC.

142

Studyguide for the SAT Skill Review: Text Development

The answer is B. In this paragraph, the woman is compared to a palace, while the others are compared to
cottages. This emphasizes their differences in both stature and elegance.
Personification
Personification is a figure of speech in which a nonhuman thing is
spoken of as if it had human form. The object or idea is spoken of as
if it behaved the way people do. An example of this is "The slow
caress of the wind rocked the cradle back and forth." In this case,
the wind is caressing the cradle as if it were human.
Practice Exercise
Question 1.
Which is an example of personification?
(A)

Rising fuel costs are eating my money faster than I can make it.

(B)

Her hair was arranged to look like a beehive.

(C)

Looking for the answer was like trying to find a needle in a haystack.

(D)

As I tried to comprehend the ramifications, my mind felt like a


closed notebook.

The answer is A.
Answer A is the only sentence in which an idea (object) is treated as displaying human
characteristics. It "eats" money. The other choices are either similes or not figures of
speech.

Copyright 2009, TCA, LLC.

143

Studyguide for the SAT Skill Review: Text Development


Question 2.
Which sentence contains personification?

(A)

The shadow of the seagull seemed as graceful as a dolphin.

(B)

The heavy-metal music burst forth angrily from beneath her door.

(C)

He zoomed into the room, sounding like a classroom of


quarreling children.

(D)

As the foreman shouted at him, he was a tree, rooted to the


ground on which he stood.

The answer is B.
In this sentence, music takes on the human characteristics of anger. The other choices are
either similes (A, C) or metaphors (D).

Practice Exercise
Alaskan sled dogs, known as Huskies, are some of the hardest-working
dogs in the world. These are not your ordinary lap dogs. No way! These
dogs live in their own spacious quarters, not in the sled driver'sor
musher'shome.
They train relentlessly from the time they are quite young. Mushers
get these dogs used to their harness and to pulling small things like
wooden boards. The dogs have heavy fur coats and padded paws that
protect them in the cold weather conditions under which they work.
Pulling sleds is their passion and their life.
These dogs spend their lives pulling sleds, sometimes in very harsh
conditions, and according to those involved in sleddingthe dogs love
their work. These beautiful, smart, strong, and loyal animals consider
their work play.

Copyright 2009, TCA, LLC.

144

Studyguide for the SAT Skill Review: Text Development


Question 1.
The author makes her point by
(A)

comparing two things with one another

(B)

citing historical and personal experience as examples

(C)

appealing to the readers' intelligence

(D)

stating a main idea and developing examples

(E)

giving a chronological account of the topic

The answer is D.
The main idea that the dogs are hard working is supported by their
lifestyles, their training, and the conditions under which they work.

Developers are trying to buy up all of the forestland at the edge of


our town. Not only will this become an eyesore of more houses,
apartments, and shopping centers, but it will also destroy the homes
of the beautiful animals that populate this ecosystem. Where will they
go? When we destroy these homes, we can never replace them. Let's
protect the land for our children by turning it into a wilderness
preserve.
City taxes keep rising, and our schools are not well funded. Money
from the sale of the forestland will help our community in the form of
a new roof for the high school, higher pay for teachers, and new
computers in our classrooms. I agree that a park would be lovely. But
unfortunately, we cannot afford it! A vote against the park is a vote
for your children's education.

Copyright 2009, TCA, LLC.

145

Studyguide for the SAT Skill Review: Text Development


Question 2.
Both paragraphs reference children in order to
(A)

argue against the other's points

(B)

appeal to the readers' sense of family

(C)

appeal to the readers' intelligence

(D)

negate the validity of the other's arguments

(E)

vehemently oppose the beliefs of the other

The answer is B.
Each appeal mentions children, making the readers aware that the
decision they make with their vote will affect their families, as well
as their community.

Copyright 2009, TCA, LLC.

146

Studyguide for the SAT Skill Quiz A: Critical Reading


In this passage, the main character, James Thornton, revisits
Pinnacle Rock in Peach Bottom, Pennsylvania, where he and
his friend Mo spent many summer weekends with high school
friends. James, now a college graduate, hasn't been in touch
with Mo since Mo joined the Marines. He reflects on the
carefree days of his youth with fond memories and a more
mature outlook.
James sat on Pinnacle Rock staring into space. Thirty
feet beneath him the Susquehanna River lapped against the
granite rocks surrounding Devil's Hole. Behind him the old
railroad tracks cut through the vertically layered strata
(5) entering the first of the twin tunnels along the north bank.
Each tunnel was about five hundred feet, not long enough to
black out the sunlight at either end, but long enough to make
the jagged, damp interior seem as black as a coal mine. This
is where he and Mo and their friends liked to hang out on
(10) weekends in the summer. It was like no other place, this rock.
It was a rite of passage, a place to think, a place to lose oneself
in the moment.
A gray mist fell over the river now, and memories
of those carefree days washed across his mind like the half(15) sunken log being nudged along the river with its makeshift
buoy bobbing in and out of the colorless waves. It would reach
its end at the Conowingo Dam where debris from storms and
riverbanks and Sunday boaters all washed into elbows on
either side of the dam. It had no place else to go once it
(20) reached the cement walls of the locks. It had no choice but to
pile up in the wings like toys tossed in a rubble pile until they
were forgotten and rusted and no longer meaningful.
James could barely see the toy-sized cars that
traveled across the horizontal stretch of highway above the
(25) dam. He remembered how on sunny, summer days, Mo would
drive his father's nineteen-foot Sea Ray past the sign that read,
"Danger: No Boats Past This Point," all the while screaming,
"Hold on!" to James as he opened the throttle full speed and
whipped him to the outside of the boat at what seemed like
(30) ninety miles an hour on skis. Motorists passing along Route
One would sometimes wave hello or give them a thumbs up as
they gazed down at the sleek, aquamarine pleasure boat
streaking through the crystal glare of sun on water. In those
moments, James felt like he was on top of the world. Cutting
(35) through the waves, his skis casting a spray that formed a
perfect rainbow under the sharp, blue sky, he, James, was in
control. He had his whole future ahead of him. He was young,
he was American, and he could be whatever he wanted to be.

Copyright 2009, TCA, LLC.

147

Studyguide for the SAT Skill Quiz A: Critical Reading

(40)

(45)

(50)

(55)

(60)

(65)

(70)

(75)

(80)

(85)

It seemed like hardly a year had gone by since the


last time he and Mo had been here together. In fact it was in
late August of 1997, the last time they would be seeing each
other before going away to college, Mo to Penn State, and
James to the University of Maryland. They took turns diving off
the ledge and spotting for others. Some days, there were as
many as thirty people there and some of them didn't know that
huge rocks and stumps were just beneath the surface on either
side of the sixty-foot-deep crevice of water called Devil's
Hole. They had to be shown. It was a dangerous dive, but
everyone who knew about it, knew what to do. Each dive was
a thrill and a challenge. Each dive was survival, closely
guarded by friendship.
So much had happened since then. Mo had joined the
Marines after finishing only one year at Penn State. He said it
was the best way to get an education and have Uncle Sam pay
for it. Besides, for Mo, being a Marine was a life-defining
experience. Each day was a new dive into Devil's Hole, a new
chance to be intrepid and steadfast. It seemed that Mo had
been preparing all his life to be a Marine.
James sat on Pinnacle Rock remembering the days
when he and Mo would jog through the tunnels just ahead of
the sound of oncoming trains. They were slow freights with
flatbeds full of scrap metal and coal cars coming from
Pittsburgh or going to West Virginia. Some trains were over
seventy-five cars long and slower than a bicycle on the slight
grade uphill. Still, it was a dangerous thing to do, running on
the tracks through the tunnels just to prove it could be done.
They did lots of things without thinking of the consequences.
Jumping into Devil's Hole or racing against each other
through the narrow, dark tunnels were summer pastimes that
filled their days with the thrills of impetuous, audacious youth.
Over and over they took risks on the spur of the moment.
Weekend after weekend they repeated the same dares, as if by
doing it again they would beat bigger odds and be better for it.
Never once did they consider their own mortality.
The gray sky over the Susquehanna folded into the
thickening mist until James could no longer see the dam or the
opposite shore. A light rain fell past the bare branches of trees
overhanging the precipice, dampening the rocks and adding to
the chill in the March breeze. Another half-sunken log
bumped along the water close to the rocks and then turned out
into the mist and disappeared. James tossed a palm-sized piece
of granite into Devil's Hole and watched its impact create an
ephemeral pattern of concentric circles, which quickly
disappeared into the backwash of waves. Youth was like that,
here one day, gone the next.

Copyright 2009, TCA, LLC.

148

Studyguide for the SAT Skill Quiz A: Critical Reading


Soon it would be spring again and the local kids who
knew about Pinnacle Rock would gather here for all the
weekends of their summer youth, just like he and Mo. There
would be dares and bets and fights and parties and friendships
(90) made and broken. Then, one day years from now, they would
return to Pinnacle Rock and everything would look the same,
only nothing would ever be the same again.

Copyright 2009, TCA, LLC.

149

Copyright 2009, TCA, LLC.

150

Studyguide for the SAT Skill Quiz A: Critical Reading

Quiz Time!
A

Question 1.
Which of the following titles best summarizes the content
of the passage?
(A)

Looking Back at Pinnacle Rock

(B)

Survival of the Fittest

(C)

Rites of Passage on the Susquehanna

(D)

Friendships Made at Devil's Hole

(E)

Summer Pastimes

Question 2.
In line 57, the word "intrepid" most nearly means
(A)

bold

(B)

craven

(C)

brave

(D)

derisive

(E)

quixotic

Question 3.
In line 70, the word "impetuous" most nearly means

(A)

sophomoric

(B)

intentional

(C)

juvenile

(D)

immature

(E)

impulsive

Copyright 2009, TCA, LLC.

151

Studyguide for the SAT Skill Quiz A: Critical Reading

Question 4.
In the last paragraph, the author uses the word "ephemeral" in line 83 to express
(A)

how the weight of the rock was too heavy to leave


a strong pattern of circles

(B)

how the choppy water splashing against Pinnacle


Rock made the circles appear larger and foreboding

(C)

how the fleeting circles dissipated and disappeared into the waves

(D)

how the impact of the rock on the water created a


glittering effect on the concentric circles

(E)

how the moment of impact created a supernatural


sensation as he watched the concentric circles

Question 5.
The author implies that
(A)

Mo failed in his freshman year of college

(B)

Mo was the instigator and he was the follower

(C)

he and Mo dissolved their friendship

(D)

he did not meet his goal

(E)

Pinnacle Rock is a place where young people took


unnecessary risks to prove their courage

Question 6.
The passage as a whole suggests that
(A)
(B)
(C)
(D)
(E)

Copyright 2009, TCA, LLC.

looking back, the author realizes he took careless,


unnecessary risks just to prove himself
the author has mixed feelings about his friendship
with Mo
everyone needs to go through a rite of passage into
adulthood
memories remain untouched by time
the author misses the thrill of diving into Devil's Hole

152

Studyguide for the SAT Skill Quiz A: Critical Reading

Question 7.
The author's tone in the passage is best described as
(A) pessimistic
(B) nostalgic
(C) indifferent
(D) ambivalent
(E) flippant

Question 8.
Which of the following statements concerning the author's awareness can
be inferred from the passage as a whole?
(A) He is obsessed with returning to the carefree
and daring days of his youth.
(B) The best days of his youth were spent at Pinnacle Rock,
and now he is an unhappy adult who can't go back to the past.
(C) He feels betrayed by Mo because he enlisted in the Marines
rather than continuing his college education as they had planned.
(D) He believes that the thrills and dares he and Mo
experienced helped them to become better adults.
(E) He realizes that his youthful experiences are part of the past
and that he can never look at Pinnacle Rock the same way again.

Copyright 2009, TCA, LLC.

153

Copyright 2009, TCA, LLC.

154

Studyguide for the SAT Skill Quiz A: Critical Reading

Hints!
A

Hint for Question 1:


Reread lines 1 and 59.
Hint for Question 2:
Read the entire sentence. What was Mo trying to prove?
Hint for Question 3:
Read the sentences before and after line 70.
Search for contextual clues in lines 67 and 71.
Hint for Question 4:
Read the last sentence of the paragraph on line 85.
Look for the contextual clue.
Hint for Question 5:
Think about which statement relates to the overall passage.
Do not be distracted by statements that are too narrow.
Hint for Question 6:
Reread paragraph 6.

Hint for Question 7:


Reread lines 8-12 and 37-38.
Note that the author has positive feelings.
Eliminate answers with negative connotations.
Hint for Question 8:
Reread the last sentence of the last paragraph.

Copyright 2009, TCA, LLC.

155

Copyright 2009, TCA, LLC.

156

Studyguide for the SAT Skill Quiz A: Critical Reading

Answers!
A

Question 1.
The answer is (A).
Explanation for Question 1:

The answer is (A).


The author refers throughout the passage to his
experiences at Pinnacle Rock. Answers (B) and (E) are
too vague and do not mention the focal point of the
passage. Answer (C) is too broad. Answer (D) is too
narrow.

Question 2.
The answer is (C).
Explanation for Question 2:
The answer is (C).
The word "intrepid" most nearly means brave or steadfast.
This is a positive word. Answers (B), (D), and (E) are
all negative. Answer (A) is a distractor that is close
in meaning; however, line 57 provides the contextual clue
for the most accurate answer, which is "brave" or "steadfast."

Question 3.
The answer is (E).
Explanation for Question 3:
The answer is (E).
The author and his friend did things for the thrill of it without thinking
of the consequences. The word that most nearly means impetuous, hasty, or
sudden is "impulsive."

Copyright 2009, TCA, LLC.

157

Studyguide for the SAT Skill Quiz A: Critical Reading


Question 4.
The answer is (C).
Explanation for Question 4:
The answer is (C).
In lines 83-85 of the passage, the context clue for the word "ephemeral"
is found after the comma: the circles "quickly disappeared." The word
"fleeting" contained in answer (C) most nearly means the same as "ephemeral."

Question 5.
The answer is (E).
Explanation for Question 5:
The answer is (E).
Evidence in the passage points to the risks they took on Pinnacle Rock.
There is no evidence to prove any of the other statements are true.

Question 6.
The answer is (A).
Explanation for Question 6:
The answer is (A).
Evidence in the passage points to the author's more mature attitude
regarding the thrill-seeking actions of his youth. In Paragraph 6, he
states, "Never once did they consider their own mortality." There is no
evidence in the passage to prove any of the other statements are true.
Question 7.
The answer is (B).
Explanation for Question 7:
The answer is (B).
Contextual clues throughout the passage indicate that the author has fond,
positive memories of his experiences. The third paragraph is one example.
Answers (A) and (D) have negative connotations. Although the author is
sentimental in some parts, the reading as a whole indicates a nostalgic
view of his past, especially his friendship with Mo.

Copyright 2009, TCA, LLC.

158

Studyguide for the SAT Skill Quiz A: Critical Reading


Question 8.
The answer is (E).
Explanation for Question 8:
The answer is (E).
The author states in the last sentence of the passage that nothing would
ever be the same again. He can never go back to being the same daring youth
he was in the past. Answers (A), (B), (C), and (D) are not supported by
evidence in the passage.
The term "Renaissance" characterizes the period or
movement that marks the transition from the medieval to the
modern world in Western Europe. During this time, poets,
artists, writers, and intellectuals, inspired by the works of
antiquity, strove to attain new creative heights. This
passage examines the characteristics of that period.

Copyright 2009, TCA, LLC.

159

Copyright 2009, TCA, LLC.

160

Studyguide for the SAT Skill Quiz B: Critical Reading


Scholars have long debated how and when the
Renaissance began. One thing, however, that most scholars
agree upon is the significance of this historical period as an
evolutionary movement toward individualism in art and
(5) thought. Although there is no historical consensus on the exact
chronological sequence of events leading to the Renaissance,
many historians agree that visionaries of the fifteenth century
had a sense of belonging to a movement far greater than
themselves and so far-reaching as to supersede the artistic and
(10) literary forms of the classical era. This phenomenon
ultimately produced the Renaissance, a period when art,
literature, language, and philosophy found new and
extraordinary interpretations in the hands of the Humanists.
Humanists believed in the importance of the
(15) humanities or "humane letters" rather than "Divine letters," or
the study of Scripture, to inspire them. Ultimately, intellectuals
of this period became aware of man's inner potential, one that
was not bound by the study of scripture as the only means to
reach creative potential.
(20)
The fourteenth century Italian poet Petrarch
characterized the early phase of this movement as the New
Age. In fact, it was his term, "Rinascita," which comes from
the Latin "renasci," to be reborn, that eventually led to the
term Renaissance that we use today.
(25)
Petrarch believed that the destruction of the Roman
Empire by barbarian invasions in 476 A.D. marked the end of
classical antiquity and the beginning of a thousand years of
darkness, which he characterized as "the Dark Ages." The
awakening from this period of creative void was a time of
(30) rebirth, which he distinguished as the New Age.
Petrarch's view limited the Rinascita to the revival of
Latin and Greek classics from antiquity based on
unadulterated translations of ancient text. Most significant
was his conviction that the pagans of antiquity were
(35) profoundly inspired by the pursuit of arts and literature in an
effort to reach the highest potential of perfection on earth. This
perception led to a new sense of individualism and
enlightenment that characterized the age of the Renaissance as
one that refuted the beliefs of medieval society that man's
(40) earthly life was merely a function of the social order. From the
fourteenth century through the early seventeenth century, this
new individualism grew steadily in Western European culture
with a momentum fed by man's desire to define and evaluate
the essential values between men of antiquity and the
(45) emerging modern man.

Copyright 2009, TCA, LLC.

161

Studyguide for the SAT Skill Quiz B: Critical Reading


Not only did artists and writers turn to classical
culture for inspiration, they set new goals. They explored the
relationship between ancient cultures and their own cultures,
and by doing so, they discovered a new vision of human life,
(50) of man as a glorious creature capable of developing his mind
and body to reach his fullest potential toward perfection.
They did not merely endeavor to duplicate Graeco-Roman
works as a means of recapturing the glory of the ancients; they
chose instead to surpass the classics with a new individualism
(55) that had no boundaries. No longer would poets and writers,
artists and philosophers base their intellectual arguments
solely on traditional beliefs, but they would seek new theories
and embrace new convictions outside of traditional beliefs,
trying all the while to reconcile Christianity with the scholarly
(60) views of antiquity. The result was tension, conflicting ideas,
exciting debates, prolific creativity, and the birth of Modern
Man.

Copyright 2009, TCA, LLC.

162

Studyguide for the SAT Skill Quiz B: Critical Reading

Quiz Time!
A

Question 1.
Which of the following statements best represents
the major idea of the passage?
(A)
(B)
(C)
(D)
(E)

Humanists turned to "Divine" letters as a source of creative


inspiration.
The pagans of antiquity lived in the Dark Ages.
A major goal of Humanists was to duplicate Graeco-Roman
works.
The Renaissance marked the beginning of a new
individualism.
Medieval writers were not enlightened.

Question 2.
In line 31, Petrarch's use of the word "Rinascita"
is used to capture the
(A)
(B)
(C)
(D)
(E)

beginning of a new social order


significance of the pursuit of arts and literature
awakening of modern man
rebirth of the classics from antiquity
revival of the individualism of ancient times

Question 3.
Which of the following statements concerning Humanism can be
inferred from the passage?
(A)
(B)
(C)
(D)
(E)

Copyright 2009, TCA, LLC.

Humanists questioned traditional ideologies and beliefs.


Humanism was confined to imitating Graeco-Roman art.
Humanists were atheists.
Humanists inspired the emergence of modern man.
Humanism was geographically limited to the Italian
Renaissance.

163

Studyguide for the SAT Skill Quiz B: Critical Reading

Question 4.
In line 33, the word "unadulterated" most nearly means
(A)
(B)
(C)
(D)
(E)

untainted
unmolested
impure
infected
untenable

Question 5.
The phrase, "humane letters," in line 15 is used to indicate
an emphasis on inspiration drawn from
(A)
(B)
(C)
(D)
(E)

secular sources
religious convictions
creative notions
historical documents
arts and letters from antiquity

Question 6.
The passage as a whole suggests that
(A)
(B)
(C)
(D)
(E)

man did not aspire to reach his greatest creative


potential prior to the fourteenth century
the Renaissance took place predominantly in Italy
humanists, inspired by a new sense of individualism,
aspired to realize their greatest potential
the French poet Petrarch best characterized the historical
significance of the Renaissance
the Renaissance evolved over a period of two centuries

Question 7.
In line 61, "prolific" most nearly means
(A)
(B)
(C)
(D)
(E)

Copyright 2009, TCA, LLC.

essential
productive
diffident
sparse
energetic

164

Studyguide for the SAT Skill Quiz B: Critical Reading

Hints!
A

Hint for Question 1:


Read lines 40-45.
Hint for Question 2:
Read line 33.
Hint for Question 3:
Read lines 49-62.
Hint for Question 4:
Answers (C) and (D) are antonyms.
Hint for Question 5:
Read lines 14-19. Note the use of "rather than" to indicate opposites.
Hint for Question 6:
Read the last paragraph.
Hint for Question 7:
The word "prolific" is a positive word.
Two of the answers are negatives. Use
the remaining three choices in the sentence from line 61.

Copyright 2009, TCA, LLC.

165

Copyright 2009, TCA, LLC.

166

Studyguide for the SAT Skill Quiz B: Critical Reading

Answers!
A

Question 1.
The answer is (D).
Explanation for Question 1:
The answer is (D).
Answers (A), (B), and (C) contradict the evidence in the passage. Answer
(E) is too narrow. The passage is not about medieval writers. Answer
(D) is the only answer supported by evidence in the passage.

Question 2.
The answer is (D).
Explanation for Question 2:
The answer is (D).
Line 33 indicates that Petrarch's view was based on the
rebirth of translations of ancient text. Renascita comes
from the Latin "renasci" that means to be reborn, as
noted in line 23 in the previous paragraph.

Question 3.
The answer is (D).
Explanation for Question 3:
The answer is (D).
Answer (A) is stated implicitly in the passage. It is not inferred, so it does
not answer the question. A conclusion that can be drawn from evidence in
the passage or "inferred" is that Humanists inspired the emergence of
modern man, answer (D). Answers (B), (C), and (E) are refuted in the
passage and, therefore, are not true.

Copyright 2009, TCA, LLC.

167

Studyguide for the SAT Skill Quiz B: Critical Reading


Question 4.
The answer is (A).
Explanation for Question 4:
The answer is (A).
"Unadulterated" suggests a positive connotation. Answers (C), (D), and
(E) can be discarded because they have negatives. Answer (B) does not
make sense when inserted in the sentence from line 33. Answer (A)
makes sense and most nearly means unadulterated.

Question 5.
The answer is (A).
Explanation for Question 5:
The answer is (A).
The author clearly identifies "humane letters" as the opposite
of "Divine letters," which are based on Scripture.
The word "secular" means nonreligious.

Question 6.
The answer is (C).
Explanation for Question 6:
The answer is (C).
Lines 37-40 and 49-53 repeat the concept that Humanists aspired to
realize their greatest potential on earth. Answers (A), (B), and (E) are
false, based on evidence in the passage. Answer (D) is too narrow and
does not cover the main idea of the passage as a whole. Only Answer (C)
focuses on the passage as a whole.

Question 7.
The answer is (B).
Explanation for Question 7:
The answer is (B).
(B) "Prolific" is a positive word. Answers (C) and (D) are negative
words. Answer (B) is positive and most nearly means the same as
prolific.

Copyright 2009, TCA, LLC.

168

Studyguide for the SAT Skill Quiz C: Critical Reading


The 1988 Supreme Court ruling in the Hazelwood School
District v. Kuhlmeier case had a significant impact on the
freedom of student press. Before the Hazelwood ruling, the
interpretation of the First Amendment doctrine upheld by
courts was that students do not shed their constitutional rights
at the schoolhouse gates. In the following passage, the
author, a former high school journalism instructor and
advisor, reacts to the impact of the new ruling on the student
press.
Until the Supreme Court's landmark decision in the
Hazelwood case, student journalists had been entrusted with
the inalienable right of free speech; the right to voice their
opinions responsibly about issues that affect their lives and the
(5) right to investigate the truth and present it in a fair and
balanced news story. They modeled their actions on the same
codes as professional journalists, and they submitted to the
authority of editors and their publication's editorial board as
watchdogs of responsible reporting. Now, it had been decided
(10) by the highest court in the land that someone else would
decide the appropriateness of their work, namely, appointed
authorities of the educational institution.
On January 14, 1988, in a five-to-three ruling, the
Supreme Court justices stated that school officials have the
(15) authority to censor school newspapers, plays and other
"school-sponsored expressive activities." Under the ruling,
students' First Amendment rights stop at the schoolhouse gate,
where once inside, they no longer have the right to express
themselves freely within the boundaries of the Constitution.
(20) Instead, school administrators have the Supreme Court's
backing to request prior review of publications and to hold
advisors accountable for not censoring stories that may be
interpreted as undesirable or inconsistent with the goals of the
educational institution.
(25)
By attributing information to reliable sources,
researching and documenting the truth, and balancing points
of view, the student reporter embraces the ethical codes of
professional journalism. By avoiding the pitfalls of libel,
negligence, and invasion of privacy, and by recognizing the
(30) difference between inciting disruption and raising awareness,
the student reporter exercises the right to free speech within
the framework of the Constitution. It is true that without
experience and guidance, these criteria are not always
discernible to inexperienced reporters; however, strong student
(35) editors with sound knowledge of press law, supported by
guidance from the advisor, are the watchdogs of sound
reporting and ethical behavior on the part of the entire staff.
Censorship under these circumstances is unnecessary because
responsible reporting is self-directed rather than dictated.

Copyright 2009, TCA, LLC.

169

Studyguide for the SAT Skill Quiz C: Critical Reading


(40)

It is understandable that the high court would


question the ability of student journalists to distinguish
between the public's right to know and the citizen's right to
privacy. Indeed, this seems to be the issue at the heart of the
Supreme Court's ruling on the Hazelwood case. Without
(45) experience, young reporters often justify their right to publish
the truth by claiming the public's right to know and their First
Amendment right to speak freely. Less clearly defined is the
gray area of invasion of privacy. It is often difficult for
professional journalists, let alone students, to discriminate
(50) between public and private information. The actions of a
public person or a person in a public place are not protected
from the public's right to know under the law; whereas the
actions of a private person or a person protected within the
confines of a private place are protected. Crossing this line
(55) can lead to lawsuits that no publication wishes to have
delivered to their door. The Constitution protects our privacy
just as vehemently as it protects our right to free speech.
By censoring articles on teenage pregnancy and
family divorce in the student paper at Hazelwood East High
(60) School in suburban St. Louis, administrators acted to protect
the school from potential lawsuits based on the fear that
private information revealed in the stories could be found to
be offensive and harmful to minors and their parents. Students
argued that their free speech rights were violated and that the
(65) stories contained true information from reliable sources about
issues that high school journalists had a right to cover.
Using common sense about approaches to coverage
and the expertise of informed advisors and press law sources,
students should be able to resolve conflicts over what is fit to
(70) print without the need for censorship. Although it is
unfortunate that this case moved all the way to the Supreme
Court for a resolution, both students and school officials
involved in this historic hearing were intrepid in their pursuit
of a clear interpretation of the law. Because of their
(75) indefatigable efforts, both professional and student journalists
were reminded more than ever that covering the news isn't
always as simple as it seems.

Copyright 2009, TCA, LLC.

170

Studyguide for the SAT Skill Quiz C: Critical Reading

Quiz Time!
A

Question 1.
The main focus of this passage is on
(A)
(B)
(C)
(D)
(E)

the codes of good journalism modeled by professional


journalists
the responsibilities of the student press
protecting student reporters from lawsuits
students' inalienable rights under the Constitution
the effect of the Supreme Court's ruling on the Hazelwood
case on the student press

Question 2.
It can be inferred from this passage that the author
(A)
(B)
(C)
(D)
(E)

vehemently opposes the Supreme Court ruling


sides with the students in the Hazelwood case
believes that censorship was necessary under the
circumstances in the Hazelwood case
supports the high court's decision that school authorities have
the right to censor student work
believes that the Hazelwood decision was regrettable but that
students and school officials involved were valiant in their
efforts to get an interpretation of the law

Question 3.
The author's reaction toward the Supreme Court's ruling is best
described as
(A)
(B)
(C)
(D)
(E)

Copyright 2009, TCA, LLC.

disappointment
support
optimism
relief
sarcasm

171

Studyguide for the SAT Skill Quiz C: Critical Reading

Question 4.
In line 25, the word "attributing" most nearly means
(A)
(B)
(C)
(D)
(E)

ascribing
characterizing
explaining
providing
gathering

Question 5.
The author implies that
(A)
(B)
(C)
(D)
(E)

advisors have the right to censor student work


informed student editors and a knowledgeable advisor are the
first vanguard of the student press
school administrators have no right to question students' rights
to express themselves freely
the First Amendment protects everyone's right to free speech
under any circumstances
the Constitution does not protect the right to privacy as much
as it protects freedom of speech

Question 6.
Which of the following headlines best summarizes the content of
the passage?
(A)
(B)
(C)
(D)
(E)

Students React to Censorship


Supreme Court Teaches a Lesson
Prior Review Favors Schools
Hazelwood Ruling Underestimates Student Press
Administrators Rejoice Over Hazelwood Ruling

Question 7.
The word "discernible" in line 34 most nearly means
(A)
(B)
(C)
(D)
(E)

Copyright 2009, TCA, LLC.

amenable
apparent
avertable
preventable
arguable

172

Studyguide for the SAT Skill Quiz C: Critical Reading

Question 8.
The word "vehemently" in line 58 most nearly means
(A)
(B)
(C)
(D)
(E)

Copyright 2009, TCA, LLC.

intentionally
apathetically
vigorously
cravenly
thoroughly

173

Copyright 2009, TCA, LLC.

174

Studyguide for the SAT Skill Quiz C: Critical Reading

Hints!
A

Hint for Question 1:

Read the first sentence of the passage.


Hint for Question 2:

Read the last sentence of the passage.


Hint for Question 3:

Read the last paragraph.


Hint for Question 4:

Insert each of the answers into the sentence.


Which makes the most sense?
Hint for Question 5:

Read lines 35-40.


Hint for Question 6:

Remember the focus in the first sentence of the passage.


Combine that with the point the author makes about the
capabilities of student editors.
Hint for Question 7:

Answers (A), (C), and (D) are not related.


Read the sentence using answer choices in
place of the word "discernible."

Copyright 2009, TCA, LLC.

175

Studyguide for the SAT Skill Quiz C: Critical Reading


Hint for Question 8:

Answers (B) and (D) are negative.


Look for a positive word that best completes
the meaning of the sentence when replacing
the word "vehemently."

Copyright 2009, TCA, LLC.

176

Studyguide for the SAT Skill Quiz C: Critical Reading

Answers!
A

Question 1.
The answer is (E).
Explanation for Question 1:
The answer is (E).
Answer (E) is the only answer that mentions the Hazelwood case, which
is referenced in the first sentence of the first paragraph and is woven
throughout the passage as the main topic.

Question 2.
The answer is (E).
Explanation for Question 2:
The answer is (E).
Answer (E) is the most accurate and is supported by evidence in the last
paragraph. Answer (A) indicates a vigorous opposition to the ruling.
There is no evidence to suggest to what degree the author disagrees.

Question 3.
The answer is (A).
Explanation for Question 3:
The answer is (A).
Answers (B), (C), and (D) suggest a positive reaction. Answer (E) is
unrelated. The only answer that reflects the negative connotation of the
word, "unfortunate," in the last paragraph is (A).
Question 4.
The answer is (A).
Explanation for Question 4:
The answer is (A).
Answers (C) and (D) make no sense since a reporter would not explain or
provide information to reliable sources based on the context of line 25.
Answers (B) and (E) are unrelated. Only answer (A) makes sense when
inserted in the sentence.

Copyright 2009, TCA, LLC.

177

Studyguide for the SAT Skill Quiz C: Critical Reading


Question 5.
The answer is (B).
Explanation for Question 5:
The answer is (B).
Answer (B) is in keeping with the author's comments in lines 35-40.
Answers (C), (D), and (E) are erroneous. Answer (A) conflicts with the
author's point of view that students should be responsible enough to
know their rights and the rights of others under the law.

Question 6.
The answer is (D).
Explanation for Question 6:
The answer is (D).
Only answer (D) covers the main topic of the passage. All of the other
headlines are not related to the overall passage.

Question 7.
The answer is (B).
Explanation for Question 7:
The answer is (B).
Only Answer (B) makes sense when inserted in the sentence in place of
the word "discernible." Answer (C), "avertable," means avoidable.
Answer (A), "amenable," means agreeable. These answers do not fit the
context of the stem sentence.
Question 8.
The answer is (C).
Explanation for Question 8:
The answer is (C).
Answers (B) and (D) are negatives. Answer (C) most nearly means the
same as "vehemently."
Lighting designers combine technology and art to create the
most effective visibility for stage productions. This passage
explores the various aspects that designers must consider in
order to integrate the dramatic elements of a play with an
artistic concept for lighting the stage.

Copyright 2009, TCA, LLC.

178

Studyguide for the SAT Skill Quiz D: Critical Reading


Lighting the stage is an art that is often
underestimated. In real life, the eye sees nuances of color and
intensities of light and shadow that create tone and mood;
however, on the stage, artificial light must create the illusion
(5) of reality. Without subtle highlights and shadows, actors, set
pieces, and props would appear dull and uninteresting to the
audience; however, through the expressive use of light, the
stage becomes a living scene where people and objects appear
to exist in natural light that indicates time of day, season,
(10) place, and mood. Like a painter, the lighting designer
endeavors to sculpt and shape perspective from the audience
view. With the stage as the canvass, a lighting designer
must artistically combine the elements of form, contrast,
harmony, balance, and color.
(15)
The designer's goal is to paint the stage with light to
create selective visibility by using an effective combination of
specific and general illumination. Specific illumination is
focused lighting with little or no spill effect while general
lighting is distributed broadly and spills over without a direct
(20) focus. The designer's challenge is to use stage lights to create
a balance between highlights, shade, and shadow while
supporting the dramatic elements of the stage production.
One type of specific illumination is form-revealing
light, which separates the actor from the setting and enhances
(25) depth perception from the audience view. The art of creating
depth perception by aiming light at certain angles and levels
and by using variables of intensity and color requires
knowledge and experience. Neophyte lighting designers often
make two common mistakes in this regard. They may
(30) indiscriminately illuminate the entire stage with insufficient
contrast, or they may create sharp distinctions between acting
areas and background settings, resulting in an artificial, twodimensional appearance. The result is either a wash of
monotonous light evenly distributed over the stage, or a harsh
(35) contrast between the intended visible area and the
surroundings. Simply put, one of the qualities of an expert
lighting designer is the ability to detect what is too much or
too little light. Focusing just the right amount of light on the
actor reveals form without distorting depth.

Copyright 2009, TCA, LLC.

179

Studyguide for the SAT Skill Quiz D: Critical Reading


(40)

Just as harmony is an important element in artistic


design, it is also essential to the relationship an actor has with
the setting, and lighting contributes to the harmonious effect
of that relationship. With subtle uses of color and intensity,
along with angle and direction of light, the designer can
(45) highlight the actor with selective visibility while enhancing
the mood of the scene. To accomplish this, the designer must
first use effective methods of distributing specific illumination
from the back, the front, and the sides. Then, general lighting
or spilled light can be used to fill in where necessary. Specific
(50) lighting on the actor and general lighting on the background is
the rule of thumb. Generally, the intensity of background
lighting should not exceed one-fourth that of the lighting on
the actor, and in most cases much less.
In addition to specific illumination, the actor's form
(55) can be shaped by the use of side lighting and backlighting. By
spotting selectively with intense, specific lighting from the
sides, the designer can avoid the use of high levels of general
illumination that cause a monotonous evenness in light. The
technique of back lighting by aiming specific illumination on
(60) the actor's head and shoulders from behind enhances the
three-dimensional perspective, keeping the actor in the
foreground while diminishing objects in the background.
Finally, the amount of light and use of color
balance the mood and tone of the scene. For example, a
(65) bright, sunny day on the plains in "Oklahoma" can be created
with warm ambers, pinks, and tints of yellow, typically used
for musical comedy. In contrast, the cold, stark shadows of a
dungeon for prisoners of the Spanish Inquisition in "Man of
La Mancha" would require a palette of blue-greens to deepen
(70) dark corners and amber yellow to suggest candlelight in
specific areas.
Choosing the most aesthetically pleasing and
effective color mediums to light every scene can be an
arduous task. Not only are there costumes and makeup
(75) considerations, but lighting must also support the dramatic
elements of the play without calling attention to itself. With
training and experience, the lighting designer can illuminate
the acting area with subtle variations of just four colors amber, daylight blue, lavender, and pink.
(80)
By using cool and warm variants of color and by
artistically choosing the intensity, angle, and level of lighting
to create depth perception, the designer can achieve an
aesthetically pleasing result that serves to enhance the scene
and focus the audience's attention while embellishing the
(85) dramatic elements of the play.

Copyright 2009, TCA, LLC.

180

Studyguide for the SAT Skill Quiz D: Critical Reading

Quiz Time!
A

Question 1.
Which of the following statements best represents the major idea of
the passage?
(A)
(B)
(C)
(D)
(E)

Color is not as important to artistic


lighting as specific illumination.
Harmony is essential to the relationship
an actor has with the setting.
Lighting acting areas is generally done from the front.
Lighting design is an art that requires expertise to create
aesthetic results without calling attention to itself.
Form-revealing light is used to enhance objects in the
background.

Question 2.
The author uses the word, "indiscriminately," in line 30 to express
this approach to illuminating the stage:
(A)
(B)
(C)
(D)
(E)

deliberately
irresponsibly
inaccurately
indiscreetly
unsystematically

Question 3.
In line 5, the word "subtle" most nearly means
(A)
(B)
(C)
(D)
(E)

Copyright 2009, TCA, LLC.

subordinated
artistic
inconspicuous
soft
artificial

181

Studyguide for the SAT Skill Quiz D: Critical Reading

Question 4.
In the third paragraph, the author presents evidence to show that
(A)
(B)
(C)
(D)
(E)

the art of creating depth perception through lighting requires


knowledge and experience
expert lighting designers often make two common mistakes
illuminating the entire stage with sufficient contrast results in
monotonous light
depth perception is more important than form-revealing light
illuminating the stage with specific light can create harsh
contrasts between the actor and the set

Question 5.
Which of the following best summarizes the content of the passage?
(A)
(B)
(C)

(D)
(E)

Specific lighting and general illumination require balance and


artistic combination.
Lighting design is an art that requires technical expertise with
equipment.
Lighting designers are artists who shape the audience's
perspective with expressive use of light that supports the
dramatic elements of the play.
Unlike a painter, the lighting designer does not consider the
elements of form and color so much as the intensity of light and balance.
Angle and direction are the two most important elements of
lighting.

Question 6.
The author implies that
(A)
(B)
(C)
(D)
(E)

Copyright 2009, TCA, LLC.

without effective lighting, a stage production is missing an


important aesthetic aspect
lighting the stage without careful use of intensity can result in
a two-dimensional appearance
most painters could easily become lighting designers
there is little need to consider color in lighting design if one
uses a four-color palette
most lighting designers do not have experience

182

Studyguide for the SAT Skill Quiz D: Critical Reading

Question 7.
In line 28, the word "neophyte" most nearly means
(A)
(B)
(C)
(D)
(E)

inadequate
unskilled
seasoned
unpretentious
novice

Question 8.
In line 74, the phrase "arduous task" is used to mean
(A)
(B)
(C)
(D)
(E)

Copyright 2009, TCA, LLC.

demanding job
artistic requirement
collaborative endeavor
laborious burden
complicated decision

183

Copyright 2009, TCA, LLC.

184

Studyguide for the SAT Skill Quiz D: Critical Reading

Hints!
A

Hint for Question 1:

The main idea covers the scope of the entire passage.


Eliminate answers that narrowly apply to only one paragraph.
Hint for Question 2:

Answer (A) conveys a positive connotation.


Look for a negative word that best means
the same as "indiscriminately" when used
in the sentence.
Hint for Question 3:

Answers (B) and (C) are unrelated.


Hint for Question 4:

Read lines 25-28.


Hint for Question 5:

Answers (A) and (E) are too narrow.


Read lines 10-14.
Hint for Question 6:

Read the last paragraph.


Hint for Question 7:

Answer (C) most nearly means the


opposite of "neophyte."
It can be immediately eliminated.

Copyright 2009, TCA, LLC.

185

Studyguide for the SAT Skill Quiz D: Critical Reading


Hint for Question 8:

Discard answers with negative phrases.


Substitute each answer choice in place of the
phrase, "arduous task" in the first sentence
of the paragraph on line 74.

Copyright 2009, TCA, LLC.

186

Studyguide for the SAT Skill Quiz D: Critical Reading

Answers!
A

Question 1.
The answer is (D).
Explanation for Question 1:
The answer is (D).
Answers (A), (C), and (E) are erroneous. They contradict the content of
the passage. Answer (B) is a valid statement that is supported by the
passage; however, it is a minor statement in comparison with the major
point in answer (D).

Question 2.
The answer is (E).
Explanation for Question 2:
The answer is (E).
Answer (A) is a positive word. The context of the sentence indicates a
negative word is needed. Both (B) and (C) do not fit the context of the
sentence when used in place of the stem word, nor do they mean
"indiscriminately." Answer (D) suggests carelessness or recklessness.
The author points out that these are mistakes based on inexperience,
thus recklessness does not apply. Answer (E), "unsystematically," most
nearly means the same as "indiscriminately."

Question 3.
The answer is (C).
Explanation for Question 3:
The answer is (C).
The author uses the word "subtle" to suggest use of highlights and
shadows that are not exaggerated, but rather "expressive." Read the entire
sentence. In this context, the word "subtle" most nearly means
"inconspicuous."

Copyright 2009, TCA, LLC.

187

Studyguide for the SAT Skill Quiz D: Critical Reading


Question 4.
The answer is (A).
Explanation for Question 4:
The answer is (A).
Answer (B) is tricky. Note that the first word is "expert," not
"inexperienced," lighting designers. Answers (C), (D), and (E) are
erroneous and are not supported by the paragraph content. Only answer
(A) is supported with evidence in the paragraph to explain how lighting
designers without experience can make common mistakes.

Question 5.
The answer is (C).
Explanation for Question 5:
The answer is (C).
The first and last paragraphs support answer (C). All other answers are
either too narrow or are false statements not supported by the passage.

Question 6.
The answer is (A).
Explanation for Question 6:
The answer is (A).
Answer (B) is true; however, it is clearly stated rather than implied in the
passage. Answers (C), (D), and (E) are false assumptions not supported
by the passage. The only statement implied by the author is answer (A),
which is supported in the first and last paragraphs.

Copyright 2009, TCA, LLC.

188

Studyguide for the SAT Skill Quiz D: Critical Reading


Question 7.
The answer is (E).
Explanation for Question 7:

The answer is (E).


Answer (C) is an antonym for "neophyte." Answers (A) and (D) are
unrelated to the context of the sentence. Answer (B) is close in meaning;
however, the word "novice" fits more appropriately into the sentence in
place of "neophyte." Always insert your last choices into the sentence to
determine the most accurate meaning.

Question 8.
The answer is (A).
Explanation for Question 8:
The answer is (A).
Substituting the word "task" for the second word in the answer choices
automatically eliminates answers (D) and (E). Neither a burden nor a
decision closely means "task." Answer (C) reads too much into the
phrase. By replacing the stem phrase with answer choices (A) and (B),
one can determine that choice (A) most nearly means the same as
"arduous task."

Copyright 2009, TCA, LLC.

189

Copyright 2009, TCA, LLC.

190

Studyguide for the SAT Skill Quiz E: Critical Reading


These passages discuss the arrival of the first Americans on the North
American continent. The author of Passage 1 believes that the
Clovis people were the first to arrive. The author of Passage 2
explores recent advances in science to refute the "Clovis First" theory.
Passage 1
According to the "Clovis First" theory, the search for the first
Native Americans begins roughly c. 11,500 B.C. near the end of the
Pleistocene epoch. Around that time, those archaeologists and
paleoanthropologists who subscribe to this model for the peopling of the
(5) Americas believe that the earliest Paleo-Indian culture found in the
Americas, that of the fluted-point hunters, had made their way from
Asia across the Bering land bridge known as Beringia. Proponents of
this theory believe that these first migratory people arrived on the vast
North American continent and found it devoid of any other humans.
(10)
Support for this arrival date is often based upon the extent of
glaciation during the Pleistocene epoch. Either these people had to
arrive before 30,000 B.C. or after 11,000 B.C. because between these
dates, the ice-free corridor along the Pacific coast was closed during the
height of glaciation.
(15)
Since there is no concrete archaeological evidence to support
an arrival before 30,000 B.C., those who believe in the "Clovis First"
model state that these Asian migrants crossed the land bridge from
Siberia and began moving south some time after 11,000 B.C. along the
open Pacific coast, following game and gathering nuts and berries.
(20) Further, this theory promotes that the journey to the tip of South
America was accomplished by 9000-8500 B.C., this is in light of Clovis
people sites discovered there and dated to this period.
At the heart of this theory is the belief that what appears to be
archaeological evidence of a culture earlier than 11,500 B.C. has been
(25) misdated. This includes the discoveries at Meadowcroft Rock Shelter in
southwestern Pennsylvania, Saltville and Cactus Hill in Virginia, and
the Topper site in South Carolina. The dig at Meadowcroft Rock
Shelter revealed human remains dated to 14,500 B.C., but there
continues to be strong disagreement within the scientific community as
(30) to the validity of the data collected. The "Clovis First" proponents feel
certain that the exact date remains in serious question, thus giving
credence to their settlement timetable. In addition, the finds in Virginia
and South Carolina, believed by some to be as old as 12,000 B.C., are
also viewed with a skeptical eye.

Copyright 2009, TCA, LLC.

191

Studyguide for the SAT Skill Quiz E: Critical Reading


Passage 2
(35)

Scientists who continue to give unflagging allegiance to the


"Clovis First" theory regarding the earliest Paleo-Indian migration and
ensuing cultural development in the Americas may soon find
themselves alone in the face of mounting archaeological evidence, as
well as other possible suggestions for routes of arrival on the North and
(40) South American continents. Accumulating evidence clearly reflects a
much earlier migration of the people who would eventually come to be
known as the first Native Americans.
One has only to study the results of pre-Clovis archaeological
digs at Cactus Hill in Virginia, Meadowcroft Rock Shelter in
(45) Pennsylvania, and Monte Verde in Chile, to recognize that artifacts and
residue from Paleolithic fire pits indicate a far earlier human
occupation of the Americas than that of the Clovis people.
Radiocarbon dating of bones and plant material found at these sites
reveals dates of 12,000 B.C. to as early as 30,000 B.C. Still, despite this
(50) mounting evidence, many scientists hold to the theory that dates earlier
than 12,000 B.C are subject to question and require further testing and
extensive investigation.
For many in the scientific community, it seems illogical to
assume that those early Asian people who found their way across the
(55) land bridge of Beringia found no other people on the two vast
continents. In addition, the speed at which they would have had to
travel from Alaska to the tip of South America also appears to be
highly suspect. If the "Clovis First" theory is to be believed, such a
lengthy migration took place in the course of a few hundred years!
(60)
Now add to this mix of evidence to support a pre-Clovis
settlement the possibility of other routes of arrival than just from Asia.
Presently, hypotheses exist suggesting an arrival via Europe to eastern
North America in boats sailed across the North Atlantic Ocean as well
as arrivals on the coast of South America by boats sailed across the
(65) Pacific from Polynesia.
Perhaps what is needed to resolve the present disagreement
over the origins of all Native Americans is a combination of two
disparate things: ancient archaeological evidence in the form of human
and animal bones yet to be discovered combined with modern scientific
(70) understanding of mitochondrial DNA and Y-chromosome family trees.
As more evidence is collected by finding additional pre-Clovis
sites, and this evidence is subjected to the most up-to-date biological
analysis, the existing enigma of who settled the Americas and, more
importantly, when they arrived, will be solved.

Copyright 2009, TCA, LLC.

192

Studyguide for the SAT Skill Quiz E: Critical Reading

Quiz Time!
A

Question 1.
The author of Passage 1 attributes proof for the "Clovis
First" theory primarily to the fact that
(A)
(B)
(C)
(D)
(E)

the land bridge across the Bering Strait didn't


exist after 11,000 B.C.
earlier archaeological evidence has been
misdated
settlements have been found in South America
analysis of early Native American bones reveals
an Asian connection
the fluted point people were game hunters

Question 2.
In lines 10-14, the author of Passage 1 primarily stresses
(A)
(B)
(C)
(D)
(E)

the dates when the ice-free corridor along the


Pacific coast was closed
the length of the Pleistocene epoch
the extent of glaciation on the North American
continent
the lack of evidence to support an arrival prior to
11,000 B.C.
the discovery of fluted point spear tips in Virginia

Question 3.
In line 7, the word "proponents" most nearly means
(A)
(B)
(C)
(D)
(E)

Copyright 2009, TCA, LLC.

arguments against
testimony for
advocates for
detractors of
supporters of

193

Studyguide for the SAT Skill Quiz E: Critical Reading

Question 4.
As used in line 32, the word "credence" most nearly means
(A)
(B)
(C)
(D)
(E)

doubt
credibility
support
proof
foundation

Question 5.
It can be inferred that the primary purpose of the dates cited
in Passage 1 was to
(A)
(B)
(C)
(D)
(E)

refute all pre-Clovis settlement theories


support evidence of earlier arrivals in the
Americas
prove the existence of Beringia
locate early settlements along the Pacific coast
support the hunter-gatherer theory

Question 6.
In lines 25-27, the sites at Meadowcroft Rock Shelter,
Saltville, and Cactus Hill are mentioned mainly as
(A)
(B)
(C)
(D)
(E)

Copyright 2009, TCA, LLC.

evidence of settlements that pre-date the Clovis


people
major support to disprove the "Clovis First"
theory
examples of radiocarbon dating accuracy
examples of radiocarbon dating inaccuracy
evidence of the use of fire by early people

194

Studyguide for the SAT Skill Quiz E: Critical Reading

Question 7.
In Passage 2, the author suggests that supporters of the
"Clovis First" theory are
(A)
(B)
(C)
(D)
(E)

leaders in the discovery of Paleo-Indian artifacts


misguided in their disregard for other theories
unable to find any earlier evidence of settlements
unwilling to support their theory with evidence
skeptical of DNA research

Question 8.
In line 35, the word "unflagging" is used to mean
(A)
(B)
(C)
(D)
(E)

tireless
loyal
stolid
stoic
unwavering

Question 9.
The author of Passage 2 implies that the "Clovis First"
theory does not recognize
(A)
(B)
(C)
(D)
(E)

Copyright 2009, TCA, LLC.

the authenticity of artifacts found at Meadowcroft


Rock Shelter
the importance of radiocarbon dating
the possibility of routes other than from Asia
the evidence of settlements in South America
after 8,500 B.C.
the Y-chromosome research for family trees

195

Studyguide for the SAT Skill Quiz E: Critical Reading

Question 10.
Which of the following would the author of Passage 2 most
likely say regarding the belief that no other people were
found in the Americas when the first Asians crossed the
land bridge to Alaska, as suggested in Passage 1?
(A) The Clovis people play only a minor role in
the settlement of the Americas.
(B) There is no evidence to support a land bridge from Asia to Alaska.
(C) No early evidence of boats has been found to
support an Atlantic crossing theory.
(D) Samples collected from Mesa Verde have been misdated.
(E) Mounting evidence strongly suggests arrivals in
the Americas much earlier than 11,500 B.C.

Question 11.
Which of the following statements is most clearly supported
by both passages?
(A)
(B)
(C)
(D)
(E)

We must remain open to many other theories of


arrival.
We must retest all available evidence for accurate
dates.
We should ignore theories involving Africa.
We will discover traces of Paleo-Indians before
30,000 B.C.
We must find more evidence at Meadowcroft
Rock Shelter.

Question 12.
Both supporters of the "Clovis First" theory and supporters
of a pre-Clovis culture would agree that a final resolution of
the quest for the first Native Americans will be determined
by
(A)
(B)
(C)
(D)
(E)

Copyright 2009, TCA, LLC.

locating other types of spear points


using a dating system other than radiocarbon dating
finding new sites
using every modern scientific analysis method
possible
DNA analysis only

196

Studyguide for the SAT Skill Quiz E: Critical Reading

Hints!
A

Hint for Question 1:


Review the passage for the main point of disagreement with pre-Clovis
theories.
Hint for Question 2:
Reread paragraph 2.
Hint for Question 3:
Which choice is the best synonym?
Hint for Question 4:
Which choice is the best synonym?
Hint for Question 5:
Consider the main point of
disagreement with pre-Clovis theories.
Hint for Question 6:
Reread the last paragraph.
Hint for Question 7:
Consider the tone used in paragraph 1.
Hint for Question 8:
Consider the most accurate synonym.
Hint for Question 9:
Which choice is the only one
"Clovis First" theorists refute?
Hint for Question 10:
Which choice is the main
idea of Passage 2?

Copyright 2009, TCA, LLC.

197

Studyguide for the SAT Skill Quiz E: Critical Reading


Hint for Question 11:
Which choice is the central point of
disagreement between the two passages?
Hint for Question 12:
Which choice is fundamental to all scientific investigation?

Copyright 2009, TCA, LLC.

198

Studyguide for the SAT Skill Quiz E: Critical Reading

Answers!
A

Question 1.
The answer is (B).
Explanation for Question 1:
The answer is (B).
The author of Passage 1 repeatedly uses the fact that earlier
archaeological evidence to support a pre-Clovis settlement is based on
dating that is not accurate.

Question 2.
The answer is (A).
Explanation for Question 2:
The answer is (A).
By rereading paragraph 2, it is clear that the use of dates is
significant evidence to stress that earlier migrations would have been impossible
before 30,000 B.C. because of the total extent of glaciation on the
North American continent. No Pacific coast corridor would have been
open.

Question 3.
The answer is (C).
Explanation for Question 3:
The answer is (C).
It is always important to select the most accurate synonym for a
word. Subtle shading of meaning will help to determine the correct
choice. Although choice (E) is close, the most accurate choice for the
meaning of "proponents" is (C).

Copyright 2009, TCA, LLC.

199

Studyguide for the SAT Skill Quiz E: Critical Reading


Question 4.
The answer is (B).
Explanation for Question 4:
The answer is (B).
An analysis of the word "credence" reveals that it comes from the
Latin root credere , meaning to trust or to believe;
thus credibility is the most accurate synonym.

Question 5.
The answer is (A).
Explanation for Question 5:
The answer is (A).
Although many points of disagreement concerning when and where the first
arrivals were made are expressed, the word "accurate," as opposed to
"questionable," dates is primarily used to imply that pre-Clovis
theories are highly questionable.

Question 6.
The answer is (D).
Explanation for Question 6:
The answer is (D).
The last paragraph of Passage 1 includes dates that are earlier
than the "Clovis First" theory recognizes. These dates are said to be
inaccurate and in need of further testing.

Question 7.
The answer is (B).
Explanation for Question 7:
The answer is (B).
The only correct choice according to the author of Passage 2 would
have to be (B). The author would agree that proponents of the
"Clovis First" theory qualify as leaders in Paleo-Indian research,
and the passage does not support any of the other inferences.

Copyright 2009, TCA, LLC.

200

Studyguide for the SAT Skill Quiz E: Critical Reading


Question 8.
The answer is (E).
Explanation for Question 8:
The answer is (E).
The choice of the best synonym is always difficult. The
connotative meaning of a word is often more powerful than its
simple denotative meaning. Although choice A appears to be
correct, the most accurate choice based on the connotation of the
word "unflagging" would have to be (E).

Question 9.
The answer is (C).
Explanation for Question 9:
The answer is (C).
Proponents of the "Clovis First" theory do recognize all of the
choices except (C). They do not question the authenticity of artifacts,
only their dates.

Question 10.
The answer is (E).
Explanation for Question 10:

The answer is (E).


Because the author of Passage 2 believes that "mounting
archaeological evidence as well as other possible suggestions for
routes of arrival on the North and South American continents"
continue to be found, the correct choice must be (E). This is the
main idea of Passage 2 and all details are used to support it.

Copyright 2009, TCA, LLC.

201

Studyguide for the SAT Skill Quiz E: Critical Reading


Question 11.
The answer is (B).
Explanation for Question 11:

The answer is (B).


Because the major point of disagreement between the two passages
involves the accuracy of pre-Clovis dates, it can be assumed that both
authors would agree that further, more accurate testing must be
conducted to authenticate the dates.

Question 12.
The answer is (D).
Explanation for Question 12:
The answer is (D).
Fundamental to all scientific investigation is the use of the
scientific method, founded on the principle that testing and retesting
using all currently available scientific methods is the best way to certify
the validity of an original hypothesis. Only choice (D) recognizes the
importance of all methods of analysis.

Copyright 2009, TCA, LLC.

202

Studyguide for the SAT Skill Quiz F: Critical Reading


This passage is taken from an article written to commemorate
the 80th anniversary of the Nobel Prize in Physics.
Planck, like several other scientists, was interested in the
subject of black body radiation, which is the name given to the
electromagnetic radiation emitted by a perfectly black object
when it is heated. (A perfectly black object is defined as one
(5) that does not reflect any light, but completely absorbs all light
falling on it.) Experimental physicists already had made careful
measurements of the radiation emitted by such objects, even
before Planck started working on the problem. Planck's first
achievement was his discovery of the fairly complicated
(10) algebraic formula that correctly describes the black body
radiation. This formula, which is frequently used in theoretical
physics today, neatly summarized the experimental data. But
there was a problem: the accepted laws of physics predicted a
quite different formula.
(15)
Planck pondered deeply on this problem and finally came
up with a radically new theory: radiant energy is only emitted in
exact multiples of an elementary unit that Planck called the
quantum. According to Planck's theory, the magnitude of a
quantum of light depends on the frequency of the light (i.e., on
(20) its color), and is also proportional to a physical quantity that
Planck abbreviated, "h," but that is now called Planck's constant.
Planck's hypothesis was quite contrary to the then-prevalent
concepts of physics; however, by using it he was able to find an
exact theoretical derivation of the correct formula for black
(25) body radiation.
Planck's hypothesis was so revolutionary that it doubtless
would have been dismissed as a crackpot idea had not Planck
been well-known as a solid, conservative physicist. Although
the hypothesis sounded very strange, in this particular case it
(30) did lead to the correct formula.
At first, most physicists (including Planck himself)
regarded his hypothesis as no more than a convenient
mathematical fiction. After a few years, though, it turned out
that Planck's concept of the quantum could be applied to
(35) various physical phenomena other than black body radiation.
Einstein used the concept in 1905 to explain the photoelectric
effect, and Niels Bohr used it in 1913 in his theory of atomic
structure. By 1918, when Planck was awarded the Nobel Prize,
it was clear that his hypothesis was basically correct, and that it
(40) was of fundamental importance in physical theory.

Copyright 2009, TCA, LLC.

203

Copyright 2009, TCA, LLC.

204

Studyguide for the SAT Skill Quiz F: Critical Reading

Quiz Time!
A

Question 1.
Planck's algebraic formula was
(A)
(B)
(C)
(D)
(E)

similar to accepted laws of physics


a provider of erroneous calculations on
black body radiation
relatively uncomplicated, yet difficult
to communicate
contrary to accepted laws of physics
a demonstration of physics and conjecture
working in tandem

Question 2.
The best title for the passage is
(A)
(B)
(C)
(D)
(E)

Hypothesis: The Road to Fact


Planck's Contributions to Theoretical Physics
Planck's Constant
Black Body Radiation
A Conservative Physicist Triumphs

Question 3.
You can infer from the passage that prior to Planck's hypothesis,
the prevalent concepts in physics asserted
(A)
(B)
(C)
(D)
(E)

Copyright 2009, TCA, LLC.

essentially the same concepts as Planck's, but


without his theoretical derivation
that arithmetic, rather than complicated algebraic
formulas, can be used to correctly describe black body radiation
that the then-accepted laws of physics were in
complete accord with Planck's formula for black body radiation
that radiant energy is not emitted in exact
multiples of any elementary unit
all of the above can be inferred from this passage

205

Studyguide for the SAT Skill Quiz F: Critical Reading

Question 4.
Planck's reputation
(A)
(B)
(C)
(D)
(E)

hindered the acceptance of his hypothesis


was that of an ultra conservative inflexible scientist
benefitted the acceptance of an otherwise
unconventional idea
suffered due to the complexity and fallibility
of his hypothesis
was deemed to be mired in questionable
practices early in his career

Question 5.
In line 22, the word "prevalent" most nearly means
(A)
(B)
(C)
(D)
(E)

ideal
accepted
steady
controlling
convincing

Question 6.
In line 35, the word "phenomena" could best be replaced with
(A)
(B)
(C)
(D)
(E)

Copyright 2009, TCA, LLC.

paragons
events
qualities
peculiarities
mysteries

206

Studyguide for the SAT Skill Quiz F: Critical Reading

Answers!
A

Question 1.
The answer is (D).
Question 2.
The answer is (B).
Question 3.
The answer is (D).
Question 4.
The answer is (C).
Question 5.
The answer is (B).
Question 6.
The answer is (B).

Copyright 2009, TCA, LLC.

207

Copyright 2009, TCA, LLC.

208

Studyguide for the SAT Skill Quiz G: Critical Reading


These passages were written for art museum newsletters.
The authors were commenting on touring exhibitions
featured at the museums.
Passage 1
The Impressionists were the first generation of painters to
treat the eye as an instrument--as an apparatus--and to take its
strengths and weaknesses consciously into account. There were
two chief results. In the first place, they broke down mixed
(5) colors into their primary tones and communicated them in this
way to the eye. Thus the eye itself does the mixing, and hence
the quite unique freshness of the Impressionists' color effects.
(It should be remembered, though, that individual artists
handled this technique of color separation in a variety of ways
(10) and degrees--no real principle was established until Georges
Seurat [1859-91] introduced his Pointillism.) Secondly, in
scientific spirit, they considered the eye as an impartial
instrument, receptive to colors, lines, and dots, but in no way
judging what it sees, either morally or by any other criteria. The
(15) Impressionists' aim is to be only an eye, and for them the
highest praise was: "Seulement un oeil, mais quel oeil!" ("Only
an eye, but what an eye!")
In line with these two tenets, the Impressionist artist
looked for particular subject matter: it must have potential for
(20) exploitation of color and light; and it must be as far as possible
neutral and impartial, having nothing to do with the old scale of
values, either religious or social. For Claude Monet, for
example, an appropriate subject was the Gare St-Lazare, a new
building at the time and therefore free of traditional
(25) preconceptions, seen as a secular "study of morning." The artist
was obviously drawn to the power and splendor of the new
form of transport, but he was elated even more by his
experience of color, particularly by his exploration of color
where previously none would have been expected or noticed.
(30) The cloud billowing forth from the locomotive, bluish in the
shadow of the station, white in the open light, is the main
theme. The purely visual is the basis of this art, possibly
precisely because no meaning is implied.

Copyright 2009, TCA, LLC.

209

Studyguide for the SAT Skill Quiz G: Critical Reading


Passage 2
Auguste Rodin has been called the Moses of sculpture. He
(35) led it out of the nineteenth-century wilderness of pretty marble
nymphs, solemn allegorical figures and pompous national heroes
into a promised land where ordinary human beings could be
made heroic without being idealized by the sculptor's hands.
Their heroism lies in the unconquerable human spirit that
(40) dignifies the mortal flesh.
Modeling the human body, he sometimes exaggerated
things that were judged ugly by the conventions of his timewrinkled skin, heavy features, contorted muscles. But it is the
spirit, the irresistible life-force, that emerges from his vibrant,
(45) often anguished figures.
Rodin's technical approach to form derived from the
Renaissance, and, in some instances, directly from
Michelangelo. But the way he handled his surfaces was new.
His aim was not only to convey actual physical movement, but
(50) also, by leaving his bronze surface rough so that it caught the
play of light and shade, to heighten the illusion of restless
vitality. Marble, on the other hand, he hated to scar, so he made
the uninterrupted flow of the material suggest its own
movement.
(55)
Living flesh was so important to Rodin as a symbol of life
that he hated to clothe it. Besides, he feared that changing
fashions might give his work a dated look. In his statue of the
great Balzac he found a solution by clothing the writer in the
dateless Dominican robe he wore when he worked.
(60)
But Rodin didn't even need a whole body to project his
enormous concern with humanity. He could model just a hand,
with tense fingers reaching out into space, and make it a symbol
of aspiration, tenderness, or wrath.

Copyright 2009, TCA, LLC.

210

Studyguide for the SAT Skill Quiz G: Critical Reading

Quiz Time!
A

Question 1.
Which of the following would be the best title for Passage 1?
(A)
(B)
(C)
(D)
(E)

The Eye as an Instrument


The Foundation of Impressionist Art
Two Chief Tenets of Impressionist Art
Georges SeuratFather of Impressionist Art
The Scientific Basis for Impressionist Art

Question 2.
According to Passage 1, the "two tenets" of Impressionist art are:
(A)
(B)
(C)
(D)
(E)

The eye should be relied on to mix colors;


the artist must not let any moral criteria influence the art.
Scenes to be painted must have dramatic colors
and lighting.
The eye is an instrument; the eye does not judge
what it sees.
The artist is only an eye; the artist must be
receptive to colors, lines, and dots.
The eye is an instrument; its strengths and
weaknesses should be compensated for.

Question 3.
It can be inferred from Passage 1 that the Gare St-Lazare is
(A)
(B)
(C)
(D)
(E)

Copyright 2009, TCA, LLC.

a church
a new locomotive train
a train station
an office building
an art museum dedicated to nontraditional art

211

Studyguide for the SAT Skill Quiz G: Critical Reading

Question 4.
The author of Passage 1 would most likely agree with
which of the following statements?
(A)
(B)
(C)
(D)
(E)

Claude Monet was more concerned with


power and splendor than color.
Implied meaning is the basis of good
Impressionist art.
Religion was frequently the subject of
Impressionist art.
Natural scenes with subtle color differences would
be a suitable subject for an Impressionist artist.
The freshness of color in Impressionist art
stems from the artists' careful mixing of colors.

Question 5.
In line 25, the word "secular" most nearly means
(A)
(B)
(C)
(D)
(E)

traditional
spiritual
colorful
worldly
artistic

Question 6.
The tone of Passage 2 is best described as
(A)
(B)
(C)
(D)
(E)

Copyright 2009, TCA, LLC.

heroic
complimentary
disparaging
dated
allegorical

212

Studyguide for the SAT Skill Quiz G: Critical Reading

Question 7.
It can be inferred from Passage 2 that if you were to compare
two statues created by Rodin, one bronze and one marble,
an obvious difference would be that
(A)
(B)
(C)
(D)
(E)

the marble statue would have a


rougher texture than the bronze statue
the bronze statue would be much more
colorful than the marble statue
the marble statue would suggest more
vitality than the bronze statue
the marble statue would show more
contortion than the bronze statue
the bronze statue would have a rougher
surface than the marble statue

Question 8.
The author of Passage 2 refers to Moses in line 1 primarily to
(A)
(B)
(C)
(D)
(E)

show how heroic Rodin was


compare Rodin and Moses as men
indicate that Rodin led an important new movement
compare the importance of religion and sculpture
indicate that Rodin sculpted nymphs

Question 9.
You can infer from Passage 2 that Rodin's statues often
(A)
(B)
(C)
(D)
(E)

Copyright 2009, TCA, LLC.

were pretty nymphs


depicted the fashions of the day accurately
depicted idealized, relaxed hands
depicted little, if any, clothing
depicted monks and religious leaders

213

Studyguide for the SAT Skill Quiz G: Critical Reading

Question 10.
In lines 41-45, the author of Passage 2 suggests that Rodin
exaggerated wrinkled skin and contorted muscles primarily to
(A)
(B)
(C)
(D)
(E)

support the assertion that ordinary people's heroism


lies in the unconquerable human spirit
show the age of his subjects
indicate that heroes are usually beautiful
imply that Rodin himself was ugly
indicate the wrinkles in the human spirit

Question 11.
A common theme shared by both passages could be expressed by the following:
(A)
(B)
(C)
(D)
(E)

Great artists attempt to render their subject as


accurately as possible.
Great artists generally prefer subjects of
great importance, such as religion, kings, or generals.
New techniques are established by great artists.
Great artists are not always popular during their lifetime.
Most great artists emphasize color.

Question 12.
The author of Passage 1 would most likely characterize Rodin as he is
described in Passage 2 as
(A)
(B)
(C)
(D)
(E)

Copyright 2009, TCA, LLC.

a traditional artist unworthy of study


a pompous, egotistical man
an artist who was ahead of his time
an artist who was a kindred spirit to the
Impressionists described in Passage 1
an artist who did not sufficiently explore the
use of color

214

Studyguide for the SAT Skill Quiz G: Critical Reading

Answers!
A

Question 1.
The answer is (B).
Question 2.
The answer is (C).
Question 3.
The answer is (C).
Question 4.
The answer is (D).
Question 5.
The answer is (D).
Question 6.
The answer is (B).
Question 7.
The answer is (E).
Question 8.
The answer is (C).
Question 9.
The answer is (D).
Question 10.
The answer is (A).
Question 11.
The answer is (C).
Question 12.
The answer is (D).

Copyright 2009, TCA, LLC.

215

Copyright 2009, TCA, LLC.

216

Studyguide for the SAT


These passages were taken from an anthology of commentaries on
Plato's philosophy.
Passage 1
As far as getting things done is concerned, experience
does not seem to differ at all from art; in fact, we observe that
those who have experience meet with more success than those
who have grasped the principles of the subject without having
(5) any experience. The reason for this is that experience is
knowledge of individuals, whereas art is knowledge of
universals, and all activities and processes have to do with
individuals.
The doctor does not treat "man" except accidentally; he
(10) treats Callias or Socrates, or someone else described in this
way, who is accidentally "man." So, if someone has grasped
the principles of the subject without having any experience, and
thus knows the universal without knowing the individuals
contained in it, he will often fail in his treatment; for it is the
(15) individual that has standing as belonging more to art than to
experience; and we regard those who possess an art as wiser
than those who just have experience, on the grounds that in
every case wisdom follows on knowledge. We have this
attitude because those who possess art know causes, whereas
(20) the others do not. Men who have experience know that a thing
is so, but not why it is so; those who know why a thing is so
also know its cause.
This is why we regard the master craftsmen in any field
as more deserving of respect, more knowledgeable, and wiser
(25) than manual workers: because they know the causes of the
things being done; the manual workers are like certain
inanimate objects in that they do things without knowing what
they are doing--fire, for instance, burns in this way; however,
whereas inanimate objects do all of these things somehow by
(30) nature, manual workers do them by habit. Thus it is not on the
grounds of their greater success in doing things that we judge
some people to be wiser than others, but because of their grasp
of principles and knowledge of causes.
In general, too, what distinguishes the man who has
(35) knowledge from the man who does not is the ability to teach,
and this is why we regard art as being more truly knowledge
than experience: those who possess art teach, those who do not
cannot.

Copyright 2009, TCA, LLC.

217

Studyguide for the SAT Skill Quiz H: Critical Reading


Passage 2
The best form of government, Plato suggests, is an
(40) aristocracy. By this he means not an hereditary aristocracy,
or a monarchy, but an aristocracy of merit--that is, rule by the best
and wisest persons in the state. These persons should be chosen
not by a vote of the citizens, but by a process of co-optation.
The persons who are already members of the ruling, or guardian
(45) class should admit additional persons to their ranks purely on
the basis of merit.
Plato believed that all persons, both male and female,
should be given the chance to demonstrate their fitness to be
members of the guardian class. (Plato was the first major
(50) philosopher, and for a long time virtually the only one, to
suggest the basic equality of the sexes.) To ensure equality of
opportunity, Plato advocated the rearing and education of all
children by the state. Children should first receive a thorough
physical training; but music, mathematics, and other academic
(55) disciplines should not be neglected. At several stages,
extensive examinations should be given. The less successful
persons should be assigned to engage in the economic activity
of the community, while the more successful persons should
continue to receive further training. This additional education
(60) should include not only the normal academic subjects, but also
the study of "philosophy," by which Plato means the study of
his metaphysical doctrine of ideal forms.
At age thirty-five, those persons who have convincingly
demonstrated their mastery of theoretical principles are to
(65) receive an additional fifteen years of training, which should
consist of practical working experience. Only those persons
who show that they can apply their book learning to the real
world should be admitted into the guardian class. Moreover,
only those persons who clearly demonstrate that they are
(70) primarily interested in the public welfare are to become
guardians.
Membership in the guardian class would not appeal to all
persons. The guardians are not to be wealthy. They should be
permitted only a minimal amount of personal property, and no
(75) land or private homes. They are to receive a fixed (and not very
large) salary, and may not own either gold or silver. Members
of the guardian class should not be permitted to have separate
families, but are to eat together, and are to have mates in
common. The compensation of these philosopher-kings should
(80) not be material wealth, but rather the satisfaction of public
service. Such, in brief, was Plato's view of the ideal republic.

Copyright 2009, TCA, LLC.

218

Studyguide for the SAT Skill Quiz H: Critical Reading

Quiz Time!
A

Question 1.
Passage 1 indicates that
(A)
(B)
(C)
(D)
(E)

a person who possesses an art is no wiser


than a person who has experience
experience is not knowledge of individuals, but
knowledge of processes
inanimate objects possess knowledge
a person who knows a subject well does not need
the experience of working, literally, in that subject area
a person who knows a subject well still
needs the practical experience of that subject to grasp it fully

Question 2.
It can be inferred from Passage 1 that
(A)
(B)
(C)
(D)
(E)

people who experience an event, feeling, or incident


know that it existed as well as why it happened
people who experience an event, feeling, or incident
know that it existed, but not necessarily why it happened
people, in general, have little aptitude for
knowledge of events
people who are considered wise also are
considered to be artists
people who are teachers are not accomplished
in an art

Question 3.
The author of Passage 1 maintains that
(A)
(B)
(C)
(D)
(E)

Copyright 2009, TCA, LLC.

From knowledge comes the benefit of wisdom


Based on some people's greater success in
accomplishing goals, we can judge them to be wise
The ability to teach is inherent
Possessing an art is preferable to all other qualities
From wisdom comes the benefit of knowledge

219

Studyguide for the SAT Skill Quiz H: Critical Reading

Question 4.
In line 9 of Passage 1, the author uses the word "accidentally"
(A)
(B)
(C)
(D)
(E)

to refer to an unfortunate experience that leads


a patient to seek medical care
to mean "in principle"
to refer to the incidental fact that a specific patient
also happens to be part of mankind
to convey the sense that treating "man" is in
the doctor's nature
to mean "out of kindness"

Question 5.
The author of Passage 2 implies that
(A)
(B)
(C)
(D)
(E)

theoretical principles were secondary to


practical working experience
metaphysics and theology are
inextricably combined
simplification of the training process occurs
by educating youth
men are, potentially, the backbone of
civil service
the training of children as future
guardian class members was critical

Question 6.
In Passage 2, the author distinguishes Plato as
(A)
(B)
(C)
(D)
(E)

Copyright 2009, TCA, LLC.

a sexist
an opponent of guardian class equality
a proponent of socialization
the first philosopher-advocate of equality for
males and females
the father of material compensation

220

Studyguide for the SAT Skill Quiz H: Critical Reading

Question 7.
According to the author of Passage 2, Plato maintained that
(A)
(B)
(C)
(D)
(E)

earning a position through hard work and


inherent intelligence was just
a vote of the citizens would be the most fair
determinant of who rose to leadership
co-optation was a means to limit membership
in the guardian class
book learning paled in comparison with
real life experience
guardians needed a measure of wealth to
command the respect of the common man

Question 8.
In line 57, the word "engage" most nearly means
(A)
(B)
(C)
(D)
(E)

hire
marry
entangle
secure
work

Question 9.
Based on Passages 1 and 2, the author of Passage 1 would most likely
charaterize one of Plato's Guardians
(A)
(B)
(C)
(D)
(E)

Copyright 2009, TCA, LLC.

as a manual worker for the community


as someone lacking practical experience and,
therefore, lacking in true knowledge
as a person who performs his or her function
out of habit
as someone wise enough to teach
as someone who is usually unable to get
important things done

221

Studyguide for the SAT Skill Quiz H: Critical Reading

Question 10.
The views expressed in Passage 1 and the views of Plato
expressed in Passage 2 are most in agreement about
(A)
(B)
(C)
(D)
(E)

Copyright 2009, TCA, LLC.

the importance of experience in achieving success


the equality of men and women
society's need for artists
the wisdom of people who teach
the importance of the individual

222

Studyguide for the SAT Skill Quiz H: Critical Reading

Answers!
A

Question 1.
The answer is (E).
Question 2.
The answer is (B).
Question 3.
The answer is (A).
Question 4.
The answer is (C).
Question 5.
The answer is (E).
Question 6.
The answer is (D).
Question 7.
The answer is (A).
Question 8.
The answer is (E).
Question 9.
The answer is (D).
Question 10.
The answer is (A).

Copyright 2009, TCA, LLC.

223

Copyright 2009, TCA, LLC.

224

Studyguide for the SAT Skill Quiz I: Critical Reading


This passage is taken from an article on vitamins
written by a nutritionist.
Vitamins are certain substances found in food in minute
amounts. They are needed for the regulation of chemical
processes inside the body, and through this have an important
role in growth and development and in protection against illness
(5) and disease. The presence of vitamins in the diet is essential;
most of them cannot be made by the body.
The role of vitamins in nutrition was discovered only in
the present century, but there are now known to be about 40, of
which 12 or more are essential in the diet. Because of the
(10) haphazard process of their discovery, they originally formed a
jumbled list of alphabetic names (A, B1, B6, etc.). But now that
their chemical structures have been identified, chemical names
are often used for many of them. Identification has also meant
that they can now be made artificially.
(15)
Chemically, in fact, they are proving to be an equally
mixed bag--sharing only the characteristic of being complex
substances needed by the body in tiny amounts. For example, a
man needs only an ounce of thiamin in a lifetime--despite the
vital importance of that ounce. Above an average day-to-day
(20) requirement, increased amounts of a vitamin do no further
good, and in some cases are actually harmful.
Vitamins in the diet can be divided into two classes:
those soluble in fat (vitamins A, D, E, and K), and those soluble
in water (vitamins C and the B vitamin complex).
(25)
VITAMIN A is found in halibut and cod liver oil, milk,
butter, and eggs. It is destroyed by cooking and sunlight. It
plays a role in the formation of bone and of enamel and dentine
in teeth. It is also responsible for the ability to see in dim light.
VITAMIN D is found in eggs, milk, butter, and fish liver
(30) oils. It is also synthesized in the skin during exposure to
sunlight. It plays a part in the digestive absorption of some
minerals, such as calcium, and phosphorus. It is also necessary
for retaining calcium in bones.
VITAMIN E is found in wheat germ, oil, lettuce,
(35) spinach, watercress, etc. There is no definite evidence that it is
essential to humans, but it does help in the healing of skin
wounds, and may also be connected with fertility.
VITAMIN K is found mainly in green plants such as
spinach, cabbage, and kale. But it is also synthesized in the gut
(40) by the action of bacteria. It is a necessary factor in the bloodclotting mechanism, as it is needed for the production of
prothrombin.

Copyright 2009, TCA, LLC.

225

Studyguide for the SAT Skill Quiz I: Critical Reading


VITAMIN C is found in fresh fruit and vegetables,
especially lemons, oranges, black currants, tomatoes, and
(45) watercress. Human milk also contains vitamin C. This vitamin
is easily destroyed by cooking, especially if the food has been
chopped up. One of its most important functions in the body is
to control the formation of dentine, cartilage, and bone. It also
helps the formation of red blood cells, and the correct healing of
(50) wounds and broken bones. There is no conclusive evidence that
vitamin C prevents colds.
VITAMIN B is in fact a complex of fifteen different
substances, but they are classed together because they occur
together in the same types of food, such as yeast and wheat
(55) germ. Unlike the other vitamins, at least some vitamins of the
B group are found in all living plants and animals.

Copyright 2009, TCA, LLC.

226

Studyguide for the SAT Skill Quiz I: Critical Reading

Quiz Time!
A

Question 1.
According to the passage, the discovery of vitamins
(A)
(B)
(C)
(D)
(E)

was accidental
was conclusive evidence that preventative
medicine was an up-and-coming science
was the scientific link to chemical alterations
of the body
was not the result of an organized quest
none of the above

Question 2.
The author implies that vitamins
(A)
(B)
(C)
(D)
(E)

play a major role in preventing the cold virus


from replicating
for the most part, cannot be formed by the body
but must be ingested
all are derivatives of the originally
discovered Vitamin A
cannot be created artificially
are all essential to every human body

Question 3.
The author's tone in the passage is
(A)
(B)
(C)
(D)
(E)

Copyright 2009, TCA, LLC.

subjective
instructional
intuitive
pedantic
precautionary

227

Studyguide for the SAT Skill Quiz I: Critical Reading

Question 4.
The author suggests all of the following EXCEPT
(A)
(B)
(C)
(D)
(E)

the more of a vitamin you consume, the better


vitamins can help protect against disease
some vitamins are made in the body
spinach contains vitamins E and K
cooking can destroy some vitamins

Question 5.
In line 30, the word "synthesized" most nearly means
(A)
(B)
(C)
(D)
(E)

organized
manufactured
destroyed
detected
organic

Question 6.
The best title for this passage is
(A)
(B)
(C)
(D)
(E)

The Chemical Makeup of Vitamins


All Foods Contain Vitamins
Vitamin C: Our Most Important Discovery
The Sources and Uses of Vitamins
How Much of Each Vitamin Do You Need

Question 7.
Even if you cannot define "prothrombin" (line 42), you can
infer from the passage that it
(A)
(B)
(C)
(D)
(E)

Copyright 2009, TCA, LLC.

is produced by certain bacteria


is found in blood
is found in green plants
is the chemical name for vitamin K
is found in fresh fruit

228

Studyguide for the SAT Skill Quiz I: Critical Reading

Answers!
A

Question 1.
The answer is (D).
Question 2.
The answer is (B).
Question 3.
The answer is (B).
Question 4.
The answer is (A).
Question 5.
The answer is (B).
Question 6.
The answer is (D).
Question 7.
The answer is (B).

Copyright 2009, TCA, LLC.

229

Copyright 2009, TCA, LLC.

230

Studyguide for the SAT Skill Quiz J: Critical Reading


Do-it-yourselfers can do a good turn for the environment
and their bank accounts by giving useful second lives to some
materials that normally go into trash cans. Below are two
examples of things that can be recycled for home maintenance
(5) or workshop uses.
Large plastic soft-drink bottles with reinforced bottoms
can be converted to first-rate containers for nails, screws, or
small parts. Use a sharp utility knife to cut off the reinforced
bottom and discard the top portion. The bowl-like bottoms have
(10) smooth, half-sphere interiors that make it easy to scoop out
even the tiniest objects.
Large plastic bags, including trash bags, make good
dropcloths to catch paint drips and spills. Slit open the sides of
bags to make large sheets. The thicker the plastic, the better,
(15) because it is less prone to blow around in a breeze when used
outdoors (one solution is to weight corners with stones or pieces
of wood).

Copyright 2009, TCA, LLC.

231

Copyright 2009, TCA, LLC.

232

Studyguide for the SAT Skill Quiz J: Critical Reading

Quiz Time!
A

Question 1.
Which of the following best expresses the main idea of this passage?
(A)
(B)
(C)
(D)
(E)

Plastic bottles and bags have several uses


People can save money by recycling plastic
The environment is helped by recycling plastic
People can save money, and help the
environment, by finding good uses for trash
Home projects for the do-it-yourselfer

Question 2.
You can infer from the passage that the author MOST wants people to
(A)
(B)
(C)
(D)
(E)

save time
save money
help the environment
build things
paint things

Question 3.
The most likely reason that the author mentions "bank accounts"
in line 2 is
(A)
(B)
(C)
(D)
(E)

Copyright 2009, TCA, LLC.

to alert the reader that the article contains some math


to give the reader a personal incentive to recycle
to promote "do-it-yourselfers"
to provoke environmentalists
to remind the reader that disposing of garbage is expensive

233

Studyguide for the SAT Skill Quiz J: Critical Reading

Question 4.
The word "prone" in line 15 could best be replaced by:
(A)
(B)
(C)
(D)
(E)

lying
likely
thin
inclined
horizontal

Question 5.
The tone of this passage is
(A)
(B)
(C)
(D)
(E)

Copyright 2009, TCA, LLC.

condescending
angry
pedantic
evangelical
informative

234

Studyguide for the SAT Skill Quiz J: Critical Reading


Passage for Questions 6-10
During the twenties, diners appeared in every crossroads
town, serving the motorist as the depot restaurant had served the
rail traveler of the last century. By the end of the decade, they
were firmly established as good, inexpensive places to eat, a
(5) reputation that carried on into the thirties, when manufacturers
were touting the running of a diner as a "depression-proof
business" that could bring a successful operator upwards of
$12,000 a year.
The look of the diner changed during the late twenties
(10) and thirties. Transoms borrowed from a railroad-car design
replaced Tierney's barrel roof, and stainless steel took the place
of wood. When the railroads introduced streamlining, the
diners followed suit. This sort of ingenuous imitation gave rise
to the persistent legend that diners were reconditioned railroad
(15) cars.
The diner went through its final transformation in the
years following World War II. At first, it simply grew larger;
but eventually the old form was abandoned in favor of Moorish
and Mediterranean buildings, made of concrete and surrounded
(20) by huge parking lots. This expensive refurbishing meant an
increase in the price of the meal. The resulting vacuum was
filled by the fast-food franchise chains, which gave those who
were comforted by such things as the illusion of having the
same cheap food in the same room whether they were eating in
(25) Fresno or Bangor. Today the franchisers have swept the field;
the diner is being upgraded out of existence.
Of course, the diner is one of our humbler traditions. But
when the last one gives way to a Mediterranean fantasy with
gold-veined mirrors and two-dollar cheeseburgers, something
(30) singularly American will have vanished.

Copyright 2009, TCA, LLC.

235

Studyguide for the SAT Skill Quiz J: Critical Reading

Question 6.
The function of the land-based diner was derived from
(A)
(B)
(C)
(D)
(E)

Moorish traditionalists
Mediterranean fundamentalists
unused land at crossroads in small towns
railway depot restaurants
big-city eateries

Question 7.
The purpose of this passage is
(A)
(B)
(C)
(D)
(E)

to enlighten the reader about objectionable


architectural practices as applied to diners
to lament the metamorphosis of the diner from folksy
affordable tradition to expensively marketed commercialism
to educate people about the benefits of diner cuisine
to explain the trend of refurbishing
architecturally archaic edifices
to describe the architectural metamorphosis
of the diner

Question 8.
You can infer from the passage that the author considers diners
(A)
(B)
(C)
(D)
(E)

an important American architectural style


better than the depot restaurants they replaced
an important part of American history
as extinct as dinosaurs
a small, but unique American tradition

Question 9.
In line 21, "resulting vacuum" refers to
(A)
(B)
(C)
(D)
(E)

Copyright 2009, TCA, LLC.

the machines used to clean fast-food restaurants


the need for cheap, good food
the abandonment of old forms of Moorish architecture
the expensive decor in the new restaurants
the huge, empty parking lots

236

Studyguide for the SAT Skill Quiz J: Critical Reading

Question 10.
In line 6, "touting" most nearly means
(A)
(B)
(C)
(D)
(E)

Copyright 2009, TCA, LLC.

soliciting
informing
betting
advertising
voting

237

Copyright 2009, TCA, LLC.

238

Studyguide for the SAT Skill Quiz J: Critical Reading

Answers!
A

Question 1.
The answer is (D).
Explanation for Question 1:

D is the best answer.


Answer A is close, but the first paragraph indicates a broader focus, with
plastic bottles and bags merely being two specific examples.
Question 2.
The answer is (C).
Question 3.
The answer is (B).
Explanation for Question 3:

The best answer is B. Read the hint if you don't see why.

Copyright 2009, TCA, LLC.

239

Studyguide for the SAT Skill Quiz J: Critical Reading


Question 4.
The answer is (B).
Explanation for Question 4:

Answer AAlthough "prone" can mean "lying" (as in the phrase


"they found the body prone on the floor") this sentence does not use
prone to mean lying. It uses another meaning of prone...
Answer B"Likely" fits in the sentence, but keep trying each choice to
find the best choice.
Answer CNo. Just because the sentence says "The thicker the
plastic...because it is less prone " does not imply that prone means
the opposite of thick.
Answer D"Inclined" as in "inclined to do something" is a possible
answer, but keep checking the choices.

Answer ENo, "horizontal" is a synonym for a different meaning of


prone, just as answer choice A is. You can eliminate E.
Okay, how do you choose between B and D?
The difference is subtle, but important. Let's look again at answer D,
"inclined." In common English usage, only people can be inclined to do
something because "being inclined to do something" implies a conscious
choice. Now look again at answer B, "likely." An object (such as the
plastic bag) or a person can be "likely" to have something happen to it...
in the wind, a plastic bag is likely to blow around. So answer choice B is
the best.

Copyright 2009, TCA, LLC.

240

Studyguide for the SAT Skill Quiz J: Critical Reading


Question 5.
The answer is (E).
Explanation for Question 5:

Answer ANo, the author does not "talk down" to the reader.
Answer BNo, there is no apparent emotion expressed in the passage.
Answer CNo, the passage offer suggestions, not detailed step-by-step
lessons, which is what pedantic suggests.
Answer DNot really. Although we can assume that the author feels
strongly that recycling is important, the tone of the passage is not
"crusading" or "devout," which are two other words for evangelical.
Answer EYes.

Question 6.
The answer is (D).
Question 7.
The answer is (E).
Question 8.
The answer is (E).
Question 9.
The answer is (B).
Question 10.
The answer is (D).

Copyright 2009, TCA, LLC.

241

Copyright 2009, TCA, LLC.

242

Critical Reading Lesson #3


Vocabulary

Triumph College Admissions


Skill Lesson

Quiz Time!
A

Hints!

Answers!
A

*Throughout this documentation, and the software: College Board and SAT are registered trademarks of the College Entrance Examination Board. PSAT/NMSQT
is a trademark of the College Entrance Examination Board and National Merit Scholarship Corporation. ACT is a trademark of ACT, Inc. None of these entities are
affiliated with the production of, nor endorse these materials.
Copyright 2009, TCA, LLC.

243

Copyright 2009, TCA, LLC.

244

Studyguide for the SAT Skill Review: Vocabulary

First the bad news: your vocabulary cannot be improved with a simple
one-night crash course. The good news: you can boost your vocabulary
in the next few weeks, if you follow our student-tested advice.
Every week, for the next three to five weeks (depending upon how much time you
have before your SAT) you should do the following:

Review your flashcards at least once a day. Do a few at a


time whenever you have a spare moment, such as on the bus, in line at
the cafeteria, when you get to class a minute early, etc.

After you have mastered the words on your flashcards start taking the
vocabulary skill quizzes, to test how well you know each word.
When you miss a word, put an asterisk next to the word on your flashcard so
you'll be sure to review that word a little harder.

Using Flashcards
Flashcards really do workin fact, nearly 90 percent
of students at top schools
like Stanford and MIT use flashcards to help them learn new material.

Another Tip:
Sometimes, look at the word and try to guess the definition;
sometimes, look at the definition and try to guess the word.
Doing it both ways helps the words sink in faster.

Copyright 2009, TCA, LLC.

245

Studyguide for the SAT Skill Review: Vocabulary


SAT Success "Complete" SAT Vocabulary Lists
Obviously, the only truly "complete" list of words that might appear on the
SAT is a very thick dictionary. And who has time to learn 2,000 new words?
However, we HAVE narrowed the list down to
* The words that have a 25%-50% chance of appearing
on any given SAT, plus
* Common prefixes and roots you should know.
We find that our students benefit dramatically from reviewing these word
lists. Don't try to memorize the entire listbut do scan the lists for words
you don't know and add at least 50 to 100 additional words to your
vocabulary.

The complete vocabulary list is divided into two parts. One part lists the
common prefixes, roots, and suffixes you should know. The other lists the
words most likely to appear on the SAT.
Even if you don't have time to learn all these words and word parts,
you can significantly improve your vocabulary by learning just 40 new
prefixes and roots and the most commonly tested words.

Copyright 2009, TCA, LLC.

246

Studyguide for the SAT Vocabulary: 192 Words You Need to Know_______
The Golden Dozen
1.

aesthetic appealing to the sense of beauty


Some cell phones are designed more for their aesthetic appeal than for their functionality.

2.

eclectic choosing from a variety of sources


A person with eclectic taste in music would like Joss Stone, Alicia Keys, Shania Twain,
and Mozart.

3.

enigma a puzzle; a riddle


Mona Lisa's enigmatic smile has puzzled art lovers for centuries.

4.

anomalous deviating from a general rule; out of place; incongruous


In the movie Legally Blonde, Elle originally stands out because she is an anomaly among
the students enrolled at Harvard Law School.

5.

caustic; vitriolic biting; stinging; cutting; scathing


Simon is famous for his caustic and vitriolic criticism of American Idol contestants.

6.

paradox a seemingly contradictory statement that nonetheless


expresses a truth
In their song "Tearin' Up My Heart," the boys from *NSYNC express a classic paradox:
Its tearin' up my heart when I'm with you. But when we are apart, I feel it too.

7.

reprimand; castigate to scold sharply


In the movie Billy Madison, Ms. Vaughn castigates Billy for making fun of a third- grade
student who was having trouble reading.

8.

alleviate; mitigate; mollify; assuage to relieve; to lessen


Do you ever take Aleve to alleviate a headache?

9.

tirade; diatribe sharp criticism; bitter speech


In the movie The Nutty Professor, Dean Richmond is livid and launches into a lengthy
diatribe at Professor Klump for alienating the colleges wealthy donors.

10.

insightful; discerning; perspicacious very perceptive; astute; penetrating


In Star Wars: The Phantom Menace, Yoda is perspicacious when he perceives that young
Anakin Skywalker has a dark side that could transform him into the villainous Darth
Vader.

11.

laud; extol; acclaim to praise


"Everwood" is lauded by many as one of the top television shows.

12.

brusque; curt blunt in manner or speech, often to the point of being rude
On the show The Apprentice, Donald Trump is curt and even brusque when he tells
each weeks losing apprentice, YOURE FIRED!

______________________________________________________________________________
Copyright 2009, TCA, LLC.

247

Studyguide for the SAT Vocabulary: 192 Words You Need to Know_______
You Meet the Most Interesting People on the SAT
13.

stalwart a loyal follower or adherent


A stalwart shows great loyalty. The Trojan armies were stalwart supporters of Prince
Hector in Troy. But many female viewers preferred to be stalwart supporters of the Greek
hero Achilles (Brad Pitt)!

14.

connoisseur an expert; authority


In the movie Legally Blonde, Elle is a connoisseur of high fashion.

15.

sage a person who is known for his/her wisdom; a wise person


In Star Wars Yoda is a perspicacious sage who is known for his wisdom.

16.

anarchist a person who opposes laws and government


An anarchist therefore opposes order. Sacco and Vanzetti were two very famous
anarchists who were executed because of their alleged terrorist acts.

17.

prodigy a person with great talent; a young genius


Prince, Doogie Howser MD and U.S. swimming sensation Michael Phelps are all
prodigies. Several of the fifth graders in the School of Rock were musical prodigies.

18.

itinerant; vagabond both are people who wander


Do you remember these lyrics from The Lion King?
And can you feel the love tonight?
It is where we are
Its enough for this wide-eyed wanderer
That we got this far
And can you feel the love tonight
How its laid to rest?
Its enough to make kings and vagabonds
Believe the very best

19.

automaton a person who is devoid of feeling or acts in a mechanical way


Many of the Nazis who committed genocide during the Holocaust behaved like unthinking
automatons.

20.

swindler a person who elicits trust and then betrays that trust; a con man or con woman;
someone who defrauds
In the movie Thelma and Louise, Brad Pitt plays the character of swindler J.D., who
befriends the women and then steals their money.

21.

mentor an advisor; teacher; guide


Obi-Wan Kenobi was Luke Skywalker's mentor as Luke became a Jedi knight. In Mean
Girls, Regina George was supposedly a mentor who showed Cady how to dress and act
like a Plastic.

______________________________________________________________________________
Copyright 2009, TCA, LLC.

248

Studyguide for the SAT Vocabulary: 192 Words You Need to Know_______
22.

virtuoso a person with great artistic talent


A virtuoso is very skilled. Everyone agrees that Michelangelo's Sistine Chapel ceiling is
the work of a virtuoso.

23.

devotee an ardent follower, supporter, or enthusiast


In School of Rock, Dewey Finn was an ardent, passionate devotee of rock music.

24

boor a person who is rude and insensitive


A boor lacks refinement and civility. In the movie Animal House Bluto was such a boor
that one person described him as a P-I-G pig!

25.

recluse a person who leads a secluded or solitary life; a hermit


Shrek was a recluse who preferred to live alone.

26.

pragmatist a person who is practical and thus not given to flights of fancy
A pragmatic senior should always have a safety school when applying to college.

27.

neophyte; novice; greenhorn a beginner


Cody Banks was a neophyte secret agent. Patch Adams was a neophyte intern when he
began using humor as a way to help his patients.

28.

skeptic a person who has doubts


A skeptic asks questions and lacks faith. In the movie Men in Black, Edwards was
originally a skeptic, who did not believe that aliens were living in New York City. In the
movie Bruce Almighty, Bruce was originally skeptical that the man he met really was
God.

29.

iconoclast literally someone who smashes icons or established images


An iconoclast opposes tradition. Eminem's fans argue that he is an iconoclast
who has made fun of pop icons such as Carson Daly, Britney Spears, and
Moby.

30.

charlatan a fake; fraud; cheat


In The Wizard of Oz, the Wizard turns out to be a charlatan, not a real wizard.

31.

pundit an authority who expresses his/her opinions on topics


Political pundits are already making their predictions about the next presidential
election.

32.

sycophant a person who tries to win favor by flattery


In "The Simpsons" Smithers is a sycophant who uses flattery to
gain influence with Mr. Burns.

______________________________________________________________________________
Copyright 2009, TCA, LLC.

249

Studyguide for the SAT Vocabulary: 192 Words You Need to Know_______
Lack of Words
33.

ambivalent lacking conviction (strong belief) because of mixed feelings


In the movie Gladiator, Emperor Commodus is ambivalent about giving the order to kill
Maximus in the Coliseum. He wants to eliminate a rival, but he also wants to please the
crowd.

34.

paltry of little value; trifling, and therefore lacking significance


The convicted criminals paltry apology did little to lessen his victims pain.

35.

ambiguous unclear; not certain


The meaning of Juliets line Wherefore art thou Romeo? is ambiguous to students who
do not know that wherefore means why. She is asking, Why are you Romeo?
(enemy of her family), not Where are you, Romeo?

36.

incoherent lacking organization


Remember Cher's incoherent speech at the beginning of the movie Clueless? She starts
out talking about Haitian immigrants and then abruptly changes the topic to her fathers
birthday party.

37.

disinterested neutral and thus lacking a bias


Although Judge Judy often becomes irritated with the guests on her show, she remains
disinterested when it comes to looking at the facts and handing down a judgment.

38.

merciless; ruthless cruel and therefore lacking in compassion


In the movie Shrek, Lord Farquaad is a merciless tyrant who threatens to eat the
Gingerbread Man's gumdrop buttons.

39.

callous lacking sensitivity, concern and solicitude


Dennis Hopper plays callous bomber Howard Payne in the movie Speed.

40.

lethargic; anemic; languid; slothful lacking in energy, vitality, and vigor


Video games such as Dance Dance Revolution are a good way to get lethargic individuals
off the couch and exercising while having fun at the same time.

41.

disdain lack of respect; contempt


Napoleon Dynamite feels disdain for his lazy Uncle Rico and threatens to call the police
to have him arrested. Uncle Rico tells him, Well then do it! Go on! Napoleon replies,
Maybe I will. GOSH!

42.

impulsive; impetuous; precipitous acting in a rash, hasty manner without fully


considering the consequences
In the movie Pirates of the Caribbean, Captain Jack Sparrow repeatedly urges Will to stop
being so impulsive and wait for the opportune moment. In the movie Finding Nemo, Nemo
impulsively swims out into the ocean toward the diving ship.

43.

intrepid; dauntless brave and therefore lacking fear


In Pirates of the Caribbean: the Curse of the Black Pearl, the British ship is named the
Dauntless. Spider-Man is intrepid in Spider-Man 2 when he places himself between an
out-of-control train and the end of the tracks.
______________________________________________________________________________
Copyright 2009, TCA, LLC.

250

Studyguide for the SAT Vocabulary: 192 Words You Need to Know_______
44.

precarious lacking stability


In the movie Dirty Dancing, Baby balances precariously on a log as she learns how to
dance.

45.

superficial shallow and therefore lacking in depth


The movie Shallow Hal could have been called Superficial Hal because at first he
judged people solely on physical appearance.

46.

diffident lacking self-confidence


Both Alex (Flashdance) and Sarah (Save the Last Dance) were originally diffident about
their dancing abilities.

47.

sophomoric; callow immature and therefore lacking maturity or sophistication


In the movie Billy Madison, Adam Sandler plays a character who is initially very
sophomoric and callow. And in Mall Rats, Holdens girlfriend calls him callow in a
letter breaking their relationship. But Holden thinks "callow" is a compliment until a
friend tells him "Dude, callow is not good!"

48.

extravagant excessive and therefore lacking restraint


In Home Alone 2, Kevins father thinks Kevins $967 hotel room service bill is
extravagant.

49.

apathetic showing little or no feeling or emotion; spiritless; lacking emotion


In the movie The Wizard of Oz, the apathetic Tin Man joins Dorothy and the Scarecrow
in hopes that the Wizard will give him a heart.

50.

erratic - lacking consistency


Be sure to give plenty of space to drivers who are driving erratically and not paying
attention to the road.

51.

trite, clich commonplace; ordinary and therefore lacking originality; humdrum


Students should use fresh, original sentences in their writing instead of trite
expressions such as as pretty as a picture.

______________________________________________________________________________
Copyright 2009, TCA, LLC.

251

Studyguide for the SAT Vocabulary: 192 Words You Need to Know_______
The Mighty Prefix De- (De is going down!)
52.

denounce to put down by condemning or criticizing


The student council denounced the decision to close the campus at lunchtime.

53.

decry to criticize; denounce; condemn


The mayor decried the deplorable living conditions of the citizens on the
outskirts of town.

54.

depreciate to go down in value


In a bear market, stocks depreciate.

55.

despondent; dejected; depressed all mean feeling very down and discouraged
In the movie The Lion King, Simba is despondent when he is unable to save his father.

56.

deleterious things are going down in the sense of being harmful, hurtful and detrimental
Smoking cigarettes is deleterious to your health.
Be- (Really Important Prefix)

57.

belittle to make someone feel really little; to put someone down; to disparage
Simon often belittles contestants on American Idol if they sing poorly.

58.

belie to lie in the sense of contradicting


In the movie 10 Things I Hate About You, the words in Kat's poem actually belie her true
feelings about Patrick.

______________________________________________________________________________
Copyright 2009, TCA, LLC.

252

Studyguide for the SAT Vocabulary: 192 Words You Need to Know_______
Law and Order
59.

plausible believable; credible


Although the plot of Jurassic Park may seem unlikely at first, author Michael
Crichton makes the story sound plausible by explaining how dinosaur DNA could be
extracted from amber.

60.

incorrigible incapable of being reformed


Bart Simpson is an incorrigible rascal.

61.

defamation an abusive attack on a persons reputation


Hollywood stars sue for defamation not as a means of making money but to clear their
names in the press and punish the tabloids that print the incorrect information.

62.

innuendo a veiled hint or insinuation, especially a negative reflection on a


persons character or reputation
The gossip column was filled with innuendo about famous actors.

63.

travesty a miscarriage of justice; a distortion of proper procedure


When the judge dismissed the case against the obviously guilty defendant, the
townspeople called it a travesty.

64.

exculpate; exonerate - both mean to free from blame; to declare innocent


In the movie My Cousin Vinny, Marisa Tomeis character, Lisa, uses her expert
automotive knowledge on the witness stand to exonerate the two youths.

65.

fabricate to make up or invent a story


Dr. Doolittles family thought he had fabricated the story about being able to talk
to animals.

66.

unprecedented without previous example


Lance Armstrong has won the Tour de France an unprecedented seven times.

67.

immaterial of no importance; irrelevant and therefore lacking significance


Her testimony was immaterial and was, therefore, not admitted into evidence.

68.

incontrovertible not open to question; indisputable


The videotape of the robbery provided incontrovertible evidence against the suspecthe
was obviously guilty.

______________________________________________________________________________
Copyright 2009, TCA, LLC.

253

Studyguide for the SAT Vocabulary: 192 Words You Need to Know_______
Degree
69.

adept very skillful; adroit


Coach Bombay's innovative teaching techniques helped the Mighty Ducks
become more adept at playing hockey.

70.

arduous very difficult to accomplish


Harry Potter has the arduous task of defeating a dragon in Harry Potter and the
Goblet of Fire.

71.

empathetic very compassionate; very sympathetic


In the movie Freaky Friday, Tess and Anna learn to be empathetic when they are forced to
walk in one anothers shoes, ultimately gaining respect and understanding for the other's
point of view.

72.

assiduous very diligent; very persistent


You should assiduously study each word on this list.

73.

furtive; clandestine very secretive; very stealthy


On Survivor, contestants make clandestine plans to sabotage one
another in order to get others voted off the show.

74.

contemplative very thoughtful; meditative; pensive


Hamlet is contemplative as he tries to decide if his life is worth living.

75.

trepidation great fear


Dorothy and her friends are filled with trepidation as they approach the Wizard of Oz.

76.

destitute very poor; indigent


In the movie Trading Places, Eddie Murphy plays a character who is originally
destitute but becomes wealthy.

77.

distraught; inconsolable very distressed; very worried


Marlin is distraught when Nemo is captured by a diver.

78.

despicable vile; very contemptible


The Green Goblin is a despicable villain who commits horrible crimes.

79.

exasperated very irritated; very annoyed


Avril Lavigne is exasperated with her boyfriend when she demands to know why
he is making things so complicated.

80.

ecstatic; exuberant; elated; euphoric feeling great happiness


Contestants are usually ecstatic when they see the results of their extreme home makeover.

81.

garish excessively vivid in color


Television home makeover shows do an excellent job of turning garish, tacky
rooms into tastefully decorated living spaces.

______________________________________________________________________________
Copyright 2009, TCA, LLC.

254

Studyguide for the SAT Vocabulary: 192 Words You Need to Know_______
82.

ostentatious fond of conspicuous display; pompous; pretentious


Some ostentatious celebrities try to attract attention with their glittering jewels
and flashy cars.

83.

vigilant very watchful


The officers aboard the Titanic were not vigilant.

84.

prodigious very large; vast; huge


The earthquake triggered a tsunami that sent prodigious waves across the
island.

85.

munificent very generous


Oprah is known for bestowing munificent gifts on members of her studio audience.

86.

parsimonious very stingy; miserly


Microsoft founder Bill Gates and his wife Melinda are NOT parsimonious. They donate
millions of dollars every year in an effort to improve health and education around the
world.

87.

convivial fond of social interaction


In Renoirs famous painting Luncheon of the Boating Party, the convivial host and his
guests are enjoying a pleasant afternoon lunch.

88.

affable friendly
The hostess was the life of the party because she was so affable.

89.

sanguine very optimistic; hopeful


President Reagan was renowned for his sanguine outlook on life.

90.

unctuous too smooth; too suave and thus giving a false sense of earnestness
In the movie School of Rock, the unctuous Dewey Finn tries to persuade his principal to
give him permission to take his class on a field trip.

91.

sanctimonious making a show of being devout and thus giving a sense of false
piousness
A sanctimonious person comes across as being holier than thou. In School of Rock,
Patty (Neds wife) is a sanctimonious busybody, who reveals Deweys true identity.

92.

lucrative very profitable


Both the Spider-Man 2 and Shrek 2 franchises are lucrative.

93.

scrutinize to very closely or very carefully examine a person or thing


On Oscar night, the press loves to scrutinize outfits of stars such as Nicole Kidman, Halle
Berry, and Jennifer Lopez as they walk the red carpet.

94.

irate; livid showing great anger and rage


In the movie Mean Girls, the junior girls at North Shore High School are livid when they
read what the Plastics said about them in the Burn Book.

______________________________________________________________________________
Copyright 2009, TCA, LLC.

255

Studyguide for the SAT Vocabulary: 192 Words You Need to Know_______
95.

circumspect very cautious; very careful; prudent


A circumspect person looks before leaping.

History and Geography


96.

repudiate to reject; disavow


The Hollywood couple repudiated claims in the press that their marriage was over;
however, a month later they filed for divorce.

97.

anachronism a person or event that is chronologically in the wrong time


The Back to the Future movies contain a number of amusing anachronisms.

98.

accord an agreement
In the movie Pirates of the Caribbean: The Curse of the Black Pearl, Captain Jack
Sparrow and Will reach an accord: Will agrees to free Sparrow and Sparrow will help
Will find Elizabeth.

99.

capitulate to surrender; to give up


In her defiant song White Flag, Dido refuses to capitulate:
Well, I will go down with this ship
And I wont put my hands up and surrender
There will be no white flag above my door
Im in love and always will be

100. enlighten to inform; instruct; illuminate and thus remove ignorance


Writers such as Voltaire enlightened European society by urging people to use science
and reason instead of blindly following their prejudices.
101. appeasement giving in to someones demands
Great Britain and France appeased Hitler at the Munich Conference.
102. stipulate to specify; set forth
The contract with the decorator stipulates the maximum amount he can charge.
103. pretext an excuse; an alleged reason
In Dr. Seuss book The Cat in the Hat Comes Back, the Cat talks his way into the
childrens house under the pretext that he would like to come in to get out of the
snow.
104. watershed a turning point; an event that is of critical importance
President Truman's decision to drop the atomic bomb was a watershed event in 20th
century history.
105. autonomy self-rule; independence
In the movie Men in Black, the MIB is an autonomous government organization.

______________________________________________________________________________
Copyright 2009, TCA, LLC.

256

Studyguide for the SAT Vocabulary: 192 Words You Need to Know_______
106. manifesto a public declaration of beliefs
Published in 1848, the Communist Manifesto publicly declared the radical economic
beliefs of Karl Marx and Friedrich Engels.
107. dovish favoring peace
During the Vietnam War, America was divided between Hawks who supported the war
and Doves who opposed it.
108. enfranchise to grant voting rights
The Voting Rights Act of 1965 enfranchised African-Americans.
109. potentate any powerful ruler; sovereign
Hitler was a brutal potentate.

Science
110. astral; celestial pertaining to the stars
The Hubble Telescope enables astronomers to view celestial bodies over 12 billion light
years away.
111. arboreal pertaining to trees
Arbor Day is tree appreciation day.
112. catalyst an agent that provokes or speeds significant change or action
Gasoline can act as a catalyst for spreading fire.
113. osmosis movement of a solvent through a semipermeable membrane
Osmosis is a process of gradual absorption. It would be easy if students could learn SAT
vocabulary words through osmosis instead of diligent studying.
114. synthesis; amalgam; conglomeration combination or fusion of different elements
The scientist synthesized the new medication in the laboratory.
115. hermetic airtight
Be sure to create a hermetic seal when storing food in the freezer.
116. indigenous native to an area
Cardinals are indigenous to the East Coast. The buffalo is indigenous to the Great
Plains.
117. archipelago a chain of islands
Hawaii is an archipelago.
118. rivulet a small stream
Several rivulets feed into the Potomac River.

______________________________________________________________________________
Copyright 2009, TCA, LLC.

257

Studyguide for the SAT Vocabulary: 192 Words You Need to Know_______
Language Arts
119. anthology a collection of poems and stories
Your literature textbook is an anthology.
120. epilogue a closing section added to a novel or play
A writer may use an epilogue to provide additional information about what
happens to the characters after the end of the story.
121. prologue an introductory section added to a poem, novel, or story
In the prologue to Romeo and Juliet, the chorus introduces the story of the starcrossed lovers.
122. satire a literary work that ridicules and derides human vices and weaknesses
Authors use satire to make fun of situations they feel are unjust.
123. vignette; anecdote short story; brief sketch
Earl Woods tells an anecdote about the time he unstrapped his ten-month-old
son Tiger from his high chair. Tiger then toddled over to a putter, picked it up, and
hit a ball into a net.
124. epic a lengthy story about the heroic deeds of a legendary hero
Compared with Homer's epic poem The Iliad, Wolfgang Petersens movie Troy seems to
be more like a vignette.
125. farce a humorous, comical show; a comedy
The Monty Python movies are excellent examples of farce because they use satire and
improbable situations.
126. raconteur a person skilled at telling a story
Homer was a blind poet who was a renowned raconteur.
127. memoir an autobiography; personal journal
Bill Clinton's memoir has had record-breaking sales.

______________________________________________________________________________
Copyright 2009, TCA, LLC.

258

Studyguide for the SAT Vocabulary: 192 Words You Need to Know_______
The Toughest Words on the SAT
128. indefatigable; unflagging tireless
You must be indefatigable and unflagging as you prepare for the upcoming SAT.
129. indomitable invincible; incapable of being conquered
You must have an indomitable will as you take the SAT.
130. vacillate to waver; hesitate; swing back and forth; be indecisive
Hamlet vacillates over which course of action to follow: "To be, or not to be.
131. hubris exaggerated pride; arrogance
In Greek tragedy, hubris is often the heros tragic flaw, which causes his
downfall.
132. excoriate to denounce scathingly
In the movie The Nutty Professor, Dean Richmond excoriates Professor Klump for
alienating all of the colleges wealthy donors.
133. innocuous harmless; inoffensive
In The Wizard of Oz we learn that water is innocuous to scarecrows but toxic to wicked
witches.
134. capricious; mercurial very changeable; fickle; constantly shifting moods
Marie knew she could not depend on her capricious little sister because she
constantly changed her mind.
135. supplant to take the place of
Happy Gilmore supplants Shooter McGavin as the biggest star on the golf tour.
136. laconic; succinct very brief; concise
In the poem The Raven by Edgar Allan Poe, the raven gives a laconic one-word
answer, Nevermore, to all of the narrators questions.
137. ubiquitous present everywhere at once; omnipresent; prevalent; widespread
The popularity of portable digital music has made the iPod ubiquitous.
138. archaic obsolete; old-fashioned; out-of-date and thus not in, with it, or
contemporary
The students felt that their schools dress code was archaic and outdated.
139. reciprocal shared and expressed by both sides; mutual
In the movie Rush Hour, Carter shows Lee how to move with the music. Lee then
reciprocates by showing Carter some new karate moves.
140. alacrity great speed; promptness
In the movie The Waterboy, Bobby is eager to help the team and responds with
alacrity to the coachs invitation to play.
______________________________________________________________________________
Copyright 2009, TCA, LLC.

259

Studyguide for the SAT Vocabulary: 192 Words You Need to Know_______
141. implacable incapable of being appeased
In the movie Rocky III, Rocky and Clubber Lang are implacable enemies.
142. grandiloquent using lofty, pompous language
The Munchkin mayor made a grandiloquent speech when he welcomed Dorothy and
thanked her for killing the Wicked Witch.
143. motley very different; very heterogeneous
The variety of outfits worn by the Continental Army soldiers gave them a motley
appearance compared with the British soldiers, who dressed uniformly.
144. juxtapose to place side by side
In television makeover shows, before-and-after pictures are juxtaposed to
highlight the dramatic changes.
145. arcane obscure; mysterious; esoteric
Shania Twain sings about being unimpressed with men who bore her with arcane facts.
146. cosmopolitan very worldly; sophisticated
Constantinople was a very cosmopolitan city as a result of its location at the
crossroads of trade.
147. ineffable incapable of being put into words; indescribable
In the movie Jerry McGuire, Rod's feelings are ineffable when he finds out that he has just
been offered an $11.2 million contract.
148. inundate to fill with water; flood
The flood waters from Hurricane Katrina inundated many communities. (It can also mean
to overwhelm or saturate.) The visiting astronaut was inundated with speaking
invitations. The students were inundated with homework.
149. rudimentary basic; elementary
Harry Potter learns that levitation is one of a wizard's rudimentary skills.
150. atone to make amends; make up for
Patrick atones for hurting Kat in the movie 10 Things I Hate About You by buying her a
guitar.
151. machinations crafty schemes
In the movie The Lion King, Simba is at first unaware of Scars evil machinations to get
rid of Mufasa and make himself king.
152. quandary a dilemma; predicament
Spider-Man finds himself in a quandary: should he save the woman he loves or the tram
filled with innocent children?
153. pernicious very injurious; destructive; deadly
Doctors were able to discover the cause of the pernicious disease.
______________________________________________________________________________
Copyright 2009, TCA, LLC.

260

Studyguide for the SAT Vocabulary: 192 Words You Need to Know_______
154. prevaricate to lie
In the movie Pirates of the Caribbean: The Curse of the Black Pearl, Will thinks Jack is
prevaricating when he tells him that Wills father was a pirate.
155. quagmire a difficult, precarious position; a predicament
In the movie Orange County, Shaun finds himself in a quagmire when he isnt accepted at
Stanford because his counselor sent in the wrong transcript and he has not applied to any
safety schools.
156. synergy the positive energy that results when two different elements are
combined
There was surprising synergy when Aerosmith and Run DMC combined to perform the
song Walk This Way.
157. eschew to abstain from; avoid; engage in abstinence
People on the Atkins diet eschew foods high in carbohydrates.
158. scintillating sparkling; dazzling; brilliant
Melissa Etheridge gave a scintillating rendition of Janis Joplins classic Piece of My
Heart at the Grammy Awards show.
159. pejorative belittling; disparaging; putting someone down
Jay Leno often makes pejorative remarks about politicians in his opening
monologue on The Tonight Show.
160. maxim a short, concise wise saying
The maxim Dont put off until tomorrow what you can do today is good advice for
those studying for the SAT!
161. consensus a general agreement
After much discussion, the members of the senior class advisory committee
reached a consensus on the location of the Senior Ball.
162. contrite; penitent expressing remorse, regret, sorrow for misdeeds
Although Denise said she was sorry, she did not act contrite.
163. heinous hatefully evil; abominable
In the movie The Patriot, the British officer commits a heinous act when he burns down
the church.
164. excise; expurgate to remove; delete; censure
The producer excised three scenes from his movie in order to receive a PG-13
rating.
165. futile doomed to failure and therefore pointless
In Star Trek the Borg always announced, Resistance is futile before conquering a
new planet.

______________________________________________________________________________
Copyright 2009, TCA, LLC.

261

Studyguide for the SAT Vocabulary: 192 Words You Need to Know_______
166. aplomb poise; confidence; self-assurance
Kelly, Reuben, and Clay all became American Idols because they had talent and
performed with aplomb.
167. paradigm a model of thought
In the geocentric paradigm, the earth is the center of the solar system. But in the
heliocentric paradigm, the sun is the center of the solar system.
168. innate inborn; present at birth
Linguists believe that humans have an innate ability to learn language.
169. penchant a preference or liking
The people of Belgium have a penchant for chocolate.
170. inured to become accustomed to accept something undesirable
The pioneers became inured to the hardships of living on the frontier.
171. venerate; revere to hold in high esteem; respect; honor
In the movie Hitch, Albert venerates Will Smiths character, Alex, because Alex
teaches him how to win the affection of the rich and beautiful Allegra.
172. enthralled fascinated; spellbound
The audience was enthralled with the special effects in the movie The Lord of the Rings:
The Return of the King.
173. subtle slight; delicate; elusive
There is a big difference between a drastic change which most people will
oppose and a subtle change which most people will barely notice.
174. sumptuous; ornate very splendid; luxurious; magnificent
Shrek and Fiona eat a sumptuous meal at her parents royal palace. The Hall of Mirrors
is the most ornate room in the Versailles Palace.
175. cathartic emotionally cleansing
In the movie Mean Girls, the teachers encouraged the junior girls to do a cathartic
exercise to release their negative feelings toward one another.
176. chicanery; disingenuousness trickery; fraud; deception
Napoleon Dynamites dishonest Uncle Rico uses chicanery to sell his defective
products.
177. premonition a forewarning; hunch; feeling ahead of an event
In the movie Star Wars, Leia has a premonition that Luke is in danger.
178. provocative thought provoking
The teacher wrote the senators provocative statement on the board in order to start a
class discussion.
179. vilify to denounce; defame; turn into a villain
Darth Vader is vilified by the Rebel Alliance in the Star Wars movies.
______________________________________________________________________________
Copyright 2009, TCA, LLC.

262

Studyguide for the SAT Vocabulary: 192 Words You Need to Know_______
180. magisterial learned and authoritative
In the Harry Potter books, Dumbledore serves as the magisterial headmaster of
Hogwarts.
181. oblivious unaware
In the movie Freaky Friday, Ryan is oblivious to the fact that mom Tess and daughter
Anna have switched bodies.
182. obstinate; adamant; obdurate very stubborn
In the movie 10 Things I Hate About You, a remake of Shakespeares play The Taming of
the Shrew, Julia Stiles plays the bitter, obstinate Katarina.
183. incessant constant; continuous; unceasing
On road trips young children often drive their parents crazy by incessantly asking, Are
we there yet?
184. cajole; coerce to coax or entice someone into doing something
SpongeBob always tries to cajole Squidward into going jellyfishing.
185. pedestrian commonplace; ordinary
Critics blasted the new play, calling it pedestrian and undistinguished.
186. improvised; impromptu done on the spur of the moment without preparation
In the beginning of School of Rock, Deweys lessons were all improvised; they
lacked preparation.
187. palpable; tangible capable of being perceived
There was a palpable air of excitement as Olympic swimming fans awaited the showdown
between Michael The Shark Phelps and the Australian Thorpedo Ian Thorpe.
188. vituperation; invective verbal abuse; a tongue-lashing; severe scolding
Her simple mistake hardly merited such vituperation.
189. tenet an article of faith; a belief
Charity is a tenet of many religions.
190. inexorable relentless; unstoppable
Gollum has an inexorable desire to regain the ring in The Lord of the Rings trilogy.
191. equanimity evenness of mind, especially under stress
Driving instructors must handle their jobs with equanimity if they are to survive the stress
of teaching young people to drive.
192. ameliorate to make better; improve
In the movie Remember the Titans, Denzel Washingtons character, Coach Boone, works
hard to ameliorate the poor relationship that exists between the players on his football
team.

______________________________________________________________________________
Copyright 2009, TCA, LLC.

263

Studyguide for the SAT Prefixes


PREFIX: The Mighty DE:
debacle
decry
demolish
deplete
deplorable
depreciate
derogatory
despondent
devoid

a big disaster, things really go down the tubes,


a fiasco
to criticize, denounce, condemn,
to put someone or something down
to tear down, destroy, raze
to exhaust, when the supply of something is depleted
it goes down
wretched, awful, miserable.
The destitute family lived in a deplorable slum.
to go down in value.
to belittle, to say unfavorable things about someone
feeling very down, discouraged
to go down in the sense of being empty,
lacking content, as the play was utterly devoid
of value and meaning

PREFIX: the Big E


eccentric
elusive
eminent
exorbitant
expound
extricate
extrovert

off center, a bit odd, weird, peculiar


out of reach, hard to catch, evasive
outstanding, illustrious, very prominent,
notable
literally out of orbit, hence unreasonable
to elaborate, to explain in great detail
to get out of an entanglement or difficulty
an outgoing person

PREFIX: MIS (wrong/bad)


misanthrope
miscreant
misnomer

a person who hates or distrusts mankind


one who behaves criminally, an evildoer, a malefactor
wrong name

PREFIX: BENE (good/well)


benediction
benefactor
benevolent
benign

Copyright 2009, TCA, LLC.

a blessing
one who has given help, especially financial
inclined to do good, kind
good natured, kindly, favorable, not malignant

264

Studyguide for the SAT Prefixes


PREFIX: UN (not)
unfazed,
unfettered
unflappable
unfounded
unorthodox
unparalleled
unremitting
unscathed
unsubstantiated,
untenable
unwarranted

calm, not disturbed not easily upset, calm


free from restraints, liberated
calm, not disturbed not easily upset, calm
groundless, without substance, false
not following established ways of thinking
having no parallel, unequaled
unrelieved, relentless, ceaseless
unhurt, unharmed
not proven, lacking justification
not able to be defended
not proven, lacking justification

PREFIX: AB (away from/off)


abdicate
aberration
abhor
amorphous
anarchy

to give up, resign


a departure from what is normal or typical
todislike intensely, loathe, despise
having no form or shape
having no government, chaotic

PREFIX: A (no,not)
anomaly
atheist

something that does not follow the norm


aperson who does not believe in God

PREFIX: RE (back,again)
redundant
refurbish
rejuvenate
repatriate
resilient
revitalize
revoke

repetitive
to make new again, renovate
to make young again
to return to one's country of origin
able to bounce back from adversity or change
to give new life to
to take back

PREFIX: AMI (friend)


amicable, amity,
amiable

Copyright 2009, TCA, LLC.

pleasant, friendly, peaceful


affable

265

Studyguide for the SAT Prefixes


PREFIX: SUPER (above)
supercilious
supersede

believing oneself to be above others, arrogant, haughty


replace, take the place of

PREFIX: MAL (bad)


malefactor
malediction
malevolent
malicious
malignant

an evildoer or criminal
a curse
wishing evil or harm to others
causing pain, injury, or distress to another
having an evil influence, very harmful

PREFIX: IN/IM (not)


inauspicious
incorporeal
incorrigible
indefatigable
immutable
impartial
impious
insatiable
intrepid
inviolable

not favorable
without material form or substance
incapable of being reformed
incapable of being fatigued, having great stamina
unchanging
unbiased, treating all equally
lacking reverence, disrespectful
incapable of being satisfied
having no fear, dauntless
secure and thus unable to be violated

Other Common Prefixes


adanteantiaquautobicircumconcontradisenfingenhypo-

Copyright 2009, TCA, LLC.

to, toward
before
against
water
self
two
around
with, together
against
not, apart
in
end, limit
birth, kind, origin
under, beneath, less than

admit, adjoin
antediluvian, antedate
antibiotic, antifreeze
aquarium, aqueduct
autocrat, autograph
bisect, bipartisan
circumlocution, circumspect
convene, convention
contradict, contraband
discharge, disarm
engulf, endanger, enfeeble
final, indefinite
generation, gender
hypodermic, hypocrite

266

Studyguide for the SAT Prefixes


intermononeononparapolypostpreproretrosemisubtranstriultrauni-

Copyright 2009, TCA, LLC.

between
one
new
not
beside, beyond
many
after, behind
before
before, forward,
acting for
backward
half, partly
under
across, beyond
three
beyond
one

interdisciplinary, intercede
monorail, monologue
neoclassic, neolithic
nondescript, nonbeliever
parallel, paradox
polygon, polysyllabic
postwar, postscript
preclude, precedent
progress, pronoun, prognosis
retroactive, retrorocket
semicircle, semiprecious
subhuman, subordinate
transcontinental, transport
triathlon, trimester
ultraviolet, ultrasound
unicycle, universe

267

Studyguide for the SAT Suffixes


Common Suffixes
capable
like, having nature of,
act of
-ance, -ence act, quality
-ant
being
state of being
-ary
relating to
-ate
to cause to become
-en
little, made of
-er
doer, maker
-ful
full of
-ic
dealing with, caused by,
showing, person or thing
-able, -ible
-al, -ial

-ion, -tion,
-ation
-ity, -ty
-ive
-less
-ly
-ment
-ness
-ous, -ose
-y

Copyright 2009, TCA, LLC.

comfortable, edible
refusal, congenial
subsistence, existence, defiance
attendant, servant
temporary, legendary
invalidate, sublimate
kitten, earthen, golden
batter, logger, complainer
boastful, cheerful
photographic, symphonic, manic, heretic

act or state of being

subtraction, addition, valuation

state or quality of being


relating to
without
used to form adverbs
from adjectives
result, action
quality,
condition of being
abounding in
full of, containing

activity, entity
legislative, communicative
guiltless, sugarless
abstractly, slowly
improvement, pavement
preparedness, willingness
voluminous, verbose
soupy, sticky

268

Studyguide for the SAT Roots


ROOT: FID (faith)
perfidious
treacherous, untrustworthy
diffident
lacking faith in oneself, lacking confidence
fidelity, infidelity faithfulness, unfaithfulness
ROOT: FLU (flow)
affluent
flowing abundantly, wealthy
confluence
to flow together, converge
superfluous
flowing above, hence exceeding what is necessary
ROOT: CHRON (time)
anachronism
something that is not happening in its proper time
chronological
arranged in the order of occurrence
synchronize
to occur at the same time, simultaneous
ROOT:GREG(group/people)
gregarious
enjoying the company of groups, affable
segregate
to separate into different groups
egregious
outside of the group, outstandingly bad
ROOT: LUC (light)
elucidate
to make clear, clarify
lucid
clear, readily understood
pellucid
very clear, transparent
translucent
permitting the passage of light
ROOT: ACRI/ACER(very sharp)
acrid
unpleasantly pungent in taste or odor
acrimonious
full of spite, bitter, nasty
acuity
keenness of perception
acumen
mental sharpness
acute
sharp, very keen
exacerbate
to sharpen or aggravate, inflame
ROOT: PUG (fist, fight)
impugn
to challenge the accuracy or honesty of something
pugnacious
combative, quick to fight
repugnant
offensive, very distasteful, repulsive
ROOT:PLAC (calm)
placid
outwardly calm, composed
complacent
so calm as to be self-satisfied, smug
implacable
incapable of being calmed, relentless
placate
to calm, appease
ROOT:PATHOS(feeling)
antipathy
feeling against someone or something, dislike
empathy
feeling the same thing as someone else

Copyright 2009, TCA, LLC.

269

Studyguide for the SAT Roots


ROOT: SPEC/SPIC (see)
specious
seemingly fair or true, but actually not so,
deceptive
circumspect
taking a look around, cautious
perspicacity
keen vision, perceptive, astuteness

Other Common Roots

aud, audit
brev
cred
dem
duc
err
gram
loc, loqu
nov
orth
ped

hear
short, brief
belief, trust
people
to lead
wander, stray
write, writing
talk, speech
new
straight, correct
foot

auditorium, audible
brevity, abbreviation
credibility, incredible
democracy, demagogue
induct, conduct
erratic, aberration
telegram, grammatical
locution, colloquial
novel, novice
orthopedic, orthodox
pedestrian, pedal

phil
phon
prim
scrib
secut, sequ
ten
tract
ver
vita

like, love
sound
first, early
write
follow
hold
draw, pull
true
life

philanthropy, philosophy
phonograph, telephone
primary, primitive
scribble, inscribe
sequential, persecute
tenacious, tenant, tenure
tractor, distraction
verify, verisimilitude
vital, vitamin

Copyright 2009, TCA, LLC.

270

Writing Lesson #1

Grammar Usage and Mechanics

Triumph College Admissions


Skill Lesson

Quiz Time!
A

Hints!

Answers!
A

*Throughout this documentation, and the software: College Board and SAT are registered trademarks of the College Entrance Examination Board. PSAT/NMSQT
is a trademark of the College Entrance Examination Board and National Merit Scholarship Corporation. ACT is a trademark of ACT, Inc. None of these entities are
affiliated with the production of, nor endorse these materials.
Copyright 2009, TCA, LLC.

271

Copyright 2009, TCA, LLC.

272

Studyguide for the SAT Skill Review: Grammar Usage and Mechanics

GRAMMAR, USAGE, AND MECHANICS


These are the principal ways the SAT tests to see if you can recognize when
sentences are grammatically correct:
Verb Tense
Subject-Verb Agreement
Noun/Pronoun Agreement
Pronoun Reference
Ambiguous Reference
Adjective/Adverb convention
Possessive Form
Double Negative
Diction

Incorrect Verb Tense


You will see two types of errors in verb tense on the SAT.
One type is the incorrect verb tense in a sentence.
Incorrect: The boys were ask to play. (Since the auxiliary
verb "were" is in the past tense, "ask" should also be in the past
tense.)
Correct: The boys were asked to play.
Incorrect: Construction of a new school was began yesterday. (The past
participle of "begin" is "begun," not "began.")
Correct: Construction of a new school was begun yesterday.
The other type is a shift in verb tense.
Examples:
Incorrect: Sheila constantly loses her keys and then
asked me to drive her home. ("loses" is in the present tense, but
"asked" is in the past tense.)
Correct: Sheila constantly loses her keys and then asks
me to drive her home. (Both verbs are in the present tense.)
Incorrect: He came to Las Vegas and plays the slot
machines. ("came" is in the past tense, but "plays" is in the present
tense.)
Correct: He came to Las Vegas and played the slot
machines. (Both verbs are in the past tense.)

Copyright 2009, TCA, LLC.

273

Studyguide for the SAT Skill Review: Grammar Usage and Mechanics
Practice Exercise:
Choose the correct sentence.
Question 1.
(A) I would not have called if I had knew you were sleeping.
(B) I would not have called if I had known you were sleeping.
The answer is B. The past participle of "know" is "known."
Question 2.
(A) At the end of the play, the entire cast
comes onstage and takes a bow.
(B) At the end of the play, the entire cast
comes onstage and will take a bow.
The answer is A. Like "comes," it is in the present tense.
Pronoun Shifts
This type of error occurs when pronouns shift from one form to
another.
Here are some examples:
If you want a good reading score on your SAT, one should
read more books. (Incorrect)
If you want a good reading score on your SAT, you should
read more books. (Correct)
Senior citizens like the governor's style of campaigning, but
younger voters have no opinion of him. (Incorrect)
Senior citizens like the governor's style of campaigning, but
younger voters have no opinion of it. (Correct)
The pronoun "it" refers to the governor's style not the governor.
Subject-Verb Agreement
The verb must agree with the subject in number.
The test writers often put a phrase between the subject and the verb
to try to throw you off.
Example:
Incorrect: The dog, along with several cats, were
chasing cars. (The subject of the sentence, "dog" is singular and
needs a singular verb.)
Correct: The dog, along with several cats, was chasing cars.

Copyright 2009, TCA, LLC.

274

Studyguide for the SAT Skill Review: Grammar Usage and Mechanics
The phrases "in addition to" and "as well as" are often placed between
the subject and verb, as "along with" is in the sentences above. To
choose the correct form of the verb, read the sentence without the
phrase.
Here's another example:
Incorrect: Cancellation of your tickets generate an
automatic refund. (The subject of the sentence, "cancellation," is
singular and needs a singular verb.)
Correct: Cancellation of your tickets generates an
automatic refund.
Practice Exercise:
Choose the correct sentence.
Question 1.
(A) Carmen's painting, unlike the other
artworks, was abstract.
(B) Carmen's painting, unlike the other
artworks, were abstract.
The answer is A. The subject of the sentence, "painting," is singular
and needs a singular verb.
Question 2.
(A) My aunts, as well as my mom, are from Spain.
(B) My aunts, as well as my mom, is from Spain.
The answer is A. The subject of the sentence, "aunts," is plural and
needs a plural verb.
Noun or Pronoun Agreement
A pronoun must agree in number with its antecedent. The antecedent is
the word, usually a noun, to which the pronoun refers.
Examples:
Incorrect: Every one of the boys needs to get their
hair cut. (The antecedent "one" is singular and needs a singular
pronoun.)
Correct: Every one of the boys needs to get his hair cut.
Incorrect: The students had lots of good ideas for a field
trip, like a trip to the beach, but the teacher ignored it. (The
antecedent "ideas" is plural and needs a plural pronoun.)
Correct: The students had lots of good ideas for a field
trip, like a trip to the beach, but the teacher ignored them.

Copyright 2009, TCA, LLC.

275

Studyguide for the SAT Skill Review: Grammar Usage and Mechanics
Practice Exercise:
Choose the correct sentence.
Question 1.
(A) The legislature will hold their final
session tomorrow.
(B) The legislature will hold its final session
tomorrow.
The answer is B. The antecedent "legislature" is singular and needs a
singular pronoun.
Question 2.
(A) Each player must buy their own uniform.
(B) Each player must buy her own uniform.
The answer is B. The antecedent "player" is singular and needs a
singular pronoun.
If a singular noun refers to people of both genders, use "his or her,"
never "their."
Example:
Each player must buy his or her own uniform.
Pronoun Reference
A pronoun must be in the correct case. The most common type of pronoun
case error is the use of the subjective case when the objective case
is needed.
Examples:
Incorrect: Let's keep this between you and I. (The pronouns
are objects of the preposition "between," and they should be in the
objective case. However, "I" is in the subjective case.)
Correct: Let's keep this between you and me.
Incorrect: No one knows about it except you and she. (The
pronouns are objects of the preposition "except," and they should be
in the objective case. However, "she" is in the subjective case.)
Correct: No one knows about it except you and her.

Copyright 2009, TCA, LLC.

276

Studyguide for the SAT Skill Review: Grammar Usage and Mechanics
Note that the pronoun "you" has the same form in the subjective case
and in the objective case.
Sometimes an error will appear as a pronoun in the subjective case in
front of a gerund. The pronoun should be in the possessive case.
Example:
Incorrect: She winning the lottery was a great shock.
Correct: Her winning the lottery was a great shock.
Exercise:
Choose the correct sentence.
Question 1.
(A) The voice on the phone was muffled, but it
could have been he.
(B) The voice on the phone was muffled, but it
could have been him.
The answer is A. In this sentence, the pronoun is a predicative
nominativea noun or a pronoun that follows a linking verb and
explains or identifies the subject of the sentence ("voice"). A
predicate nominative is always in the subjective case.

Question 2.
(A) The librarian gave Clem and I a lecture about eating in
the library.
(B) The librarian gave Clem and me a lecture about eating in
the library.
The answer is B. The pronoun is the indirect object of the verb "gave"
and needs to be in the objective case.

Copyright 2009, TCA, LLC.

277

Studyguide for the SAT Skill Review: Grammar Usage and Mechanics
Ambiguous Reference
A pronoun must refer clearly to its antecedent. When a pronoun could
refer to either of two nouns, the meaning of the sentence is
ambiguous.
Examples:
Ambiguous: Joe told Bill that he was lucky. (he" could refer to
either Joe or Bill.)
Clear: Joe told Bill that Bill was lucky. (the pronoun has been
replaced by a noun.)
Clear: Joe said to Bill, " You are lucky." (The sentence is
constructed as dialogue.)
Ambiguous: Sue saw Sally when she was waiting for the bus. ("she"
could refer to either Sue or Sally.)
Clear: While Sue was waiting for the bus, she saw Sally. (The
pronoun and its antecedent have been moved to make their relationship
clear.)

Practice Exercise:
Choose the sentence in which the relationship between the pronoun and
its antecedent is clear.
Question 1.
(A) Dave missed the game, which caused quite a
controversy.
(B) Dave's absence from the game caused quite a
controversy.
The correct answer is B. In Sentence A, it could be the game itself or
the fact that Dave missed the game that caused the controversy.
Question 2.
(A) When the remote-controlled model airplane
hit the window, it broke.
(B) The window broke when the remotecontrolled model airplane hit it.
The correct answer is B. In Sentence A, it could be the window or the
airplane that broke.

Copyright 2009, TCA, LLC.

278

Studyguide for the SAT Skill Review: Grammar Usage and Mechanics
Possevive Pronouns
Possessive pronouns replace possessive nouns. The possessive pronouns
are:

First Person
Second Person
Third Person

Singular
my, mine
your, yours
his, her, hers,

Plural
our, ours
your, yours
its their, theirs

Like other forms of personal pronouns, possessive pronouns must agree


with their antecedent in number and in gender.
Examples:
Adela asked Jake to critique her essay.
If Troy and Kevin win the contest, the prize will be theirs .
Most possessive pronouns differ in form depending on whether they
modify a noun or stand alone.
Examples:
Max is my dog. Max is mine.
The pronouns in italics above always stand alone. The third-person
singular possessive pronouns his and its are the same whether
they modify a noun or stand alone.

Copyright 2009, TCA, LLC.

279

Studyguide for the SAT Skill Review: Grammar Usage and Mechanics

Pay particular attention to its and their . These two confuse many students.
Its is a possessive pronoun.
Do not confuse it with the contraction it's (which means it is).
Incorrect: Its a beautiful day today.
Correct: It's a beautiful day today.
Incorrect: The kitten opened it's eyes.
Correct: The kitten opened its eyes.
Their is a possessive pronoun.
Do not confuse it with the contraction they're (which means there are) or the
adverb there (which shows location).
Incorrect: There hoping to move they're offices their.
Correct: They're hoping to move their offices there.

Practice Exercise

Question 1.
(A) Kaya and Sarah are going to do there
homework together.
(B) Kaya and Sarah are going to do their
homework together.
The correct answer is B. Use the possessive pronoun "their."

Question 2.
(A) The city is celebrating their 100th
anniversary.
(B) The city is celebrating its 100th
anniversary.

The correct answer is B. Use a singular possessive pronoun "its" to agree with the
singular noun "city."

Copyright 2009, TCA, LLC.

280

Studyguide for the SAT Skill Review: Grammar Usage and Mechanics
Adjective/Adverb Convention
Make sure that you use adjectives and adverbs correctly. An adjective
modifies a noun or a pronoun. An adverb modifies a verb, an adjective,
or another adverb.
Examples:
Incorrect: We drove cautious because the roads were wet.
("drove" is a verb and must be modified by an adverb, while "cautious"
is an adjective.)
Correct: We drove cautiously because the roads were wet.
("cautiously" is an adverb.)

Incorrect: He worked slow and deliberate . ("worked" is


a verb and must be modified by adverbs, while "slow" and "deliberate"
are adjectives.)
Correct: He worked slowly and deliberately . ("slowly"
and "deliberately" are adverbs.)
Watch out for "good" and "well." "Good" is an adjective, and "well" is
an adverb.
Example:
Correct: To get a good grade you must do well on your tests.
The adjective "good" modifies the noun "grade," and the adverb "well"
modifies the verb "do."

Copyright 2009, TCA, LLC.

281

Studyguide for the SAT Skill Review: Grammar Usage and Mechanics
Practice Exercise:
Choose the correct sentence.
Question 1.
(A) Pete pitched outstanding in last night's game.
(B) Pete pitched outstandingly in last night's game.
The answer is B. "Pitched" is a verb that must be modified by an
adverb.

Question 2.
(A) The vet said that the kitten's prospects
for recovery are well.
(B) The vet said that the kitten's prospects
for recovery are good.

The answer is B. A word that follows a linking verb (such as "are")


and modifies a noun ("prospects") is called a predicate adjective.
Double Negative
You probably won't see the double negative "not no" on the SAT. But
you will see errors using the phrases "hardly no," "barely no,"
"scarcely no," and "couldn't hardly."
Examples:
Incorrect: There were scarcely no jobs available.
("scarcely no" is a double negative.)
Correct: There were scarcely any jobs available.
Incorrect: The children couldn't hardly wait to go on
vacation. ("couldn't hardly" is a double negative.)
Correct: The children could hardly wait to go on vacation.

Copyright 2009, TCA, LLC.

282

Studyguide for the SAT Skill Review: Grammar Usage and Mechanics
When a double negative appears in a sentence improvement section, you
must pay close attention to the intended meaning in order to correct
the sentence. For example, "hardly no" could be corrected by "hardly
any" or by "no."
Examples:
Incorrect: The bird population declined because there was
hardly no food.
Correct: The bird population declined because there was
hardly any food. (the double negative has been corrected to show
the most likely intended meaning. Since some birds stayed, there
probably was some food.)
Incorrect: All the birds left the area because there was
hardly no food.
Correct: All the birds left the area because there was no
food. (The double negative has been corrected to show the most likely
intended meaning. Since all the birds left, there probably was no
food.)
Practice Exercise:
Choose the best word or phrase to replace the underlined portion of
each sentence.
Question 1.
(A) The girl's answers to our questions didn't
make any sense.
(B) The girl's answers to our questions hardly
made no sense.
The answer is A. "Hardly made no" is a double negative.
Question 2.
(A) We don't need but $20 for gas.
(B) We need only $20 for gas.
The correct answer is B. "Don't need but" is a double negative.

Copyright 2009, TCA, LLC.

283

Studyguide for the SAT Skill Review: Grammar Usage and Mechanics
Diction
The SAT tests your ability to choose the correct word for a given
context. Words that are commonly misused and confused will show up
several times in each SAT.
Examples:
accept, except (Accept means receive. Except means exclude
or excluding.)
The local college accepted everyones application except his.
affect, effect (Affect, meaning to influence or change," is used as a
verb. Effect is most commonly used as a noun meaning "result," but
it sometimes is used as a verb to mean "bring about" or "cause.)
allusion, illusion (Allusion means "indirect reference."
Illusion means "a false image or idea.")
eminent, imminent (Eminent means "distinguished." Imminent
means "about to occur.")
The arrival of the eminent professor was imminent.
farther, further (Farther denotes physical distance;
further denotes distance in time, degree, extent, or quantity.)
Arizona is farther from here then I thought.
Do you need any further assistance?
former, latter (Former refers to the first of two items
mentioned; latter refers to the second.)
principal, principle (Principal, as a noun means "person in
charge" or "sum of money." As an adjective it means "main" or "most
important." Principle means "rule, law, or fundamental truth.")
The principal at our high school made sure that we learned the
principles of good citizenship.
than, then (Than is used in comparisons; then refers to
time.)
He went to the library rather than the mall and then drove home.
Make sure you know the meanings of these words and how to use them
correctly.

Copyright 2009, TCA, LLC.

284

Studyguide for the SAT Skill Review: Grammar Usage and Mechanics
Practice Exercise:
Choose the correct sentence.
Question 1.
(A) The game had to be postponed because a
thunderstorm was imminent.
(B) The game had to be postponed because a
thunderstorm was eminent.
The correct answer is A.

Question 2.
(A) The short story makes an illusion to an
event that actually occurred.
(B) The short story makes an allusion an event
that actually occurred.
The correct answer is B.
Weak Passive Verb
Verbs in the active voice make sentences more effective and emphatic.
My skin was punctured by a nail. (Passive)
A nail punctured my skin. (Active)
Global warming is being studied by climatologists. (Passive)
Climatologists are studying global warming. (Active)
Three bears were seen by Joe near the lake. (Passive)
Joe saw three bears near the lake. (Active)
The movie will be seen by only a few of her fans. (Passive)
Only a few of her fans will see the movie. (Active)

Copyright 2009, TCA, LLC.

285

Copyright 2009, TCA, LLC.

286

Studyguide for the SAT Skill Quiz A: Grammar Usage and Mechanics

Quiz Time!
A

Question 1.
We've collected several items to auction: Rick's watch, Tony's bike, and Ross's
A
B
golf clubs, but most of the merchandise is your's . No error
C
D
E

(A)

(B)

(C)

(D)

(E)

Question 2.
I love to cook Italian dishes made from scratch , but my specialty are pasta dishes
A
B
C
with a variety of sauces. No error
D
E

(A)

(B)

(C)

(D)

(E)

Question 3.
Danielle stayed late and practiced reading Macbeth with Kathy because she
A
B
wanted to make sure their performance would be flawless. No error
C
D
E

(A)

Copyright 2009, TCA, LLC.

(B)

(C)

(D)

(E)

287

Studyguide for the SAT Skill Quiz A: Grammar Usage and Mechanics

Question 4.
My plate came with two enchiladas; however , I could not eat both, so I had the
A
B
C
waiter wrap one and took them home to eat later. No error
D
E

(A)

(B)

(C)

(D)

(E)

Question 5.
The mayor, along with several prominent businesses and council members, have
A
B
given a generous contribution to local artists . No error
C
D
E

(A)

(B)

(C)

(D)

(E)

Question 6.
Our teacher wants us to conduct interviews with people who were
formerly in the service and wrote the results in narrative form.

(A)

wrote the results

(B)

will write the results

(C)

will have written the results

(D)

write the results

(E)

can write the results

Copyright 2009, TCA, LLC.

288

Studyguide for the SAT Skill Quiz A: Grammar Usage and Mechanics

Question 7.
John, Heather, and Kim got into an inane argument because they did not
agree on who is the funniest comedian : Buster Keaton or Jerry
Lewis.

(A)

on who is the funniest comedian

(B)

on who is the most funniest comedian

(C)

about who is the funniest comedian

(D)

about who the funniest comedian is

(E)

on who is the funnier comedian

Question 8.
By the time I wash the car, walk the dog, and drop off my little
brother at his friend's sleepover party, my friends will have
already left on their camping trip.

(A)

will have already left

(B)

will already be leaving

(C)

will already left

(D)

had already left

(E)

had already be leaving

Copyright 2009, TCA, LLC.

289

Studyguide for the SAT Skill Quiz A: Grammar Usage and Mechanics

Question 9.
The fog was so thick that Monica couldn't hardly see anything;
nevertheless, she was driving too quickly and ran into a parked
vehicle.

(A)

couldn't hardly see anything; nevertheless,

(B)

couldn't hardly see anything; in addition,

(C)

could hardly see anything; nevertheless,

(D)

couldn't hardly see everything; as a result,

(E)

could hardly see nothing; nevertheless,

Question 10.
Having been bred and trained as workers and companions for
thousands of years, most domesticated dog breeds do not have the
skills necessary to survive in the wild.

(A)

Having been bred and trained

(B)

Despite the fact that they are bred and trained

(C)

To have been bred and trained

(D)

To be bred and trained

(E)

In order to breed and train

Copyright 2009, TCA, LLC.

290

Studyguide for the SAT Skill Quiz A: Grammar Usage and Mechanics

Hints!
A

Hint for Question 1:


One apostrophe is not needed.
Hint for Question 2:
What is the specialty?
Hint for Question 3:
Who wants a flawless performance?
Hint for Question 4:
How many enchiladas were left?
Hint for Question 5:
The mayor is the subject of the sentence.
Hint for Question 6:
Look for a shift.
Hint for Question 7:
How many people are being compared?
Hint for Question 8:
When are the speaker's friends leaving?
Hint for Question 9:
How many negative words are in this sentence?
Hint for Question 10:
The first half of the sentence modifies "breeds."

Copyright 2009, TCA, LLC.

291

Copyright 2009, TCA, LLC.

292

Studyguide for the SAT Skill Quiz A: Grammar Usage and Mechanics

Answers!
A

Question 1.
The answer is (D).
Explanation for Question 1:
The answer is D.
Possessive pronouns do not need apostrophes.
Question 2.
The answer is (C).
Explanation for Question 2:
The answer is C.
The verb should agree with the subject: "specialty is."
Question 3.
The answer is (B).
Explanation for Question 3:
The answer is B.
"She" could refer to Danielle or Kathy. The pronoun should be replaced
with one of the girls' names.
Question 4.
The answer is (D).
Explanation for Question 4:
The answer is D.
Only one enchilada gets wrapped, so the writer took it home.
Question 5.
The answer is (B).
Explanation for Question 5:
The answer is B.
A phrase will not change the number of the verb. The mayor has donated money.
Question 6.
The answer is (D).
Explanation for Question 6:
The answer is D.
The verb "wrote" should be "write" to keep the verb tense consistent
with "conduct."

Copyright 2009, TCA, LLC.

293

Studyguide for the SAT Skill Quiz A: Grammar Usage and Mechanics
Question 7.
The answer is (E).
Explanation for Question 7:
The answer is E.
"Funnier" compares two things, while "funniest" compares three or more.
Question 8.
The answer is (A).
Explanation for Question 8:
The answer is A.
The sentence is correct as it is.
Question 9.
The answer is (C).
Explanation for Question 9:
The answer is C.
This is a double negative. Try "Monica could hardly see anything" or
"Monica couldn't see anything."
Question 10.
The answer is (A).
Explanation for Question 10:
The answer is A.
The sentence is correct as it is.

Copyright 2009, TCA, LLC.

294

Studyguide for the SAT Skill Quiz B: Grammar Usage and Mechanics

Quiz Time!
A

Question 1.
By next semester, all seniors will have been send several entrance applications
A
B
to colleges of their choice . No error
C
D
E
(A)

(B)

(C)

(D)

(E)

Question 2.
Before takeoff, the flight attendant asked each passenger to read the brochure for
A
B
their safety in the event of an emergency . No error
C
D
E

(A)

(B)

(C)

(D)

(E)

Question 3.
After the principal announced that several cars in the parking lot had their lights
A
B
on, Alice told the teacher that her car lights were still on. No error
C
D
E

(A)

Copyright 2009, TCA, LLC.

(B)

(C)

(D)

(E)

295

Studyguide for the SAT Skill Quiz B: Grammar Usage and Mechanics

Question 4.
If you are in training, energy bars or fruit are a great snack that
A
B
will keep you satisfied until your next full meal. No error
C
D
E

(A)

(B)

(C)

(D)

(E)

Question 5.
Although the hailstorm damaged several metal roofs, hardly no damage
A
B
C
was evident on shingled roofs. No error
D
E

(A)

(B)

(C)

(D)

(E)

Question 6.
Yesterday, my brother learns that he is not required to wear a suit and tie to his
job interview next week.

(A)

my brother learns that he is not

(B)

my brother is learning that he is not

(C)

my brother will learn that he will not be

(D)

my brother learned that he was not

(E)

my brother learned that he is not

Copyright 2009, TCA, LLC.

296

Studyguide for the SAT Skill Quiz B: Grammar Usage and Mechanics
Question 7.
Before obtaining your driver's license, you must take driver education, get your
learner's permit, and the rules of the road must be learned.
(A)

you must take driver education, get your learner's permit, and the rules of
the road must be learned.

(B)

you must take driver education, get your learner's permit, and learn the
rules of the road.

(C)

you must take driver education and get your learner's permit
and learn the rules of the road.

(D)

you must take driver education, get your learner's permit,


then you must learn the rules of the road.

(E)

you must take driver education, get your learner's permit, and
learning the rules of the road.

Question 8.
The method by which the party control the actions of its members is known as
political influence.

(A)

control the actions of its members

(B)

control the actions of it's members

(C)

controls the actions of it's members

(D)

controls the actions of its members

(E)

control the actions of they're members

Copyright 2009, TCA, LLC.

297

Studyguide for the SAT Skill Quiz B: Grammar Usage and Mechanics
Question 9.
I was supposed to take Mandy and Lisa to the beach in my car, but they refused,
claiming that I desperately needed to wash it.

(A)

claiming that I desperately needed to wash it.

(B)

claiming desperately that I needed to wash it.

(C)

desperately claiming that I needed to wash it.

(D)

claiming that I needed to wash it desperately.

(E)

claiming that I needed to desperately wash it.

Question 10.
Failing the reach the summit, morale of the hikers was low.

(A)

Failing the reach the summit, morale of the hikers was low.

(B)

Failing to reach the summer, morale of the hikers is low.

(C)

The morale of the hikers was low when they failed to reach the summit.

(D)

The hikers' morale was low failing to reach the summit.

(E)

When the summit could not reach reached by them, the hiker's morale was
low.

Copyright 2009, TCA, LLC.

298

Studyguide for the SAT Skill Quiz B: Grammar Usage and Mechanics

Hints!
A

Hint for Question 1:


When does the action take place?
Hint for Question 2:
Whose safety is at issue?
Hint for Question 3:
Who forgot to turn off the car lights?
Hint for Question 4:
Check the verbs.
Hint for Question 5:
Avoid double negatives.
Hint for Question 6:
When did the brother get the information?
Hint for Question 7:
Are all the phrases in the same (parallel) form?

Hint for Question 8:


Check for subject/verb agreement.
Hint for Question 9:
What word in the sentence is modified by the word "desperately"?
Hint for Question 10:
Does the first part of the sentence correctly modify the subject?

Copyright 2009, TCA, LLC.

299

Copyright 2009, TCA, LLC.

300

Studyguide for the SAT Skill Quiz B: Grammar Usage and Mechanics

Answers!
A

Question 1.
The answer is (A).
Explanation for Question 1:
The answer is A.
All seniors "will have sent" would be correct. The action will end before the end of the semester.

Question 2.
The answer is (C).
Explanation for Question 2:
The answer is C.
The pronoun "their" is plural; "each passenger" is singular. The pronoun should be singular: "his or her
safety" is correct.

Question 3.
The answer is (D).
Explanation for Question 3:
The answer is D.
It is difficult to know whose car lights are still onAlice's or the teacher's.

Question 4.
The answer is (B).
Explanation for Question 4:
The answer is B.
When a singular and a plural subject are joined by "or," the verb agrees with the subject nearer the verb.
Question 5.
The answer is (C).
Explanation for Question 5:
The answer is C.
"Hardly" and "no" are both negative words. "Hardly any damage" corrects this double negative.

Copyright 2009, TCA, LLC.

301

Studyguide for the SAT Skill Quiz B: Grammar Usage and Mechanics
Question 6.
The answer is (E).
Explanation for Question 6:
The answer is E.
The event happened yesterday. The correct verb should be "learned."
Question 7.
The answer is (B).
Explanation for Question 7:
The answer is B.
In order to make all three phrases parallel, change the passive "the rules of the road must be learned" to
"learn the rules of the road." Answer D makes it parallel, but it contains a comma splice.
Question 8.
The answer is (D).
Explanation for Question 8:
The answer is D.
The singular "party" takes the singular verb "controls." Answer D also uses the correct possessive
pronoun "its."
Question 9.
The answer is (A).
Explanation for Question 9:
The answer is A.
"Desperately" should modify "needed." It should not modify "wash" or "claiming."
Question 10.
The answer is (C).
Explanation for Question 10:
The answer is C.
As it is written, "morale" failed to reach the summit. Answer C corrects this dangling modifier.

Copyright 2009, TCA, LLC.

302

Studyguide for the SAT Skill Quiz C: Grammar Usage and Mechanics

Quiz Time!
A

Question 1.

You will arrive latter if you miss the shortcut


A
B
C
near King Street. No error
D
E
(A)

(B)

(C)

(D)

(E)

(D)

(E)

Question 2.

Tell them that their dinner will be ready when


A
B
their finished with their chores. No error
C
D
E
(A)

(B)

(C)

Question 3.
People would be more careful with their money, if
its the principal reason that couples
B
C
D
argue. No error
E
(A)

(B)

(C)

they knew
A

(D)

(E)

(D)

(E)

Question 4.

They told me he never excepted


A
B
C
what happened to his daughter. No error
D
E
(A)

Copyright 2009, TCA, LLC.

(B)

(C)

303

Studyguide for the SAT Skill Quiz C: Grammar Usage and Mechanics
Question 5.

He didn't seem to be effected


A
B
C
by the loss of his job. No error
D
E
(A)

(B)

(C)

(D)

(E)

Question 6.

Please be careful with that china, because it could very easily get busted .
(A)

could very easily get busted

(B)

could very easily get broken

(C)

could very easily be broken

(D)

is extremely fragile

(E)

could get broken very easily

Question 7.
The Senate will be considering a bill to protect the environment.

(A)

will be considering a bill

(B)

will consider a bill

(C)

shall be considering a bill

(D)

will be taking under consideration

(E)

will themselves be considering a bill

Copyright 2009, TCA, LLC.

304

Studyguide for the SAT Skill Quiz C: Grammar Usage and Mechanics
Question 8.
Bill told Jeff that he should have taken the train.
(A)

that he should have taken the train

(B)

that the train should have been taken

(C)

that he should of taken the train

(D)

what he should have done was to take the train

(E)

to take the train next time

Question 9.
California suffered substantial economic effects from the winter storms.
(A)

suffered substantial economic effects from

(B)

suffered substantially, especially economic effects, from

(C)

suffered substantial economic affects from

(D)

substantially suffered economic effects from

(E)

suffered economic effects, substantially from

Question 10.
The bill for the dinner was paid by I.

(A)

The bill for the dinner was paid by I

(B)

The bill for the dinner were paid by me

(C)

I paid the bill for the dinner

(D)

The bill for the dinner was paid for by me

(E)

The bill was paid by me, for the dinner

Copyright 2009, TCA, LLC.

305

Copyright 2009, TCA, LLC.

306

Studyguide for the SAT Skill Quiz C: Grammar Usage and Mechanics

Hints!
A

Hint for Question 1:


One of the words is not quite right.
Hint for Question 2:
Look for an incorrectly used "their."
Hint for Question 3:
The wrong answer? It's a pronoun.
Hint for Question 4:
One of the words is not acceptable.
Hint for Question 5:
How will you be affected if you miss this one?
Hint for Question 6:
Which is the most concise answer?
Hint for Question 7:
Avoid passive voice.
Hint for Question 8:
Who rode the train?
Hint for Question 9:
What affected California's economy?
Hint for Question 10:
Avoid passive voice.

Copyright 2009, TCA, LLC.

307

Copyright 2009, TCA, LLC.

308

Studyguide for the SAT Skill Quiz C: Grammar Usage and Mechanics

Answers!
A

Question 1.
The answer is (B).
Explanation for Question 1:
(B) is the correct choice.
"Later" is correct. "Latter" means the second of two.

Question 2.
The answer is (C).
Explanation for Question 2:
(C) "their" is incorrect.
They're is the correct form.
Question 3.
The answer is (B).
Explanation for Question 3:
(B) its is incorrect.
It's, a contraction for "it is," is correct.
Question 4.
The answer is (C).
Explanation for Question 4:
(C) is incorrect. "Accepted" would be the correct word.
Question 5.
The answer is (C).
Explanation for Question 5:
(C) is incorrect.
"Effected" means "caused"; "affected" means "influenced" or "changed." If a
person is affected by an event, that event had an effect on the person.
Question 6.
The answer is (D).
Explanation for Question 6:
(D) is the best answer.
It eliminates the wordiness.

Copyright 2009, TCA, LLC.

309

Studyguide for the SAT Skill Quiz C: Grammar Usage and Mechanics
Question 7.
The answer is (B).
Explanation for Question 7:
(B) avoids using the passive voice.
Question 8.
The answer is (E).
Explanation for Question 8:
Only choice (E) resolves the ambiguity.
Question 9.
The answer is (A).
Explanation for Question 9:
(A) is correct. The economic effects were substantial.
Question 10.
The answer is (C).
Explanation for Question 10:
(C) is the best choice.

Copyright 2009, TCA, LLC.

310

Writing Lesson #2
Sentence Contruction

Triumph College Admissions


Skill Lesson

Quiz Time!
A

Hints!

Answers!
A

*Throughout this documentation, and the software: College Board and SAT are registered trademarks of the College Entrance Examination Board. PSAT/NMSQT
is a trademark of the College Entrance Examination Board and National Merit Scholarship Corporation. ACT is a trademark of ACT, Inc. None of these entities are
affiliated with the production of, nor endorse these materials.
Copyright 2009, TCA, LLC.

311

Copyright 2009, TCA, LLC.

312

Studyguide for the SAT Skill Review: Sentence Construction

Sentence Construction
The SAT test will test your ability to recognize errors in standard
English conventions and sentence construction. These will include:
Parallel Structure
Misplaced/Dangling Modifiers
Sentence Fragments
Run-ons and Comma Splices
Idioms
Faulty Comparisons
Wordiness

Parallel Structure
A series of words, phrases, or clauses used in a sentence should have
parallel structure. Non-parallel structure is a sentence construction
error.
Examples:
Incorrect: The students read, wrote, and they recited. (The
first two items in the series are single words, while the last item is
a clause.)
Correct: The students read, wrote, and recited. (The three
items are parallel in their construction.)
Incorrect: He was a stickler for spelling, grammar rules, and
using punctuation correctly. (Each item in the series has a different
form.)
Correct: He was a stickler for spelling, grammar, and
punctuation. (The three items are parallel in their construction.)

Copyright 2009, TCA, LLC.

313

Studyguide for the SAT Skill Review: Sentence Construction


Practice Exercise:
Choose the sentence in each pair that has parallel structure.
Question 1.
(A) Activities at the picnic included pitching
horseshoes, riding horses, and softball games.
(B) Activities at the picnic included pitching
horseshoes, riding horses, and playing softball.
The answer is B. All three activities are described in the same form,
a gerund followed by a direct object.
Question 2.
(A) Jessica took classes in karate, tai chi,
and yoga.
(B) Jessica took classes in karate, tai chi,
and learned yoga.
The correct answer is A. All three activities are described using only
a noun.
Misplaced/Dangling Modifiers
A modifying phrase or clause should be placed as close as possible to
the word it modifies. A misplaced modifier makes the meaning of the
sentence unclear.
Examples:
Unclear: I baked cookies for my friends with nuts in them. (The
modifying phrase "with nuts in them" seems to modify the closest noun,
"friends.")
Clear: I baked cookies with nuts in them for my friends. (The
modifying phrase has been moved so that it is closest to the noun it
modifies, "cookies.")
Unclear: We bought sandwiches in a gas station that must have been two
weeks old. (The modifying clause "that must have been two weeks old"
seems to modify the closest noun, "station.")
Clear: We bought sandwiches that must have been two weeks old in a
gas station. (The modifying clause has been moved so that it is
closest to the noun it modifies, "sandwiches.")
A dangling modifier does not modify any noun in the sentence.

Copyright 2009, TCA, LLC.

314

Studyguide for the SAT Skill Review: Sentence Construction


Examples:
Incorrect: Though warned of the storm, no action was taken.
(The modifier "though warned of the storm" does not refer to any noun
in the sentence.)
Correct: Though warned of the storm, officials took no action.
(The phrase modifies "officials.")
Incorrect: To take History 205, History 101 is a prerequisite.
(The modifier "to take history 205" does not refer to any noun in the
sentence.)
Correct: History 101 is a prerequisite for students wishing to
take History 205. (The phrase modifies "students.")
Practice Exercise:
Choose the sentence in which the modifier is used correctly.
Question 1.
(A) I served cake after the graduation
ceremony that I made myself.
(B) I served cake that I made myself after the
graduation ceremony.
The correct answer is B. The modifier "that I made myself" is closest
to the noun it modifies, "cake."
Question 2.
(A) After I had waited for weeks, the letter
finally arrived.
(B) After waiting for weeks, the letter
finally arrived.
The correct answer is A. In Sentence B, the modifier "after waiting
for weeks" does not refer to any noun in the sentence.

Copyright 2009, TCA, LLC.

315

Studyguide for the SAT Skill Review: Sentence Construction


Faulty Comparisons
One kind of faulty comparison occurs when the superlative form of an
adjective or adverb is substituted for the comparative form. Use the
comparative form, such as "better," "more," or "less," when comparing
two people or things. Use the superlative form, such as "best,"
"most," or "least," when comparing three or more.
Examples:
Incorrect: Of the two candidates, the incumbent was the
least nervous. (The comparative form "less" should be used to
compare two candidates.)
Correct: Of the two candidates, the incumbent was less
nervous.
Incorrect: I like apples, pears, and oranges, but I like apples
better . (The superlative form "best" should be used to compare
three fruits.)
Correct: I like apples, pears, and oranges, but I like apples
best.
Another kind of faulty comparison occurs when a thought is not quite
completed.
Examples:
Incorrect: The gifts we bought in town were less expensive than
the mall. (The sentence compares gifts to the mall.)
Correct: The gifts we bought in town were less expensive than
the ones we bought at the mall. (The sentence compares gifts
bought at two different places.)
Mixed metaphors are a third type of faulty comparison.
Examples:
Mixed metaphor: Bill was clouded in a sea of confusion. (The idea of
being "clouded" is related to the sky, not to the sea.)
Better: Bill was drowning in a sea of confusion.

Copyright 2009, TCA, LLC.

316

Studyguide for the SAT Skill Review: Sentence Construction


Practice Exercise:
Choose the correct sentence.
Question 1.
(A) Paulo and Emma sang solos, and Emma's was
better.
(B) Paulo and Emma sang solos, and Emma's was
best.
The correct answer is A. The comparative form is used to compare two
things.
Question 2.
(A) The population of Arizona is older than
it is in California.
(B) The population of Arizona is older than
that of California.
The correct answer is B. The phrase makes clear that two populations
are being compared.
Senetence Fragments
A sentence fragment is a group of words that masquerades as a
sentence. It has capitalization and punctuation but lacks some
essential element of a sentence. Remember, a sentence must have both a
subject and a predicate, and it must express a complete thought.
There are two basic types of fragments.
Type 1 fragments are phrases; they lack a subject, a predicate, or
both. Fragment: Like to eat fresh seafood. (This phrase has a
predicate but no subject). Sentence: I like to eat fresh seafood. (A
subject was added to make a sentence.)
Fragment: At the beach. (This prepositional phrase has neither a
subject nor a predicate.)
Sentence: I like to eat fresh seafood at the beach. (Both a subject
and a predicate were added to make a sentence.)

Copyright 2009, TCA, LLC.

317

Studyguide for the SAT Skill Review: Sentence Construction


Type 2 fragments are clauses that have a subject and predicate but
that do not express a complete thought. Such fragments are introduced
by a subordinating conjunction or a relative pronoun.
Fragment: When I jog in the morning.
Sentence: I jog in the morning. (The subordinating conjunction was
dropped so that the sentence expresses a complete thought.)
Sentence: I listen to music when I jog in the morning. (The fragment
was added to a sentence to make one complete thought.)
Fragment: That Jeff brought back from Mexico.
Sentence: This is the carving that Jeff brought back from Mexico. (The
fragment was added to a sentence to make one complete thought.)
Practice Exercise:
Identify the fragment in each pair.
Question 1.
(A) Because of the weather.
(B) The weather worsened.

The correct answer is A. The fragment is a phrase that has neither a


subject nor a predicate.
Question 2.
(A) Left over an hour ago.
(B) He has arrived.
The correct answer is A. The fragment has a predicate but not a
subject.

Copyright 2009, TCA, LLC.

318

Studyguide for the SAT Skill Review: Sentence Construction


Run-ons and Comma Splices
When two sentences are run together without the needed punctuation
and/or conjunction, the result is a run-on sentence.
Examples:
Run-on: The wind died the kite could not fly. (Two sentences have been
joined with no punctuation or conjunctions.)
Correct: The wind died, so the kite could not fly. (The two
clauses have been joined with a comma and a coordinating conjunction.)
Correct: The wind died; the kite could not fly. (The two
clauses have been joined with a semicolon.)
Correct: When the wind died, the kite could not fly.
(Two clauses have been joined by adding a subordinating conjunction
before the first clause and a comma after it.)
A comma splice is a type of run-on sentence in which two sentences are
joined with only a comma.
Comma Splice: Sarah invited me to dinner, she made lasagna.
Correct: Sarah invited me to dinner, and she made lasagna.
(The two sentences are joined with a comma and a coordinating
conjunction.)

Copyright 2009, TCA, LLC.

319

Studyguide for the SAT Skill Review: Sentence Construction


Practice Exercise:
Choose the sentence in each pair that is written correctly.
Question 1.
(A) Dark clouds gathered, soon it began to
rain.
(B) Dark clouds gathered, and soon it began to
rain.

The correct answer is B. Sentence A is a comma splice.


Question 2.
(A) Ty brought his tennis racquet; Jen loaned
us a can of balls.
(B) Ty brought his tennis racquet and Jen
loaned us a can of balls.

The correct answer is A. Sentence B needs a comma before the


conjunction "and."
Unacceptable Idiom
There are many idiomatic and informal expressions that are considered
unacceptable in academic writing. Here is a short list of some
expressions and constructions that you should avoid.
Incorrect Substitution
"of" for "have"
"on" for "for"
"being as" for "because"
"suspicion" for "suspect"

Copyright 2009, TCA, LLC.

Examples
would of, could of
a protest on civil rights
being as I was late
I suspicioned he was guilty.

320

Studyguide for the SAT Skill Review: Sentence Construction

Non-idiomatic
(incorrect)
angry at (a person)
authority about
capable to
compare against
different than
doubt if
in accordance to
independent from
intend on
plan on
prior than
similar than
superior than
try and

Idiomatic
(correct)
angry with
authority on
capable of
compare to or with
different from
doubt that
in accordance with
independent of
intend to
plan to
prior to
similar to
superior to
try to

Practice Exercise:
Choose the correct sentence.
Question 1.
(A) The second semester of chemistry was very
different than the first.
(B) The second semester of chemistry was very
different from the first.
The correct answer is B.
Question 2.
(A) I plan on going to the party.
(B) I plan to go to the party.
The correct answer is B.

Copyright 2009, TCA, LLC.

321

Studyguide for the SAT Skill Review: Sentence Construction


Wordiness
Wordiness errors usually show up as incorrect choices in the Improving
Sentences and Improving Paragraphs sections. When a single word
expresses an idea as well as a phrase, use a single word.

Examples:
Wordy
at the present time
easily broken
not very long after that
the reason being
living in poverty

Better
now
fragile
soon
because
poor

Practice Exercise:
Choose the correct sentence.
Question 1.
(A) The company has no job openings at the
present time.
(B) The company has no job openings now.
The answer is B.
Question 2.
(A) Classes were held outdoors, the reason
being that the air conditioning was broken.
(B) Classes were held outdoors because the
air conditioning was broken.
The answer is B.

Copyright 2009, TCA, LLC.

322

Studyguide for the SAT Skill Quiz A: Sentence Construction

Quiz Time!
A

Question 1.
After the legislators met for a special session, they finally voted on, approved,
and passing an important amendment to the law.

(A)

they finally voted on, approved, and passing an important


amendment to the law.

(B)

they finally voted on, approved, and passed an important


amendment to the law.

(C)

they finally voted, approved on and passed an important


amendment to the law.

(D)

they finally voted on, approved upon, and passed on an


important amendment to the law.

(E)

they finally voted and approved and passing on an important


amendment to the law.

Copyright 2009, TCA, LLC.

323

Studyguide for the SAT Skill Quiz A: Sentence Construction


Question 2.
One of the worst catastrophes in space travel, the science class read about the
Space Shuttle explosion.
(A)

One of the worst catastrophes in space travel, the science


class read about the Space Shuttle explosion.

(B)

The science class, one of the worst catastrophes in space


travel, read about the Space Shuttle explosion.

(C)

The science class read about the Space Shuttle explosion, one
of the worst catastrophes in space travel.

(D)

The Space Shuttle explosion was read about by the science


class, one of the worst catastrophes in space travel.

(E)

One of the worst catastrophes in space travel, the Space


Shuttle explosion, the science class read about.

Question 3.
Many theatergoers paid a premium for their tickets. A sold-out event.

(A)

Many theatergoers paid a premium for their tickets. A soldout event.

(B)

Many theatergoers paid a premium for their tickets: for a


sold-out event.

(C)

Many theatergoers paid a premium for their tickets, and a


sold-out event.

(D)

Many theatergoers paid a premium for their tickets; it was a


sold-out event.

(E)

Many theatergoers paid a premium for their tickets, because a


sold-out event.

Copyright 2009, TCA, LLC.

324

Studyguide for the SAT Skill Quiz A: Sentence Construction


Question 4.
If I had known that it was going to rain,I would of brought a rain poncho to the
game.
(A)

I would of brought a rain poncho to the game.

(B)

I would had brought a rain poncho to the game.

(C)

I would have brought a rain poncho to the game.

(D)

I would bring a rain poncho to the game.

(E)

I would brought a rain poncho to the game.

Question 5.
Most people in Latin America speak Spanish, some also speak a native Indian
language.

(A)

Most people in Latin America speak Spanish,

(B)

Since most people in Latin America speak Spanish,

(C)

Astonishingly, most people in Latin America speak Spanish,

(D)

Most people in Latin America speak Spanish; however,

(E)

Many people, speaking Spanish in Latin America,

Copyright 2009, TCA, LLC.

325

Studyguide for the SAT Skill Quiz A: Sentence Construction


Question 6.
It is a well-known fact that my grandfather grows the juiciest tomatoes in all of
Greene County.
(A)

my grandfather grows the juiciest tomatoes in all of Greene County.

(B)

my grandfather grows the most juiciest tomatoes in all of Greene


County.

(C)

my grandfather grows the most juicy tomatoes in all of Greene


County.

(D)

my grandfather grows the juicier tomatoes in all of Greene County.

(E)

my grandfather grows the juicy tomatoes in all of Greene County.

Question 7.
Listening to soft music is recommended for patients recovering from surgery
reason being that it speeds up the healing process.

(A)

the reason being that it speeds up the healing process.

(B)

the reason is that it speeds up the healing process.

(C)

because it speeds up the healing process.

(D)

in the event that that it speeds up the healing process.

(E)

due to the fact that it speeds up the healing process.

Copyright 2009, TCA, LLC.

the

326

Studyguide for the SAT Skill Quiz A: Sentence Construction


Question 8.
Although many workers ride the commuter train to work and taking buses , the
city still has inadequate parking facilities.
(A)

Although many workers ride the commuter train to work and


taking buses

(B)

Although many workers ride the commuter train to work and


are taking buses

(C)

Although many workers ride the commuter train to work and


took buses

(D)

Although many workers are riding the commuter train to


work and take buses

(E)

Although many workers ride the commuter train to work and


take buses

Question 9.
We determined how much money we made from the concession sales twenty
percent of the proceeds will finance the band trip to Washington D.C.

(A)

the concession sales twenty percent of the proceeds will


finance

(B)

the concession sales: twenty percent of the proceeds will


finance

(C)

the concession sales, twenty percent of the proceeds will


finance

(D)

the concession sales; twenty percent of the proceeds will


finance

(E)

the concession sales. twenty percent of the proceeds will finance

Copyright 2009, TCA, LLC.

327

Studyguide for the SAT Skill Quiz A: Sentence Construction


Question 10.
Stuffing them with meat and rice, green peppers can make a complete and
tasty meal.

(A)

Stuffing them with meat and rice,

(B)

If you stuff them with meat and rice,

(C)

Because stuffing them with meat and rice,

(D)

Having them stuffed with meat and rice,

(E)

To stuff them with meat and rice,

Copyright 2009, TCA, LLC.

328

Studyguide for the SAT Skill Quiz A: Sentence Construction

Hints!
A

Hint for Question 1:


Find the answer choice that has parallel structure.

Hint for Question 2:


A phrase should be close to the word or words it is modifying.

Hint for Question 3:


Fix that fragment.

Hint for Question 4:


Idioms are not acceptable for writing.

Hint for Question 5:


How should these two ideas be joined?

Hint for Question 6:


How many tomatoes are grown in Greene County?

Hint for Question 7:


Strive for clear and concise sentences.

Hint for Question 8:


Look for an error in parallel structure.

Hint for Question 9:


Correct the run on sentence with punctuation.

Copyright 2009, TCA, LLC.

329

Studyguide for the SAT Skill Quiz A: Sentence Construction


Hint for Question 10:
Who is doing the stuffing?

Copyright 2009, TCA, LLC.

330

Studyguide for the SAT Skill Quiz A: Sentence Construction

Answers!
A

Question 1.
The answer is (B).
Explanation for Question 1:
The answer is B.
The verbs "voted," "approved," and "passed" all need to have parallel structure.

Question 2.
The answer is (C).
Explanation for Question 2:
The answer is C.
It corrects the misplaced modifier by moving it to the end of the sentence.
Question 3.
The answer is (D).
Explanation for Question 3:
The answer is D. Adding subject and verb ("it was") creates a complete sentence,
which can then be separated from the first sentence by a semicolon.

Question 4.
The answer is (C).
Explanation for Question 4:
The answer is C.
People may use idioms in daily speech, but they are not permitted in writing.

Question 5.
The answer is (D).
Explanation for Question 5:
The answer is D.
By adding a subordinating conjunction and punctuation, the comma splice is
corrected.

Copyright 2009, TCA, LLC.

331

Studyguide for the SAT Skill Quiz A: Sentence Construction


Question 6.
The answer is (A).
Explanation for Question 6:
The answer is A. More than two tomatoes are being compared. "Most juicy" in not
the correct superlative of the word "juicy."

Question 7.
The answer is (C).
Explanation for Question 7:
The answer is C.
It eliminates the wordiness.

Question 8.
The answer is (E).
Explanation for Question 8:
The answer is E.
"Ride the commuter train" and "take buses" have parallel structure.

Question 9.
The answer is (D).
Explanation for Question 9:
The answer is D. A semicolon can be used to separate two complete sentences. If
you use a period, as in choice E, the next sentence should begin with a capital
letter.

Question 10.
The answer is (B).
Explanation for Question 10:
The answer is B.
The dangling modifier is now an introductory clause.

Copyright 2009, TCA, LLC.

332

Studyguide for the SAT Skill Quiz B: Sentence Construction

Quiz Time!
A

Question 1.
When I visited the Grand Canyon, it was winter, the canyon was
A
B
C
lightly dusted with snow. No error.
D
E
(A)

(B)

(C)

(D)

(E)

Question 2.
Many people who suffer from arthritis benefit from
A
B
taking pain medication, staying physically active, and to receive massage
D
D
therapy. No error.
E
(A)

(B)

(C)

(D)

(E)

Question 3.
Donna, a new worker in the New York office, told a
A
B
story about visiting Italy during her lunch hour.
No error.
C
D
E
(A)

(B)

(C)

(D)

(E)

Question 4.
Tightened security policies in the United States have created
A
longer waiting periods at airports, the wait to check in can be over one
B
C
D
hour. No error.
E
(A)

Copyright 2009, TCA, LLC.

(B)

(C)

(D)

(E)

333

Studyguide for the SAT Skill Quiz B: Sentence Construction


Question 5.
The coaches, as well as the players, debated which sport
A
B
is the most physically demanding, football or soccer. No error.
C
D
E
(A)

(B)

(C)

(D)

(E)

Question 6.
After vacationing in Tahiti, she decided she loved it
A
B
C
more than any place on earth. No error.
D
E
(A)

(B)

(C)

(D)

(E)

Question 7.
Although the sequel to

the movie was much different than what


A
B
she had expected , she enjoyed it nonetheless . No error.
C
D
E
(A)

(B)

(C)

(D)

(E)

Question 8.
Thomas and I decided that we would spend our summer
A
B
rock climbing and to take a white-water river trip before returning to school in the fall. No error.
C
D
E
(A)

(B)

(C)

(D)

(E)

Question 9.
In an effort to increase cancer awareness as well as promote
A
B
healthy habits, the school sponsored a poster contest. No error.
C
D
E
(A)

Copyright 2009, TCA, LLC.

(B)

(C)

(D)

(E)

334

Studyguide for the SAT Skill Quiz B: Sentence Construction


Question 10.
The students practiced hard all season, as a result,
A
B
their hard work paid off, and
C
D
they won the state competition. No error.
E
(A)

Copyright 2009, TCA, LLC.

(B)

(C)

(D)

(E)

335

Copyright 2009, TCA, LLC.

336

Studyguide for the SAT Skill Quiz B: Sentence Construction

Hints!
A

Hint for Question 1:


How do you join two independent clauses?

Hint for Question 2:


Are all parts of the sentence parallel?

Hint for Question 3:


Check for a misplaced modifier.

Hint for Question 4:


Check punctuation.

Hint for Question 5:


How many things are being compared?

Hint for Question 6:


Check the comparison.

Hint for Question 7:


Check for correct idiomatic expressions.

Hint for Question 8:


Look for an error in parallel structure.

Hint for Question 9:


Read the sentence carefully.

Copyright 2009, TCA, LLC.

337

Studyguide for the SAT Skill Quiz B: Sentence Construction


Hint for Question 10:
Check punctuation.

Copyright 2009, TCA, LLC.

338

Studyguide for the SAT Skill Quiz B: Sentence Construction

Answers!
A

Question 1.
The answer is (C).
Explanation for Question 1:
The answer is C.
The comma splice is corrected by adding a coordinating conjunction.

Question 2.
The answer is (D).
Explanation for Question 2:
The answer is D.
To maintain parallel structure, "to receive" should be written as "receiving." All three
phrases must be in the same form.

Question 3.
The answer is (D).
Explanation for Question 3:
The answer is D.
The misplaced modifier "during her lunch hour" should be placed before "Donna."
Otherwise, the sentence would mean that she visited Italy during her lunch hour. .z
Question 4.
The answer is (C).
Explanation for Question 4:
The answer is C.
This sentence contains a comma splice. You cannot join two complete sentences with
only a comma.

Question 5.
The answer is (C).
Explanation for Question 5:
The answer is C.
When comparing two things, use "better."

Copyright 2009, TCA, LLC.

339

Studyguide for the SAT Skill Quiz B: Sentence Construction


Question 6.
The answer is (D).
Explanation for Question 6:
The answer is D. The sentence contains a faulty comparison. It should be "more than any other place."
Otherwise, the sentence says that she likes Tahiti better than any place, which includes
Tahiti.
Question 7.
The answer is (B).
Explanation for Question 7:
The answer is B.
The correct idiomatic expression is "different from," not "different than."

Question 8.
The answer is (C).
Explanation for Question 8:
The answer is C.
"To take" should be changed to "taking" to be parallel with "climbing."

Question 9.
The answer is (E).
Explanation for Question 9:
The answer is E.
This sentence contains no error. Don't assume that just because the sentence is
long that it contains an error.

Question 10.
The answer is (B).
Explanation for Question 10:
The answer is B.
This sentence contains a comma splice. You cannot join two complete sentences
with only a comma.

Copyright 2009, TCA, LLC.

340

Studyguide for the SAT Skill Quiz C: Sentence Construction

Quiz Time!
A

Question 1.
To bolster their images as charitable enterprises, many corporations
A
B
have initiated relief programs to aid and help victims of natural disasters. No error
C
D
E
(A)

(B)

(C)

(D)

(E)

Question 2.
has
It seems that each one of the ten manufacturers claims that it
A
B
the better quality product on the market . No error
C
D
E
(A)

(B)

(C)

(D)

(E)

Question 3.
During the remodeling all offices will be closed, our temporary
A
offices will be open during regular business hours with special late
B
C
hours on Thursday . No error
D
E
(A)

(B)

(C)

(D)

(E)

Question 4.
The new computer did not improve the performance of the graphics
A
software in any of the ways we had expected , but it did speed up our
B
C
word processing capabilities and increasing our memory. No error
D
E
(A)

Copyright 2009, TCA, LLC.

(B)

(C)

(D)

(E)

341

Studyguide for the SAT Skill Quiz C: Sentence Construction


Question 5.

Listening carefully, the judge instructed the jury to deliberate


A
B
C
carefully before reaching a verdict. No error
D
E
(A)

(B)

(C)

(D)

(E)

Question 6.
The government is altering its stance on what is acceptable Internet content,
web hosting services are taking a closer look at their clients' materials.

(A)

The government is altering its stance on what is acceptable


Internet content,

(B)

The government, altering its stance on what is acceptable Internet


content,
The government is altering its stance on what is acceptable
Internet content, as a result

(C)

(D)

Since the government is altering its stance on what is


acceptable Internet content,

(E)

The government, having altered its stance on what is


acceptable Internet content,

Copyright 2009, TCA, LLC.

342

Studyguide for the SAT Skill Quiz C: Sentence Construction


Question 7.
Hoping to win the contest, the pie was baked to perfection.

(A)

the pie was baked to perfection.

(B)

Robert baked the pie to perfection.

(C)

the pie had been baked to perfection.

(D)

the pie Robert was baking to perfection.

(E)

since the pie Robert baked was to perfection.

Question 8.
Most hiring managers are now taking flexible positions on benefits; the reason
being to allow them to draw from a wide segment of the working
population.

(A)

benefits; the reason being to allow them to draw

(B)

benefits in order to draw

(C)

benefits, the reason is to allow them to draw

(D)

benefits and the reason is to allow them to draw

(E)

benefits; that was allowing them to draw

Copyright 2009, TCA, LLC.

343

Studyguide for the SAT Skill Quiz C: Sentence Construction


Question 9.
Using them cleverly, screwdrivers can perform many household chores.

(A)

Using them cleverly,

(B)

If they are used cleverly,

(C)

Because of cleverness,

(D)

Because of their clever use,

(E)

Having used them cleverly,

Question 10.
We collected fewer than 25 shells when we went and drove to the beach
last weekend.

(A)

fewer than 25 shells when we went and drove

(B)

less than 25 shells when we went and drove

(C)

less than 25 shells when we were driving

(D)

less than 25 shells driving to the beach

(E)

fewer than 25 shells when we went

Copyright 2009, TCA, LLC.

344

Studyguide for the SAT Skill Quiz C: Sentence Construction

Hints!
A

Hint for Question 1:


Look for redundancy.

Hint for Question 2:


Which answer would be best?
Hint for Question 3:
Where does one idea stop and another begin?
Hint for Question 4:
Check for parallel verb structure.
Hint for Question 5:
Check for dangling modifiers.
Hint for Question 6:
Check punctuation.
Hint for Question 7:
Check for dangling modifiers.
Hint for Question 8:
Look for a fragment.
Hint for Question 9:
Check for dangling modifiers.
Hint for Question 10:
Check for redundancy.

Copyright 2009, TCA, LLC.

345

Copyright 2009, TCA, LLC.

346

Studyguide for the SAT Skill Quiz C: Sentence Construction

Answers!
A

Question 1.
The answer is (D).
Explanation for Question 1:
The answer is D.
The phrase "aid and help" is redundant; both words mean the same thing.

Question 2.
The answer is (C).
Explanation for Question 2:
The answer is C.
There are more than two companies; therefore, "best" should be used in place of
"better."
Question 3.
The answer is (A).
Explanation for Question 3:
The answer is A.
A semicolon between the two independent clauses corrects this comma splice.
Question 4.
The answer is (D).
Explanation for Question 4:
The answer is A.
The verb "increasing" should be changed to "increase" to put it in the same form as the
verb "speed."
Question 5.
The answer is (A).
Explanation for Question 5:
The answer is A.
"Listening carefully" is a dangling modifier. It is meant to modify the jury, who are
listening to the judge, but as it is written, it incorrectly says that the judge was listening.
Question 6.
The answer is (D).
Explanation for Question 6:
The answer is D.
Only Answer D corrects the comma splice.

Copyright 2009, TCA, LLC.

347

Studyguide for the SAT Skill Quiz C: Sentence Construction


Question 7.
The answer is (B).
Explanation for Question 7:
The answer is B.
"Hoping to win the contest" is a dangling modifier. It is meant to modify Robert, but as
written, it means the pie was hoping to win the contest.
Question 8.
The answer is (B).
Explanation for Question 8:
The answer is B You must have a complete sentence on both sides of a semicolon. The second part of this
question is a fragment. The phrase "in order to draw" concisely corrects this error.

Question 9.
The answer is (B).
Explanation for Question 9:
The answer is B
"Using them correctly" is a dangling modifier. It is meant to modify the implied word
"people," but as it is written, it means that the screwdrivers are the ones doing the
"using."
Question 10.
The answer is (E).
Explanation for Question 10:
The answer is E.
The phrase "went and drove" is redundant. Also note that "fewer" is correct. Use "fewer"
with numbers and "less" with amounts (fewer sugar cubes, less sugar ).

Copyright 2009, TCA, LLC.

348

Writing Lesson #3

Organization and Development

Triumph College Admissions


Skill Lesson

Quiz Time!
A

Hints!

Answers!
A

*Throughout this documentation, and the software: College Board and SAT are registered trademarks of the College Entrance Examination Board. PSAT/NMSQT
is a trademark of the College Entrance Examination Board and National Merit Scholarship Corporation. ACT is a trademark of ACT, Inc. None of these entities are
affiliated with the production of, nor endorse these materials.
Copyright 2009, TCA, LLC.

349

Copyright 2009, TCA, LLC.

350

Studyguide for the SAT Skill Review: Organization and Development

ORGANIZATION AND DEVELOPMENT


There are two ways the SAT tests whether or not you can recognize that
sentences or paragraphs are logical and consistent:
Faulty Logic or Structure within a Sentence
Lack of Continuity between Sentences and Paragraphs

Faulty Logic or Structure within a Sentence


Coordination and Subordination
Coordination and subordination are methods of combining two or more
ideas in a single sentence.
Examples:
Coordination: Ed watered the neighbors' garden, and I fed their
pets. (Two ideas are combined using a coordinating conjunction.)
Subordination: Ed watered the neighbors' garden while I fed their
pets. (Two ideas are combined using a subordinating conjunction.)
When the link between the combined ideas is not clear, the result is
weak subordination or coordination.
Examples:
Weak Coordination: John had a fender-bender in the parking lot, and
he failed the English test. (The two ideas do not seem to be related.)
Stronger Coordination: John had a terrible day: not only did he have a
fender-bender in the parking lot, but he also failed his test. (The
added clause links the two ideas.)
Two ideas also can be combined using correlative conjunctions, which
are always used in pairs.
Examples:
Either we will practice today, or we will practice Saturday.
Dala's essay was neither as long as mine nor as well written as Jenna's.

Copyright 2009, TCA, LLC.

351

Studyguide for the SAT Skill Review: Organization and Development


Practice Exercise:
Choose the sentence in each pair that best combines two ideas.
Question 1.
(A) The Civil War began and Kansas attained statehood.
(B) In 1861, the Civil War began and Kansas attained statehood.
The correct answer is B. The introductory phrase links the two ideas.
Question 2.
(A) There is only one hose, so either Scott
will mow the lawn, and I will wash the car.
(B) There is only one hose, so either Scott
will mow the lawn, or I will wash the car.
The correct answer is B. Correlative conjunctions such as "either/or"
are always used in pairs.
Unity and Coherence
Occasionally in the Improving Paragraphs section you will be asked to
insert a sentence to improve the unity and coherence of the paragraph.
Such a sentence is sometimes called a transition.
In the paragraph below, Sentence 2 creates coherence by providing a
transition between the sentence that comes before it and the ones that
follow. Try reading the paragraph both with and without Sentence 2.
(1) Growing up near the Florida Everglades, my friends and I loved to
frighten one another with tales of huge, slimy swamp creatures. (2)
But that is hardly surprising, since every region seems to have its
larger-than-life mythical beasts. (3) In Washington State, people have
long told stories of bands of giant ape-like monsters that live in the
shadow of Mount Saint Helens. (4) Farther north one risks running into
the Abominable Snowman, and across the sea in Scotland, it's the Loch
Ness Monster.

Copyright 2009, TCA, LLC.

352

Studyguide for the SAT Skill Review: Organization and Development


Practice Exercise:
Choose the sentence below that should be inserted to provide a
transition between Sentence 1 and Sentence 2.
(1) The role of vitamins in nutrition was only discovered in this
century, but there are now known to be about 40, of which 12 or more
are essential in the diet. (2) But now that their chemical structures
have been identified, chemical names are often used for many of them.

(A)

(B)

Because of the haphazard process of their


discovery, they originally formed a jumbled list of
alphabetic names.
Most of the chemical elements had already
been discovered.

The correct answer is A. Sentence A links the discovery of vitamins with the current
naming convention.

Elaboration
"Elaboration" means adding details to writing to make it more
complete, clear, and interesting. Here are some kinds of details that
writers use to elaborate their ideas.
facts and statistics (who, what, when, where, why, how, how many, etc.)
sensory details (sights, sounds, smells, etc.)
examples and anecdotes
definitions
quotations
reasons and explanations
analyses (comparisons, contrasts, causes, effects, etc.)
Writers choose the kinds of details that best clarify and support their ideas.

Copyright 2009, TCA, LLC.

353

Studyguide for the SAT Skill Review: Organization and Development


Examples:
Humans are taking a toll on the environment. When I was a child, my
family and I would go to the beach and find clean, white sand as far
as the eye could see; the water and beaches were clear and pristine. I
visited the same beach last year to find soda cans, candy wrappers and
cigarette butts half buried in the sand, and the water had the
telltale swirling rainbows of petroleum floating on it. The air
smelled heavy and congested. (Sensory details elaborate the idea
stated in the first sentence.)
Overcrowding in our schools is becoming a major problem. So many new
students are moving into our district that schools are bursting at the
seams. District-wide, the student population grew by 624more than 7
percentlast year alone. Every elementary school holds some of its
classes in portable classrooms, and 29 more portables will be built
this summer. (Facts and statistics support the idea stated in the
first sentence.)

Practice Exercise:
correct answer.

Question 1.
Idea: When the New York Yankees have a losing streak, many fans of other teams
are guilty of schadenfreude .
(A)

The Yankees may be both the most loved


and the most hated team in all of baseball.

(B)

In fact, it may have been the Yankees who


drove English-speakers to borrow the German word meaning "pleasure
taken from the misfortunes of others."

The correct answer is B. Since schadenfreude is a seldom-used loan


word from another language, many readers will need a definition.

Copyright 2009, TCA, LLC.

354

Studyguide for the SAT Skill Review: Organization and Development


Question 2.
Idea: My father says his old hound dog can eat anything without ill effect.
(A)

I personally have seen Muppett devour a


plate of spaghetti marinara, an entire pumpkin pie, a cantaloupe
(rind and all), and a stick of butterthough not all at one time.

(B)

Muppett is 11 years old, weighs 87


pounds, and wakes my father promptly at 6:30 every morning for his
walk.

The correct answer is A. The idea is best supported with examples of


what the dog ate rather than general statistics.

Copyright 2009, TCA, LLC.

355

Copyright 2009, TCA, LLC.

356

Studyguide for the SAT Skill Quiz A: Organization and Development

Quiz Time!
A

(1)The Impressionists were the first generation of painters to


treat the eye as an instrumentas an apparatusand to take its
strengths and weaknesses consciously into account. (2) There were
two chief results. (3) They broke down mixed
colors into their primary tones and communicated them in this
way to the eye.(4) Thus the eye itself does the mixing, and hence
the quite unique freshness of the Impressionists' color effects.
(5)(It should be remembered, though, that individual artists
handled this technique of color separation in a variety of ways
and degreesno real principle was established until Georges
Seurat [1859-91] introduced his Pointillism.) (6) Many paintings
by Impressionists are in museums in Paris. (7) Secondly, in
scientific spirit, they considered the eye as an
instrument, receptive to colors, lines, and dots, but in no way
judging what it sees, either morally or by any other criteria.
Question 1.
Which of the following improves sentence 3?
They broke down mixed colors into their primary tones and
communicated them in this way to the eye.
(A)
(B)
(C)
(D)
(E)

Copyright 2009, TCA, LLC.

They broke down mixed colors


In the first place, they broke down mixed colors
Firstly, they broke down mixed colors
The mixed colors were broke down first
Each and every one of them broke down mixed colors

357

Studyguide for the SAT Skill Quiz A: Organization and Development


Question 2.
Which of the following improves the underlined portion
of sentence 7?
Secondly, in scientific spirit, they considered the eye as an
instrument , receptive to colors, lines, and dots, but in no way
judging what it sees, either morally or by any other criteria.

(A)
(B)
(C)
(D)
(E)

they considered the eye as an instrument


they considered the eye as an impartial instrument
they considered the eye like an instrument
they considered the eye as being an instrument
they considered that the eye was an instrument

Question 3.
Which of the following is the best revision of sentence 4 ?
Thus the eye itself does the mixing , and hence
the quite unique freshness of the Impressionists' color effects.

(A)
(B)
(C)
(D)
(E)

Thus the eye itself does the mixing


Thus the eye its ownself does the mixing
Thus the eye all by itself does the mixing
The mixing, done by the eye itself
The mixture is done with the eye thusly

Question 4.
Which of the following should be deleted to improve the
coherence of the paragraph?
(A)
(B)
(C)
(D)
(E)

Copyright 2009, TCA, LLC.

Sentence 2
Sentence 3
Sentence 4
Sentence 5
Sentence 6

358

Studyguide for the SAT Skill Quiz A: Organization and Development


Question 5.
Which of the following provides the best conclusion to the paragraph ?
(A)
(B)
(C)
(D)

(E)

Copyright 2009, TCA, LLC.

Ironically, several Impressionist painters


were colorblind.
Combine sentences 6 and 7.
The eye mixes colors by combining impulses from the
retina in the brain.
The Impressionists' aim is to be only an eye, and
for them the highest praise was
"Seulement un oeil, mais quel oeil!"
("Only an eye, but what an eye!").
No change.

359

Copyright 2009, TCA, LLC.

360

Studyguide for the SAT Skill Quiz A: Organization and Development

Hints!
A

Hint for Question 1:


Line 7 should give you a clue.
Hint for Question 2:
What is another word for "not judging"?
Hint for Question 3:
How does your choice improve the sentence?
Hint for Question 4:
Which sentence does not belong in this passage?
Hint for Question 5:
Which sentence sums up the main idea of the paragraph?

Copyright 2009, TCA, LLC.

361

Copyright 2009, TCA, LLC.

362

Studyguide for the SAT Skill Quiz A: Organization and Development

Answers!
A

Question 1.
The answer is (B).
Explanation for Question 1:
Since line 7 uses the word "secondly," choice (B)
with its use of the words "In the first place" fits best.
(A) is okay but it does not reflect the order of first then second.
(C) "Firstly" is not a good word.
(D) "were broke" should be "were broken."
(E) "Each and every one" is too wordy.
Question 2.
The answer is (B).
Explanation for Question 2:
(B) is the best choice; the word impartial means "not judging."
Question 3.
The answer is (A).
Explanation for Question 3:
(A) is the correct answer.
(B) "its ownself" is not standard English.
Choices (C) and (D) confuse the meaning of the sentence.
(E) is an awkward construction.
Question 4.
The answer is (E).
Explanation for Question 4:
Sentence 6 does not fit with the discussion of the eye and
the Impressionists.
Question 5.
The answer is (D).
Explanation for Question 5:
Choice (D) reiterates the main themes of the passage.
(A) and (B) have little to do with rest of the passage.
(C) Sentence 6 should be deleted.

Copyright 2009, TCA, LLC.

363

Copyright 2009, TCA, LLC.

364

Studyguide for the SAT Skill Quiz B: Organization and Development

Quiz Time!
A

(1) Submarine technology entered the global


conscience during World War I, but by then,
submarines had already existed for several hundred
years. (2) The first recorded successful test of an
underwater vessel occurred in England in 1620. (3)
A submarine was used successfully in combat
during the American Civil War, and the
Confederate submarine Hunley sank the Union
ship Housatonic. (4) By the beginning of
World War I, submarine technology had
undergone vast improvements, making the
machines sleeker, faster, quieter, and much more
effective, taking international warfare beneath the
ocean's surface. (5) Sunken submarines can be
found in almost all of the world's oceans today.

Question 1.
Which of the following would be most suitable to insert between sentences 2 and 3?

(A)

During the 1700's, 14 patents for submarine designs were granted.

(B)

The German government's interest in submarines grew in the


mid-1800's, spurred on by a builder named Sebastian Bauer.

(C)

In that year, a Dutch inventor named Cornelis Drebbel


navigated a craft beneath the surface of the Thames River.

(D)

Submarine technology has made an enormous impact on global events.

(E)

The first underwater vessel to be used in the United States


Navy was the Plunger , in 1900.

Copyright 2009, TCA, LLC.

365

Studyguide for the SAT Skill Quiz B: Organization and Development


Question 2.
Which of the following best improves the underlined portion of sentence 3?
A submarine was used successfully in combat during the American Civil War,
and the Confederate submarine Hunley sank the Union ship Housatonic.

(A)

A submarine was used successfully in combat during the


American Civil War, and

(B)

Submarines were first used successfully in combat during the


American Civil War when

(C)

Because submarines were first used successfully in combat


during the American Civil War, then

(D)

Despite the fact that a submarine was used successfully in


combat during the American Civil War,

(E)

It so happens that submarines were successfully used in


combat during the American Civil War because

Question 3.
Which of the following sentences should be deleted to improve the focus of the
paragraph?

(A)

Sentence 1

(B)

Sentence 2

(C)

Sentence 3

(D)

Sentence 4

(E)

Sentence 5

Copyright 2009, TCA, LLC.

366

Studyguide for the SAT Skill Quiz B: Organization and Development


Question 4.
Which of the following sentences would be most suitable to insert between
sentences 4 and 5?

(A)

Some experts say that the submarines used in World Wars I


and II altered the courses of both wars.

(B)

Periscopes enabled submarines to see across the surface of


the water while remaining hidden.

(C)

Submarines submerge by filling giant onboard tanks with water.

(D)

The Hunley had to get so close to Housatonic to fire


its torpedo that it also sank in the ensuing explosion.

(E)

When World War I began, Germany had fewer submarines


than any of the world powers.

Question 5.
Which of the following would be a logical elaboration of the statement, "submarine
technology had undergone vast improvements"?

(A)

Scientists have used underwater vessels to conduct deep-sea


research.

(B)

Several hundred years passed between the submarine's


invention and its use in warfare.

(C)

Submarines were used very effectively to break blockades


constructed by opposing forces.

(D)

Developments in electric and diesel motors enabled speeds of


up to 8.7 knots submerged.

(E)

Engineers and builders used scientific principles to make


submarines one of the greatest weapons of the 20th century.

Copyright 2009, TCA, LLC.

367

Copyright 2009, TCA, LLC.

368

Studyguide for the SAT Skill Quiz B: Organization and Development

Hints!
A

Hint for Question 1:


Which sentence provides details about the information presented in sentence 2?

Hint for Question 2:


What is the relationship between the two thoughts in the sentence?

Hint for Question 3:


Which sentence doesn't belong in this passage?

Hint for Question 4:


Look for an answer that makes a transition from the specific information mentioned in
sentence 4 to the general information mentioned in sentence 5.

Hint for Question 5:


Look for an answer that includes examples of "vast improvements."

Copyright 2009, TCA, LLC.

369

Copyright 2009, TCA, LLC.

370

Studyguide for the SAT Skill Quiz B: Organization and Development

Answers!
A

Question 1.
The answer is (C).
Explanation for Question 1:
The answer is C.
This sentence elaborates on the previous sentence, providing more details about the test
in England.

Question 2.
The answer is (B).
Explanation for Question 2:
The answer is B.
The second part of the sentence is added information to the main idea; therefore the
second part makes more sense as a subordinate clause. The word when in answer B
shows the correct relationship.

Question 3.
The answer is (E).
Explanation for Question 3:
The correct answer is E.
This sentence discusses the locations of submarines today while the rest of the paragraph
is about the history of submarines.

Question 4.
The answer is (A).
Explanation for Question 4:
The correct answer is A.
Only answer A is general enough to make a smooth transition between the two sentences.
All of the other options discuss very specific information.

Copyright 2009, TCA, LLC.

371

Studyguide for the SAT Skill Quiz B: Organization and Development


Question 5.
The answer is (D).
Explanation for Question 5:
The answer is D.
This detail supports the idea that technological advancements made submarines faster.

Copyright 2009, TCA, LLC.

372

Studyguide for the SAT Skill Quiz C: Organization and Development

Quiz Time!
A

(1) Brandon elbowed me sharply in the ribs.


(2) "Can you believe we're actually here?" he
whispered hoarsely, grinning and gazing up starryeyed at the rows of seats that towered above our
heads. (3) I conceded that it was an incredible
experience: after spending so much time in English
class toiling over Shakespeare's plays, seeing a play
at the Globe Theatre in London seemed like a
dream fulfilled. (4) I had enjoyed reading All's
Well That Ends Well the best.
(5) We stood there like that for some time,
looking up at the Gallery(a series of wooden,
bleacher-like benches that rose up to our sides and
behind us and faced the still-empty stage. (6) We
had decided to save our money and buy passes to
the Groundling area. (7) As Groundlings, we were
located on the ground between the stage and the
Gallery. (8) The tickets were cheap and the view
was great, we'd have to remain standing for the
entire three-hour performance.
Question 1.
Which of the following best improves the underlined portion of sentence 3?
I conceded that it was an incredible experience: after spending so much time
in English class toiling over Shakespeare's plays, seeing a play at the Globe
Theatre in London seemed like a dream fulfilled.

(A)

Leave it as it is.

(B)

incredible experience, after spending so much time

(C)

incredible experience since after spending so much time

(D)

incredible experience because after spending so much time

(E)

incredible experience after having spent so much time

Copyright 2009, TCA, LLC.

373

Studyguide for the SAT Skill Quiz C: Organization and Development


Question 2.
Which of the following should be deleted to improve the unity of the paragraph?

(A)

Sentence 1

(B)

Sentence 2

(C)

Sentence 4

(D)

Sentence 6

(E)

Sentence 8

Question 3.
What would be the best place to insert the following sentence?
"In Shakespeare's time, only the nobility could afford tickets to the
Gallery; even hard, wooden benches were considered a luxury in those
days."

(A)

Between sentences 3 and 4

(B)

Before sentence 5

(C)

Between sentences 5 and 6

(D)

Between sentences 6 and 7

(E)

After sentence 8

Copyright 2009, TCA, LLC.

374

Studyguide for the SAT Skill Quiz C: Organization and Development


Question 4.
Which is the best way to combine sentences 6 and 7?
We had decided to save our money and buy passes to the Groundling area. As
Groundlings, we were located on the ground between the stage and the Gallery.

(A)

Leave it as it is.

(B)

We had decided to save our money and buy passes to the Groundling area.
We were located on the ground between the stage and the Gallery.

(C)

We had decided to save our money and buy passes to the Groundling area,
and as Groundlings, we were located on the ground between the stage and the Gallery.

(D)

We had decided to save our money and buy passes to the


Groundling area, a spot located on the ground between the stage and the
Gallery.

(E)

We had decided to save our money and buy passes to the Groundling area, as
Groundlings, we were located on the ground between the stage and the Gallery.

Question 5.
Which of the following best improves the underlined portion of sentence 8?
The tickets were cheap and the view was great, we'd have to remain standing
for the entire 3-hour performance.

(A)

the view was great, we'd have to remain standing

(B)

the view was great, and we'd have to remain standing

(C)

the view was great; moreover, we'd have to remain standing

(D)

the view was great, because we'd have to remain standing

(E)

the view was great, but we'd have to remain standing

Copyright 2009, TCA, LLC.

375

Copyright 2009, TCA, LLC.

376

Studyguide for the SAT Skill Quiz C: Organization and Development

Hints!
A

Hint for Question 1:


How are the thoughts in the sentence related?

Hint for Question 2:


Which sentence doesn't contribute to the excited tone in this passage?
Hint for Question 3:
Which sentences discuss seating options?
Hint for Question 4:
Look for the most concise way to combine these two sentences.
Hint for Question 5:
What is the relationship between the two parts of the sentence?

Copyright 2009, TCA, LLC.

377

Copyright 2009, TCA, LLC.

378

Studyguide for the SAT Skill Quiz C: Organization and Development

Answers!
A

Question 1.
The answer is (A).
Explanation for Question 1:
The answer is A.
The original sentence correctly separates the general idea from its explanation
by using a colon.

Question 2.
The answer is (C).
Explanation for Question 2:
The correct answer is C.
This sentence names the narrator's favorite play but does not particularly
contribute to the excited tone in the passage.

Question 3.
The answer is (C).
Explanation for Question 3:
The answer is C.
This sentence elaborates on the previous sentence about the Gallery and sets up
the next sentence about opting for cheaper tickets.

Question 4.
The answer is (D).
Explanation for Question 4:
The answer is D.
Answer D effectively combines these two sentences by deleting unnecessary words.
Answer B is choppy, answer C is wordy, and answer E contains a comma splice.

Question 5.
The answer is (E).
Explanation for Question 5:
The answer is E. The first part of the sentence lays out the good part of being
a Groundling, and the second part states the big drawback. That contrast is best
represented with "but."

Copyright 2009, TCA, LLC.

379

Copyright 2009, TCA, LLC.

380

Writing Lesson #4
Essay Overview

Triumph College Admissions


Skill Lesson

Quiz Time!
A

Hints!

Answers!
A

*Throughout this documentation, and the software: College Board and SAT are registered trademarks of the College Entrance Examination Board. PSAT/NMSQT
is a trademark of the College Entrance Examination Board and National Merit Scholarship Corporation. ACT is a trademark of ACT, Inc. None of these entities are
affiliated with the production of, nor endorse these materials.
Copyright 2009, TCA, LLC.

381

Copyright 2009, TCA, LLC.

382

Studyguide for the SAT Skill Review: Essays

OVERVIEW OF THE NEW SAT ESSAY


The Importance of Writing
Have you ever heard the expression "reading, writing, and
arithmetic"? Writing is one of the famous three R's because it is a
core skill. Writing and speaking are the primary ways we express our
thoughts, feelings, and ideas. It would be impossible for the SAT to
test speaking skills, but it can test writing skills by requiring you
to compose an essay. The College Board strongly believes that writing
is needed for success in both college and the workplace. They also
believe that writing correlates to how well you will perform in
college. Because the SAT is first and foremost a college admission
test, it now includes an essay.
The SAT Essay Question
When your teachers give you written work, they probably call it an
assignment. The SAT test writers are also going to give you a writing
assignment. They will begin with what the test writers call a
"prompt," which introduces the topic you will be asked to write about.
For example, you may be given two quotes on the importance of secrecy.
One author believes that some secrecy is necessary while the second
author believes that secrecy and a free society are incompatible. The
quotes are then followed by your actual assignmentfor example,
"Should we always keep secrets or is keeping secrets harmful?" You are
then asked to take a point of view on this issue. You are free to
support your position with examples from your reading, studies,
experience, or observations.
Is this discussion of prompts and assignments starting to worry you?
Don't let it. Like all SAT essay topics, secrecy is a very general subject.
College Board readers don't care what position you take as long as you have
a point of view that is supported by examples.

Copyright 2009, TCA, LLC.

383

Studyguide for the SAT Skill Review: Essays


The Scoring Guide
Two trained readers will score your essay. Readers include
both high school teachers and college professors. The readers will use
a detailed scoring guide to evaluate your essay. This guide uses a
6-point scale, with 1 being the lowest and 6 being the highest. The
scores from your two readers are then combined for a total score of 2-12.
The College Board scoring guide is designed for its professional
readers. What do phrases such as "insightfully develops a point of
view" and "facility in the use of language" really mean?

Score of 6
The essay effectively and insightfully develops a position on the subject
that is supported by clearly relevant examples, evidence, and logic. The
writer uses excellent critical thinking, and the central idea is clearly
defined and developed.
The essay is well-organized and stays focused on the topic. Ideas are
logically and clearly developed, and paragraphs are carefully constructed
and organized. The essay is a unified presentation of the theses and has a
clear introduction, body and conclusion.
The writer has versatility and mastery in use of the language and a
comprehensive vocabulary. Words are well chosen, and transitional devices
create a smooth flow from sentence to sentence.
The writer uses variety in sentence structure to enhance communication.
The essay has no serious errors in grammar, usage, and mechanics. Accepted
conventions are followed.

Copyright 2009, TCA, LLC.

384

Studyguide for the SAT Skill Review: Essays

Score of 5
The essay effectively develops a position on the subject that is
supported by good and appropriate examples, evidence and reasons. The essay
demonstrates strong critical thinking, and the central idea is welldeveloped.
The essay is well-organized and focused. Ideas are logically developed.
Paragraphs are well-developed and unified. The essay has good development
and closure.
The essay demonstrates ease and comfort using language, with the
appropriate vocabulary.
The essay has a good variety of sentence structure.
The essay has only a few minor errors in grammar and punctuation. Accepted
conventions are followed.

Score of 4
The essay develops a position on the issue, and competent critical thinking
is used. The writer uses generally appropriate examples, evidence and
reasoning. The central idea is developed adequately but may be too
general.

The essay is generally coherent, organized and focused. Some ideas in the
essay may not entirely relate to the subject. The structure of the essay
may be somewhat incomplete.
The use of language in the essay is adequate but inconsistent; the writing
may be occasionally vague or unidiomatic. Generally appropriate vocabulary
is used.
The essay contains some variety in sentence structure, but can be
occasionally repetitive or awkward.
There are occasional minor errors in grammar, usage and mechanics.

Copyright 2009, TCA, LLC.

385

Studyguide for the SAT Skill Review: Essays

Score of 3
The essay develops a position on the subject and shows critical
thinking skills, but may be inconsistent in logic or lack adequate
examples, reasons and details. The central idea is developed too generally
or vaguely.
The essay may be somewhat arbitrarily organized or lack focus. Some ideas
may be irrelevant or not adequately developed.
The essay shows a growing understanding of the use of language, but may
indicate a weak vocabulary. Word choice may be inappropriate.
There are some problems with sentence structure, and only a limited variety
in sentence structure is used.
The essay contains some serious errors in grammar or punctuation and a
number of less serious errors.

Score of 2
The essays position is unclear or limited. The critical thinking in the
essay is weak and not supported by sufficient or relevant examples and
details. The central idea is poorly defined.
The essay is poorly organized and focused. Extraneous information may be
included; the essay lacks coherence. Ideas are not well developed.
The essay shows little command of the language. The vocabulary is limited,
and the word choice is often incorrect.
Sentence structure is frequently problematic.
The essay contains enough serious errors in grammar, usage and mechanics to
make some parts of the essay unclear.

Copyright 2009, TCA, LLC.

386

Studyguide for the SAT Skill Review: Essays

Score of 1
There is either no sustainable position on the subject or a lack of
examples or reasoning to support the position.
The essay is unfocused, arbitrarily organized, and lacks continuity. Ideas
presented in the essay are often not related or developed.
The essay contains serious errors in word usage. Word choice is generally
incorrect or unidiomatic.
There are fundamental flaws in sentence structure.
The essay contains numerous mistakes in grammar, usage, or mechanics that
often interfere with meaning.
The All-Important Point of View
The scoring guide gives the impression that readers will carefully
study and debate your essay. They won't. Believe it or not, readers
will spend no more than two minutes on your essay. In fact, they are
specifically instructed not to reread or analyze an essay.
When College Board readers evaluate your essay, the first and most
important element they will look for is your point of view. Readers
don't care what point of view you take as long as you have one. Your
point of view will then guide your selection of examples.
Essay Length Counts
We've analyzed a large number of essays and have found that the length
of your essay does matter. Your essay sheet will contain 46 lines, 22
on the front and 24 on the back. Students who write 22 lines or less
will most likely receive scores of 1 to 3. In contrast, students who
write 35-46 lines will most likely receive scores of 4-6. There is a
good reason for this: the College Board readers reward your ability to
develop your point of view. A well-developed essay will be given a
high score, while a poorly developed essay will be penalized.

Copyright 2009, TCA, LLC.

387

Studyguide for the SAT Skill Review: Essays


Pause, Think, and Outline
You will have 25 minutes to write your essay. It will take less than a
minute to read the prompt and the assignment. You should then devote
2-3 minutes to planning your essay. Here is a list of questions that
will help guide your planning:
1.
2.
3.
4.

What is my point of view on the topic?


Have I read a novel or short story that I can use to develop my
point of view?
Is there a historic or current event I can use to develop my point
of view?
Do I have a personal experience that I can use to develop my point
of view?

Once you have answered these questions, jot down a brief outline to
guide your writing. The importance of a brief outline cannot be
overemphasized. Students who fail to do this often lose their focus
and digress from their point of view. A lack of focus is one of the
leading reasons essays receive scores of 1, 2, or 3.
Sample Essays with Commentary
This section of our program provides you with carefully selected
examples of essays at each of the six levels. Each essay is
accompanied by a detailed commentary. Taken together, these essays
and commentaries should give you a good idea of what to do and,
equally important, what to avoid.

Level 6 Essay
Topic: Good choices can be costly.
Essay:
"Promise, promise me that you won't tell anyone!" Amanda's plea
pounded on my conscience. "I knew what I did was wrong," Amanda
continued. "But the exam was right there on the teacher's desk. I need
a good grade. I made a copy and then returned it. No one knows." My
hands gripped the quilt on my bed as I listened intently to Amanda
imploring for my support and for my silence. What could I do? What
should I do? Somehow I managed to be calm and composed, even as she
placed our friendship in jeopardy, even as she asked me to compromise
my integrity, even as I wanted to reach across my bed and shake her,
screaming "Why did you do it?"

Copyright 2009, TCA, LLC.

388

Studyguide for the SAT Skill Review: Essays

I was a junior in high school and a member of our school's Honor


Council. Elected by the student body, the Honor Council is responsible
for enforcing our school's honor code. Amanda was a friend, but she
was also a student. Her confession placed me in a terrible
predicament. Sitting on my bed looking at her tear-streaked face I
could feel my conflicting responsibilities bearing down on my chest,
heavy and oppressive. My mind vacillated. I did not know what to do. I
desperately wanted to protect my friend. After all she returned the
exam. No one else knew. No one else would be hurt. But, I was elected
to the Honor Council and I had given my word to enforce our school's
code of ethics.

Finally, after what seemed like an eternity, Amanda left. I promised


we would talk again tomorrow. I spent a long sleepless night thinking
about the importance of friendship and also thinking about the
importance of my responsibility to the Honor Council. What would the
other students think of me? How could I face the other members of the
Honor Council? How could I look at my teachers, at my family? I
finally decided that I had to report Amanda even if it meant losing a
friend, even if it meant being seen as a traitor. So I reported her.
"I hate you!" Amanda screamed at me. "They're giving me a zero on the
exam. I'll fail the course. What college will accept me now? You've
ruined my life!" Amanda's words hurt; they hurt deeply. But I had to
make a choice. I made what I felt was a good choice. But, good choices
can be costly. I lost a friend.

Copyright 2009, TCA, LLC.

389

Studyguide for the SAT Skill Review: Essays

Level 6 Commentary
1. Point of view on the issue
Fully developed: This essay presents a fully developed discussion on a
good but costly choice. Note that the author never digresses from her
topic. Each detail contributes to the overall narration.
Insightful: Insight is one of the hallmarks of a Level 6 essay. Insight
means more than just presenting a story. Demonstrating insight means
clearly presenting different viewpoints. The author of this passage does a
superb job of taking us inside her mind as she grapples with a hard
decision. As readers we clearly understand the conflict she faces
between her loyalty to a friend and her responsibility to uphold the
school's honor code. Outstanding insight also means recognizing the
consequences of taking a stand on a difficult issue. In the final paragraph,
the author does a particularly effective job of showing us that she
understands the consequences of her decision and that she will grow from
this experience.

2. Organization
The essay is well organized, clearly focused and provides the reader with a
smooth progression of ideas. Paragraphs 1 and 2 grab the reader's attention.
The author has a difficult problem. What will she do? In paragraphs 3 and 4
the author goes back and fills in the gaps in her story. We learn details
about the author that help us understand why she is so conflicted. In
paragraph 5 the author reaches a final, irrevocable decision. And finally, in
paragraph 6 the author describes the consequences of her decision.

Copyright 2009, TCA, LLC.

390

Studyguide for the SAT Skill Review: Essays

3. Language
A. Skillful use of language
1. Quotes: The author does an outstanding job of integrating quotes
into her essay. The essay begins with a vivid, attention-grabbing
quote. In the final paragraph, the author once again uses a quote to start a
paragraph, and once again it is appropriate and effective.
2. Parallel structure: The author does a particularly outstanding job
of using parallel structure. In paragraph 2, sentence 4 she vividly conveys her
growing sense of urgency by using a series of phrases all starting with the word
"even."
3. Placement of modifiers: Look closely at sentence 5 in paragraph 3. In the last
three words of the sentence ("heavy and oppressive") the author once again
displays sophistication in the use of language by placing her modifiers after the
word they are modifying. The placement of "heavy and oppressive" helps to
paint an image for the reader that conveys her conflicting emotions.

B. Varied, accurate and apt vocabulary


1. Implore: In the first sentence of the second paragraph, the author
"listened intently to Amanda imploring for my support and for my silence."
Implore means to beg or plead with. The use of the word "implore" perfectly
conveys Amanda's urgent plea for help.
2. Predicament: A predicament is a trying situation, a dilemma. The
use of this word in paragraph 3, sentence 4 accurately conveys the author's
dilemma. While dilemma would work, predicament is a much stronger and more
apt word.
3. Vacillate: The author opens paragraph 4 by saying, "My mind
vacillated." Vacillate means to waver. The author's use of the word "vacillate"
aptly and succinctly conveys the back and forth questioning that is taking place in
her mind.

Copyright 2009, TCA, LLC.

391

Studyguide for the SAT Skill Review: Essays

4. Meaningful variety in sentence structure


The author does a truly outstanding job of employing a meaningful
variety in her sentence structure. This is particularly evident in her use
of questions and short powerful sentences.
1. Use of questions: Paragraphs 2 and 5 contain excellent examples of
how the author uses questions as a tool to vary her sentence structure and convey
the ongoing mental debate that is at the heart of the essay.
2. Short powerful sentences: Sentences do not all have to be long.
Short sentences can be very effective. For example, reread paragraph 5. The fourword
sentence that ends the paragraph packs a powerful punch. The author's
mental debate is over; she has reached a decision.

5. Grammar, usage and mechanics


An essay does not have to be perfect to receive a 6. There can be a few minor
errors. For example, the author uses the conjunction "but" to start two of
the final three sentences in paragraph 6. Although this is a minor
grammatical error, it does not take away from the author's powerful
insights.

Level 5 Essay
Topic: True wisdom means _____.
Essay:
True wisdom means learning from mistakes. I learned this lesson the
hard way. It all started when I was a young girl. My best friend Jen
lived just down the street. We did everything together. We played
together, danced together, and our families even vacationed together.
As we grew older Jen and I became closer or so I thought. High School
meant noticing and being noticed by boys. I had my first boyfriend
when I was a sophomore. I was so happy. I shared everything with Jen.
I told her all about Scott and all about the good times we had. And
when we had our first quarrels of course I told Jen. She knew
everything. I trusted her.

Copyright 2009, TCA, LLC.

392

Studyguide for the SAT Skill Review: Essays

I thought my life was perfect. Scott was my boyfriend and Jen was my
best girlfriend. But I was mistaken. Scott suddenly broke up with me.
I was devastated. But the worst was yet to come.
Heartbroken, I shared my pain with Jen. At first she seemed
empathetic. I cried and she listened. Yet somehow I sensed something
was not quite right. One day I went to the mall. Imagine my shock when
I saw Jen hanging out with Scott. I felt hurt and even worse I felt
betrayed. When I confronted her, Jen said that their relationship
"just happened." She thought it would be alright for her to go out
with Scott. After all Scott and I were no longer a couple. But, it
wasn't alright. It was all wrong!

This painful experience taught me a valuable lesson about wisdom. I


had been nave and was thus easily duped. Wisdom is all about learning
from mistakes. My mistake was thinking that friendship is forever. It
isn't. People can be duplicitous. I leaned this lesson the hard way.

Level 5 Commentary
1. Point of view on the issue
A. Effectively developed versus Fully developed: This essay presents
an effectively developed discussion on the author's belief that
"wisdom means learning from mistakes." Like the Level 6 essay. The
author never digresses from her topic. Each detail contributes to the
overall narration. While the passage is "effectively developed," it is
not fully developed. For example, additional details about her
confrontation with Jen would have strengthened the essay by helping us
understand the basis for her conclusion.

Copyright 2009, TCA, LLC.

393

Studyguide for the SAT Skill Review: Essays

B. Strong Critical Thinking versus Insight: The author


demonstrates strong Critical Thinking skills. In her opening sentence
she states that "true wisdom means learning from mistakes." She then
provides an appropriate example that concretely and effectively
illustrates her point. The author then clearly states the conclusion
she reaches from her mistake: "My mistake was thinking that friendship
is forever." Compare this level of critical thinking with the insight
shown in the Level 6 passage. The author of the Level 6 essay takes us
inside her mind as she grapples with a difficult decision. This mental
debate is missing in the Level 5 passage. The author could have added
a mental debate by providing a more insightful discussion of why
seeing Jen with Scott was "all wrong!"

2. Organization
The essay is well organized, focused, and demonstrates coherence and
progression of ideas. Paragraph 1 clearly states the author's
viewpoint and introduces Jen. Paragraph 2 deepens the relationship
between the author and Jen and introduces Scott. Paragraph 3 describes
the breakup between the author and Scott while also foreshadowing
trouble between the author and Jen. Paragraph 4 describes the author's
reaction to her breakup with Scott and to Jen's betrayal. And finally,
paragraph 5 exp lains the author's mistake and what she learned from
it.

3. Language
A. Facility in the use of language
The author does an effective job of using parallel structure. For
example, in the last sentence of paragraph 1 she writes that, "We
played together, danced together, and our families even vacationed
together." She also does an effective job of conveying her emotions
when she writes (paragraph 4): "I felt hurt and even worse I felt
betrayed."

Copyright 2009, TCA, LLC.

394

Studyguide for the SAT Skill Review: Essays

B. Appropriate vocabulary
1. Nave and duped: "Nave" means innocent and "duped" means easily
fooled. In the second sentence of her concluding paragraph, the
author does a very nice job of correctly using both words to describe
her mistake.
2. Duplicitous: "Duplicitous " means to be dishonest, deceitful.
Using this excellent descriptive word strengthens the author's link
between her view of people and her conclusion about the nature of
friendship.

4. Variety in sentence structure


The author does an excellent job of punctuating her essay with short
declarative sentences. For example, she concludes paragraph 2 by
writing, "I trusted her." And in her concluding paragraph the two word
sentence, "It isn't," provides a particularly emphatic way of
explaining her mistake and introducing her conclusion, "People can be
duplicitous."
5. Grammar, usage and mechanics
This essay is free of errors in grammar, usage and mechanics.

Copyright 2009, TCA, LLC.

395

Studyguide for the SAT Skill Review: Essays

Level 4 Essay
Topic: The wisest person is someone who _________.
Essay:
There are many people in the world who are wise. However, to me
the wisest person is someone who is always prepared. People have to
deal with everyday situations a lot, and yet often find themselves
unprepared unprepared for school, for sports, almost anything. A
wise person to me is someone who is always prepared for something, and
if they aren't they can make a quick adjustment to their plans.
A prepared student is a wise student. If you study for a test,
then you don't have to worry as much if you're going to pass it or
not, or if you do your homework and are prepared for class then you'll
get a good grade. Sometimes, though, people forget things, as everyone
makes mistakes, but often times the student can work themselves out of
the problem. For example, once I forgot to do my history homework and
didn't have my book so I borrowed a book from a friend and did my
homework at lunch, It was a wise choice because our homework was
checked that day, and I was glad I was prepared.

Preparation is also wise in sports. A manager of a baseball team


has to prepare before a game each day. They have to look at who the
opposing team's pitcher is and which of their own athletes is best
against them. They also look at how their team is hitting so they can
decide batting order, and they take into account who's injured or
who's playing well. If a manager didn't really prepare for a game he
or she would send out their players in any order, not caring, and it
could turn out for the worst. Also managers continue to prepare for
pitching changes, and occasionally send in pinch hitters if a batter
is very good against a pitcher. Not being prepared wouldn't be wise as
it could lead to the team losing the game.

Copyright 2009, TCA, LLC.

396

Studyguide for the SAT Skill Review: Essays

People need to be prepared in everyday life for little things too.


One time I was going to bake a cake, only to find out we only had one
egg and I needed two. It would have been wiser if I had looked to see
if we had what I needed before hand so I could have gotten what I
didn't have, but forgot and couldn't go out and buy more. So instead I
adjusted my plans and made brownies instead because I only needed one
egg for those.
To me the wisest person is the person who is always prepared.
From being prepared for schoolwork, or a baseball game or cooking the
wisest thing you can do is be prepared for anything. It also helps to
be flexible and be able to change your plans slightly if things don't
go the exact way they're supposed to. Being wise is being prepared.

Level 4 Commentary
1. Point of View
A. Develops a point of view versus Effectively develops a point of view
This essay argues that "the wisest person is someone who is always
prepared." The author develops this point of view by presenting three
examples. However, at the same time, the author also notes that if a
wise person is not prepared then he/she should demonstrate the ability
to "make a quick adjustment to their plans." This exception detracts
from the author's primary thesis and prevents him from fully or
effectively developing a point of view.
B. Competent critical thinking versus Strong critical thinking
The author demonstrates competent critical thinking by presenting a
thesis and then supporting it with adequate examples. While the
examples are adequate, they are overly generalized. Compare these
examples with the focused example in the Level 5 essay and the
insightful examples in the Level 6 essay.

Copyright 2009, TCA, LLC.

397

Studyguide for the SAT Skill Review: Essays

2. Organization
This essay is generally well organized. The first paragraph states the
author's thesis. The author then provides three examples to illustrate
his thesis. The final paragraph restates the author's position and
summarizes the supporting examples. Although this organization is
competent, it is flawed by examples in paragraphs 2 and 4 of
improvised work. This prevents a smooth and therefore effective
progression of ideas.
3. Language
The author exhibits adequate but inconsistent facility in the use of
language. Although the author's vocabulary is appropriate, he does not
demonstrate facility in the use of language. For example, the author
could have strengthened the second sentence in paragraph 3 by writing,
"The manager of a baseball team must meticulously prepare for each
game." "Meticulous " is a strong descriptive word that aptly describes
the precision needed by managers as they prepare for each game.

4. Some variety in sentence structure


The author does demonstrate some variety in sentence structure. For
example, the third sentence in paragraph 1 utilizes some parallel
structure. However, unlike the Level 5 and Level 6 essays the author
fails to punctuate her essay with short declarative sentences.
5. Some errors in grammar, usage, and mechanics
The passage contains some grammatical errors. For example, in the
third sentence of paragraph 3 the author uses the plural pronoun
"they" to refer to the singular noun "manager" in the preceding
sentence. In addition, the author uses a number of lengthy, wordy
sentences. For example, in paragraph 4 the author states, "It would
have been wiser if I had looked to see if we had what I needed before
hand so I could have gotten what I didn't have, but forgot and
couldn't go out and buy more." A more succinct statement would be, "It
would have been wiser if I had checked out our supply of eggs before
deciding to bake a cake."

Copyright 2009, TCA, LLC.

398

Studyguide for the SAT Skill Review: Essays

Level 3 Essay
Topic: Good choices can be costly.
Essay:
Every human being has their own history filled with choices of
all sorts. The outcome of their decision may better them or hurt them.
Not every choice is easy to make, and many of the most important ones
are costly. We can see this in history during the Civil War and many
times we see it in our own lives.
Robert E. Lee's decision to attack the North on its own soil and
seize Harrisburg was brilliant in theory and with the South dominating
the Eastern theater of the war he expected a decisive victory and an
end to the war. He couldn't have been more wrong. His forces met
General Meade's forces at Gettysburg and defeated his army soundly in
a battle that changed the tide of the war. With heavy casualties
sustained to his already thin ranks he was forced to fight a defensive
war the rest of the way. Lee's decision worked on paper, but ended up
having costly consequences for the Southern effort.

Personally I've seen the downfall of a good choice. One day, one
of my friends asked if I wanted to cheat on a project with them. Being
a rather assiduous student I declined. This friend took the declining
as a sign that I was insinuating that he was a bad person with no
moral judgment and did not speak to me for a while after that. My
decision not to cheat, which was made with good intentions, resulted
in my ending an important friendship in my life with someone with whom
I was close.
Whether it be in history or in the present day, making decisions is
always difficult and some decisions made with good intentions can
sometimes end up causing more harm than good.

Copyright 2009, TCA, LLC.

399

Studyguide for the SAT Skill Review: Essays

Level 3 Commentary
1. Point of View
A. Weak development
The author begins by stating that every human being makes choices that
may be costly. While this point of view is on topic, it is very
general. This lack of focus results in an inadequate level of
development.
B. Weak examples
The author presents two very different examples to support his thesis.
Taken separately each example is interesting and capable of being
expanded. However, the two examples are completely unrelated. This
creates a lack of focus and coherence that weakens the essay.

2. Limited organization
The organization of this essay is limited. The first paragraph
presents a very general thesis. The author then presents two unrelated
examples. The concluding paragraph briefly restates the author's
thesis. This limited organization prevents a smooth and coherent
presentation of ideas that is one of the key characteristics of level
4, 5, and 6 essays.

3. Language
The essay displays weak vocabulary and inappropriate word choice. For
example, in paragraph 2, the author states that Lee's defeat had
"costly consequences" for the South. A stronger choice and more apt
choice of words would be to write that Lee's defeat had disastrous
consequences for the South. The author also displays inappropriate
word choice in the third paragraph when he writes: "Being a rather
assiduous student I declined." Assiduous means hardworking, diligent.
"Rather" is a poor choice because it weakens the meaning of assiduous.
A more forceful statement would be: "Since I take pride in being an
assiduous student, I firmly declined."

Copyright 2009, TCA, LLC.

400

Studyguide for the SAT Skill Review: Essays

4. Problems in sentence structure


The author fails to use variety in sentence structure. In addition, he
uses a number of lengthy, awkward sentences. For example, the opening
sentence in the second paragraph could be split into the following
three sentences: "Robert E. Lee boldly decided to invade the North and
seize Harrisburg. Although a gamble, the strategy could work. Since
the South was dominating the Eastern theatre a decisive victory on
Union soil could end the war."
5. Accumulation of grammatical errors
The passage contains a number of grammatical errors. For example, the
author uses a confusing sequence of pronouns in the second and third
sentences of paragraph 2 when he writes: "He couldn't have been more
wrong. His forces met General Meade's forces at Gettysburg and defeated
his army soundly in a battle that changed the tide of the war." As it is
currently written it sounds as if General Lee defeated his own army. A
judgment. His army clashed with General Meade's forces at Gettysburg.
Meade won a pivotal victory that forever changed the war."

Level 2 Essay
Topic: Good choices can be costly.
Essay:
Decisions aren't always hard to make. I disagree that decisions
are never easy because it is just common sense. Some decisions have
obvious answers, like choosing your favorite ice cream or deciding
whether or not you like baseball. Not all decisions are easy because
there are some that are difficult to make.
There are many examples of this. For example, choosing not to smoke
may lose you friends, but at the same time it will keep you healthy.
This decision cost you friends in exchange for health. Another example
of a good decision being costly is when America rebelled against Great
Britain in the 1700's. This good decision led to America's great
independence. But it cost many lives in order to get there.
Good decisions don't always have to be costly. This is just common
sense as well. Decisions aren't always hard and good decisions can
sometimes be costly. It just depends on the circumstance.

Copyright 2009, TCA, LLC.

401

Studyguide for the SAT Skill Review: Essays

Level 2 Commentary
1. Vague point of view
Reread the first and the last paragraphs in the essay. What do you
think the topic is? Is the topic, "decisions aren't always hard to
make?" Or is the topic, "Not all decisions are easy?" In reality the
topic of this essay is supposed to be: "Good choices can be costly."
The author's failure to stick to the topic results in a vague point of
view and thus a seriously limited essay.

2. Poor organization
The essay begins with a vague introduction. The author then presents
two examples that are poorly developed. There is little coherence and
no progression of ideas. The conclusion, like the introduction, is
vague and lacks focus.
3. Little facility in the use of language
The author displays very little facility in the use of language. Sentences
follow sentences in a disordered parade of ideas. There are no descriptive
adjectives or interesting word choices.

4. Problems in sentence structure


The author uses weak, choppy sentences at the end of the third
paragraph. A better sentence structure would be, "This good decision
led to American independence; however, freedom is neither easy nor
inexpensive. The colonists paid a heavy price in lives for their
independence."
5. Serious errors in grammar
Look at the last sentence in the second paragraph. Can you spot the
grammatical error? The subject and verb must agree. The sentence
should read: "This decision costs (not cost) you friends...."

Copyright 2009, TCA, LLC.

402

Studyguide for the SAT Skill Review: Essays

Level 1 Essay
Topic: Good choices can be costly.
Essay:
I decided to bring a puzzle up here to school one day to let my fellow
classmates try and solve it. It was a good idea because its funny
watching them try and solve it and fail. But it was also bad because
people did not wish to part with it and would try anything to keep it
longer. Also some teacher got upset when they saw students trying so
hard to solve it and not on the class,

Level 1 Commentary
1. No viable point of view
This one-paragraph essay lacks a viable point of view. Short, oneparagraph essays almost always receive a score of 1. The reasons are
obvious. It is impossible to state a point of view, illustrate it and
draw conclusions in one short paragraph.
2. Absence of organization
The essay is unfocused. There is no introduction or conclusion. The author
simply provides a brief vignette.
3. Absence of vocabulary
The passage lacks descriptive vocabulary. For example, the author tells us
that "some teachers got upset." Upset is a vague descriptive word. Other
word choices that would better describe the teacher's reaction could include
"frustrated," "aggravated", and "exasperated."

Copyright 2009, TCA, LLC.

403

Studyguide for the SAT Skill Review: Essays

4. Severe flaws in sentence structure


The essay contains four declarative sentences. As a result, there is no
variety in sentence structure.
5. Errors in grammar
Can you spot the grammatical errors in the final sentence? First, the author
uses the plural pronoun they to refer to the singular subject teacher. This
part of the sentence should read, "In addition, some teachers got upset when they
saw..." The same sentence contains a second serious error. The phrase "and
not on the class" is too vague. It should read: "when they should have been
paying attention to the lesson."

Copyright 2009, TCA, LLC.

404

Writing Lesson #5
Point of View

Triumph College Admissions


Skill Lesson

Quiz Time!
A

Hints!

Answers!
A

*Throughout this documentation, and the software: College Board and SAT are registered trademarks of the College Entrance Examination Board. PSAT/NMSQT
is a trademark of the College Entrance Examination Board and National Merit Scholarship Corporation. ACT is a trademark of ACT, Inc. None of these entities are
affiliated with the production of, nor endorse these materials.
Copyright 2009, TCA, LLC.

405

Copyright 2009, TCA, LLC.

406

Studyguide for the SAT Essay Point of View

Point of View and Critical Thinking


Point of View
A point of view is a standpoint on an issue or problem. Your point of
view is your own opinion. Consider these sentences.
"Michelangelo's paintings and sculptures had a profound
impact on Renaissance art."
"Donatello created sculptures during the Renaissance period of art."
Both sentences above discuss Renaissance-period artists. The
difference between them, however, is that the first sentence states
the author's point of view, while the second sentence merely states a
well-known fact that has been known for centuries.
A clearly stated point of view tells the reader exactly what you think
about an issue. Your point of view cannot be right or wrongit is
your own opinion.
To create a point of view, you must have strong convictions about your
statement. By truly believing in what you have written, your statement
is much easier to write about and your argument will be much more
convincing. The key to writing a good point of view essay for the SAT
is to support your statement with examples from:
Personal experience
Literature
Historical events
These sources help provide insight into the topic you are writing
about. By providing insightful examples, you are strengthening your
argument and making your point of view seem more convincing.

Copyright 2009, TCA, LLC.

407

Studyguide for the SAT Essay Point of View


The Personal Essay
To write a personal essay, you will tie your own experience to the
prompt in order to prove why your point of view is correct. Personal
experience can be taken from relationships with your family and peers,
school, work, camp, or anything else that you have encountered in your
life. As long as your experience relates to the topic and provides
insight into how you view the issue, you can use it.
Tying in Literature
When writing your essay, you can use examples from literature to
support your point of view. To do this, you will explain the
significance of a piece of work in relation to the essay prompt. It is
important to establish a clearly stated point of view in response to
the prompt, and then use specific examples from literature to explain
how your point of view is correct.
Historical Events
In response to an essay prompt, you may provide an example of a
historical event that directly correlates with your point of view on
the issue. You may use a specific part of an event, the relationships
between key players in the event, or the entire event as an example to
support any claims you made.
Writing the Essay
Whichever type of example you decide to use, you must continuously
stick to your point of view(even if you change your mind halfway
through. Your grade will be based on how well you can defend a clearly
established point of view.

Copyright 2009, TCA, LLC.

408

Studyguide for the SAT Essay Point of View


Sample Prompts
1. Carefully consider your point of view in response to the
quotation and assignment below:
"Although the world is full of suffering, it is full also of the
overcoming of it." Helen Keller
Assignment:
Can people overcome suffering to choose their own destiny? Write and
develop an essay in which your point of view on this issue is clearly
defined. Support your point of view with examples and reasoning taken
from your studies, personal experience, or observations.
What is your point of view on this issue?

Sample:
Our destiny is not something that happens to us despite our actions
and attitudes. Everything we do(the way we interact with people, the
way we view obligations to family, friends, work, and ourselves, and
the way we react to adversity and obstacles thrown in our path is
within our ability to change. If we take the time to look around us,
we see that people can and do overcome suffering to choose their own
destiny.
Note: This sample response is NOT an introduction. It is simply an
example of the "mental conversation" taking place in the writer's head
while attempting to choose a point of view on the subject.

Copyright 2009, TCA, LLC.

409

Studyguide for the SAT Essay Point of View

2. Carefully consider your point of view in response to the


quotation and assignment below:
"A slip of the foot you may soon recover, but a slip of the tongue you
may never get over." Benjamin Franklin
"A lie can travel halfway around the world while the truth is putting
on its shoes." Mark Twain
Assignment:
Is slander (lie intended to damage a person's reputation) ever
justified? Write and develop an essay in which your point of view on
this issue is clearly defined. Support your point of view with
examples and reasoning taken from your studies, personal experience,
and observations.
What is your point of view on this issue?

Sample:
Slander is an action that, more often than not, comes back to hurt its
originator more than it hurts its intended target. Slander can ruin
careers, reputations, and even entire lives. The cost of such an
action is far greater than anything that can be gained from it; people
who use slander to attack another person have no justification for
doing so.

Copyright 2009, TCA, LLC.

410

Studyguide for the SAT Essay Point of View

3. Carefully consider your point of view in response to the


quotation and assignment below:
"Forget injuries; never forget kindnesses." Confucius
Assignment:
Should wrongdoings always be forgiven? Write and develop an essay in
which your point of view on this issue is clearly defined. Support
your point of view with examples and reasoning taken from your
studies, personal experience, and observations

What is your point of view on this issue?

Sample:
Life is far too short for every person to dwell on the mistakes and
misdeeds of the people they interact with. If everybody carried around
all of the anger and resentment they've ever felt, they wouldn't be
able to forge new relationships and make old ones continue to work.
People make mistakes and do others wrong; it is up to each individual
to forgive and forget.

Copyright 2009, TCA, LLC.

411

Studyguide for the SAT Essay Point of View


4. Carefully consider your point of view in response to the
quotation and assignment below:
"Imagination is more important than knowledge..." Albert Einstein
Assignment:
Can books teach you everything you need to know? Write and develop an
essay in which your point of view on this issue is clearly defined.
Support your point of view with examples and reasoning taken from your
studies, personal experience, and observations.
What is your point of view on this issue?

Sample:
Books can teach us many things and are a useful tool in disseminating
information to many people across many generations. However, books are no
substitute for experience, emotion, and imagination. Without these things,
humankind would never have reached the point we are at today. Books only
provide us with a very limited view of our world.

Copyright 2009, TCA, LLC.

412

Studyguide for the SAT Essay Point of View


5. Carefully consider your point of view in response to the
quotation and assignment below:
"It is a mistake to try to look too far ahead. The chain of destiny
can only be grasped one link at a time." Sir Winston Churchill
Assignment:
Is patience the greatest human virtue? Write and develop an essay in
which your point of view on this issue is clearly defined. Support
your point of view with examples and reasoning taken from your
studies, personal experience, and observations.

What is your point of view on this issue?

Sample:
While patience is indeed a great virtue, it is not the greatest.
Patience is essentially a passive act in order to be patient, a
person must sit still without acting. Far greater virtues are the
ability to use foresight to predict the future and motivation to
manipulate the future.

Copyright 2009, TCA, LLC.

413

Studyguide for the SAT Essay Point of View


Choosing Thesis Statements
The thesis statement is the sentence that states your point of view on
an issue. A thesis statement should be one complete sentence. It is
usually found near the beginning of your essay. It's important to put
the thesis near the beginning of your essay so that your point of view
does not get lost in the middle of your other ideas.

For each of the following sample prompts, choose the thesis statement
that works best:
"Although the world is full of suffering, it is full also of the
overcoming of it." Helen Keller
Assignment:
Can people overcome suffering to choose their own destiny? Write and
develop an essay in which your point of view on this issue is clearly
defined. Support your point of view with examples and reasoning taken
from your studies, personal experience, or observations.
Which thesis statement would work best in stating a point of view?
(A)

Most people cannot overcome suffering, but sometimes very


remarkable people do.

(B)

Fiction is full of people who make choices about where


they're going.

(C)

Suffering strengthens character and gives people the


motivation to shape their own future.

The correct answer is C. The third thesis statement relates directly


to the issue at hand. The point of view of the author is clearly
indicated in this statement. Based upon this statement, which is one
complete sentence, the author can write a full essay. The thesis
statement tells the reader that people can shape their own future,
that they can "triumph over adversity," to which the original issue
refers.
Answer A takes both sides of the issue; it does not decide on just one
point of view. Answer B is totally unrelated to the issue and does not
discuss suffering or destiny in any form; it indirectly refers to a
geographic or philosophical direction.

Copyright 2009, TCA, LLC.

414

Studyguide for the SAT Essay Point of View


"A slip of the foot you may soon recover, but a slip of the tongue you
may never get over." Benjamin Franklin
"A lie can travel halfway around the world while the truth is putting
on its shoes." Mark Twain
Assignment:
Is slander (lie intended to damage a person's reputation) ever
justified? Write and develop an essay in which your point of view on
this issue is clearly defined. Support your point of view with
examples and reasoning taken from your studies, personal experience,
and observations.
Which thesis statement would work best in stating a point of view?

(A)

While lying to protect a friend can have


its place in time, slandering a friend is never the right thing to do.

(B)

Politicians often use slander against


their opponents, which can be productive. However, slander itself is
bad and should not be used.

(C)

Slander is very common in every aspect of


American social and political life.

The correct answer is A. The first statement is one complete thought.


The statement relates to the topic(slander(and clearly provides the
author's point of view on the topic. In addition, the statement leaves
room for the author to relate the virtues of friendship in the absence
of slander back to the quotes by Benjamin Franklin and Mark Twain.
Statement B takes both sides of the issue. A thesis statement needs to
give the author's opinion one way or another, not both. Answer C does
not state the author's opinion regarding the justification of slander
at all. It does state that slander is common, but it doesn't state
whether slander is a justifiable action.

Copyright 2009, TCA, LLC.

415

Studyguide for the SAT Essay Point of View

"Forget injuries; never forget kindnesses." Confucius


Assignment:
Should wrongdoings always be forgiven? Write and develop an essay in
which your point of view on this issue is clearly defined. Support
your point of view with examples and reasoning taken from your
studies, personal experience, and observations
Which thesis statement would work best in stating a point of view?

(A)

Forgiving a person is one thing;


forgetting a wrongdoing is another.

(B)

There are always ways to forgive a person


for something he or she has done wrong.

(C)

Sometimes you can forgive a person, but


sometimes a person cannot be forgiven.

The correct answer is B. Answer B provides a thesis statement that


clearly states that people should be forgiven for their wrongdoings.
The sentence parallels the structure of the prompt by using the adverb
"always." The thesis leaves room to relate back to the quote and
provide personal, historical, or literary support.
Choice A does not answer the question asked in the prompt. Choice C
takes both sides of the issue(a big mistake in SAT essay writing.

Copyright 2009, TCA, LLC.

416

Studyguide for the SAT Essay Point of View


"Imagination is more important than knowledge..." Albert Einstein
Assignment:
Can books teach you everything you need to know? Write and develop an
essay in which your point of view on this issue is clearly defined.
Support your point of view with examples and reasoning taken from your
studies, personal experience, and observations.

Which thesis statement would work best in stating a point of view?


(A)

Learning is best achieved through a


variety of sources(print media, listening, and hands-on experience;
books are not enough.

(B)

Newspapers and books can provide people


with a supportive learning environment.

(C)

Daily use of books will teach you


everything you need to know, but you need to learn some things from
hands-on experience as well.

The correct answer is A. The first statement takes a stand on the


issue presented. The thesis statement tells the reader that the author
believes that books are not the only source people need to find out
everything they need to know.
The second statement, choice B, does not relate back to the issue,
even though it contains the word "books." The issue presented
discusses whether or not books can teach you everything, not whether
books provide you with a supportive learning environment. While answer
C is one that most people would probably agree with, it takes both
sides of the issue and is therefore bad for the purposes of this
exercise.

Copyright 2009, TCA, LLC.

417

Studyguide for the SAT Essay Point of View


"It is a mistake to try to look too far ahead. The chain of destiny
can only be grasped one link at a time." Sir Winston Churchill
Assignment:
Is patience the greatest human virtue? Write and develop an essay in
which your point of view on this issue is clearly defined. Support
your point of view with examples and reasoning taken from your
studies, personal experience, and observations.

Which thesis statement would work best in stating a point of view?

(A)

Patience is humankind's greatest virtue,


as is sympathy.

(B)

Only when faced with a dilemma do humans


make good decisions.

(C)

Empathy is a human's greatest virtue.

The correct answer is C. It directly relates to the issue being


discussed. While the answer does not include the virtue of "patience,"
the author shows through his or her thesis statement that patience is
not the greatest human virtue.
Answer A is a little tricky. The author takes two points of view in
this statement: If patience is humankind's greatest virtue, then
sympathy cannot be the greatest virtue as well. Answer B is
completely unrelated to the topic. A dilemma and making good decisions
have nothing to do with patience in this context.

Copyright 2009, TCA, LLC.

418

Studyguide for the SAT Essay Point of View

Choosing Insightful Examples


When you create your own point of view and thesis statement concerning
an issue, you have to provide examples to back up your point of view.
Examples strengthen your argument and enable the reader to see why
your point of view is correct. To persuade readers to consider your
viewpoint, you most prove your point of view.
Examples can be taken from personal experience, literature, or
historical events. The primary role of examples in your essay is to
show that you are able to think critically about the issue presented
and relate the issue to different sources of information.
You demonstrate critical thinking and insight in your essay by showing
that you have a deep, clear understanding of the topic and have
reflected on it from different perspectives in developing your own
point of view. When choosing your examples, try to be as fresh and
original as you can. Remember, the readers are scoring hundreds of
essays, and you want yours to stand out (in a good way!).

Copyright 2009, TCA, LLC.

419

Studyguide for the SAT Essay Point of View


Look at the following prompts and examples. Choose the example that shows the best use
of critical thinking and insight.
"Although the world is full of suffering, it is full also of the
overcoming of it." Helen Keller
Assignment:
Can people overcome suffering to choose their own destiny? Write and
develop an essay in which your point of view on this issue is clearly
defined. Support your point of view with examples and reasoning taken
from your studies, personal experience, or observations.
Which of the following examples shows the best use of critical
thinking and insight?

(A)

Abraham Lincoln grew up in a very poor


family. Through hard work and dedication, Lincoln learned to read and
write, doing all he could to educate himself. After several political
losses, he won the race for President of the U.S. in 1860.

(B)

In the play by Shakespeare, Prince Hamlet


struggles with his anger over his uncle Claudius' murder of Hamlet's
father, and Claudius' quick marriage to Hamlet's mother. Hamlet slips
deeper and deeper into insanity and finally resolves to kill Claudius
in revenge.

(C)

People blame their parents for everything,


which is why parents should not be responsible for how their children
turn out.

The correct answer is A. This example explains how Abraham Lincoln


suffered in poverty as a child but was able to overcome circumstances
through his own perseverance. This directly relates to the issue of
suffering and choosing one's own destiny, which shows insight into the
topic.
Choice B is not an example of a person overcoming suffering. Rather,
Hamlet's grief and anger consume him and eventually drive him crazy.
His resolution to seek revenge by murdering Claudius results in a
chain of events that ends in tragedy.
Choice C does not talk about the individual at all. Instead it focuses
on the parents and their role in a child's development. If the author
has an idea of how this example relates to the thesis, it is not clear
here.

Copyright 2009, TCA, LLC.

420

Studyguide for the SAT Essay Point of View


"A slip of the foot you may soon recover, but a slip of the tongue you
may never get over." Benjamin Franklin
"A lie can travel halfway around the world while the truth is putting
on its shoes." Mark Twain
Assignment:
Is slander (lie intended to damage a person's reputation) ever
justified? Write and develop an essay in which your point of view on
this issue is clearly defined. Support your point of view with
examples and reasoning taken from your studies, personal experience,
and observations.
Which of the following examples shows the best use of critical
thinking and insight?

(A)

Once in a while it is okay to talk badly


about someone.

(B)

My friend Linda did not make the


softball team because the coach thought that she was a terrible
player. Linda then thought it was okay to talk badly about the coach,
since he passed her over for the team.

(C)

When my ball bounced into the street, I


ran after it. A driver swerved to avoid hitting me, and yelled out the
window, "Get out of the way!"

The correct answer is B. Slander means damaging a person's reputation


by lying or talking badly about them, which is what example B
addresses. Notice that the example is very specific, not only
mentioning a person's name, but also including a location and
description of the event. When creating your own example, make sure
that you provide the reader with enough information to understand the
whole situation you are using as an example.
Answer A is a thesis, not an example. Remember: Examples support a
point of view, not state it. While it may be an example for another
issue, answer C has nothing to do with slander. The driver was yelling
angrily, but he was not lying about the other person.

Copyright 2009, TCA, LLC.

421

Studyguide for the SAT Essay Point of View

"Forget injuries; never forget kindnesses." Confucius


Assignment:
Should wrongdoings always be forgiven? Write and develop an essay in
which your point of view on this issue is clearly defined. Support
your point of view with examples and reasoning taken from your
studies, personal experience, and observations
Which of the following examples shows the best use of critical
thinking and insight?
(A)

My father left when I was two. For years


after that, my mother told me that he had to work in a different town.
By the time I turned eight, it was obvious that my father was not
coming back. I knew my mom was only trying to protect me and keep me
from feeling abandoned, so I forgave her for her lie. Forgiving my
father for leaving, however, was not possible.

(B)

Sometimes people do really bad things in


the hopes that they will someday be better off. Napoleon Bonaparte
killed many people in his efforts to conquer other lands. The world
has not forgotten his actions, which affected much of modern-day
Europe.

(C)

I try not to do anything wrong. In fact, I


don't even step on ants when they are in the house. I bring them
outside so that they can live where they want. By not doing any harm
to others, hopefully I can avoid having to be forgiven for anything.

The correct answer is A. This provides a clear example that is


specific to the issue at hand. The example defends the point of view
that wrongdoings should not always be forgiven.
Answer choice B relates somewhat to the thesis statement but does not
provide a concrete example to defend the author's point of view. The
world may not have forgotten Napoleon, but this still does not show
how wrongdoings have or have not been forgiven in this instance.
Answer C is not relevant to the issue. The author does not address the
question.

Copyright 2009, TCA, LLC.

422

Studyguide for the SAT Essay Point of View


"Imagination is more important than knowledge..." Albert Einstein
Assignment:
Can books teach you everything you need to know? Write and develop an
essay in which your point of view on this issue is clearly defined.
Support your point of view with examples and reasoning taken from your
studies, personal experience, and observations.
Which of the following examples shows the best use of critical
thinking and insight?
(A)

Books taught me how to read, write, and


even how to play the piano. There are books available to teach
almost every skill we need to accomplish things in our day-to-day
lives.

(B)

My favorite book is Camille by


Alexander Dumas. Many elements in Camille are things that we
see and experience regularly in everyday life.

(C)

In To Kill a Mockingbird by Harper


Lee, Scout and Jem learn an important lesson about racism and
integrity through their father Atticus Finch's actions. Only by
experiencing the trial do Scout and Jem learn how destructive racist
attitudes can be and how difficult and rewarding it can be to do the
right thing.

The correct answer is C. The author gives specific examples of things


Scout and Jem could not learn in a book. Just by reading this example,
we can tell that the author feels that books are insufficient to
convey all knowledge.
Choice A includes things that the author has learned from books, but
it does not encompass everything, nor does it provide a response to a
point of view on the issue. Answer choice B does not provide an
example in relation to the issue. This choice is simply stating what
the author's favorite book is, not relating back to the thesis
statement.

Copyright 2009, TCA, LLC.

423

Studyguide for the SAT Essay Point of View


"It is a mistake to try to look too far ahead. The chain of destiny
can only be grasped one link at a time." Sir Winston Churchill
Assignment:
Is patience the greatest human virtue? Write and develop an essay in
which your point of view on this issue is clearly defined. Support
your point of view with examples and reasoning taken from your
studies, personal experience, and observations.

Which of the following examples shows the best use of critical


thinking and insight?

(A)

The greatest gift to humankind is the


invention of the Internet. This amazing tool puts incredible power
at our fingertips, giving us access to an infinite number of valuable
resources from around the world.

(B)

My sister Laurie is four years older than


I am. Growing up, Laurie always had freedoms that I didn't. She could
stay up later, received more allowance, and wasn't limited to PG-rated
movies. Sometimes I wanted to scream that it was unfair, but I knew
that she had those advantages because she was older and more mature.
It wouldn't help to throw a tantrum; instead, I decided to wait until
I was old enough to enjoy the freedoms Laurie had earned.

(C)

Our society has evolved so that we rarely


have to wait for anything anymore. We have one-hour photo developing
and laundry services, we can research information and purchase items
without leaving the house. Because of this immediate access to things,
we have become less patient.

The correct answer is B. The author uses experience with an older


sibling to illustrate the idea that deciding to be patient and seeing
that decision through was a difficult, yet necessary step in the
maturing process. This is a good example because nothing the writer
could have done would have won him or her the freedoms Laurie had.
Answer A is completely unrelated to the issue. The author indicates
that the Internet is the greatest gift to humankind. This is a
material thing, not a virtue. In answer C, patience is discussed, but
the example does not illustrate whether the author believes that
patience is our greatest virtue.

Copyright 2009, TCA, LLC.

424

Writing Lesson #6

Organization and Focus

Triumph College Admissions


Skill Lesson

Quiz Time!
A

Hints!

Answers!
A

*Throughout this documentation, and the software: College Board and SAT are registered trademarks of the College Entrance Examination Board. PSAT/NMSQT
is a trademark of the College Entrance Examination Board and National Merit Scholarship Corporation. ACT is a trademark of ACT, Inc. None of these entities are
affiliated with the production of, nor endorse these materials.
Copyright 2009, TCA, LLC.

425

Copyright 2009, TCA, LLC.

426

Studyguide for the SAT Essay Organization and Focus

Organization and Focus


Having a well-organized essay is crucial in order to get a good score
on the SAT writing section. A succinct and effective essay requires
good planning. You will have only 25 minutes to read the essay prompt,
plan what you want to write, write the essay, and edit your work.
Managing Your Time
It's getting down to the last couple minutes of the SAT essay, and
you've only written two paragraphs. Your heart is racing, and your
hand is cramped from scribbling down a confusing mess of words. "I
could have avoided this, if only I'd practiced managing my time
better," you think.
Managing time is as important as writing an organized essay. You can
have the best ideas in the world, but if you can't transfer those
ideas to paper, you won't score well on the SAT essay. The good news
is that you can learn to manage your time wisely. There are three main
categories to managing your time wisely: preparing to write, writing
the essay, and editing your work.
Preparing to Write (5 minutes maximum)
Preparing to write should take no more than 5 minutes. Planning your
time can be broken down into the following steps:
1. Circle the topic that is presented in the prompt.
This will be a question.
2. Decide what stance you will take on the topic.
You must choose a viewpoint and stick to it! Even if you change your
mind halfway through writing, stay with your original thought. Scores
for the essay will be based on how well you defend your point of view,
not whether or not your opinion agrees with that of the scorer.
3. Write down your thesis statement.
Your thesis statement should be one complete sentence that clearly
states your position on the topic.
4. Jot down a few examples you can use to support your thesis.
Examples can be from literature, history, personal experience, or
personal observations. They must always be relevant and support your
thesis statement. Stay focused on the reason you are using that
example: to prove why your thesis statement is valid.

Copyright 2009, TCA, LLC.

427

Studyguide for the SAT Essay Organization and Focus


Writing the Essay (1518 minutes)
Writing the essay should take no more than 1518 minutes. This will
leave you a few minutes to edit your essay. By allowing a few minutes
at the end of the time period, you will feel less frazzled when
writing.
Introductory Paragraph
The essay should start with an introductory paragraph that will
contain your thesis statement. It does not have to start with your
thesis statement, however. You could start with a relevant anecdote,
quotation, analogy, or dialogue. By starting your essay in a creative
way, you draw the reader in and make him or her want to read more.
Keep your introductory paragraph short(three to four sentences. Make
sure it contains your single point of view(never defend both sides of
the issue(on the prompt presented.
Body Paragraphs: Structure
Each paragraph of the essay should be indented and clearly organized.
Good internal organization means each paragraph contains a topic
sentence, supporting details, and a transition to the next paragraph.
We will go over transitions later in the lesson.
Body Paragraphs: Sentence Structure
Vary your sentence structure within paragraphs. Use sentences of
different lengths and complexity to engage the reader; give your
writing muscle by using strong verbs and the active voice. Make sure
that details and examples are specific and that they support your
thesis statement. Using a diverse and sophisticated vocabulary will
help you only if all words are used correctly. If you aren't
completely sure of a word's meaning, it's best not to use it. Stick to
what you know.
Concluding Paragraph
The essay should have a definite conclusion, leaving your readers to
think about the stance you have presented or eager to research the
topic further.

Copyright 2009, TCA, LLC.

428

Studyguide for the SAT Essay Organization and Focus


General Essay Tips
Essays of 30 lines or longer generally get better scores than shorter
essays do, since it usually takes that much time and space to state
and sufficiently prove a thesis. Make sure your writing is legible: if
you are better at printing, print. If your cursive is clearer, write
in cursive. No matter how brilliant your thesis, examples, and
organization, scorers will give you a zero if they cannot decipher
your writing.
Editing Your Work (25 minutes)
The last two to five minutes of the test should be reserved for
editing your work. By going back and proofreading what you have
written, you may catch errors in grammar that could affect your score.
If you have time, go back and strengthen your vocabulary and sentence
structure throughout the essay.

The Writing Process


Don't feel intimidated. Many students are convinced that they cannot
think through and write a well-organized essay in so short a time. The
good news is that essay writing is a skill that can be learned with
practice. Consider the following sample quotation and assignment:
"Better to remain silent and be thought a fool than to speak out and
remove all doubt."
Abraham Lincoln
Assignment:
What is your view on the idea that people should speak their mind,
even if they look foolish to others? Write and develop an essay in
which your point of view on this issue is clearly defined. Support
your point of view with examples and reasoning taken from your
studies, personal experience, or observations.

Copyright 2009, TCA, LLC.

429

Studyguide for the SAT Essay Organization and Focus


Before you answer the prompt, practice planning your essay. This
should take no longer than five minutes. There are two parts to the
planning phase: reading/thinking and plan development.
Reading/thinking should take less than two minutes. Plan development
should take up to 3 minutes.
During the reading/thinking phase, you will:
1. Read the prompt twice. Don't worry if you do not completely
understand the quote. It's just there to help you generate ideas.
Focus on the assignment itself.
2. Circle the topic you will write about. In this case, "What is your
view on the idea that people should speak their mind, even if they
look foolish to others?" is the assigned topic.
3. Take a stance on the issue presented. Take one side or the other;
don't discuss the virtues of both sides of the issue. Depending on the
prompt, you will either agree or disagree, support or oppose the topic
presented.
4. Decide what sources you will use as examples to prove your point.
Sources may include literary works, current or historical events,
personal experience, or personal observations.
5. Decide what areas of society the issue may affect. This can include
cultural, economic, social, physical, emotional, educational, and
mental factors. By thinking about how the topic may affect different
aspects of our world, you can think of more powerful supporting
points.
The plan development phase is for planning the body paragraphs of your
essay. The essay does not have to be the standard five paragraphs, but
it should contain paragraphs with two or three supporting points for
the thesis. During the plan development phase, you should:
1. Write a strong, one-sided thesis statement in response to the essay
prompt. For the example above, you may write: "People should voice
their own opinion, regardless of what others think." Make sure the
thesis statement is one complete sentence.
2. Write down your ideas for two or three paragraphs. For example, you
may talk about the ethical implications of speaking your own mind. You
may use an example from literature that illustrates how a character
spoke his or her mind and affected a future event positively.

Copyright 2009, TCA, LLC.

430

Studyguide for the SAT Essay Organization and Focus


3. Jot down a topic sentence for each paragraph. A topic sentence for
the thesis above could be: "Individuals who voiced unpopular opinions
brought about equal rights for African Americans."
4. Write transition words that you can use to connect your topic
sentence to the example in the paragraph. See the lesson on
transitions.
5. Right after your transition comes an example.
The above exercise may initially take you 1015 minutes, but once you
get the hang of it, you should be able to complete the thesis
statement and supporting examples within 3 minutes. Practice this
exercise well before the SAT testing date.

Copyright 2009, TCA, LLC.

431

Studyguide for the SAT Creating an Outline


Creating an Outline
Outlining can be a quick and effective way to organize your essay.
Writing an outline before you write your essay will tell you what
information each paragraph will contain.
Since you have only a short amount of time to create your outline, you
will not be making a formal one with Roman numbers and capital
letters. Instead, yours needs to be a short sentence or phrase for
each of your topic sentences with phrases indented under each one for
the examples.
Use any blank space on the essay prompt page to jot down a brief
outline and refer to it to help you stay focused on your point of
In the following examples, you will see formal outlines to give you
practice in organizing your information. To create an outline, start
with the topic of the essay and then create sub-points. Those subpoints are further divided into supporting details.
The following is a good outline for the "voicing your own opinion" topic:
I. People should voice their own opinion
A. Martin Luther King, Jr.
1. The Civil Rights Movement
2. Created controversy, but helped bring about lasting change
B. The Women's Suffrage Movement
1. United and empowered women nationwide
2. Led to women's right to vote, a freedom previously unavailable
C. In my lifeprotesting overpriced school lunches
1. Led to action on the part of the school board
2. Helped demonstrate that students have a voice too
3. Set an example for other students to take action on issues they
are concerned about
In the outline, there would be three main points and seven supporting points of one
or two sentences each. Your outline should contain at least three main points to support
the thesis of the essay. Remember: While you should always write your essay in
complete sentences, your outline need only consist of phrases that remind you what to
write about.

Copyright 2009, TCA, LLC.

432

Studyguide for the SAT Creating an Outline


Practice Exercise
Consider the following essay prompts, thesis statements, and the outlines that follow.
Choose the best outline for the thesis statement that is presented.
Question 1.
"Be great in act, as you have been in thought."
William Shakespeare
Assignment:
Are people's actions more important than their words or thoughts? Write and
develop an essay in which your point of view on this issue is clearly defined.
Support your point of view with examples and reasoning taken from your studies,
personal experience, and observations.

Thesis:
A person's value is defined by his or her actions, not by thoughts or words.
Which outline would work best with this topic?
(A)
I. A person's value is defined by his or her actions, not by thoughts or words
A. I have always wanted to visit Japan
1. I am interested in Japanese art and music
2. I would like to learn more about Buddhism
B. Focusing on goals is extremely important
1. My mother wants to paint more but lacks the time
2. My brother plays trombone and hopes to make all-state in the band
competition

(B)
I. A person's value is defined by his or her actions, not by thoughts or words
A. Montgolfier brothers
1. Dreamed of flying
2. Through trial error, invented hot air balloon
3. Still credited with first flight today
B. President Washington
1. Knew where he wanted to take this country
2. Took action to fight the British
3. Known for becoming the first leader of this country
C. Me
1. My lifelong ambition to attend State University
2. Study hard to get good grades
3. Practicing to do well on SAT

Copyright 2009, TCA, LLC.

433

Studyguide for the SAT Creating an Outline


(C)
I. A person's value is defined by his or her actions, not by thoughts or words
A. I auditioned and was cast in the school musical
1. To get there, I had to work hard on my audition piece
2. My range isn't great, but I did lots of other things to be worthy of
the part
a. Took voice lessons
b. Worked on facial expressions
c. Learned lines thoroughly
d. Researched the characters
B. My trip to Bermuda
1. Did biological research on sea urchins
2. Learned a lot about the flora and fauna of the island
3. Went with 5 other students in honors biology

Outline B is the best. It starts with the essay thesis and then draws
main points from historical figures and from personal experience. Each
main point has relevant supporting details, which helps create a solid
essay.
The first outline is too general. The author plans to discuss
different people's aspirations, but doesn't relate it back to the
thesis statement or main idea of the essay. The third outline is too
specific. It focuses only on the author's experiences, and does not
draw in multiple sources to support the main point of the essay.

Copyright 2009, TCA, LLC.

434

Studyguide for the SAT Creating an Outline


Question 2.
"I am still determined to be cheerful and happy, in whatever situation I may be;
for I have also learned from experience that the greater part of our happiness
or misery depends upon our dispositions, and not upon our circumstances."
Martha Washington
Assignment:
Can people choose to be happy regardless of their circumstances? Write and
develop an essay in which your point of view on this issue is clearly defined.
Support your point of view with examples and reasoning taken from your studies,
personal experience, and observations.

Thesis:
Even in the direst of circumstances, people can choose how they feel about their
current situation.
Which outline would work best with this topic?

(A)
I. Even in the direst of circumstances, people can choose how they feel about their
current situation.
A. War veterans
1. Some are never able to regain life after war
2. Others come home and move on to other things
B. My friend with a terminal cancer
1. Happiest person I've ever met
2. Focuses on acceptance and a sense of humor
3. Is happy for each day she is granted(chooses to focus on the
small things
C. The Secret Life of Bees
1. Lily grew up in bad family situation, but she had hope
2. Never gave up on the possibilities(imagination

(B)
I. Feelings about a situation
A. Can get over a bad situation at some point
B. Acceptance of a situation
1. Accepting something you can't change helps
2. Sometimes acceptance and hope can go together
C. Learning to deal with things is an important lesson of life

Copyright 2009, TCA, LLC.

435

Studyguide for the SAT Creating an Outline


(C)
I. People can choose how they feel about their current situation
A. The Diary of Anne Frank
1. Jewish girl during Holocaust
2. She was hidden from the Nazis in a family's attic
3. In order to pass the time and keep a record of her life, Anne
wrote in a diary
a. Diary is still read worldwide
b. She is a symbol of strength and courage during a trying
time
c. Diary gives insight into the strength and character of a
Jewish girl during the Holocaust
d. Anne was eventually found
The best choice is A. The outline shows main points to support the
thesis, from history, literature, and the student's personal
experience. The main points are supported with details, creating a
well-supported thesis statement.
The second outline shows only the author's personal experience. In
order to have a strong essay, you need to have supporting points from
at least two outside sources. The third outline is too specific. It
focuses just on The Diary of Anne Frank, and does not provide
other examples to support the thesis. While The Diary of Anne
Frank is a good source to use, it should not be the only evidence
used to support the thesis statement.

Copyright 2009, TCA, LLC.

436

Studyguide for the SAT Creating an Outline


Question 3.
"Only the educated are free."
Epictetus, Discourses
Assignment:
Is education the key to freedom? Write and develop an essay in which your point
of view on this issue is clearly defined. Support your point of view with
examples and reasoning taken from your studies, personal experience, and
observations.
Thesis: Well-educated people have more freedoms than non-educated people.
Which outline would work best with this topic?
(A)

(B)

Copyright 2009, TCA, LLC.

I. Well-educated people have more freedoms than non-educated people


A. Don't have to work for minimum wage
1. Fast food
2. Waiting tables
3. Picking fruit
4. Can work in a field you enjoy
a. Electrician
b. Librarian
c. Teacher

I. Well-educated people have more freedoms than non-educated people


A. Going to college is a positive thing
B. Getting a college degree can boost earning power
C. People without college degrees have fewer options
1. Working for minimum wage
2. Can afford fewer things

437

Studyguide for the SAT Creating an Outline

(C)

I. Well-educated people have more freedoms than non-educated people


A. Less educated have fewer job choices
1. My older brother, a 10-year firefighter, lacks a college degree
2. A guy who came in after him was offered a senior position first
because of his college degree
3. He rejected the promotion because he liked what he was doing
B. General perception of educated is favorable
1. My mother has always been gifted advertising writer
2. Had difficult time finding work without college degree
3. Got a degree, now has lots of job offers
4. Same skills, but diploma is more impressive
C. Education leads to higher pay, which enables more freedoms
1. Person with college degree makes about 30% more than person
without degree in same job
2. People with money take more vacation, retire earlier, live longer

The best outline for this thesis is C. Examples are taken from the
student's observations of his or her family. The student also cites
statistics that enable solid support of the thesis.
Outline A is too general, and focuses on different types of jobs, of
which there is a wide range. The second outline is too specific,
because it focuses only on the benefits that college offers, not on
the freedom of the individual to choose a job that they enjoy.

Copyright 2009, TCA, LLC.

438

Studyguide for the SAT Creating an Outline


Question 4.
"Envy is the ulcer of the soul."
Socrates
Assignment:
Is there ever justification for feeling envious of another person? Write and
develop an essay in which your point of view on this issue is clearly defined.
Support your point of view with examples and reasoning taken from your studies,
personal experience, and observations.

Thesis: There are times when feelings of envy can yield positive results.
Which outline would work best with this topic?

(A)
I. There are times when feelings of envy can yield positive results
A. Jacob Have I Loved
1. Two sisters: Caroline pretty and musically gifted, Louise plainer
and envious of sister
2. Louise finds talent as doctor to escape sister's shadow
3. Envy caused Louise to strive for greatness
B. Scientific competition
1. Scientists who are close to a discovery compete to be the first
2. They want to be remembered
3. Result is scientific advancements
C. Basketball team tryouts
1. My twin brother made the varsity basketball team and I was still
on junior varsity
2. Began jogging and practicing more
3. I made the varsity team in the middle of last year

(B)
I. There are times when feelings of envy can yield positive results.
A. My brother and I compete constantly
1. Sports
2. Grades
3. End result: We do better in school and on sports teams
B. Competing for attention from parents
1. Do more chores for higher allowance
2. Time spent volunteering in mom's office
3. Time spent with dad mowing lawn
4. End result: better relationship with parents
C. Competing for more freedoms
1. Better behavior, good grades mean later curfew

Copyright 2009, TCA, LLC.

439

Studyguide for the SAT Creating an Outline


(C)
I. There are times when feelings of envy can yield positive results.
A. Envy leads to feelings of insecurity
1. Insecurity can make a person work harder to be accepted
2. A person who works harder can accomplish more
B. Feeling envious can affect a person's social life
1. Can make a person more self-aware
2. Can make a person work harder to make and keep friends
C. Feeling envious can cause tension in relationships
1. Envy may initially cause problems
2. A smart person learns from those problems and works to change
3. End result: Better person who has learned from his or her
mistakes

Outline A has sources taken from history, literature, and personal


experience. These sources help to prove that the thesis statement for
the essay is valid. The outline is not overly specific, nor is it too
general.
Outline B is too specific. It focuses just on envy within the family
structure, and does not bring in any outside sources. When creating an
essay for the SAT, you should make sure to include at least two to
three outside sources to support your thesis statement. Outline C is
too general. It discusses envy in general, and does not provide
specific examples in relation to the thesis statement.

Copyright 2009, TCA, LLC.

440

Studyguide for the SAT Creating an Outline


Question 5.
"Nearly all men can stand adversity, but if you want to test a man's character,
give him power."
Abraham Lincoln
Assignment:
Does power change people fundamentally? Write and develop an essay in which your
point of view on this issue is clearly defined. Support your point of view with
examples and reasoning taken from your studies, personal experience, and
observations.
Thesis: Anyone who is given too much power can become a danger to others.
Which outline would work best with this topic?

(A)
I. Anyone who is given too much power can become a danger to others
A. Power causes people to feel superior to others
1. They begin making rules without regard for other people
2. They begin to judge people based on their personal preferences
B. People forget their family and friends
1. They put their own needs before those of others
2. They imagine that their success was achieved without any help or
support
C. People are never satisfied with the amount of power they have
1. Power makes people feel entitled
2. There is never enough power for people who crave it

(B)

Copyright 2009, TCA, LLC.

I. Anyone who is given too much power can become a danger to others
A. Joseph Stalin
1. Executed thousands of his citizens
2. Caused thousands of people to starve
3. Kept his citizens under Communist rule
B. Communist rule led to uniformity of thought
1. No room for individuality
2. People who did not follow government rule were executed

441

Studyguide for the SAT Creating an Outline


(C)
I. Anyone who is given too much power can become a danger to others
A. Napoleon Bonaparte
1. Took over many countries as his craving for power expanded
2. Ruled Europe with unlimited power as he spread east to Russia
3. Armies only stopped by the Russian winter
B. Lord of the Flies by William Golding
1. Boys living on island with no rules or laws
2. Gradually become more savage and violent
3. Human nature is base; those with power don't have to obey rules
that keep that nature in check
C. My junior-high gymnastics instructor
1. Thought only about his own reputation, not what was best for the
health of his athletes
2. Led to athletes getting hurt
3. Lasting damage to his athletes' careers and his own

Outline C is the best option. It shows specific examples of how power


has led to corruption through a historical figure, literary figures
and figure in the athletic community.
Outline A focuses on power in general, and does not give specific
examples of how too much power results in dangerous behavior and
corruption. Outline B talks specifically about Joseph Stalin and
Communism, but does not give other examples in relation to the thesis
statement.

Copyright 2009, TCA, LLC.

442

Studyguide for the SAT Starting Your Essay


Starting Your Essay
An introduction should draw the reader into the essay and make him or
her want to read more. It should also contain your thesis statement.
The introduction should be briefone short paragraph. It can start
with a quotation, anecdote, question, or analogy. However you start
your essay, it should always lead up to the essay's thesis statement.
The thesis statement should be visible and easy for the reader to
find.

Practice Exercise
Read the following prompts and sample introductions. Choose the best
introduction out of the three choices.
Question 1.
"Be great in act, as you have been in thought."
William Shakespeare
Assignment:
Are people's actions more important than their words or thoughts? Write and
develop an essay in which your point of view on this issue is clearly defined.
Support your point of view with examples and reasoning taken from your studies,
personal experience, and observations.
Which introduction would be most effective?
(A) I was backing slowly away from the dog, whose teeth were
bared in anger. My friend Manny and I were delivering the newspaper
to Mrs. Waterhouse, who had left my weekly check on her back porch.
The dog lunged at me and bit down on my forearm. In a moment, Manny
was wrestling the dog to the ground and seconds later, the dog had run
away. To say that courage is important is one thing; to face danger
courageously is another. This is an example of how actions are more
important than words or thoughts.

Copyright 2009, TCA, LLC.

443

Studyguide for the SAT Starting Your Essay


(B) Actions do speak louder than words. Every day, people are
faced with certain decisions they must make, whether to take action or
just talk about what they want to do. What people actually accomplish
matters more than what they want to accomplish.
(C) I once went to the beach and got caught in the tide. I
was carried a half mile out to sea, where the current moved me
swiftly farther and farther from my family. I tried to remember
everything I had ever heard about situations like this, but all I
could manage was to keep my head above water. That was the scariest
event I ever experienced.
Introduction A is the best choice. It draws the reader in by providing
an anecdote that relates to the thesis statement. Notice that the
story is a few sentences long and that the thesis statement sums up
the story nicely.
Introduction B is boring. It starts with a clear thesis statement, but
does not effectively draw the reader in to the essay. It speaks in
very broad terms, without creating a single picture or idea that
readers can keep in their minds. Introduction C is interesting, but
never gets to a thesis statement. At the end of it, the reader still
cannot tell whether the writer things thoughts or actions are more
important. Maintain focus in your essay, without going off on a
tangent. Provide an immediate and specific response to the essay
prompt.

Copyright 2009, TCA, LLC.

444

Studyguide for the SAT Starting Your Essay


Question 2.
"I am still determined to be cheerful and happy, in whatever situation I may be;
for I have also learned from experience that the greater part of our happiness
or misery depends upon our dispositions, and not upon our circumstances."
Martha Washington
Assignment:
Can people choose to be happy regardless of their circumstances? Write and
develop an essay in which your point of view on this issue is clearly defined.
Support your point of view with examples and reasoning taken from your studies,
personal experience, and observations.
Which introduction would be most effective?
(A) For me, the carnival is the happiest place in the world.
When I go to the town carnival each October, I feel a sense of joy
when riding the Ferris wheel, eating candied apples, and seeing all
of the animals at the 4H center. I have attended the carnival every
year since I was five years old, and plan to continue attending it for
as long as I can. The carnival has the unique ability to make me feel
as wondrous and excited as a five-year-old again.
(B) People can choose to be happy, no matter what their
circumstances. Many people, when faced with adversity, succumb to
depression or a feeling of helplessness. There are many people,
however, who live in dire circumstances but find a way to get by.
Happiness is a choice, not something that requires money or a good
situation to create.
(C) My grandmother took pride in being a tailor for four
decades. She worked until crippling arthritis in her hands forced
her to retire. We worried that she would become depressed, but
Grandmother immediately started volunteering at a local homeless
shelter. She says that keeps her busy and gives her all the pride she
needs. My grandmother's shift in attitude demonstrates that people can
choose to be happy, regardless of what is happening in their lives.

Copyright 2009, TCA, LLC.

445

Studyguide for the SAT Starting Your Essay


Introduction C is the best choice. It starts with a short story that
relates directly to the question presented in the prompt. The writer
builds up to the definitive thesis statement, preparing the reader for
a concise and deliberately stated opinion.
The first introduction is unrelated to the prompt. The writer may feel
happy when he or she attends a carnival, but the prompt refers to
happiness during difficult times. This anecdote has nothing to do with
trying circumstances. Option B is mundane, starting with a regular
thesis statement but not intriguing the reader. SAT Essay scorers like
to see creativity in an essay, starting with the essay's introduction.

Copyright 2009, TCA, LLC.

446

Studyguide for the SAT Starting Your Essay


Question 3.
"Only the educated are free."
Epictetus, Discourses
Assignment:
Is education the key to freedom? Write and develop an essay in which your point
of view on this issue is clearly defined. Support your point of view with
examples and reasoning taken from your studies, personal experience, and
observations.
Which introduction would be most effective?
(A) Education has been regarded as a priority for humankind for
centuries. Education has opened up opportunities for countries and the
people who reside in them. Education has enabled people to become
self-sufficient and to bring their living standards up to those of
those in other nations. Education has enabled the freedom of many
people throughout the world.
(B) "Are you finished with your application yet? The deadline is in
two weeks." My father has been hounding me about going to college,
saying that it would give me more control over my life as an adult. I
usually ignore my father's "life advice," but looking at the
employment section of the newspaper changed my mind. Every job that
sounded interesting required at least a B.A.; those that didn't tended
to pay much lower. With an education, I'll have a lot more choices and
opportunities(a lot more freedom.
(C) Plato, a student of Socrates thousands of years ago was an
important philosopher and man of letters. His ideas permeated
important circles of thought at the time, and are still taught and
studied to this day. Since his time, philosophers have built on
Plato's ideas and used them as a model, to shape society.
Option B is the best, starting with a short glimpse into the author's
life. Dialogue is one creative way to open up the essay and lead to
the thesis statement.
Option A, while not disastrous, could stand improvement. It discusses
education in general, but is not specific enough in depicting how
education has led to greater freedoms in different parts of the world.
Option C is not directly related to the topic. Plato was a student of
Socrates, but the introduction does not create a relationship between
education and freedom.

Copyright 2009, TCA, LLC.

447

Studyguide for the SAT Starting Your Essay


Question 4.
"Envy is the ulcer of the soul."
Socrates
Assignment:
Is there ever justification for feeling envious of another person? Write and
develop an essay in which your point of view on this issue is clearly defined.
Support your point of view with examples and reasoning taken from your studies,
personal experience, and observations.
Which introduction would be most effective?

(A) It's easy to feel envious of people who have things we


don't have. The gap between the very wealthy and very poor in our
society is huge, and evidence of that gap is everywhere: in
advertisements, in popular culture, and driving down the street in
expensive cars. While reminders of our limitations can spur us on to
achieve more, envy is usually a useless emotion.

(B) People do amazing things every day. From doctors to


pianists to diplomats, people accomplish feats(creating beautiful
music or saving a life or working for peace between nations(that are
truly unique and that benefit our world. Whether they have the
admiration of others is immaterial: they do what they do because they
love it, and it fulfills them.

(C) My closest friend growing up was slightly better than I


was at everything. Neil was always a half-inch taller, a faster
runner, a better art student. As we grew older, I envied Neil more and
more. My family moved away and I visited him two years later. In those
years when we were apart, I had found my self-confidence and learned
to appreciate my own strengths without comparing them to anyone
else's. Envy is an unjustifiable emotion that keeps people from
achieving their full potential.

Copyright 2009, TCA, LLC.

448

Studyguide for the SAT Starting Your Essay


Introduction C is the most effective. It starts with a concise, yet
heartfelt anecdote of the author's previous insecurity, and then ties
the story into the author's conclusion: that envy is unjustifiable and
destructive.
Option A falls short of the kind of introduction you should strive
for. It isn't exactly riveting, since it lacks an image or idea that
strikes the reader. Its biggest weakness, however, is the thesis
statement, which fails to answer the question, instead wavering on
both sides of the issue. It is unclear how option B addresses the
prompt at all. It starts off with examples of people who contribute to
society, but then only weakly ties in the idea of envy.

Copyright 2009, TCA, LLC.

449

Studyguide for the SAT Starting Your Essay


Question 5.
"Nearly all men can stand adversity, but if you want to test a man's character,
give him power."
Abraham Lincoln
Assignment:
Does power change people fundamentally? Write and develop an essay in which your
point of view on this issue is clearly defined. Support your point of view with
examples and reasoning taken from your studies, personal experience, and
observations.
Which introduction would be most effective?
(A) "Power corrupts. Absolute power corrupts absolutely."
Lord Acton is famous for that quote. His feelings about the
relationship between power and character are clear. I disagree.
Character is the most basic part of a person, something that stays
constant throughout our lives. Whether power corrupts us depends on
whether we let it corrupt us. Power does not change people
fundamentally; it just shows us more clearly what kind of character
lies beneath our words and actions.
(B) Power changes people by making them feel that they can
get away with anything. Not only does power give a person authority
over other people's lives, but it gives a person the ability to change
the quality of life that many people lead. Many people who have that
kind of power find that they lack the judgment to handle it. They
begin to lose perspective and forget that they are human, with a
human's limitations and foibles.
(C) The most powerful person in the world is nothing without
her or his cabinet. The support staff of a powerful person is what
defines how well a person does in office. Without a support staff, not
much can get accomplished on a daily basis. So a single person really
doesn't have that much power, but surrounds him or herself with people
whose job it is to help them make the best decisions.
The best choice is option A. It starts with a well-known quote that
relates directly to the topic in the prompt. The writer then evaluates
that quote and proceeds to explain why he or she disagrees with it.
The paragraph culminates in a clear thesis statement that states the
writer's opinion about the question asked.
Option B begins with the idea that power corrupts people, but it ends
making the same people sound like victims of power. In between those
two extremes, it tends to ramble about changing people's lives. This
paragraph lacks a clear direction. Option C is not directly related to
the topic, since it discusses the importance of a support staff, not
how power changes or does not change people.

Copyright 2009, TCA, LLC.

450

Studyguide for the SAT Transitions


Transitions
Transitions move the reader from one idea to the next between or
within paragraphs. They show how ideas are related to one another.
These words indicate relationships between ideas in terms of
chronology, order of importance, and development. Transitions that
show order of importance include:
least important
more important
most important
finally
also
even greater
greatest
great

least significantly
more significantly
more significantly
most significantly
for one reason
next
even more
last

Transitions that show chronological order include:


after
afterward
as soon as
at that moment
before
earlier
during
now
eventually
finally
formerly
gradually
immediately
initially
then
whenever

Copyright 2009, TCA, LLC.

in the course of
while
in the meantime
last
later
long ago
meanwhile
moments later
next
presently
since
soon
until
when

451

Studyguide for the SAT Transitions


Transitions that show developmental order include:
accordingly
along with
also
and
another
as a result
as an illustration
as well
because
besides
consequently
finally
for example
for instance
in like manner
in the same way

for this reason


furthermore
in addition
in fact
indeed
moreover
namely
next
on account of
on the other hand
so
then
therefore
thus
too
in contrast

Transition words are used as part of a sentence, both between and


within paragraphs. To make a smooth connection between paragraphs,
include a transition in the first sentence of the second paragraph to
link back to the first paragraph.
The following example shows transitions both within and between paragraphs:
Computers insinuate themselves into virtually every aspect of our lives.
Not only do we write, play games, and communicate using computers, but
we also interact with them in ways we aren't always aware of. Computers
control the programs that air on our televisions, the commercials that come on
between songs on the radio, the stoplights at busy intersections, as well as the
power supplies to our homes, schools and businesses.
As computers establish their prominence in everyday tasks , scientists
are increasingly becoming aware of their immense learning potential.
In the 1950's, Alan Turing created a computer that could play a game
of chess against a human. Over the years, computers improved at the
game and eventually, computer programs were just as adept at chess as
most people were.

Copyright 2009, TCA, LLC.

452

Studyguide for the SAT Transitions


In the example, transitions within the paragraph include "not
only...but also" and "as well as." The transition between the
paragraphs is underlined in green in the first sentence of the second
paragraph. The phrase "establish their prominence in everyday tasks"
links back to the topic in the first paragraph of computers being in
"every aspect of our lives."
As demonstrated in the sample, transitions can be used within a
sentence to indicate order or between paragraphs to move from one
thought to the next. Transitions help with the flow of the essay.
Without transitions from one idea to the next, the essay will seem
choppy and ideas will lack connection.

Copyright 2009, TCA, LLC.

453

Studyguide for the SAT Transitions


Practice Exercise
Read the following sample prompts and paragraphs that require a
transition sentence to link them. Decide what sentence should be
filled in, and write it in the box.
Question 1.
"Be great in act, as you have been in thought."
William Shakespeare
Assignment:
Are people's actions more important than their words or thoughts? Write and
develop an essay in which your point of view on this issue is clearly defined.
Support your point of view with examples and reasoning taken from your studies,
personal experience, and observations.
Dr. Martin Luther King, Jr. is indeed famous for his actions. For most
of his life, he took part in rallies for Civil Rights across the
South. He marched and picketed and demonstrated with thousands of
others to make his visions for America known.
Many people still live today who heard Dr. King speak and who remember
clearly the strength of his words and the conviction with which he
delivered them. His "I Have a Dream" speech is still held up as an
example of brilliant oratory that drives home a powerful message.
________________________________________________________________________
________________________________________________________________________
________________________________________________________________________
________________________________________________________________________

A sample transition sentence is shown underlined below:


However, Dr. King's words had just as great an impact on American
society as his actions did. Many people still live today who
heard Dr. King speak and who remember clearly the strength of his
words and the conviction with which he delivered them. His "I Have a
Dream" speech is still held up as an example of brilliant oratory that
drives home a powerful message.

Copyright 2009, TCA, LLC.

454

Studyguide for the SAT Transitions


Question 2.
"I am still determined to be cheerful and happy, in whatever situation I may be;
for I have also learned from experience that the greater part of our happiness
or misery depends upon our dispositions, and not upon our circumstances."
Martha Washington
Assignment:
Can people choose to be happy regardless of their circumstances? Write and
develop an essay in which your point of view on this issue is clearly defined.
Support your point of view with examples and reasoning taken from your studies,
personal experience, and observations.

Sometimes it's simply impossible to maintain one's happiness in the


face of extreme adversity. Life is often full of pain that, as humans
living in human societies, we must bear. When I was in the eighth
grade, a classmate of mine lost his older brother, a senior in high
school, to meningitis, a fierce disease that can kill a seemingly
healthy person in 12 hours. Could I have gone to my classmate and
said, "You're choosing to be sad right now. All you have to do is make
up your mind to be happy and you will be"? Of course not! His grief
consumed him, which is natural.
The book of Job is the story of a good man who God decides to test by
slowly taking away everything that is important to him, including
wealth and then family. Job responds by grieving and complaining
bitterly that he is a good person and does not deserve to suffer in
such a way.
________________________________________________________________________
________________________________________________________________________
________________________________________________________________________
________________________________________________________________________

A sample transition sentence is shown underlined below:


The link between circumstance and sadness is also the subject of one
of the best-known biblical stories. The book of Job is the story
of a good man who God decides to test by slowly taking away everything
that is important to him, including wealth and then family. Job
responds by grieving and complaining bitterly that he is a good person
and does not deserve to suffer in such a way.

Copyright 2009, TCA, LLC.

455

Studyguide for the SAT Transitions


Question 3.
"Only the educated are free."
Epictetus, Discourses
Assignment:
Is education the key to freedom? Write and develop an essay in which your point
of view on this issue is clearly defined. Support your point of view with
examples and reasoning taken from your studies, personal experience, and
observations.
Mobabai lived in a small village in Nigeria. No one in his village had
ever traveled more than thirty miles outside of the village. Mobabai
was the first from his village to attend a university. Upon
graduating, Mobabai returned to find that wealthy poachers were
killing valuable livestock that belonged to the villagers.
The government banned the poachers from entering village lands. This illustrates how
education led to freedom from great poverty for the people of a small Nigerian village.
________________________________________________________________________
________________________________________________________________________
________________________________________________________________________
________________________________________________________________________

A sample transition sentence is shown underlined below::


Since Mobabai had gone to a university and had experience working
with the law in his classes at the university, he was able to enlist
the help of the Nigerian government to stop the poachers. The
government banned the poachers from entering village lands. This
illustrates how education led to freedom from great poverty for the
people of a small Nigerian village.

Copyright 2009, TCA, LLC.

456

Studyguide for the SAT Transitions


Question 4.
"Envy is the ulcer of the soul."
Socrates
Assignment:
Is there ever justification for feeling envious of another person? Write and
develop an essay in which your point of view on this issue is clearly defined.
Support your point of view with examples and reasoning taken from your studies,
personal experience, and observations.
Envy is a useless emotion that causes people to lose sight of their
goals. Envy is all too common among adolescents and teens in our
schools. There is often so much focus on the latest fashions and
technology that students sometimes forget the greater goal of getting
a quality education. Unfortunately, kids tend to accept or reject
others based on their material possessions, creating a culture of
haves and have-nots and creating the impression that a person's value
is based on the t-shirt they're wearing to school. Rather than
focusing on upcoming tests, students instead obsess over whether their
jeans are "too blue" or if they're faded just right.

Shakespeare's play Julius Caesar illustrates excellently how envy can


lead to chaos. Declaring that Caesar is "too ambitious," Antony,
Brutus and Cassius conspire to kill the leader because they envy his
immense popularity. Their actions lead not just to Caesar's death, but
hundreds of others as well, including those of Brutus and Cassius.
________________________________________________________________________
________________________________________________________________________
________________________________________________________________________
________________________________________________________________________

A sample transition sentence is shown underlined below:


Just as envy is a common emotion that students wrestle with in dayto-day life and relationships, envy is also a topic that appears
frequently in well-known literature. Shakespeare's play Julius Caesar
illustrates excellently how envy can lead to chaos. Declaring that
Caesar is "too ambitious," Antony, Brutus and Cassius conspire to kill
the leader because they envy his immense popularity. Their actions
lead not just to Caesar's death, but hundreds of others as well,
including those of Brutus and Cassius.

Copyright 2009, TCA, LLC.

457

Studyguide for the SAT Transitions


Question 5.
"Nearly all men can stand adversity, but if you want to test a man's character,
give him power."
Abraham Lincoln
Assignment:
Does power change people fundamentally? Write and develop an essay in which your
point of view on this issue is clearly defined. Support your point of view with
examples and reasoning taken from your studies, personal experience, and
observations.
Power does not make good people bad or make bad people good; rather,
power makes a person's goodness or badness much clearer. For example,
Eleanor Roosevelt was a powerful first lady with much more influence
than any first lady had ever had in history. She used that influence
strategically to make people's lives better. She pursued women's
rights by helping female journalists get stories published at a time
when women were at a severe disadvantage in the field. She brought
attention to the issues of Civil Rights, joblessness, and poverty.
Last week, my sister and I were walking our dog in the park and found
a car with the rear door left open. It looked like the owner had
forgotten to close it. Inside, clearly visible, were some CDs and
other things. It would have been the easiest thing to reach in and
take something and nobody would have caught us, but instead, my sister
walked straight to the car, pushed the lock on the inside down, and
slammed the door shut.
________________________________________________________________________
________________________________________________________________________
________________________________________________________________________
________________________________________________________________________
A sample transition sentence is shown underlined below:
But the term "power" doesn't necessarily have to mean power over a lot
of people; it could be the opportunity, or power, to do something bad
without the risk of being caught.

Copyright 2009, TCA, LLC.

458

Studyguide for the SAT Wrapping It Up


Wrapping It Up
You're almost there! A conclusion should wrap up the essay and make the
reader want to read more. An effective conclusion will leave the reader
thinking about your ideas and deciding that your thesis was correct and
well defended. The conclusion should summarize the main idea of the
essay, and can point the reader to further resources for more
information.
There are two ways to write an effective conclusion. The first way is to
remind the reader of the main idea of your essay (your point of view),
and to review what you covered in the essay, all of which can be done in
one short paragraph. The second effective method is to create an insight
paragraph that states what you learned from a personal or observational
experience. This type of concluding paragraph can receive a higher score
if it shows depth of understanding on your part.

Copyright 2009, TCA, LLC.

459

Studyguide for the SAT Wrapping It Up


Practice Exercise
Choose the best conclusion for each of the five prompts.
Question 1.
"Be great in act, as you have been in thought."
William Shakespeare
Assignment:
Are people's actions more important than their words or thoughts? Write and
develop an essay in which your point of view on this issue is clearly defined.
Support your point of view with examples and reasoning taken from your studies,
personal experience, and observations.
Which conclusion would be most effective?
(A) The motorboat incident taught me that safety is imperative
in water sports. I used to tell my mother I'd be safe just so that she'd
stop nagging me, but I've now learned that I have to put that thought
into action in order for it to be effective.
(B) In conclusion, any person with good intentions must take
action to make it count. A person can talk about doing the right thing,
but until they put their actions behind it, it is of limited value.
(C) People can learn to change. Change is not just limited to
those who put their mind to it, but anyone can take small steps towards
lasting change each day.

Conclusion A is the best choice. It provides insight about what the


author learned in relation to the incident they presented in previous
paragraphs. This shows that the author has a deep understanding of the
topic presented in the essay.
Conclusion B is a little bit weak. It restates the thesis well, but does
not provide additional insight into the information presented in the
essay. Option C is off-topic. The essay is not about creating change; it
is about whether actions or thoughts are more important.

Copyright 2009, TCA, LLC.

460

Studyguide for the SAT Wrapping It Up


Question 2.
"I am still determined to be cheerful and happy, in whatever situation I may be;
for I have also learned from experience that the greater part of our happiness
or misery depends upon our dispositions, and not upon our circumstances."
Martha Washington
Assignment:
Can people choose to be happy regardless of their circumstances? Write and
develop an essay in which your point of view on this issue is clearly defined.
Support your point of view with examples and reasoning taken from your studies,
personal experience, and observations.

Which conclusion would be most effective?

(A) Once I was able to accept that I could not force the
admissions committee to admit me to the university, I felt a sense of
calm and happiness about my circumstances. This caused all my worries to
disappear.
(B) Happiness is therefore something that people can choose to
feel, not something that is based on circumstance. Even though the
Olivers in the above example were very poor, they bonded together as a
family and showed that happiness could be achieved through conscious
effort.

(C) The last time Daniel felt happy was when he lived in Ohio.
In Ohio, all of his family lived in the same house, and he did not feel
a sense of separation from the core of his life.

Conclusion B is the best choice, since it relates directly back to an


example in the essay. It also reviews the thesis statement, which
reminds the reader what the main point of the essay was.
Option A is too weak. It ends abruptly, without reviewing the thesis or
providing insight into an experience presented in the essay. Option C
strays from the type of content a conclusion should contain. Although
the character of "Daniel" may have felt happy in Ohio, these sentences
do not belong at the end of the essay. This segment of writing belongs
toward the beginning or middle of the essay, when an example is
presented that is directly related to the thesis statement.

Copyright 2009, TCA, LLC.

461

Studyguide for the SAT Wrapping It Up


Question 3.
"Only the educated are free."
Epictetus, Discourses
Assignment:
Is education the key to freedom? Write and develop an essay in which your point
of view on this issue is clearly defined. Support your point of view with
examples and reasoning taken from your studies, personal experience, and
observations.
Which conclusion would be most effective?
(A) The educated in Southeast Asia are therefore better off
than their less-educated countrymen.
(B) From these experiences, I learned that an education can
open up more opportunities for people than they otherwise would have.
Education is crucial to having the freedom of choice, so I plan to
continue my education. As a result, I will be able to choose an
interesting and challenging field and not be forced to do work I
dislike.
(C) Freedom rang in Liberty Square. The Declaration of
Independence had just been signed, and the new Americans could feel that
enormous, history-making change was occurring at that very moment.
Choice B is the best. This concluding paragraph shows that the author
learned a lesson from an experience. The lesson relates directly to
the main point of the essay, which is important to reiterate for
readers when concluding the essay. Choice A doesn't sum up the essay
effectively. While an example concerning Southeast Asia may have been
used in the essay, the writer ends the essay too abruptly with this
conclusion. A good strategy would have been to tie the example of
Southeast Asia back in to the thesis and to end the essay more
gradually. Conclusion C gets off topic and, while it creates a nice
image, it does not clearly relate to the thesis or sum up the writer's
point of view.

Copyright 2009, TCA, LLC.

462

Studyguide for the SAT Wrapping It Up


Question 4.
"Envy is the ulcer of the soul."
Socrates
Assignment:
Is there ever justification for feeling envious of another person? Write and
develop an essay in which your point of view on this issue is clearly defined.
Support your point of view with examples and reasoning taken from your studies,
personal experience, and observations.
Which conclusion would be most effective?

(A) The argument with Carla taught me that envy itself isn't
necessarily bad, but what a person does with it can be bad or good. My
envy of Carla's position on the soccer team drove me to look closer at
myself and decide whether I really wanted to play soccer. Carla and I
talked it out and our friendship survived, but it was tough facing my
own envy head-on.

(B) In addition to envy, I also feel that rage is a


destructive emotion that can destroy relationships. Envy and anger are
natural emotions and are unavoidable in a society in which individuals
constantly relate to one another. However, they are also emotions that
have a lot of power and are often not worth the consequences.

(C) In conclusion, a sense of envy is something that everyone


feels at one time or another. I think that by expressing what a person
feels, they can get their feelings out in the open before those
feelings turn into a deep resentment that can change the nature of the
relationship.

Conclusion A is the best option. The author relates back to an example


from the essay and provides insight into what they learned from the
experience. Conclusion B goes into a discussion of a different
emotion, without focusing solely on the main point of the essay.
Option C is only vaguely related to the topic and does not clearly
restate the thesis, answering the question, "Is envy ever justified?"
Most people would agree with the writer's last words on the subject,
but they don't really have much to do with envy.

Copyright 2009, TCA, LLC.

463

Studyguide for the SAT Wrapping It Up


Question 5.
"Nearly all men can stand adversity, but if you want to test a man's character,
give him power."
Abraham Lincoln
Assignment:
Does power change people fundamentally? Write and develop an essay in which your
point of view on this issue is clearly defined. Support your point of view with
examples and reasoning taken from your studies, personal experience, and
observations.
Which conclusion would be most effective?
(A) All people are, therefore, affected by power. It takes a
certain type of personality to be overcome by power.
(B) Power can be found from the corporate ladder to the
educational system. I've learned that people can be sneaky sometimes, so
I keep an eye out for that when I tutor kids at the elementary school.
(C) Even the people with the best intentions can therefore be
overcome by responsibility and an ego can grow when the person is placed
in a position of authority. Power changes people, but not always for the
better.
The best choice is the option C. The author has just presented an
example and wrapped it up with an allusion to what has already been
written. The first sentence summarizes the point of the essay. In the
last sentence, the author restates the thesis statement. This leaves
the reader thinking about the main idea presented in the essay. Choice
A is too weak, referring to power and an individual's personality, but
not relating directly to a thesis statement. These sentences would not
form an effective conclusion, since the second sentence would fall
somewhere in the middle of the essay. Choice B is off-topic. It
discusses the author's personal experience but does not restate in any
way the main point of the essay.

Copyright 2009, TCA, LLC.

464

Writing Lesson #7
Use of Language

Triumph College Admissions


Skill Lesson

Quiz Time!
A

Hints!

Answers!
A

*Throughout this documentation, and the software: College Board and SAT are registered trademarks of the College Entrance Examination Board. PSAT/NMSQT
is a trademark of the College Entrance Examination Board and National Merit Scholarship Corporation. ACT is a trademark of ACT, Inc. None of these entities are
affiliated with the production of, nor endorse these materials.
Copyright 2009, TCA, LLC.

465

Copyright 2009, TCA, LLC.

466

Studyguide for the SAT Skill Review: Use Of Language And Vocabulary

USE OF LANGUAGE AND VOCABULARY

College Board essay readers are trained to reward students who use a
"varied, accurate, and apt vocabulary." With so many words to choose
from, which are the best? In theory your vocabulary choices are almost
limitless. In practice, however, there is a relatively small group of
words that lend themselves to the type of essay you will be writing
for the SAT. Because these words are descriptive and precise, they
will help make your writing clear and authoritative. It is important,
though, to use such words correctly. Essay readers will not be
impressed by essays sprinkled with sophisticated words that are used
incorrectly. Learn their exact meanings, and use them correctly in
your essay.

Here is a list of descriptive words that can help


raise your score.
Words to describe personality types
1. EGOTIST a self-centered person who is devoted to his or her own interests
2. ALTRUIST a generous person who is devoted to helping others
3. INTROVERT a shy, private person
4. EXTROVERT an outgoing, sociable person
5. PRAGMATIST a practical person
Words to describe people who are honest or dishonest
6. VIRTUOUS very moral, righteous
7. CANDID very open and honest
8. VENAL very corrupt, willing to accept bribes
9. UNSCRUPULOUS very immoral, unprincipled

Copyright 2009, TCA, LLC.

467

Studyguide for the SAT Skill Review: Use Of Language And Vocabulary
Words to describe people who are mature or immature
10. LEVEL-HEADED sensible, poised
11. PRUDENT showing good judgment, thoughtful
12. SOPHOMORIC juvenile, childish
13. CALLOW infantile, very immature
Words to describe people who are brave or cowardly
14. TIMOROUS very shy
15. CRAVEN very cowardly
16. INTREPID very brave
17. AUDACIOUS very bold

Words to describe people who can and cannot make up their minds
18. DECISIVE able to reach a firm decision
19. RESOLUTE very determined
20. VACILLATE to waver back and forth
21. AMBIVALENT to have mixed feelings
Words to describe the thrill of victory and the agony of defeat
22. ELATED very happy
23. ECSTATIC delirious with joy
24. DEJECTED downcast and gloomy
25. DESPONDENT feeling that all hope is lost

Words to describe situations involving deception


26. CLANDESTINE secret, hidden
27. NEFARIOUS extremely wicked, villainous
28. PERFIDIOUS disloyal, treacherous
29. STRATAGEM a clever scheme or trick designed to attain a goal
Words to describe people who are fun to be with
30. AFFABLE very friendly, pleasant
31. VIVACIOUS filled with life, fun-loving
32. GREGARIOUS very sociable, outgoing
33. OPTIMISTIC having a hopeful outlook on life, seeing only good things

Copyright 2009, TCA, LLC.

468

Studyguide for the SAT Skill Review: Use Of Language And Vocabulary
Words to describe people who aren't fun to be with
34. PESSIMISTIC having a gloomy outlook on life, seeing only bad things
35. EXASPERATING very irritating, annoying
36. SUPERCILIOUS very arrogant, haughty, stuck-up
37. CANTANKEROUS very grouchy, cranky
Words to describe feelings
38. DISTRAUGHT filled with anxiety, very worried
39. APATHETIC uninterested, unfeeling
40. ZEALOUS very passionate, fervent
41. ANIMOSITY great dislike, ill will
42. EMPATHY great compassion, sympathy
Words to describe good and bad speakers
43. ELOQUENT very articulate, vividly expressive
44. LUCID very clear, easy to follow
45. LOQUACIOUS very talkative
46. PERSUASIVE very convincing, compelling
47. CONVOLUTED very intricate, twisted, difficult to follow
Words to describe people who have energy or lack energy
48. VIGOROUS very active, energetic
49. ARDENT filled with passion, great fervor
50. LETHARGIC lazy, drowsy

Copyright 2009, TCA, LLC.

469

Copyright 2009, TCA, LLC.

470

Studyguide for the SAT Skill Quiz A: Use of Language

Quiz Time!
A

Question 1
Which of the following words means
uninterested or unfeeling ?

(A)

apathetic

(B)

affable

(C)

clandestine

(D)

despondent

Question 2
Which of the following words means
disloyal or treacherous ?

(A)

lethargic

(B)

pessimistic

(C)

perfidious

(D)

supercilious

Question 3
Which of the following words means
very talkative?

(A)

persuasive

(B)

audacious

(C)

ecstatic

(D)

loquacious

Copyright 2009, TCA, LLC.

471

Studyguide for the SAT Skill Quiz A: Use of Language


Question 4
Which of the following words means
feeling that all hope is lost ?

(A)

ambivalent

(B)

resolute

(C)

intrepid

(D)

despondent

Question 5
Which of the following words means
a self-centered person ?

(A)

altruist

(B)

egotist

(C)

extrovert

(D)

introvert

Question 6
Which of the following words means
delirious with joy ?

(A)

ecstatic

(B)

vivacious

(C)

zealous

(D)

lucid

Copyright 2009, TCA, LLC.

472

Studyguide for the SAT Skill Quiz A: Use of Language


Question 7
Which of the following words means
lazy or drowsy?

(A)

ardent

(B)

venal

(C)

lethargic

(D)

lucid

Question 8
Which of the following words means
very moral or righteous ?

(A)

venal

(B)

virtuous

(C)

prudent

(D)

intrepid

Question 9
Choose the word that best completes this sentence:
Harry tried to coax the ______ kitten out of its hiding place.

(A)

gregarious

(B)

timorous

(C)

nefarious

(D)

zealous

Copyright 2009, TCA, LLC.

473

Studyguide for the SAT Skill Quiz A: Use of Language


Question 10
Choose the word that best completes this sentence:
Sarah is too much of an ______ to come to the party.

(A)

intrepid

(B)

egotist

(C)

ardent

(D)

introvert

Copyright 2009, TCA, LLC.

474

Studyguide for the SAT Skill Quiz A: Use of Language

Answers!
A

Question 1.
The answer is (A).
Explanation for Question 1:

"Apathetic" means "uninterested or unfeeling."


"Affable" means "very friendly and pleasant."
"Clandestine" means "secret or hidden."
"Despondent" means "feeling that all hope is lost."
The answer is A.

Question 2.
The answer is (C).
Explanation for Question 2:

"Perfidious" means "disloyal or treacherous."


"Lethargic" means "lazy or drowsy."
"Pessimistic" means "having a gloomy outlook on life."
"Supercilious" means "very arrogant or stuck-up."
The answer is C.
Question 3.
The answer is (D).
Explanation for Question 3:

"Loquacious" means "very talkative."


"Persuasive" means "very convincing or compelling."
"Audacious" means "very bold."
"Ecstatic" means "delirious with joy."
The answer is D.

Copyright 2009, TCA, LLC.

475

Studyguide for the SAT Skill Quiz A: Use of Language


Question 4.
The answer is (D).
Explanation for Question 4:

"Despondent" means "feeling that all hope is lost."


"Ambivalent" means "having mixed feelings."
"Resolute" means "very determined."
"Intrepid" means "very brave."
The answer is D.
Question 5.
The answer is (B).
Explanation for Question 5:
An "egotist" is a self-centered person.
An "altruist" is a generous person who is devoted to helping others.
An "extrovert" is an outgoing, sociable person.
An "introvert" is a shy, private person.
The answer is B.
Question 6.
The answer is (A).
Explanation for Question 6:

"Ecstatic" means "delirious with joy."


"Vivacious" means "fun-loving and filled with life."
"Zealous" means "very passionate or fervent."
"Lucid" means "very clear and easy to follow."
The answer is A.
Question 7.
The answer is (C).
Explanation for Question 7:

"Lethargic" means "lazy or drowsy."


"Ardent" means "filled with passion or great fervor."
"Venal" means "very corrupt and willing to accept bribes."
"Lucid" means "very clear and easy to follow."
The answer is C.

Copyright 2009, TCA, LLC.

476

Studyguide for the SAT Skill Quiz A: Use of Language


Question 8.
The answer is (B).
Explanation for Question 8:

"Virtuous" means "very moral or righteous."


"Venal" means "very corrupt and willing to accept bribes."
"Prudent" means "showing good judgment, or thoughtful."
"Intrepid" means "very brave."
The answer is B.
Question 9.
The answer is (B).
Explanation for Question 9:

"Timorous" means "very shy."


"Gregarious" means "very sociable and outgoing."
"Nefarious" means "extremely wicked or villainous."
"Zealous" means "very passionate or fervent."
The answer is B.
Question 10.
The answer is (D).
Explanation for Question 10:

An "introvert" is a shy, private person.


"Intrepid" means "very brave."
An "egotist" is a self-centered person.
"Ardent" means "filled with passion or great fervor."
The answer is D.

Copyright 2009, TCA, LLC.

477

Copyright 2009, TCA, LLC.

478

Studyguide for the SAT Skill Quiz B: Use of Language

Quiz Time!
A

Question 1
Which of the following words means
able to make a firm choice ?

(A)

ambivalent

(B)

decisive

(C)

optimistic

(D)

prudent

Question 2
Which of the following words means
very friendly and pleasant ?

(A)

supercilious

(B)

apathetic

(C)

callow

(D)

affable

Question 3
Which of the following words means
a practical person ?

(A)

pragmatist

(B)

extrovert

(C)

altruist

(D)

stratagem

Copyright 2009, TCA, LLC.

479

Studyguide for the SAT Skill Quiz B: Use of Language


Question 4
Which of the following words means
very sociable and outgoing ?

(A)

gregarious

(B)
(C)

nefarious
ecstatic

(D)

eloquent

Question 5
Which of the following words means
showing good judgment, or thoughtful ?

(A)

callow

(B)

prudent

(C)

intrepid

(D)

resolute

Question 6
Which of the following words means
to waver back and forth?

(A)

resolute

(B)

clandestine

(C)

exasperating

(D)

vacillate

Copyright 2009, TCA, LLC.

480

Studyguide for the SAT Skill Quiz B: Use of Language


Question 7
Which of the following words means
very arrogant or stuck-up ?

(A)

supercilious

(B)

cantankerous

(C)

convoluted

(D)

loquacious

Question 8
Which of the following words means
filled with passion or great fervor ?

(A)

lucid

(B)

affable

(C)

ardent

(D)

optimistic

Question 9
Choose the word that best completes this sentence:
The puppy was ______ and playful.

(A)

lucid

(B)

zealous

(C)

lethargic

(D)

vigorous

Copyright 2009, TCA, LLC.

481

Studyguide for the SAT Skill Quiz B: Use of Language


Question 10
Choose the word that best completes this sentence:
Michelle felt ______ after she lost her job.

(A)

nefarious

(B)

venal

(C)

dejected

(D)

unscrupulous

Copyright 2009, TCA, LLC.

482

Studyguide for the SAT Skill Quiz B: Use of Language

Answers!
A

Question 1.
The answer is (B).
Explanation for Question 1:

"Decisive" means "able to make a firm choice."


"Ambivalent" means "having mixed feelings."
"Optimistic" means "having a hopeful outlook on life."
"Prudent" means "showing good judgment, or thoughtful."
The answer is B.
Question 2.
The answer is (D).
Explanation for Question 2:

"Affable" means "very friendly and pleasant."


"Supercilious" means "very arrogant or stuck-up."
"Apathetic" means "uninterested or unfeeling."
"Callow" means "very immature."
The answer is D.

Question 3.
The answer is (A).
Explanation for Question 3:

A "pragmatist" is a practical person.


An "extrovert" is an outgoing, sociable person.
An "altruist" is a generous person who is devoted to helping others.
A "stratagem" is "a clever scheme designed to attain a goal."
The answer is A.

Copyright 2009, TCA, LLC.

483

Studyguide for the SAT Skill Quiz B: Use of Language


Question 4.
The answer is (A).
Explanation for Question 4:

"Gregarious" means "very sociable and outgoing."


"Nefarious" means "extremely wicked or villainous."
"Ecstatic" means "delirious with joy."
"Eloquent" means "very articulate or vividly expressive."
The answer is A.
Question 5.
The answer is (B).
Explanation for Question 5:

"Prudent" means "showing good judgment, or thoughtful."


"Callow" means "very immature."
"Intrepid" means "very brave."
"Resolute" means "very determined."
The answer is B.
Question 6.
The answer is (D).
Explanation for Question 6:

"Vacillate" means "to waver back and forth."


"Resolute" means "very determined."
"Clandestine" means "secret or hidden."
"Exasperating" means "very irritating or annoying."
The answer is D.
Question 7.
The answer is (A).
Explanation for Question 7:

"Supercilious" means "very arrogant or stuck-up."


"Cantankerous" means "very grouchy or cranky."
"Convoluted" means "very intricate and difficult to follow."
"Loquacious" means "very talkative."
The answer is A.

Copyright 2009, TCA, LLC.

484

Studyguide for the SAT Skill Quiz B: Use of Language


Question 8.
The answer is (C).
Explanation for Question 8:

"Ardent" means "filled with passion or great fervor."


"Lucid" means "very clear and easy to follow."
"Affable" means "very friendly and pleasant."
"Optimistic" means "having a hopeful outlook on life."
The answer is C.
Question 9.
The answer is (D).
Explanation for Question 9:

"Vigorous" means "very active and energetic."


"Lucid" means "very clear and easy to follow."
"Zealous" means "very passionate or fervent."
"Lethargic" means "lazy or drowsy."
The answer is D.
Question 10.
The answer is (C).
Explanation for Question 10:

"Dejected" means "downcast and gloomy."


"Nefarious" means "extremely wicked or villainous."
"Venal" means "very corrupt and willing to accept bribes."
"Unscrupulous" means "very immoral and unprincipled."
The answer is C.

Copyright 2009, TCA, LLC.

485

Copyright 2009, TCA, LLC.

486

Studyguide for the SAT Skill Quiz C: Use of Language

Quiz Time!
A

Question 1
Which of the following words means
a generous person who is devoted to helping others ?

(A)

extrovert

(B)

altruist

(C)

egotist

(D)

candid

Question 2
Which of the following words means
very immoral and unprincipled ?

(A)

callow

(B)

craven

(C)

unscrupulous

(D)

sophomoric

Question 3
Which of the following words means
very brave?

(A)

craven

(B)

candid

(C)

ardent

(D)

intrepid

Copyright 2009, TCA, LLC.

487

Studyguide for the SAT Skill Quiz C: Use of Language


Question 4
Which of the following words means
very articulate or vividly expressive ?

(A)

lucid

(B)

eloquent

(C)

vivacious

(D)

loquacious

Question 5
Which of the following words means
great dislike or ill will ?

(A)

animosity

(B)

empathy

(C)

callow

(D)

stratagem

Question 6
Which of the following words means
sensible and poised ?

(A)

sophomoric

(B)

level-headed

(C)

candid

(D)

resolute

Copyright 2009, TCA, LLC.

488

Studyguide for the SAT Skill Quiz C: Use of Language


Question 7
Which of the following words means
extremely wicked or villainous ?

(A)

nefarious

(B)

timorous

(C)

audacious

(D)

ambivalent

Question 8
Which of the following words means
very open and honest?
(A)

callow

(B)

candid

(C)

venal

(D)

lucid

Question 9
Choose the word that best completes this sentence:
Gary's ______ uncle complained throughout the meal.

(A)

cantankerous

(B)

apathetic

(C)

vivacious

(D)

lethargic

Copyright 2009, TCA, LLC.

489

Studyguide for the SAT Skill Quiz C: Use of Language


Question 10
Choose the word that best completes this sentence:
No one could follow the movie's ______ plot.

(A)

loquacious

(B)

ambivalent

(C)

convoluted

(D)

decisive

Copyright 2009, TCA, LLC.

490

Studyguide for the SAT Skill Quiz C: Use of Language

Question 1.
The answer is (B).
Explanation for Question 1:

An "altruist" is a generous person who is devoted to helping others.


An "extrovert" is an outgoing, sociable person.
An "egotist" is a self-centered person.
"Candid" means "very open and honest."
The answer is B.
Question 2.
The answer is (C).
Explanation for Question 2:

"Unscrupulous" means "very immoral and unprincipled."


"Callow" means "very immature."
"Craven" means "very cowardly."
"Sophomoric" means "juvenile or childish."
The answer is C.
Question 3.
The answer is (D).
Explanation for Question 3:

"Intrepid" means "very brave."


"Craven" means "very cowardly."
"Candid" means "very open and honest."
"Ardent" means "filled with passion or great fervor."
The answer is D.

Copyright 2009, TCA, LLC.

491

Studyguide for the SAT Skill Quiz C: Use of Language

Answers!

Question 4.
The answer is (B).
B C
D 4:E
Explanation forAQuestion

"Eloquent" means "very articulate or vividly expressive."


"Lucid" means "very clear and easy to follow."
"Vivacious" means "fun-loving and filled with life."
"Loquacious" means "very talkative."
The answer is B.
Question 5.
The answer is (A).
Explanation for Question 5:

"Animosity" means "great dislike or ill will."


"Empathy" means "great compassion."
"Callow" means "very immature."
A "stratagem" is "a clever scheme designed to attain a goal."
The answer is A.
Question 6.
The answer is (B).
Explanation for Question 6:

"Level-headed" means "sensible and poised."


"Sophomoric" means "juvenile or childish."
"Candid" means "very open and honest."
"Resolute" means "very determined."
The answer is B.
Question 7.
The answer is (A).
Explanation for Question 7:
"Nefarious" means "extremely wicked or villainous."
"Timorous" means "very shy."
"Audacious" means "very bold."
"Ambivalent" means "having mixed feelings."
The answer is A.

Copyright 2009, TCA, LLC.

492

Studyguide for the SAT Skill Quiz C: Use of Language


Question 8.
The answer is (B).
Explanation for Question 8:

"Candid" means "very open and honest."


"Callow" means "very immature."
"Venal" means "very corrupt and willing to accept bribes."
"Lucid" means "very clear and easy to follow."
The answer is B.
Question 9.
The answer is (A).
Explanation for Question 9:

"Cantankerous" means "very grouchy or cranky."


"Apathetic" means "uninterested or unfeeling."
"Vivacious" means "fun-loving and filled with life."
"Lethargic" means "lazy or drowsy."
The answer is A.
Question 10.
The answer is (C).
Explanation for Question 10:

"Convoluted" means "very intricate and difficult to follow."


"Loquacious" means "very talkative."
"Ambivalent" means "having mixed feelings."
"Decisive" means "able to make a firm choice."
The answer is C.

Copyright 2009, TCA, LLC.

493

Copyright 2009, TCA, LLC.

494

Studyguide for the SAT Skill Quiz D: Use of Language

Quiz Time!
A

Question 1
Which of the following words means
juvenile or childish ?

(A)

prudent

(B)

sophomoric

(C)

optimistic

(D)

craven

Question 2
Which of the following words means
very bold?

(A)

audacious

(B)

timorous

(C)

vigorous

(D)

virtuous

Question 3
Which of the following words means
very determined ?

(A)

ambivalent

(B)

pessimistic

(C)

resolute

(D)

apathetic

Copyright 2009, TCA, LLC.

495

Studyguide for the SAT Skill Quiz D: Use of Language


Question 4
Which of the following words means
secret or hidden ?

(A)

candid

(B)

callow

(C)

convoluted

(D)

clandestine

Question 5
Which of the following words means
filled with anxiety or very worried ?

(A)

apathetic

(B)

distraught

(C)

perfidious

(D)

ambivalent

Question 6
Which of the following words means
having a hopeful outlook on life ?

(A)

optimistic

(B)

elated

(C)

ecstatic

(D)

vivacious

Copyright 2009, TCA, LLC.

496

Studyguide for the SAT Skill Quiz D: Use of Language


Question 7
Which of the following words means
very irritating or annoying ?

(A)

timorous

(B)

intrepid

(C)

exasperating

(D)

affable

Question 8
Which of the following words means
an outgoing, sociable person ?

(A)

extrovert

(B)

egotist

(C)

altruist

(D)

stratagem

Question 9
Choose the word that best completes this sentence:
Maria was ______ when she won a car in the contest.

(A)

craven

(B)

dejected

(C)

elated

(D)

venal

Copyright 2009, TCA, LLC.

497

Studyguide for the SAT Skill Quiz D: Use of Language


Question 10
Choose the word that best completes this sentence:
I agreed with Sanjay because his argument was very ______.

(A)

unscrupulous

(B)

sophomoric

(C)

affable

(D)

persuasive

Copyright 2009, TCA, LLC.

498

Studyguide for the SAT Skill Quiz D: Use of Language

Answers!
A

Question 1.
The answer is (B).
Explanation for Question 1:

"Sophomoric" means "juvenile or childish."


"Prudent" means "showing good judgment, or thoughtful."
"Optimistic" means "having a hopeful outlook on life."
"Craven" means "very cowardly."
The answer is B.
Question 2.
The answer is (A).
Explanation for Question 2:

"Audacious" means "very bold."


"Timorous" means "very shy."
"Vigorous" means "very active and energetic."
"Virtuous" means "very moral or righteous."
The answer is A.
Question 3.
The answer is (C).
Explanation for Question 3:

"Resolute" means "very determined."


"Ambivalent" means "having mixed feelings."
"Pessimistic" means "having a gloomy outlook on life."
"Apathetic" means "uninterested or unfeeling."
The answer is C.

Copyright 2009, TCA, LLC.

499

Studyguide for the SAT Skill Quiz D: Use of Language


Question 4.
The answer is (D).
Explanation for Question 4:

"Clandestine" means "secret or hidden."


"Candid" means "very open and honest."
"Callow" means "very immature."
"Convoluted" means "very intricate and difficult to follow."
The answer is D.
Question 5.
The answer is (B).
Explanation for Question 5:
"Distraught" means "filled with anxiety or very worried."
"Apathetic" means "uninterested or unfeeling."
"Perfidious" means "disloyal or treacherous."
"Ambivalent" means "having mixed feelings."
The answer is B.
Question 6.
The answer is (A).
Explanation for Question 6:

"Optimistic" means "having a hopeful outlook on life."


"Elated" means "very happy."
"Ecstatic" means "delirious with joy."
"Vivacious" means "fun-loving and filled with life."
The answer is A.
Question 7.
The answer is (C).
Explanation for Question 7:

"Exasperating" means "very irritating or annoying."


"Timorous" means "very shy."
"Intrepid" means "very brave."
"Affable" means "very friendly and pleasant."
The answer is C.

Copyright 2009, TCA, LLC.

500

Studyguide for the SAT Skill Quiz D: Use of Language


Question 8.
The answer is (A).
Explanation for Question 8:

An "extrovert" is an outgoing, sociable person.


An "egotist" is a self-centered person.
An "altruist" is a generous person who is devoted to helping others.
A "stratagem" is "a clever scheme designed to attain a goal."
The answer is A.
Question 9.
The answer is (C).
Explanation for Question 9:

"Elated" means "very happy."


"Craven" means "very cowardly."
"Dejected" means "downcast and gloomy."
"Venal" means "very corrupt and willing to accept bribes."
The answer is C.
Question 10.
The answer is (D).
Explanation for Question 10:

"Persuasive" means "very convincing or compelling."


"Unscrupulous" means "very immoral and unprincipled."
"Sophomoric" means "juvenile or childish."
"Affable" means "very friendly and pleasant."
The answer is D.

Copyright 2009, TCA, LLC.

501

Copyright 2009, TCA, LLC.

502

Studyguide for the SAT Skill Quiz E: Use of Language

Quiz Time!
A

Question 1
Which of the following words means
very clear and easy to follow ?

(A)

eloquent

(B)

lucid

(C)

ardent

(D)

decisive

Question 2
Which of the following words means
very immature ?
(A)

candid

(B)

elated

(C)

callow

(D)

zealous

Question 3
Which of the following words means
great compassion ?

(A)

clandestine

(B)

stratagem

(C)

animosity

(D)

empathy

Copyright 2009, TCA, LLC.

503

Studyguide for the SAT Skill Quiz E: Use of Language


Question 4
Which of the following words means
very passionate or fervent ?

(A)

zealous

(B)

vivacious

(C)

supercilious

(D)

cantankerous

Question 5
Which of the following words means
having mixed feelings ?

(A)

resolute

(B)

distraught

(C)

ambivalent

(D)

timorous

Question 6
Which of the following words means
very cowardly?

(A)

callow

(B)

craven

(C)

venal

(D)

affable

Copyright 2009, TCA, LLC.

504

Studyguide for the SAT Skill Quiz E: Use of Language


Question 7
Which of the following words means
having a gloomy outlook on life ?

(A)

apathetic

(B)

resolute

(C)

pessimistic

(D)

cantankerous

Question 8
Which of the following words means
fun-loving and filled with life ?

(A)

vivacious

(B)

gregarious

(C)

loquacious

(D)

virtuous

Question 9
Choose the word that best completes this sentence:
The police officer was sure that the ______ mayor would accept a bribe.

(A)

venal

(B)

persuasive

(C)

convoluted

(D)

prudent

Copyright 2009, TCA, LLC.

505

Studyguide for the SAT Skill Quiz E: Use of Language


Question 10
Choose the word that best completes this sentence:
We carefully developed a ______ to trick our opponents.

(A)

lucid

(B)

pragmatist

(C)

stratagem

(D)

clandestine

Copyright 2009, TCA, LLC.

506

Studyguide for the SAT Skill Quiz E: Use of Language

Answers!
A

Question 1.
The answer is (B).
Explanation for Question 1:

"Lucid" means "very clear and easy to follow."


"Eloquent" means "very articulate or vividly expressive."
"Ardent" means "filled with passion or great fervor."
"Decisive" means "able to make a firm choice."
The answer is B.
Question 2.
The answer is (C).
Explanation for Question 2:

"Callow" means "very immature."


"Candid" means "very open and honest."
"Elated" means "very happy."
"Zealous" means "very passionate or fervent."
The answer is C.
Question 3.
The answer is (D).
Explanation for Question 3:

"Empathy" means "great compassion."


"Clandestine" means "secret or hidden."
A "stratagem" is "a clever scheme designed to attain a goal."
"Animosity" means "great dislike or ill will."
The answer is D.

Copyright 2009, TCA, LLC.

507

Studyguide for the SAT Skill Quiz E: Use of Language


Question 4.
The answer is (A).
Explanation for Question 4:

"Zealous" means "very passionate or fervent."


"Vivacious" means "fun-loving and filled with life."
"Supercilious" means "very arrogant or stuck-up."
"Cantankerous" means "very grouchy or cranky."
The answer is A.
Question 5.
The answer is (C).
Explanation for Question 5:

"Ambivalent" means "having mixed feelings."


"Resolute" means "very determined."
"Distraught" means "filled with anxiety or very worried."
"Timorous" means "very shy."
The answer is C.
Question 6.
The answer is (B).
Explanation for Question 6:

"Craven" means "very cowardly."


"Callow" means "very immature."
"Venal" means "very corrupt and willing to accept bribes."
"Affable" means "very friendly and pleasant."
The answer is B.
Question 7.
The answer is (C).
Explanation for Question 7:

"Pessimistic" means "having a gloomy outlook on life."


"Apathetic" means "uninterested or unfeeling."
"Resolute" means "very determined."
"Cantankerous" means "very grouchy or cranky."
The answer is C.

Copyright 2009, TCA, LLC.

508

Studyguide for the SAT Skill Quiz E: Use of Language


Question 8.
The answer is (A).
Explanation for Question 8:

"Vivacious" means "fun-loving and filled with life."


"Gregarious" means "very sociable and outgoing."
"Loquacious" means "very talkative."
"Virtuous" means "very moral or righteous."
The answer is A.
Question 9.
The answer is (A).
Explanation for Question 9:

"Venal" means "very corrupt and willing to accept bribes."


"Persuasive" means "very convincing or compelling."
"Convoluted" means "very intricate and difficult to follow."
"Prudent" means "showing good judgment, or thoughtful."
The answer is A.
Question 10.
The answer is (C).
Explanation for Question 10:

A "stratagem" is "a clever scheme designed to attain a goal."


"Lucid" means "very clear and easy to follow."
A "pragmatist" is a practical person.
"Clandestine" means "secret or hidden."
The answer is C.

Copyright 2009, TCA, LLC.

509

Copyright 2009, TCA, LLC.

510

Writing Lesson #8
Sentence Structure

Triumph College Admissions


Skill Lesson

Quiz Time!
A

Hints!

Answers!
A

*Throughout this documentation, and the software: College Board and SAT are registered trademarks of the College Entrance Examination Board. PSAT/NMSQT
is a trademark of the College Entrance Examination Board and National Merit Scholarship Corporation. ACT is a trademark of ACT, Inc. None of these entities are
affiliated with the production of, nor endorse these materials.
Copyright 2009, TCA, LLC.

511

Copyright 2009, TCA, LLC.

512

Studyguide for the SAT Essay Sentence Structure

SENTENCE STRUCTURE VARIETY


Your essay for the New SAT will be judged on much more than your use
of vocabulary, of course. The sentence structure you use must be as
varied and as interesting as your vocabulary.
The four sentence structures are:
1.

Simple: 1 independent clause (contains 1 subject, which can be


compound, and 1 predicate, which can be compound)
Example: I went to the store.

2.

Compound: 2 or more independent clauses, usually joined by a


coordinating conjunction
Example: Bob went to the mall, and Alice went to the drycleaners.

3.

Complex: 1 independent clause and at least 1 dependent clause (a


group of words that has both a subject and predicate but that can't
stand on its own)
Example: Although I love the movies, I passed up a trip to the
theater this afternoon.

4.

Compound-Complex: 2 or more independent clauses, usually joined by


a coordinating conjunction, and at least 1 dependent clause
Example: When I go to school each day, I drive the car that my
parents bought me, but in return, I must drive my younger brother.

You should use all of these sentence structures to give your writing
variety and clarity.
Exercise:
In the box provided, rewrite the following sentences as instructed.
answer.
Question 1
Combine these two simple sentences to make a compound sentence:
Land use is a controversial issue. Even farmers have differing
opinions about it.

Copyright 2009, TCA, LLC.

513

Studyguide for the SAT Essay Sentence Structure

Land use is a controversial issue, and even farmers have differing


opinions about it.

Question 2
Combine these two simple sentences to make a complex sentence:
Some farmers are eager to sell their land to developers. Others want
to preserve their farms and ranches.

While some farmers are eager to sell their land to developers, others
want to preserve their farms and ranches.

Question 3
Combine these three simple sentences to make a compound-complex
sentence:
My grandfather has been a farmer for more than 50 years. He believes
that farming is a good way of life. He hopes that his land will always
be farmed.

My grandfather has been a farmer for more than 50 years, and because
he believes that farming is a good way of life, he hopes that his land
will always be farmed.

Question 4
Combine these sentences to make one compound, complex, or compoundcomplex sentence:
A growing population drives up demand for land. Increased demand leads
to increased prices.

Copyright 2009, TCA, LLC.

514

Studyguide for the SAT Essay Sentence Structure

A growing population drives up demand for land, causing prices to


increase. (compound)

Question 5
Combine these sentences to make one compound, complex, or compoundcomplex sentence:
Some farmers choose to sell their land in spite of regrets. They can
become wealthy by selling their land. They cannot become wealthy by
farming it.

Despite regrets, some farmers choose to sell their land to become


wealthy, because they cannot become wealthy by farming it. (compoundcomplex)

Copyright 2009, TCA, LLC.

515

Copyright 2009, TCA, LLC.

516

Studyguide for the SAT Skill Quiz A: Sentence Variety

Quiz Time!
A

Question 1
Which of the following combines these sentences to make a complex sentence?
The American bison is a member of the bovid family. This family also includes
cattle, sheep, and goats.

(A)

The American bison is a member of the bovid family, which


also includes cattle, sheep, and goats.

(B)

The American bison is a member of the bovid family, and


this family also includes cattle, sheep, and goats.

Question 2
Which of the following combines these sentences to make a compound sentence?
The bison is the largest land animal in North America. Females are
considerably smaller than males.

Copyright 2009, TCA, LLC.

(A)

The bison is the largest land animal in North America, but


females are considerably smaller than males.

(B)

The bison is the largest land animal in North America even


though females are considerably smaller than males.

517

Studyguide for the SAT Skill Quiz A: Sentence Variety

Question 3
Which of the following combines these sentences to make a complex sentence?
One large, free-ranging herd of bison lives in Yellowstone National Park.
Visitors can see them grazing on the open plains.
(A)

One large, free-ranging herd of bison lives in Yellowstone


National Park; visitors can see them grazing on the open plains.

(B)

One large, free-ranging herd of bison lives in Yellowstone


National Park, where visitors can see them grazing on the open plains.

Question 4
Which of the following combines these sentences to make a compound sentence?
A bison's horns are permanent, bony growths, unlike a deer's antlers. Both
male and female bison have them.

Copyright 2009, TCA, LLC.

(A)

A bison's horns are permanent, bony growths, unlike a deer's


antlers, and both male and female bison have them.

(B)

A bison's horns are permanent, bony growths, unlike a deer's


antlers: both male and female bison have them.

518

Studyguide for the SAT Skill Quiz A: Sentence Variety

Question 5
Which of the following combines these sentences to make a compound-complex
sentence?
The bison was nearly extinct at one time. They are no longer endangered. Today
most bison live in national parks or on privately owned land.

(A)
land.

The bison was nearly extinct at one time, they are no longer
endangered, today most bison live in national parks or on privately owned

(B)

Although the bison was nearly extinct at one time, it is no


longer endangered, and today it lives mainly in national parks or on
privately owned land.

Question 6
Which of the following combines these sentences to make a complex sentence?
Privately owned bison herds are found in nearly every state. Historically, their
natural range was more limited.

Copyright 2009, TCA, LLC.

(A)

Privately owned bison herds are found in nearly every state


although, historically, their natural range was more limited.

(B)

Privately owned bison herds are found in nearly every state,


and, historically, their natural range was more limited.

519

Studyguide for the SAT Skill Quiz A: Sentence Variety

Question 7
Which of the following most effectively combines these sentences?
A female bison usually has just one calf at a time. Occasionally two calves are
born at the same time.

(A)

A female bison usually has just one calf at a time, and


occasionally two calves are born at the same time.

(B)

A female bison usually has just one calf at a time; however,


occasionally two calves are born at the same time.

Question 8 Which of the following most effectively combines these sentences?


Bison are massive and heavy. They are excellent swimmers. They are very
buoyant.

(A)

Bison are massive and heavy, and they are excellent


swimmers, and they are very buoyant.

(B)

Bison are massive and heavy, yet they are excellent


swimmers because they are very buoyant.

Question 9
Which of the following most effectively combines these sentences?
Bison can survive in snowy conditions. They use their hooves and heads to
scrape snow off of vegetation.

Copyright 2009, TCA, LLC.

(A)

Bison can survive in snowy conditions, and they use their


hooves and heads to scrape snow off of vegetation.

(B)

Bison can survive in snowy conditions since they use their


hooves and heads to scrape snow off of vegetation.

520

Studyguide for the SAT Skill Quiz A: Sentence Variety

Question 10
Which of the following most effectively combines these sentences?
Bison most often live on plains. They sometimes live in wooded areas. One sign
of their presence is tree bark worn smooth by a bison's rubbing.

Copyright 2009, TCA, LLC.

(A)

Bison most often live on plains, but they sometimes live in


wooded areas, where they wear the tree bark smooth by rubbing it.

(B)

Although, bison most often live on plains, they sometimes live in wooded areas,
and it is here that they wear the tree bark smooth by
rubbing it.

521

Copyright 2009, TCA, LLC.

522

Studyguide for the SAT Skill Quiz A: Sentence Variety

Hints!
A

Hint for Question 1:


What word could you use to make the second sentence a dependent clause?

Hint for Question 2:


What subordinating conjunction could be used to join the two sentences?

Hint for Question 3:


What subordinating conjunction could be used to join the two sentences?

Hint for Question 4:


Which coordinating conjunction could best join these two sentences?

Hint for Question 5:


Which sentence could best be made into a dependent clause?

Hint for Question 6:


What subordinating conjunction could be used to join the two sentences?

Hint for Question 7:


What kind of conjunction could best be used to join the two sentences?

Hint for Question 8:


Which sentence could best be made into a dependent clause?

Hint for Question 9:


What kind of conjunction could best be used to join the two sentences?

Copyright 2009, TCA, LLC.

523

Studyguide for the SAT Skill Quiz A: Sentence Variety


Hint for Question 10:
Which sentence could best be made into a dependent clause?

Copyright 2009, TCA, LLC.

524

Studyguide for the SAT Skill Quiz A: Sentence Variety

Answers!
A

Question 1.
The answer is (A).
Explanation for Question 1:
The answer is A.
The American bison is a member of the bovid family, which also includes cattle,
sheep, and goats.

Question 2.
The answer is (A).
Explanation for Question 2:
The answer is A.
The bison is the largest land animal in North America, but females are considerably
smaller than males.

Question 3.
The answer is (B).
Explanation for Question 3:
The answer is B.
One large, free-ranging herd of bison lives in Yellowstone National Park, where
visitors can see them grazing on the open plains.

Question 4.
The answer is (A).
Explanation for Question 4:
The answer is A.
A bison's horns are permanent, bony growths, unlike a deer's antlers, and both male
and female bison have them.
Question 5.
The answer is (B).
Explanation for Question 5:
The answer is B.
The bison was nearly extinct at one time, and while they are no longer endangered,
today most bison live in national parks or on privately owned land.

Copyright 2009, TCA, LLC.

525

Studyguide for the SAT Skill Quiz A: Sentence Variety


Question 6.
The answer is (A).
Explanation for Question 6:
The answer is A.
Privately owned bison herds are found in nearly every state, although historically,
their natural range was more limited.

Question 7.
The answer is (B).
Explanation for Question 7:
The answer is B.
A female bison usually has just one calf at a time, but occasionally two calves are
born at the same time. (coordinating conjunction; compound sentence)

Question 8.
The answer is (B).
Explanation for Question 8:
The answer is B.
Bison are massive and heavy, yet they are excellent swimmers because they are very
buoyant. (1 dependent and 2 independent clauses; compound-complex sentence)

Question 9.
The answer is (B).
Explanation for Question 9:
The answer is B.
Bison can survive in snowy conditions, since they use their hooves and heads to
scrape snow off of vegetation. (subordinating conjunction; complex sentence)
Question 10.
The answer is (A).
Explanation for Question 10:
The answer is A.
Bison most often live on plains, but they sometimes live in wooded areas, where one
sign of them is tree bark worn smooth by a bison's rubbing. (1 dependent and 2
independent clauses; compound-complex sentence)

Copyright 2009, TCA, LLC.

526

Studyguide for the SAT Skill Quiz B: Sentence Variety

Quiz Time!
A

Question 1
Which of the following combines these sentences to make a compound sentence?
Riding the subway is fast and inexpensive. It is not always pleasant.
(A)

Although, riding the subway is fast and inexpensive, it is not


always pleasant.

(B)

Riding the subway is fast and inexpensive, but it is not always pleasant.

Question 2
Which of the following combines these sentences to make a complex sentence?
I take the subway to school. There is a stop right across the street.

(A)

I take the subway to school since there is a stop right across the street.

(B)

I take the subway to school, and there is a stop right across the street.

Question 3
Which of the following combines these sentences to make a complex sentence?
Sometimes I stay after school. I take a cab or a bus home.

Copyright 2009, TCA, LLC.

(A)

If I stay after school, I take a cab, or sometimes I take a bus home.

(B)

If I stay after school, I take a cab or a bus home.

527

Studyguide for the SAT Skill Quiz B: Sentence Variety

Question 4
Which of the following combines these sentences to make a compound sentence?
Yesterday it was raining hard. My friend's dad drove us both to school.
(A)

Yesterday it was raining hard, so my friend's dad drove us both to school.

(B)

Because it was raining hard yesterday, my friend's dad drove


us both to school.

Question 5
Which of the following combines these sentences to make a compound-complex
sentence?

Next year I hope to get my driver's license. I plan to take driver's education
first. It will lower my insurance rate.

Copyright 2009, TCA, LLC.

(A)

Next year I hope to get my driver's license, and I plan to take


driver's education first to lower my insurance rate.

(B)

Next year I hope to get my driver's license, and I plan to take


driver's education first because it will lower my insurance rate.

528

Studyguide for the SAT Skill Quiz B: Sentence Variety

Question 6
Which of the following combines these sentences to make a compound-complex
sentence?
I have to pass driver's education. Then my mom will let me use her car. My dad
will pay for my insurance.

(A)

I have to pass driver's education for my mom will let me use


her car and my dad to pay for my insurance.

(B)

If I pass driver's education, my mom will let me use her car,


and my dad will pay for my insurance.

Question 7
Which of the following most effectively combines these sentences?
I hope to go to the University of Chicago. My mom went there. She loved it.

Copyright 2009, TCA, LLC.

(A)

I hope to go to the University of Chicago because my mom


went there, where she loved it.

(B)

I hope to go to the University of Chicago because my mom


went there and loved it.

529

Studyguide for the SAT Skill Quiz B: Sentence Variety

Question 8
Which of the following most effectively combines these sentences?
I may major in political science. I may pursue a double major in political
science and business.

(A)

I may major in political science, or I may pursue a double


major in political science and business.

(B)

I may major in political science, and I may pursue a double


major in political science and business.

Question 9
Which of the following most effectively combines these sentences?
I was a volunteer for a political campaign last year. That experience will
strengthen my application.

Copyright 2009, TCA, LLC.

(A)

I was a volunteer for a political campaign last year, an


experience which will strengthen my application.

(B)

I was a volunteer for a political campaign last year, and this


was an experience which will strengthen my application.

530

Studyguide for the SAT Skill Quiz B: Sentence Variety

Question 10
Which of the following most effectively combines these sentences?
Last week I met a business school graduate. He told me that he had four job
offers.

Copyright 2009, TCA, LLC.

(A)

When I met a business school graduate last summer, I was


told that he had four job offers.

(B)

Last summer I met a business school graduate, who told me


that he had four job offers.

531

Copyright 2009, TCA, LLC.

532

Studyguide for the SAT Skill Quiz B: Sentence Variety

Hints!
A

Hint for Question 1:


Which coordinating conjunction could best join these two sentences?

Hint for Question 2:


Which subordinating conjunction could best join these two sentences?

Hint for Question 3:


Which sentence could best be made into a dependent clause?

Hint for Question 4:


Which coordinating conjunction could best join these two sentences?

Hint for Question 5:


Which sentence could best be made into a dependent clause?

Hint for Question 6:


Which sentence could best be made into a dependent clause?

Hint for Question 7:


Which sentence could best be made into a dependent clause?

Hint for Question 8:


What word could be used to combine the two sentences?

Hint for Question 9:


What word could be used to combine the two sentences?

Copyright 2009, TCA, LLC.

533

Studyguide for the SAT Skill Quiz B: Sentence Variety


Hint for Question 10:
What word could be used to combine the two sentences?

Copyright 2009, TCA, LLC.

534

Studyguide for the SAT Skill Quiz B: Sentence Variety

Answers!
A

Question 1.
The answer is (B).
Explanation for Question 1:
The answer is B.
Riding the subway is fast and inexpensive, but it is not always pleasant.

Question 2.
The answer is (A).
Explanation for Question 2:
The answer is A.
I take the subway to school since there is a stop right across the street.
Question 3.
The answer is (B).
Explanation for Question 3:
The answer is B.
If I stay after school, I take a cab or a bus home.

Question 4.
The answer is (A).
Explanation for Question 4:
The answer is A.
Yesterday it was raining hard, so my friend's dad drove us both to school.

Question 5.
The answer is (B).
Explanation for Question 5:
The answer is B.
Next year I hope to get my driver's license, and I plan to take driver's education first
because it will lower my insurance rate.

Copyright 2009, TCA, LLC.

535

Studyguide for the SAT Skill Quiz B: Sentence Variety


Question 6.
The answer is (B).
Explanation for Question 6:
The answer is B.
If I pass driver's education, my mom will let me use her car, and my dad will pay for
my insurance.

Question 7.
The answer is (B).
Explanation for Question 7:
The answer is B.
I hope to go to the University of Chicago because my mom went there loved it. (1
dependent and 2 independent clauses; compound-complex sentence)
Question 8.
The answer is (A).
Explanation for Question 8:
The answer is A.
I may major in political science, or I may pursue a double major in political science
and business. (coordinating conjunction; compound sentence)

Question 9.
The answer is (A).
Explanation for Question 9:
The answer is A.
I was a volunteer for a political campaign last year, an experience which will
strengthen my application. (relative pronoun; complex sentence)

Question 10.
The answer is (B).
Explanation for Question 10:
The answer is B.
Last summer I met a business school graduate, who told me that he had four job
offers. (relative pronoun; complex sentence)

Copyright 2009, TCA, LLC.

536

Studyguide for the SAT Skill Quiz C: Sentence Variety

Quiz Time!
A

Question 1
Which of the following combines these sentences to make a complex sentence?

Last summer I worked at a stadium. I learned about security procedures.

(A)

Last summer I worked at a stadium, where I learned about


security procedures.

(B)

Last summer I worked at a stadium, and I learned about


security procedures.

Question 2
Which of the following combines these sentences to make a compound sentence?
A security consultant gave a seminar for employees. I was allowed to attend.

Copyright 2009, TCA, LLC.

(A)

A security consultant gave a seminar for employees, and I


was allowed to attend.

(B)

A security consultant gave a seminar for employees, which I


was allowed to attend.

537

Studyguide for the SAT Skill Quiz C: Sentence Variety

Question 3
Which of the following combines these sentences to make a complex sentence?
In the spring, I attended a game at the stadium. Then I applied for the job.

(A)

I applied for the job after I attended a game at the stadium in the spring.

(B)

I applied for the job and then attended a game at the stadium in the spring.

Question 4
Which of the following combines these sentences to make a compound-complex
sentence?
Sometimes there was nothing for me to do. I watched security officers monitor
the cameras. I learned the codes for security alerts.
(A)

There was nothing for me to do, so I watched security


officers monitor the cameras and I learned the codes for security alerts.

(B)

When there was nothing for me to do, I watched security


officers monitor the cameras, and I learned the codes for security alerts.

Question 5
Which of the following combines these sentences to make a complex sentence?
I plan to major in law enforcement. I will write about the job in my college
application essay.

Copyright 2009, TCA, LLC.

(A)

I plan to major in law enforcement; I will write about the job


in my college application essay.

(B)

Since I plan to major in law enforcement, I will write about


the job in my college application essay.

538

Studyguide for the SAT Skill Quiz C: Sentence Variety

Question 6
Which of the following combines these sentences to make a compound sentence?
I may specialize in electronic security. I may discover a different area that
interests me even more.
(A)

I may specialize in electronic security, or I may discover a


different area that interests me even more.

(B)

I may specialize in electronic security if I don't discover a


different area that interests me even more.

Question 7
Which of the following most effectively combines these sentences?
My uncle works for the FBI. My mom is a sheriff's deputy.

Copyright 2009, TCA, LLC.

(A)

My uncle works for the FBI, and my mom is a sheriff's


deputy.

(B)

My uncle works for the FBI although my mom is a sheriff's


deputy.

539

Studyguide for the SAT Skill Quiz C: Sentence Variety

Question 8
Which of the following most effectively combines these sentences?
I hope to do internships before graduation. Experience is a big help in getting a
job.

(A)

I hope to do internships before graduation because experience


is a big help in getting a job.

(B)

I hope to do internships before graduation, and experience is


a big help in getting a job.

Question 9
Which of the following most effectively combines these sentences?
Law enforcement depends on technology. Technology is changing rapidly. I will
continue to learn throughout my career.

Copyright 2009, TCA, LLC.

(A)

Law enforcement depends on technology, which is changing


rapidly, so I will continue to learn throughout my career.

(B)

Because law enforcement depends on technology, which is


changing rapidly, so I will continue to learn throughout my career.

540

Studyguide for the SAT Skill Quiz C: Sentence Variety

Question 10
Which of the following most effectively combines these sentences?
My parents support my plans to enter law enforcement. They are concerned
about my safety.

Copyright 2009, TCA, LLC.

(A)

My parents support my plans to enter law enforcement


although they are concerned about my safety.

(B)

My parents support my plans to enter law enforcement; they


are concerned about my safety.

541

Copyright 2009, TCA, LLC.

542

Studyguide for the SAT Skill Quiz C: Sentence Variety

Hints!
A

Hint for Question 1:


What subordinating conjunction could be used to combine the sentences?

Hint for Question 2:


What coordinating conjunction could be used to combine the sentences?

Hint for Question 3:


Which sentence could best be made into a dependent clause?

Hint for Question 4:


Which sentence could best be made into a dependent clause?

Hint for Question 5:


Which sentence could best be made into a dependent clause?

Hint for Question 6:


What coordinating conjunction could be used to combine the sentences?

Hint for Question 7:


What word could best be used to combine the sentences?

Hint for Question 8:


What word could best be used to combine the sentences?

Hint for Question 9:


Which sentence could best be made into a dependent clause?

Copyright 2009, TCA, LLC.

543

Studyguide for the SAT Skill Quiz C: Sentence Variety


Hint for Question 10:
What word could best be used to combine the sentences?

Copyright 2009, TCA, LLC.

544

Studyguide for the SAT Skill Quiz C: Sentence Variety

Answers!
A

Question 1.
The answer is (A).
Explanation for Question 1:
The answer is A.
Last summer I worked at a stadium, where I learned about security procedures.

Question 2.
The answer is (A).
Explanation for Question 2:
The answer is A.
A security consultant gave a seminar for employees, and I was allowed to attend.
Question 3.
The answer is (A).
Explanation for Question 3:
The answer is A.
I applied for the job after I attended a game at the stadium in the spring.

Question 4.
The answer is (B).
Explanation for Question 4:
The answer is B.
When there was nothing for me to do, I watched security officers monitor the
cameras, and I learned the codes for security alerts.

Question 5.
The answer is (B).
Explanation for Question 5:
The answer is B.
Since I plan to major in law enforcement, I will write about the job in my college
application essay.

Copyright 2009, TCA, LLC.

545

Studyguide for the SAT Skill Quiz C: Sentence Variety


Question 6.
The answer is (A).
Explanation for Question 6:
The answer is A.
I may specialize in electronic security, or I may discover a different area that interests
me even more.

Question 7.
The answer is (A).
Explanation for Question 7:
The answer is A.
My uncle works for the FBI, and my mom is a sheriff's deputy. (coordinating
conjunction; compound sentence)
Question 8.
The answer is (A).
Explanation for Question 8:
The answer is A.
I hope to do internships before graduation, because experience is a big help in getting
a job. (subordinating conjunction; complex sentence)

Question 9.
The answer is (A).
Explanation for Question 9:
The answer is A.
Law enforcement depends on technology, which is changing rapidly, so I will
continue to learn throughout my career. (1 dependent and 2 independent clauses;
compound-complex sentence)

Question 10.
The answer is (A).
Explanation for Question 10:
The answer is A.
My parents support my plans to enter law enforcement, although they are concerned
about my safety. (subordinating conjunction; complex sentence)

Copyright 2009, TCA, LLC.

546

Writing Lesson #9

Grammar, Usage and Mechanics

Triumph College Admissions


Skill Lesson

Quiz Time!
A

Hints!

Answers!
A

*Throughout this documentation, and the software: College Board and SAT are registered trademarks of the College Entrance Examination Board. PSAT/NMSQT
is a trademark of the College Entrance Examination Board and National Merit Scholarship Corporation. ACT is a trademark of ACT, Inc. None of these entities are
affiliated with the production of, nor endorse these materials.
Copyright 2009, TCA, LLC.

547

Copyright 2009, TCA, LLC.

548

Studyguide for the SAT Essay Grammar, Usage and Mechanics

Because your SAT essay will be a draft, it will probably not be


completely free of errors in grammar, usage, and mechanics. You
should, however, try to avoid the types of errors that can make your
writing less clear and lower your score. These include the following:
Sentence fragments
Incorrect example: They began to flee. Running away as fast as
they could. (The second phrase is a fragment since it has no subject.)
Correct example: They began to flee, running away as fast as
they could. (Combining the sentences eliminates the fragment, making
"they" the subject.)
Run-on sentences, including comma splices
Incorrect example: I walked the dog, Kevin washed the car.
(Each of these sentences can stand alone, with its own subject and verb.
Separating them with only a comma creates a comma splice.
Correct example: I walked the dog while Kevin washed the car.
(Combining the sentences with a conjunction eliminates the comma
splice.)
Agreement errors
A subject and verb must agree in number. If your subject is plural, the
verb that goes with it must reflect that. A pronoun must agree with its
antecedent in number and in gender.
Incorrect example: Several people in the room thinks that Jody
should be elected president.
Correct example: Several people in the room think that Jody
should be elected president. (The subject is "people," which is plural, so
its verb is "think.")
Incorrect example: One of the people think that Mackenzie
should run for vice president.

Copyright 2009, TCA, LLC.

549

Studyguide for the SAT Essay Grammar, Usage and Mechanics


Correct example ; One of the people thinks that Mackenzie should run
for vice president. (The subject is not "people," but "one," which is
singular. Therefore, its verb should be "thinks."
Incorrect example: If Joel is to run against Jody, she'll have to
win over a lot of her supporters. (This doesn't make sense. Jody would not
have to win over her own supporters; Joel would have to win them over.)
Correct example: If Joel is to run against Jody, he'll have to win
over a lot of her supporters.
Incorrect example: If someone decides to run for a position on
the board, they must first be nominated.
Correct example: If someone decides to run for a position on the
board, he or she must first be nominated. ("Someone" is singular, but the
pronoun "they" is plural. "He or she" is the correct pronoun to use with
"someone" whose gender is not specific.)
You should refer to the Multiple Choice Writing Section for more instruction
on this subject.

Copyright 2009, TCA, LLC.

550

S-ar putea să vă placă și